You are on page 1of 438

San Beda University

Mendiola, Manila
College of Law

Case Digests
In
Criminal Law 1

Submitted by:
Block 1-M A.Y. 2020 – 2021

Submitted to:
Atty. Lyan David Juanico
CRIMINAL LAW 1 CASE DIGESTS
ATTY. LYAN DAVID JUANICO
BLOCK 1-M A.Y. 2020 - 2021

MODULE 1: GENERAL PRINCIPLES OF CRIMINAL LAW 1


1. Ient v. Tullett Prebon
G.R. No. 189158 January 11, 2017
LEONARDO-DE CASTRO, J.

FACTS:
James Ient and Schulze and Villalon were employees of Tullet Prebon, but upon some frustration
with the upper management, they decided to scout for new job opportunities. Tradition Philippines
was a rival of Tullett in their mother country and is looking to expand in the Philippines. James
Ient together with Schulze and Villalon quit tullett and spearheaded the former’s expansion.
Because of this, Tullett filled a case against Ient and the others because they secretly met with
Tradition and then met with the senior brokers to convince them to join Tradition, they in turn
contend that the latter is violating the corporation code specifically Section 31 in relation to section
144. Ient and the others contended that they did not violate any laws because they were only
seeking “greener pastures” to further their careers, and that the meeting with other employees
were just a formality to bid goodbye to former workmates. The mass resignation of Tullett’s
employees were done out of their own free will and that Ient et. Al. Had nothing to do with it. The
state prosecutor dismissed the criminal complaints saying that there was no basis for such claim
that there was conspiracy to committ acts that violate Section 31 of the Corporation code. There
was an argument whether the article 144 of the corporation code speaks about imprisonment or
only civil liability. There was a clear ambiguity on hot to apply section 31 and 34 of the
Corporation code in relation to section 144.The secretary of Justice found them guilty. They
appealed the case to the CA but it affirmed the secretary’s decision and found them guilty of such
acts.
ISSUE:
Whether or not Ient and the others are liable for the said acts and should be punished by
imprisonment.
RULING:
The supreme court held that NO, they cannot be held liable for the said crimes. When interpreting
penal laws that has ambiguity such as this, they must first go back and see the spirit of the law, as
said in the deliberations of the congress on the aforesaid law, it was not intended to punish but to
impose exacting standards of fidelity and other corporate values. When congress intends to
criminalize an act, it does so in clear and plain categorical language so as not to be questionable in
its nature. Applying the rule of lenity, the Supreme court must rule in dubio pro reo. The case was
reversed and set aside in the favor of the petitioners.

1
CRIMINAL LAW 1 CASE DIGESTS
ATTY. LYAN DAVID JUANICO
BLOCK 1-M A.Y. 2020 - 2021

2. Minucher vs. Court of Appeals


G.R. No. 142396 February 11, 2003
Vitug, J.

FACTS:
Khosrow Minucher was an Iranian Exchange student back in 1974 but Iran was destabilized and
he wasn’t able to comeback. He became a selleer of Iranian products. In 1986 he met with a
potential buyer named Jose Inigo that wants to buy caviar. He entertained the said interested buyer,
thereafter the meeting, the buyer told him that his interest piqued and wanted to buy carpets also,
they haggled and reached the price of $24,000, they then proceeded to Minucher’s resident to fetch
the carpets, when suddenly men with high powered guns went inside his house handcuffed
Minucher and went in his room opened his vault and got the $24,000 together with his other
belongings. Minucher was entraped with the suspicion of having heroin under dangerous drugs act
but was later acquitted by the court. Minucher filed a case against the Agent in charge who was
Scalzo a DEA agent from the United States of America. Scalzo invoked that he has diplomatic
immunity and could not be prosecuted under the Philippine courts.
ISSUE:
Whether or not Scalzo is immune from suit because of his dimplomatic immunity.
RULING:
YES. Scalzo is immune from suit. A foreign agent if it can be established that he was sent by the
country on which it is a member of the Vienna Convention. Diplomatic immunity is afforded to
those Heads of diplomatic missions and their diplomatic staff excluding the members of the
administrative, technical and service staff of the mission are accorded diplomatic rank. In the
present case, Scalzo was a member of the Diplomatic staff, thus immune from suit.

2
CRIMINAL LAW 1 CASE DIGESTS
ATTY. LYAN DAVID JUANICO
BLOCK 1-M A.Y. 2020 - 2021

3. Liang vs People of the Philippines


G.R. No. 125865 January 28, 2000
YNARES-SANTIAGO , J.

FACTS:
Petitioner was an economist working for Asian Development bank at the time of the incident.
Joyce Cabal was an employee of ADB but had a lower rank than Liang, Liang then yelled
defamatory words at Cabal insinuating that she did something. Cabal filed a case against Liang for
the Defamatory words in the MeTC but the judge was informed by the ADB that Liang was
immune to suit because of ADB having an agreement with the Philippines. When the Judge
dismissed the case, the prosecution filed it on the RTC and the same issued a warrant of arrest
against herein petitioner. Liang contended that he was under the immunity of the Section 45 of the
Agreement by the ADB and the Philippines.
ISSUE:
Whether or not Liang is indeed immune from suit.
RULING:
No, Liang is not immune from suit, under the agreement, it is said that he will be protected from
suit ON HIS CORPORATE FUNCTIONS, slandering someone is not one of those functions, thus
he is not entitled to the immunity because he performed ulta vires acts of corporations.

3
CRIMINAL LAW 1 CASE DIGESTS
ATTY. LYAN DAVID JUANICO
BLOCK 1-M A.Y. 2020 - 2021

4. Del Socorro vs Van Wilsem


G.R. No. 193707 December 10, 2014
PERALTA, J.

FACTS:
Norma A. Del Socorro and Ernst Van Wilsem contracted marriage in Holland. They were blessed
with a son named Roderigo Norjo Van Wilsem. Unfortunately, their marriage bond ended by virtue
of a Divorce Decree issued by the appropriate Court of Holland. Thereafter, Norma and her son
came home to the Philippines. According to Norma, Ernst made a promise to provide monthly
support to their son. However, since the arrival of petitioner and her son in the Philippines, Ernst
never gave support to Roderigo. Respondent remarried again a Filipina and resides again the
Philippines particulary in Cebu where the petitioner also resides. Norma filed a complaint against
Ernst for violation of R.A. No. 9262 for the latter’s unjust refusal to support his minor child with
petitioner. The trial court dismissed the complaint since the facts charged in the information do
not constitute an offense with respect to the accused, he being an alien
ISSUE:
Whether or not a foreign national can be held criminally liable under R.A. No. 9262 for his
unjustified failure to support his minor child.
RULING:
YES. The court has jurisdiction over the offense (R.A 9262) because the foreigner is living here
in the Philippines. The status of the crime is a continuing crime, so when he went back here, he
has committed the crime here, thus, he is liable under the Philippine courts

4
CRIMINAL LAW 1 CASE DIGESTS
ATTY. LYAN DAVID JUANICO
BLOCK 1-M A.Y. 2020 - 2021

5. Gonzales vs. Abaya


G.R. No. 164007. August 10, 2006
SANDOVAL-GUTIERREZ, J.

FACTS:
On July 27, 2003 at around 1:00 a.m., more than 300 heavily armed junior officers and enlisted
men of the AFP entered the premises of the Oakwood Premier Luxury Apartments on Ayala
Avenue, Makati City. They disarmed the security guards and planted explosive devices around the
building. They declared their withdrawal of support from their Commander-in-Chief and called
for her resignation and her cabinet members. President Arroyo declared a state of rebellion
(Proclamation No. 427) and directed the AFP and PNP (Gen. Order No. 4) to suppress the
rebellion. The negotiators sent by the government succeeded in convincing a total of 321 soldiers
to lay down their arms and defuse the explosives placed around the premises of the Oakwood
Apartments. The National Bureau of Investigation (NBI) investigated the incident and
recommended that the military personnel involved be charged with coup d’etat . Pursuant to Article
70 of the Articles of War, respondent General Narciso Abaya ordered the arrest and detention of
the soldiers involved in the Oakwood incident and directed the AFP to conduct its own separate
investigation. On August 5, 2003, the DOJ filed with the Regional Trial Court (RTC), Makati City
an Information for coup d’etat against those soldiers. On the same date, respondent Chief of Staff
(Letter Order No. 625) created a Pre-Trial Investigation Panel tasked to determine the propriety of
filing with the military tribunal charges for violations of the Articles of War under Commonwealth
Act No. 408, as amended, against the same military personnel. Specifically, the charges are: (a)
violation of Article 63 for disrespect toward the President, the Secretary of National Defense, etc.,
(b) violation of Article 64 for disrespect toward a superior officer, violation of Article 67 for
mutiny or sedition, (d) violation of Article 96 for conduct unbecoming an officer and a gentleman,
and (e) violation of Article 97 for conduct prejudicial to good order and military discipline. Of the
original 321 accused, 148 filed an Omnibus Motion praying that the said trial court assume
jurisdiction over all the charges filed with the military tribunal invoking Republic Act (R.A.) No.
7055. In its Initial Report, the Pre-Trial Investigation Panel recommended that the military
personnel involved in the Oakwood incident be charged before a general court martial with
violations of Articles 63, 64, 67, 96, and 97 of the Articles of War. In its Final Pre-Trial
Investigation Report to the JAGO, it recommended that, following the "doctrine of absorption,"
those charged with coup d’etat before the RTC should not be charged before the military tribunal
for violation of the Articles of War. The DOJ, after conducting a reinvestigation, found probable
cause against only31 of the 321 accused and filed with the RTC an Amended Information, which
the RTC admitted so it dropped the charge of coup d’etat against the 290 accused. The RTC issued
an Order stating that "all charges before the court martial against the accused...are hereby declared
not service-connected, but rather absorbed and in furtherance of the alleged crime of coup d’etat."
The trial court then proceeded to hear petitioners’ applications for bail. Colonel Julius A.
Magno(OIC of the JAGO), reviewed the findings of the Pre-Trial Investigation Panel and
recommended that 29 of the officers involved in the Oakwood incident be prosecuted before a
general court martial for violation of Article 96 (conduct unbecoming an officer and a gentleman)
of the Articles of War. The recommendation was approved by the AFP top brass. Instead of
complying, the petitioners filed the instant Petition for Prohibition praying that respondents be
ordered to desist from charging them with violation of Article 96 of the Articles of War in relation
to the Oakwood incident. Petitioners maintain that since the RTC has made a determination in its
Order of February 11, 2004 that the offense for violation of Article 96 (conduct unbecoming an

5
CRIMINAL LAW 1 CASE DIGESTS
ATTY. LYAN DAVID JUANICO
BLOCK 1-M A.Y. 2020 - 2021

officer and a gentleman) of the Articles of War is not service-connected, but is absorbed in the
crime of coup d’etat , the military tribunal cannot compel them to submit to its jurisdiction.
ISSUE:
Whether or not the Conduct of Unbecoming an Officer (Article 96 of the Articles of War) is
service-connected and therefore cognizable by the court Martial.
RULING:
As to the jurisdiction of the court the general rule is Members of the AFP and other persons subject
to military law who commit crimes or offenses penalized under the Revised Penal Code (like coup
d’etat), other special penal laws, or local ordinances shall be tried by the proper civil court. The
exception is where the civil court, before arraignment, has determined the offense to be service-
connected, then the offending soldier shall be tried by a court martial. But the exception to the
exception is that the President of the Philippines, in the interest of justice, directs before
arraignment that any such crimes or offenses be tried by the proper civil court.

It bears stressing that the charge against the petitioners concerns the alleged violation of their
solemn oath as officers to defend the Constitution and the duly-constituted authorities. Such
violation allegedly caused dishonor and disrespect to the military profession. In short, the charge
has a bearing of their professional conduct or behavior as military officers. Equally indicative of
the "service-connected" nature of the offense is the penalty prescribed for the same (under Art. 96
of Articles of War) – dismissal from the service –imposable only by the military court.
The RTC, in making the declaration that Art 96 of Articles of War as “not sevice-connected, but
rather absorbed and in furthenance of the crime of coup d’etat”, practically amended the law which
expressly vests in the court martial the jurisdiction over "service-connected crimes or offenses." It
is only the Constitution or the law that bestows jurisdiction on the court, tribunal, body or officer
over the subject matter or nature of an action which can do so. Evidently, such declaration by the
RTC constitutes grave abuse of discretion tantamount to lack or excess of jurisdiction and is,
therefore, void.

6
CRIMINAL LAW 1 CASE DIGESTS
ATTY. LYAN DAVID JUANICO
BLOCK 1-M A.Y. 2020 - 2021

6. People vs Tulin
G.R. No. 111709 August 30, 2001
Ynares- Santiago, J.

FACTS:
“M/T Tabangao,” a cargo vessel loaded fuel was sailing off the coast of Mindoro near Silonay
Island when it was suddenly boarded, by seven fully armed pirates. The pirates were armed with
M-16 rifles, .45 and .38 caliber handguns, and bolos. They detained the crew and took complete
control of the vessel. “M/T Tabangao” then sailed to and anchored about 10 to 18 nautical miles
from Singapore’s shoreline where another vessel called “Navi Pride” received the cargo under the
supervision of accused-appellant Cheong San Hiong.
Accused-appellants were arrested and charged with qualified piracy for violating Presidential
Decree No. 532 (Piracy in Philippine Waters) and were convicted as principals of the crime
charged, except for accused-appellant Hiong who was convicted as an accomplice. On appeal,
Hiong ratiocinates that he cannot be convicted of piracy in Philippine waters as defined and
penalized in Sections 2(d) and 3(a), respectively of PD 532 because Republic Act No. 7659 has
impliedly superseded PD 532. He reasons out that Presidential Decree No. 532 has been rendered
“superfluous or duplicitous” because both Article 122 of the Revised Penal Code, as amended, and
Presidential Decree No. 532 punish piracy committed in Philippine waters. He maintains that in
order to reconcile the two laws, the word “any person” mentioned in Section 1 (d) of Presidential
Decree No. 532 must be omitted such that Presidential Decree No. 532 shall only apply to
offenders who are members of the complement or to passengers of the vessel, whereas Republic
Act No. 7659 shall apply to offenders who are neither members of the complement or passengers
of the vessel, hence, excluding him from the coverage of the law.
ISSUE:
Whether or not the accused-appellant Hiong was guilty of piracy?
RULING:
Yes, Hiong was guilty of piracy. Article 122 of the Revised Penal Code, before its amendment,
provided that piracy must be committed on the high seas by any person not a member of its
complement nor a passenger thereof. Upon its amendment by Republic Act No. 7659, the coverage
of the pertinent provision was widened to include offenses committed “in Philippine waters.” On
the other hand, under Presidential Decree No. 532 (issued in 1974), the coverage of the law on
piracy embraces any person including “a passenger or member of the complement of said vessel
in Philippine waters.” Hence, passenger or not, a member of the complement or not, any person is
covered by the law.
Republic Act No. 7659 neither superseded nor amended the provisions on piracy under
Presidential Decree No. 532. There is no contradiction between the two laws. There is likewise no
ambiguity and hence, there is no need to construe or interpret the law. All the presidential decree
did was to widen the coverage of the law, in keeping with the intent to protect the citizenry as well
as neighboring states from crimes against the law of nations. As expressed in one of the “whereas”
clauses of Presidential Decree No. 532, piracy is “among the highest forms of lawlessness
condemned by the penal statutes of all countries.” For this reason, piracy under the Article 122, as
amended, and piracy under Presidential Decree No. 532 exist harmoniously as separate laws.

7
CRIMINAL LAW 1 CASE DIGESTS
ATTY. LYAN DAVID JUANICO
BLOCK 1-M A.Y. 2020 - 2021

7. People vs. Lol-lo


G.R. No. 17958 February 27, 1992
Malcolm J.

FACTS:
On June 30, 1920, sixer vintas intercepted two Dutch boats which were on its way in the midst of
the islands of Buang and Bukid in the Dutch East Indies. The six vintas were manned by 24 armed
Moros. The said Dutch boats were carrying men, women and children. At first, the Moros asked
for food. But when they got on the Dutch boats, they asked for themselves all the vessel’s cargo,
attacked nearly all of the men and brutally violated two of the women by methods too tremendous
to be described. All of the persons on the Dutch boat, except the two young women, were again
placed on it and holes were made in it, the idea that it would submerge. The Moros finally arrived
at Maruro, a Dutch possession. Two of the Moro marauders were Lol-lo, who also raped one of
the women, and Saraw. At Maruro, the two women were able to escape. Lol-lo and Saraw later
returned to their home in South Ubian, Tawi-Tawi, Sulu. They were arrested there and were
charged in the Court of First Instance of Sulu with the crime of piracy.
ISSUE:
Whether or not Philippine courts have jurisdiction over the crime of piracy alleged in this case.
RULING:
Yes, the Philippine courts have jurisdiction on the case. Piracy is a villainy not against any
particular state but against all mankind. It should be tried and punished in the sufficient tribunal of
any country where the offender may be found or into which he may be carried. The jurisdiction of
piracy, unlike all other crimes, has no territorial limits.

8
CRIMINAL LAW 1 CASE DIGESTS
ATTY. LYAN DAVID JUANICO
BLOCK 1-M A.Y. 2020 - 2021

8. White Light Corporation v. City of Manila


G.R. No. 122846, January 20, 2009
Tinga, J.:

FACTS:
On December 3, 1992, City Mayor Alfredo S. Lim signed into law an Ordinance No. 7774 that
prohibits hotels, motels, lodging houses, pension houses and similar establishments from offering
short-time admission, as well as pro-rated or “wash-up” rates for such abbreviated stays in the City
of Manila.

Petitioners White Light Corporation (WLC), Titanium Corporation (TC) and Sta. Mesa Tourist
and Development Corporation (STDC) filed a motion to intervene on the complaint filed by The
Malate Tourist and Development Corporation (MTDC) on the ground that the Ordinance directly
affects their business interests as operators of drive-in-hotels and motels in Manila, and that motion
was granted by the Regional Trial Court of Manila. MTDC moved a motion to withdraw while the
three (3) other companies agreed to submit the case for decision without trial as the case involved
a purely legal question.

On October 20, 1993, the RTC rendered a decision declaring the Ordinance null and void and the
preliminary injunction issued was made permanent. The RTC declared that the Ordinance “strikes
at the personal liberty of the individual guaranteed and jealously guarded by the Constitution”.
That the Constitution itself encourages private enterprises and the incentive to need investment, as
well as the right to operate economic enterprises.

Before the Court of Appeals, the City asserted that the Ordinance was a valid exercise of police
power pursuant to the Local Government Code which gives them the power to regulate the
establishment, operation and maintenance of cafes, restaurants, beer houses, hotels, motels, inns,
pension houses, lodging houses and other similar establishments, including tourist guides and
transports. They further argued that the Ordinance was in accordance with the power granted by
the Revised Manila Charter. On the other hand, petitioners argued that the Ordinance is
unconstitutional and void since it violates the right to privacy and the freedom of movement, that
it is an invalid exercise of police power and is an unreasonable and oppressive interference in their
business.

The Court of Appeals ruled against the petitioners. It reversed the decision of the RTC and affirmed
the constitutionality of the Ordinance. It held that the Ordinance did not violate the right to privacy
or the freedom of movement as it only penalizes the owners or operators of establishments that
admit individuals for short time stays. That the lawful objective of the Ordinance is satisfied since
it aims to curb immoral activities and that there is a lawful method since the establishments are
still allowed to operate. And that was held in Ermita-Malate Motel Operators Association v. City
Mayor of Manila, liberty is regulated by law.

ISSUE:
Whether Ordinance No. 7774 is constitutional.

RULING:
No, Ordinance No. 7774 is unconstitutional.
For an ordinance to be valid and constitutional, it must not only be within the corporate powers of
the local government unit to enact and pass according to the procedure prescribed by law, but must

also conform to the following substantive requirements: 1. Must not contravene the Constitution
any statute; 2. Must not be unfair or oppressive; 3. Must not be partial or discriminatory; 4. Must

9
CRIMINAL LAW 1 CASE DIGESTS
ATTY. LYAN DAVID JUANICO
BLOCK 1-M A.Y. 2020 - 2021

not prohibit but may regulate trade; 5. Must be general and consistent with public policy; and 6.
Must not be unreasonable.

The goal of the Ordinance is to minimize the use of the covered establishments for illicit sex,
prostitution, drug use and alike, and it certainly fall within the ambit of the police power of the
State. But the means for their achievement must align with the Bill of Rights guaranteed to the
people. Right to due process is among the rights guaranteed by the Constitution. The purpose of
which is to prevent arbitrary governmental encroachment against the life, liberty and property of
individuals. It serves as a protection against arbitrary regulation or seizure. Corporations and
partnerships are protected by the guaranty insofar as their property is concerned.

The rights at stake herein fall within the same fundamental rights to liberty. In accordance with
this case, the rights of the citizen to be free to use his faculties in all lawful ways; to live and work
where he will; to earn his livelihood by any lawful calling; and to pursue any avocation are all
deemed embraced in the concept of liberty. The primary intention of the Ordinance is the
curtailment of sexual behavior, as the City asserts that the subject establishments have gained
notoriety as venue of ‘prostitution, adultery and fornications’ in Manila since they provide the
necessary atmosphere for clandestine entry, presence and exit and thus became the ‘ideal haven
for prostitutes and thrill-seekers’. But even if this is accurate, it cannot be denied that legitimate
sexual behavior among willing married or consenting single adults which is constitutionally
protected will be curtailed as well.

The Ordinance would proscribe or impair other legitimate activities. There are very legitimate uses
for a wash rate or renting the room out for more than twice a day. Entire families are known to
choose to pass the time in a motel or hotel whilst the power is momentarily out in their homes. In
transit passengers who wish to wash up and rest between trips have a legitimate purpose for
abbreviated stays in motels or hotels. Indeed any person or groups of persons in need of
comfortable private spaces for a span of a few hours with purposes other than having sex or using
illegal drugs can legitimately look to staying in a motel or hotel as a convenient alternative.

Another constitutional requisite for the legitimacy of the Ordinance as a police power measure-It
must appear that the interests of the public generally, as distinguished from those of a particular
class, require an interference with private rights and the means must be reasonably necessary for
the accomplishment of the purpose and not unduly oppressive of private rights. It must also be
evident that no other alternative for the accomplishment of the purpose less intrusive of private
rights can work. More importantly, a reasonable relation must exist between the purposes of the
measure and the means employed for its accomplishment, for even under the guise of protecting
the public interest, personal rights and those pertaining to private property will not be permitted to
be arbitrarily invaded. The Ordinance makes no distinction between places frequented by patrons
engaged in illicit activities and patrons engaged in legitimate actions. Thus it prevents legitimate
use of places where illicit activities are rare or even unheard of.

However well-intentioned the Ordinance may be, the arbitrary and whimsical intrusion into the
rights of the people cannot be overlooked. Therefore, the Ordinance is void and unconstitutional.

10
CRIMINAL LAW 1 CASE DIGESTS
ATTY. LYAN DAVID JUANICO
BLOCK 1-M A.Y. 2020 - 2021

9. Garcia v. Drilon
G.R. No. 179267, June 25, 2013
PERLAS-BERNABE, J.:

FACTS:
On March 23, 2006, private respondent Rosalie Jaype-Garcia filed a verified petition (Civil Case
no. 06-797) for issuance of a Temporary Protection Order (TPO) before the Regional Trial Court
of Bacolod City against her husband, petitioner herein Jesus C. Garcia, pursuant to R.A. 9262. She
claimed that she and their three (3) children are victims of physical abuse; emotional,
psychological, and economic violence as a result of marital infidelity on the part of petitioner, with
threats of deprivation of custody of her children and of financial support.She contends that
petitioner admitted to the affair when he was confronted. He even boasted to the household help
about his sexual relations with the bank manager of Robinson’s Bank, Bacolod City who happens
to be the godmother of one of their sons. In one of their quarrels, the petitioner grabbed the private
respondent on both arms and shook her with such force that caused bruises and hematoma. Private
respondent decided to leave the petitioner but their children begged her to stay for fear that if she
leaves, the petitioner would beat them again. All the emotional and psychological turmoil drove
private respondent to the brink of repair which brought her to attempt suicide. She was found by
her son bleeding on the floor. Seven (7) days being hospitalized, the petitioner never bothered to
visit her, nor apologized or showed pity on her.

Finding reasonable grounds to believe that an imminent danger of violence against the private
respondent and her children exists or is about to recur, the RTC issued a TPO. The TPO was
renewed and extended yet again because the petitioner continued to deprive them of financial
support, failed to comply with the TPO, and committed new acts of harassment against the private
respondent and their children.

During the pendency of the civil case, petitioner filed before the Court of Appeals a petitioner for
prohibition, with prayer for injunction and temporary restraining order (TRO), challenging the
constitutionality of R.A. 9262 for being violative of the due process and the equal protection
clause, and the validity of the modified TPO issued in the civil case for being “unwanted product
of an invalid law”.

The appellate court issued a TRO against the enforcement of TPOs. But subsequently, the CA
dismissed the petition for failure of petitioner to raise the constitutional issue in his pleadings
before the trial court in the civil case, which is clothed with jurisdiction to resolve the same.
Secondly, the challenge to the validity of R.A. 9262 through a petition for prohibition seeking to
annul the protection orders issued by the trial court constituted a collateral attack on said law.

His motion for reconsideration having been denied, the petitioner is now before the Supreme Court.

ISSUE:
Whether or not R.A. 9262 is violative of 1) equal protection clause and 2) due process clause.

RULING:

1. No, R.A. 9262 does not violate the guarantee of equal protection of the laws. Equal protection
simply requires that all persons or things similarly situated should be treated alike, both as to rights
conferred and responsibilities imposed. The equal protection of the laws clause of the Constitution
allows classification. Classification in law, as in the other departments of knowledge or practice,
is the grouping of things in speculation or practice

11
CRIMINAL LAW 1 CASE DIGESTS
ATTY. LYAN DAVID JUANICO
BLOCK 1-M A.Y. 2020 - 2021

because they agree with one another in certain particulars. All that is required of a valid
classification is that it be reasonable, which means that the classification should be based on
substantial distinctions which make for real differences; that it must be germane to the purpose of
the law; that it must not be limited to existing conditions only; and that it must apply equally to
each member of the class.

a. The law rests on substantial distinctions. The unequal power relationship between
women and men; the fact that women are more likely than men to be victims of violence; and the
widespread gender bias and prejudice against women all make for real differences justifying the
classification under the law.
b. The classification is germane to the purpose of the law. The distinction between men
and women is germane to the purpose of R.A. 9262, which is to address violence committed
against women and children, spelled out in its Declaration of Policy. Towards this end, the State
shall exert efforts to address violence committed against women and children in keeping with the
fundamental freedoms guaranteed under the Constitution and the provisions of the Universal
Declaration of Human Rights, the Convention on the Elimination of All Forms of Discrimination
Against Women, Convention on the Rights of the Child and other international human rights
instruments of which the Philippines is a party.
c. The classification is not limited to existing conditions only. The application of R.A. 9262
is not limited to the existing conditions when it was promulgated, but to future conditions as well,
for as long as the safety and security of women and their children are threatened by violence and
abuse. R.A. 9262 applies equally to all women and children who suffer violence and abuse, which
the law clearly defines. There is nothing in the definition of VAWC that is vague and ambiguous
that will confuse the petitioner in his defense. The acts enumerated above are easily understood
and provide adequate contrast between the innocent and the prohibited acts. They are worded with
sufficient definiteness that persons of ordinary intelligence can understand what conduct is
prohibited, and need not guess at its meaning nor differ in its application.
d. Applies equally to all the members. There is likewise no merit to the contention that
R.A. 9262 singles out the husband or father as the culprit. As defined above, VAWC may likewise
be committed "against a woman with whom the person has or had a sexual or dating relationship."
Clearly, the use of the gender-neutral word "person" who has or had a sexual or dating relationship
with the woman encompasses even lesbian relationships. Moreover, while the law provides that
the offender be related or connected to the victim by marriage, former marriage, or a sexual or
dating relationship, it does not preclude the application of the principle of conspiracy under the
Revised Penal Code.

2. No, R.A. 9262 is not violative of due process clause. It cannot be said that R.A. 9262
disregard protections afforded by the due process clause of the Constitution, specifically in the
issuance of Protection Orders. A protection order is an order issued to prevent further acts of
violence against women and their children, their family or household members, and to grant other
necessary reliefs. Its purpose is to safeguard the offended parties from further harm, minimize any
disruption in their daily life and facilitate the opportunity and ability to regain control of their life.
The court is authorized to issue ex parts a TPO when the life, limb or property of the victim is in
jeopardy and there is reasonable ground to believe that the order is necessary to protect the victim
from the immediate and imminent danger of VAWC or to prevent such violence, which is about
to recur.

There should not be any fear that the judge may have no rational basis to issue an ex parte
order. The victim is required not only to verify the allegations in the petition, but also to attach her
witnesses' affidavits to the petition. Moreover, the rule requires that petitions for protection order

12
CRIMINAL LAW 1 CASE DIGESTS
ATTY. LYAN DAVID JUANICO
BLOCK 1-M A.Y. 2020 - 2021

be in writing, signed and verified by the petitioner, undertaking full responsibility, criminal or
civil, for every allegation therein.
It should be recalled that the petitioner filed an Opposition to the Urgent Ex-Parte Motion for
Renewal of the TPO, and also filed a motion for modification of the TPO to allow him visitation
rights to his children. The Court gave him five (5) days to show cause why the TPO should not be
renewed and extended, but he did not comply. Having failed to do so, the petitioner may not now
be heard to complain that he was denied due process of law.

13
CRIMINAL LAW 1 CASE DIGESTS
ATTY. LYAN DAVID JUANICO
BLOCK 1-M A.Y. 2020 - 2021

10. U. S. v. Bustos
G.R. No. L-12592, March 8, 1918
MALCOLM, J.:

FACTS:
In 1915, numerous citizens of the Province of Pampanga signed a petition to the Executive
Secretary charging Roman Punsalan, justice of peace of Macabebe and Masantol, Pampanga, with
malfeasance in office and asking for his removal. Because allegedly, the justice of peace was
asking extra charges from people if they wish to win their cases. The justice of peace was notified
and denied the charges. The judge of first instance recommended to the Governor-General that
Roman Punsalan be removed from his position as justice of the peace.

Later, the justice of peace filed a motion for a new trial which was granted. He asserts that he was
the victim of prosecution and that the charges instituted was because of personal reasons. Criminal
action against the petitioners, now defendants, was instituted. That the said accused voluntarily,
illegally, and criminally and with malicious intent to prejudice and defame Roman Punsalan by
publishing a writing which was false, scandalous, malicious, and libelous against the justice of the
peace; that the latter is absolutely unfair, eminently immoral and dangerous to the community, and
consequently unworthy of the office; and that because if these false allegations, some citizens of
the town were compelled to present an administrative case against him. All of these attacked the
virtue , honor, and reputation of Roman Punsalan as justice of the peace and thus exposing him to
public hatred, contempt and ridicule.

The court found the defendants guilty and sentenced them to pay a fine.

ISSUE:
Whether or not the defendants are guilty for libel against the justice of the peace.

RULING:
No. The Constitution, the Bill of Rights specifically, guarantees the people the freedom of speech,
which grants the complete liberty to comment on the conduct of public servants. The interest of
society and the maintenance of good government demand a full discussion of public affairs. A
public officer must not be thin-skinned with reference to comment upon his official acts. Criticism
does not authorize defamation, nevertheless, as the individual is less than the State, so must expect
criticism to be born for the common good. The guarantees of a free speech and a free press include
the right to criticize judicial conduct because the administration of the law is a matter of vital public
concern. If the people cannot criticize a justice of the peace or a judge the same as any other public
officer, public opinion will be effectively muzzled.

The right to assemble and petition is also a part of free speech. Assembly means a right on the part
of citizens to meet peaceably for consultation in respect to public affairs; and petition means that
any person can apply, without fear of penalty, to the appropriate branch or office of the government
to redress grievances. This qualified privilege however may be lost by proof of malice, which lies
on the plaintiff.

In the case at bar, the plaintiff failed to prove the existence of malice as the true motive of the
defendant’s conduct. Express malice has not been proved by the prosecution. Further, although the
charges are probably not true as to the justice of the peace, they were believed to be true by the
petitioners. Good faith surrounded their action and in no way abused their privilege. Therefore, the

14
CRIMINAL LAW 1 CASE DIGESTS
ATTY. LYAN DAVID JUANICO
BLOCK 1-M A.Y. 2020 - 2021

defendants cannot be held guilty for their manner of commenting on the conduct of the justice of
peace was proper, growing out of constitutional guarantees in our bill of rights.

15
CRIMINAL LAW 1 CASE DIGESTS
ATTY. LYAN DAVID JUANICO
BLOCK 1-M A.Y. 2020 - 2021

11. Guingguing v. People


G.R. No. 128959, September 30, 2005
Tinga, J.:

FACTS:
This case originated from the criminal complaint for libel filed by broadcast journalist Cirse
“Choy” Torralba against Ciriaco “Boy” Guingguing (herein petitioner) and Segundo Lim. Lim
caused the publication of records of criminal cases filed against the broadcast journalist as well as
photographs of the latter being arrested. These were published by means of a one-page
advertisement paid for by Lim in the Sunday Post, a weekly publication edited and published by
petitioner. The Sunday Post was circulated in the province of Bohol, as well as in the Visayas and
Mindanao. In his defense, he claimed that the complainant was making defamatory attacks against
him and his family over the airwaves. And since Lim had no access to radio time, he opted for paid
advertisements via newspaper to answer the attacks, as a measure of self-defense. Lim also argued
that complainant, as a media man, occupied a position almost similar to a public functionary and
should not be onion-skinned and be Abel to absorb the thrust of public scrutiny.

The lower court and CA found that the publication was indeed libelous and the petitioners guilty.
Declaring that malice as the most important element of libel, was present in the case because every
defamatory publication prima facie implies malice on the part of the author and publisher towards
the person subjected thereof.

Petitioner Guingguing contends that as editor-publisher of the Sunday Post and as a member of the
fourth estate, the lower courts’ finding of guilt against him constitutes an infringement of his
constitutional right to freedom of speech and of the press.

ISSUE:
Whether or not the petitioner is guilty for the crime libel.

RULING:
No. Jurisprudence has affirmed that in libel cases, even if the defamatory statement is false, no
liability can attach if it relates to official conduct, unless the public official concerned proves that
the statement was made with actual malice, that is, with knowledge that it was false or with reckless
disregard of whether it was false or not.

In the case at bar, the complainant broadcast journalist is definitely qualified as a public figure. He
is a broadcast journalist hosting two radio programs aired over a large portion of Visayas and
Mindanao, that also makes his notoriety unquestionable. The newspaper in question, the Sunday
Post, is particularly in circulation in the areas where complainant’s broadcasts were aired.
Certainly, it cannot be denied that the target audience of the newspaper was the same persons who
may have listened regularly to the complainant’s broadcast. Even if the sphere of complainant’s
renown is limited in geography, it is in the same plane as the circulation of the offending
newspaper. Complainant’s volition to practice the radio broadcasting profession necessarily thrusts
him in the public sphere. The lower courts failed to recognize the constitutional guarantee of free
expression which is a leeway if the commentary on public affairs and public figures is qualified as
“justifiable motive” if not “good intention”.

Moreover, malice, as a necessary element of libel, was not proven. As it has been established that
complainant was a public figure, the prosecution has the burden of proving that actual malice on
the part of Lim and petitioner exist when the latter published the article about the complainant.
The prosecution must have established beyond reasonable doubt that the defendants knew the
statements in the advertisement were false or nonetheless proceeded with reckless disregard to

16
CRIMINAL LAW 1 CASE DIGESTS
ATTY. LYAN DAVID JUANICO
BLOCK 1-M A.Y. 2020 - 2021

publish it whether or not it was true. If the statements made against the public figure are essentially
true, then no conviction for libel can be had. Any statement that does not contain a provably false
factual connotation will receive full constitutional protection. It is clear that there was nothing
untruthful about what was published in the Sunday Post.

The information went into the very characters and integrity of the complainant to which his
listening public has a very legitimate interest. The complainant hosts a public affairs program, one
which he himself claimed was imbued with public character since it deals with "corruptions in
government, corruptions by public officials, irregularities in government in comrades.”
Therefore, Lim and petitioner cannot be held guilty for the crime libel as the actual malice was
absent and that their intention was to let the public know the character of their radio commentator,
which is constitutionally protected by the freedom of expression.

17
CRIMINAL LAW 1 CASE DIGESTS
ATTY. LYAN DAVID JUANICO
BLOCK 1-M A.Y. 2020 - 2021

12. Estrada v. Escritor


AM No. P-02-1651, June 22, 2006
PUNO, J.:

FACTS:
Complainant Alejandro Estrada requested the judge of Regional Trial Court of Las Piñas City,
where respondent Soledad Escritor work as a court interpreter, for investigation because he
believes that respondent is committing an immoral act by living with a man not her husband and
having borne a child within live-in arrangement. This tarnishes the image of the court, thus she
should not be allowed to remain employed therein as it appears that the court condones her act.
Consequently, the respondent was charged under the Revised Administrative Code for committing
“disgraceful and immoral conduct”.

The respondent admitted the allegations made by the complainant. But she testified that as a
member of the religious sect known as Jehovah’s Witnesses and the WatchTower and Bible Tract
Society, their conjugal arrangement is in conformity with their religious beliefs and has the
approval of her congregation, to which she provided the “Declaration of Pledging Faithfulness” to
support her claim. This Declaration allows members of the congregation who have been abandoned
by their spouses enter into marital relations, and is binding all over the world except in countries
where divorce is allowed. Respondent and her live-in partner’s Declarations were executed in the
usual and approved form prescribed by the Jehovah’s Witnesses, approved by elders of the
congregation, and recorded in the WatchTower Central Office; which is valid insofar as the
congregation is concerned.

Respondent is now invoking her religious freedom guaranteed under Article 3, section 5 of the
Constitution asserting that her conjugal arrangement being in accordance with practices and moral
standards of her congregation, does not constitute disgraceful and immoral conduct and therefore,
she should not be held administratively liable. The State, on the other hand, seeks to regulate her
behavior and protect its interest in marriage and family, and also the integrity of the courts where
the respondent is an employee.

ISSUE:
Whether or not the State has more compelling interest to defeat the claim of the respondent to
religious freedom.

RULING:

The Court ruled in favor of the respondent by reason of her right guaranteed by the Constitution.

Applying the two (2) religion clauses, Establishment Clause and Free Exercise Clause,
respondent’s religious freedom was established and recognized. Free Exercise Clause prohibits
government from inhibiting religious beliefs with penalties for religious beliefs and practices,
while the Establishment Clause prohibits government from inhibiting religious belief with rewards
for religious beliefs and practices. Both were intended to deny the government the power to
influence individual religious beliefs and practices, protecting and promoting freedom of
individuals including the respondent herein.

Legislative acts that purposely aids or inhibits religion will be challenged as unconstitutional. But
although legislative acts are purely secular and general in applicability, it still may incidentally
have “burdensome effect” on religious exercises. That is why the government may take into
consideration those individuals whose religious beliefs and practice would be infringed by
governmental regulations. In theory of accommodation, when the legislature fails to recognize

18
CRIMINAL LAW 1 CASE DIGESTS
ATTY. LYAN DAVID JUANICO
BLOCK 1-M A.Y. 2020 - 2021

religions and their practices in enacting laws of general application, those threatened and burdened
may turn into the courts for protection.

In this case, the respondent is entitled to protection of her right to the free exercise of religion not
only by the Constitution, but also by the court. The only limitation on this religious freedom is the
existence of a grave and present danger of a character both grave and imminent, of a serious evil
to public safety, public morals, public health or any other legitimate public interest, that the State
has a right to prevent. As to the contention that the State has a compelling interest in the
preservation of marriage and the family as basic social institutions, still, the freedom of religion is
specifically articulated as one of the fundamental rights in the Constitution. It weighs more than
the contention of “state’s interest is important” as the Constitution itself holds the right to religious
freedom as sacred.

Therefore, the respondent cannot be penalized as she is exempt from the law based on her
fundamental right to freedom of religion.

19
CRIMINAL LAW 1 CASE DIGESTS
ATTY. LYAN DAVID JUANICO
BLOCK 1-M A.Y. 2020 - 2021

13. People v. Echagaray


G.R. No. 117472, February 7, 1997
PER CURIAM:

FACTS:
This case originated from a decision rendered affirming the conviction for the crime of raping her
ten-year-old daughter, and sentencing the accused of the supreme penalty death, as the Republic
Act No. 7659 or the Death Penalty was already in effect at the time the crime was committed.

The accused discharged his defense counsel and retained the services of the Anti-Death Penalty
Task Force of the Free Legal Assistance Group of the Philippines (FLAG). The FLAG prepared
the Supplemental Motion for Reconsideration raising three (3) main issues, among which is the
legal question of the constitutionality of the Death Penalty Law.

Accused-appellant raised the constitutionality of the Death Penalty Law for having been enacted
in the absence of compelling reasons; and that death penalty for the crime of rape is severe and
excessive, cruel and unusual, because rape does not involve taking of life unlike the crime of
murder, thus, this is in violation of the constitutional proscription against cruel and unusual
punishments.

ISSUE:
Whether or not the proposition of the accused-appellant is correct.

RULING:
The Court rejected his proposition. Article 3, section 19 of the Constitution itself vests in Congress
the power to re-impose the death penalty “for compelling reasons involving heinous crimes”. This
power is subject to following limitations: 1. That Congress defines or describes what is meant by
heinous crimes; 2. That only crimes that qualify as heinous as defined by the death penalty bill, be
punished by death upon the attendance of circumstances duly proven in court that characterize the
crime to be heinous; and 3. That the enactment of this bill be singularly motivated by "compelling
reasons involving heinous crimes.” These limitations were not contravened by the Congress. The
definition or description of heinous crimes were sufficiently provided within the preamble of R.A.
No. 7659 or the Death Penalty Law.

For his second argument, the accused-appellant tried to use the Furman case where the U.S. ruled
death penalty is a cruel, degrading or inhuman punishment. But this is misleading and inapplicable
to this case because in Furman, the theory is centered on the discrimination against the black
accused in front of a white jury that is given the unconditional discretion to determine whether or
not to impose the death penalty.

In this case, the death sentence is neither cruel, unjust nor excessive when the law itself provides
a specific and well-defined criminal act. Moreover, U.S. jurisprudence already discussed that
punishments are cruel only when they involve torture or a lingering death, something inhuman and
barbarous, or something more than the mere extinguishment of life.

The Court reiterated that death penalty is imposed in heinous crimes because the perpetrators have
committed unforgivable acts that have dehumanized a persons, or/and because they have use
irreparable and substantial injury both to the victim and society and a repetition of their acts would
pose actual threat to the safety of individuals and the survival of government, that they must be
permanently prevented from doing so.

20
CRIMINAL LAW 1 CASE DIGESTS
ATTY. LYAN DAVID JUANICO
BLOCK 1-M A.Y. 2020 - 2021

14. Corpuz v. People


G.R. No. 180016, April 29, 2014
PERALTA, J.:

Facts:
It was at the Admiral Royale Casino in Olongapo City when petitioner approached private
complainant Danilo Tangcoy, who was then engaged in the business of lending money to casino
players, upon hearing that the latter had some pieces of jewelry for sale and offered to sell the said
pieces of jewelry on commission basis. Private complainant agreed, and turned over to petitioner
the following items: an 18k diamond ring for men; a woman's bracelet; one (1) men's necklace and
another men's bracelet, with an aggregate value of ₱98,000.00, as evidenced by a receipt of even
date. They both agreed that the petitioner shall remit the proceeds of the sale, and/or, if unsold, to
return the same items, within a period of 60 days. The period expired without petitioner remitting
the proceeds of the sale or returning the pieces of jewelry. When the private complainant was able
to meet the petitioner, the latter promised the former that he will pay the value of the said items
entrusted to him, but to no avail.

The Regional Trial Court found the petitioner guilty beyond reasonable doubt of crime estafa. The
accused was sentenced to suffer the penalty of imprisonment under the Indeterminate Sentence
Law of four (4) years and two (2) months of Prision Correccional in its medium period as
minimum, to fourteen (14) years and eight (8) months of Reclusion Temporal in its minimum
period as maximum; to indemnify private complainant Danilo Tangcoy the amount of ₱98,000.00
as actual damages, and to pay the costs of suit.

The case was elevated to the Court of Appeals, but the appellate court denied and affirmed the
decision of the RTC.

Issue:
Whether or not the penalties imposed were excessive.

Ruling:
The Court found that there seems to be a perceived injustice brought about by the range of penalties
that the courts continue to impose on crimes against property committed today, based on the
amount of damage measured by the value of money eighty years ago in 1932. However, this Court
cannot modify the said range of penalties because that would constitute judicial legislation. What
the legislature's perceived failure in amending the penalties provided for in the said crimes cannot
be remedied through this Court's decisions, as that would be encroaching upon the power of
another branch of the government. But with Article 5 of the Revised Penal Code, when the court
finds an act proper to be repressed, the remedy is to render the proper decision and thereafter,
report to the Chief Executive, through the Department of Justice, the reasons why the same act
should be the subject of penal legislation. While the second paragraph of the same provision
provides for the situation wherein the act is already punishable by law but the corresponding
penalty is deemed by the court as excessive.

The opinion advanced by Chief Justice Maria Lourdes P. A. Sereno echoes the view that the role
of the Court is not merely to dispense justice, but also the active duty to prevent injustice. Thus, in
order to prevent injustice in the present controversy, the Court should not impose an obsolete
penalty pegged eighty three years ago, but consider the proposed ratio of 1:100 as simply
compensating for inflation. Furthermore, the Court has in the past taken into consideration
"changed conditions" or "significant changes in circumstances" in its decisions.

21
CRIMINAL LAW 1 CASE DIGESTS
ATTY. LYAN DAVID JUANICO
BLOCK 1-M A.Y. 2020 - 2021

15. Lozano vs. Martinez


G.R. No. L-63419; December 18, 1986
Yap, J;

FACTS:
The constitutionality of B.P. 22 was assailed because of its alleged unconstitutionality for being
violative of the Constitutional provision: Non-imprisonment for debt or nonpayment of poll tax
(Art. 3, Sec. 19, 1987 Constitution).

BP 22 punishes a person "who makes or draws and issues any check on account or for value,
knowing at the time of issue that he does not have sufficient funds in or credit with the drawee
bank for the payment of said check in full upon presentment, which check is subsequently
dishonored by the drawee bank for insufficiency of funds or credit or would have been dishonored
for the same reason had not the drawer, without any valid reason, ordered the bank to
stop payment."

The statute likewise imposes the same penalty on "any person who, having sufficient funds in or
credit with the drawee bank when he makes or draws and issues a check, shall fail to keep sufficient
funds or to maintain a credit to cover the full amount of the check if presented within a period of
ninety (90) days from the date appearing thereon, for which reason it is dishonored by the drawee
bank.

An essential element of the offense is "knowledge" on the part of the maker or drawer of the check
of the insufficiency of his funds in or credit with the bank to cover the check upon its presentment.
Since this involves a state of mind difficult to establish, the statute itself creates a prima facie
presumption of such knowledge where payment of the check "is refused by the drawee because of
insufficient funds in or credit with such bank when presented within ninety (90) days from the date
of the check. To mitigate the harshness of the law in its application, the statute provides that such
presumption shall not arise if within five (5) banking days from receipt of the notice of dishonor,
the maker or drawer makes arrangements for payment of the check by the bank or pays the holder
the amount of the check.

The presumptions being merely prima facie, it is open to the accused of course to present proof to
the contrary to overcome the said presumptions.

BP 22 is aimed at putting a stop to or curbing the practice of issuing checks that are worthless, i.e.
checks that end up being rejected or dishonored for payment. The practice, as discussed later, is
proscribed by the state because of the injury it causes to public interests.

ISSUE:
Does B.P 22 offends the constitutional provision forbidding imprisonment for debt?

RULING:
No. B.P. 22 is not violative of the Constitutional provision forbidding imprisonment for debt.

The gravamen of the offense punished by BP 22 is the act of making and issuing a worthless check
or a check that is dishonored upon its presentation for payment. It is not the non- payment of an
obligation which the law punishes. The law is not intended or designed to coerce a debtor to pay
his debt. The thrust of the law is to prohibit, under pain of penal sanctions, the making of worthless
checks and putting them in circulation.

22
CRIMINAL LAW 1 CASE DIGESTS
ATTY. LYAN DAVID JUANICO
BLOCK 1-M A.Y. 2020 - 2021

16. People vs. Ferer


G.R. Nos. L-32613-14; December 27, 1972
CASTRO, J.:

FACTS:

Gravamen of the Case: The Constitutionality of Anti-Subversion Act was assailed because it was
alleged to be a Bill of Attainder which is violative to the Constitution.

The accused were charged for Anti-Subversion Act for allegedly being members of the Communist
Party of the Philippines (CPP), Kabataang Makabayan (KM), and New People’s Army (NPA);
which are considered Subversive organizations by the same law.

The accused contends that the Anti-Subversion Act is a Bill of Attainder; hence unconstitutional;
Article III, section 1 (11) of the Constitution states that "No bill of attainder or ex post facto law
shall be enacted. A bill of attainder is a legislative act which inflicts punishment without trial. Its
essence is the substitution of a legislative for a judicial determination of guilt.

ISSUE:
Is the Anti-Subversion Act unconstitutional for being Bill of Attainder?

RULING:
No. Anti-Subversion Act is not a Bill of Attainder; because it must be proved first that a person
joined an organization to overthrow the Government for alien rule, or with overt acts he knowingly
joined a subversive organization enumerated by the Anti-Subversive Act.

23
CRIMINAL LAW 1 CASE DIGESTS
ATTY. LYAN DAVID JUANICO
BLOCK 1-M A.Y. 2020 - 2021

17. US v. Diaz Conde


G.R. No. L-18208; February 14, 1922
Johnson, J;

FACTS:
That on the 30th day of December, 1915, the alleged offended persons Bartolome Oliveros and
Engracia Lianco executed and delivered to the defendants a contract (Exhibit B) evidencing the
fact that the former had borrowed from the latter the sum of P300, and (2) that, by virtue of the
terms of said contract, the said Bartolome Oliveros and Engracia Lianco obligated themselves to
pay to the defendants interest at the rate of five per cent (5%) per month, payable within the first
ten days of each and every month, the first payment to be made on the 10th day of January, 1916.

The defendants claim that at the time of the execution of the contract there was not yet a law
prohibiting usurious interest; only after the execution of such contract was Usury law passed; hence
the same cannot be given retroactive effect unless the same is favorable to the accused; applying
it conversely will amount to ex post facto law.

Art. 3, Sec. 22, 1987 of the Constitution; Ex post facto laws, unless they are favorable to the
defendant, are prohibited in this jurisdiction. Every law that makes an action, done before the
passage of the law, and which was innocent when done, criminal, and punishes such action, is an
ex post facto law.

ISSUE(S):
Does usury law amounts to Ex Post Facto Law?

HELD:
No. The acts complained of in the present case were legal at the time of their occurrence, they
cannot be made criminal by any subsequent or ex post facto legislation.

24
CRIMINAL LAW 1 CASE DIGESTS
ATTY. LYAN DAVID JUANICO
BLOCK 1-M A.Y. 2020 - 2021

17. People v. Abilong


G.R. No. L-1960; November 26, 1948
Montemayor, J;

FACTS:
That on or about the 17th day of September, 1947, in the City of Manila, Philippines, the said
accused, being then a convict sentenced and ordered to serve two years, four months and one day
of destierro during which he should not enter any place within the radius of 100 kilometers from
the City of Manila.

Counsel for the appellant contends that a person like the accused evading a sentence of destierro
is not criminally liable under the provisions of the Revised Penal Code, particularly article 157 of
the said Code for the reason that said article 157 refers only to persons who are imprisoned in a
penal institution and completely deprived of their liberty. He bases his contention on the word
"imprisonment" used in the English text of said article which in part reads as follows:

Evasion of service of sentence. The penalty of prision correccional in its medium and maximum
periods shall be imposed upon any convict who shall evade service of his sentence by escaping
during the term of his imprisonment by reason of final judgment.

Solicitor General however contends that in case of doubt the Spanish text shall govern.

ART. 157. Quebrantamiento de sentencia. — Sera castigado con prision correccional en sus grados
medio y maximo el sentenciado que quebrantare su condena, fugandose mientras estuviere
sufriendo privacion de libertad por sentencia firme;

Translation in English:

ART. 157. Breach of sentence. - The sentenced person who violates his sentence will be punished
with correctional prison in its medium and maximum degrees, if he escapres while he is suffering
deprivation of liberty by final judgment.

ISSUE:
Should the Spanish Text be followed over the English Text?

HELD
Yes. Spanish text is controlling over English language in case of doubt to its interpretation.
Conclusively; The Court find and hold that the appellant is guilty of evasion of service of sentence
under article 157 of the Revised Penal Code (Spanish text), in that during the period of his sentence
of destierro by virtue of final judgment wherein he was prohibited from entering the City
of Manila, he entered said City.

25
CRIMINAL LAW 1 CASE DIGESTS
ATTY. LYAN DAVID JUANICO
BLOCK 1-M A.Y. 2020 - 2021

18. Hernan v. Sandiganbayan


G.R. No. 217874 ; December 05, 2017
Ponente: Peralta, J;

FACTS:

The petitioner, Ophelia Hernan, a Disbursing Officer of the Department of Transportation and
Communications (DOTC) in Baguio city, was charged for Malversing Public funds when a
government transaction between the DOTC and LBP; the latter denied receiving any ₱11,300.00
deposit on September 19, 1996 from petitioner for the account of the DOTC.

Thus, the COA demanded that she pay the said amount. Petitioner, however, refused.
Consequently, the COA filed a complaint for malversation of public funds against petitioner with
the Office of the Ombudsman for Luzon which, after due investigation, recommended her
indictment for the loss of ₱11,300.00.

RA No.10951 in effect; can the same law be applicable to the accused?

ISSUE:
Can RA No. 10951 be given Retroactive effect?

HELD:
Yes. As a general rule retroactivity may be given if favorable to the accused.

the provisions of RA No. 10951 whenever it is, by reason of justice and equity, called for by the
facts of each case. Hence, said recent legislation shall find application in cases where the imposable
penalties of the affected crimes such as theft, qualified theft, estafa, robbery with force upon things,
malicious mischief, malversation, and such other crimes, the penalty of which is dependent upon
the value of the object in consideration thereof, have been reduced, as in the case at hand, taking
into consideration the presence of existing circumstances attending its commission. For as long as
it is favorable to the accused, said recent legislation shall find application regardless of whether its
effectivity comes after the time when the judgment of conviction is rendered and even if service
of sentence has already begun.

The accused, in these applicable instances, shall be entitled to the benefits of the new law
warranting him to serve a lesser sentence, or to his release, if he has already begun serving his
previous sentence, and said service already accomplishes the term of the modified sentence.
In the latter case, moreover, the Court, in the interest of justice and expediency, further directs the
appropriate filing of an action before the Court that seeks the reopening of the case rather than an
original petition filefor a similar purpose.

26
CRIMINAL LAW 1 CASE DIGESTS
ATTY. LYAN DAVID JUANICO
BLOCK 1-M A.Y. 2020 - 2021

19. People vs. Formigones


G.R. No. L-3246 ; November 29, 1950
Montemayor, J;

FACTS:

Appellant in a rage of Jealousy killed his wife; because he suspected his wife to be in an illicit
affair with his half-brother.

Despite the court should taking into consideration the presence of two mitigating circumstances of
a qualifying nature, which it can not afford to overlook, without any aggravating one, the penalty
could not be reduced to the next lower to that imposed by law, because, according to a ruling of
the court of Spain, article 80 above-mentioned does not contain a precept similar to that contained
in Rule 5 of article 81 (now Rule 5, art. 64 of the Rev. Penal Code.).

Yet, in view of the excessive penalty imposed, the strict application of which is inevitable and
which, under the law, must be sustained, this court now resorts to the discretional power conferred
by paragraph 2 of article 2 of the Penal Code.

ISSUE(S):
May the penalty for Formigines be lowered in light of prescribed but undeserved penalties under
Art. 5 of the Revised Penal Code?

HELD:
No. The Supreme court affirm the judgment by the lower courts; which sentenced the accused with
Parricide;
Appealed from with costs, and hereby order that a proper petition be filed with the executive
branch of the Government in order that the latter, if it be deemed proper in the exercise of the
prerogative vested in it by the sovereign power, may reduce the penalty to that of the next lower.

27
CRIMINAL LAW 1 CASE DIGESTS
ATTY. LYAN DAVD JUANICO
BLOCK 1-M A.Y. 2020 - 2021

20. Ladonga vs. People


G.R. No. 141066; February 17, 2005
Austria-Martinez, J;

FACTS:
Accused was charged by B.P 22 (Bouncing Check Law); the facts are as follows:
That, sometime in May or June 1990, in the City of Tagbilaran, Philippines, and within
the jurisdiction of this Honorable Court, the above-named accused, conspiring, confederating, and
mutually helping with one another, knowing fully well that they did not have sufficient funds
deposited with the United Coconut Planters Bank (UCPB), Tagbilaran Branch, did then and there
willfully, unlawfully, and feloniously, draw and issue UCPB Check No. 284743 post dated July 7,
1990 in the amount of NINE THOUSAND SEVENTY-FIVE PESOS AND FIFTY-FIVE, payable
to Alfredo Oculam, and thereafter, without informing the latter that they did not have sufficient
funds deposited with the bank to cover up the amount of the check, did then and there willfully,
unlawfully and feloniously pass on, indorse, give and deliver the said check to Alfredo Oculam by
way of rediscounting of the aforementioned checks; however, upon presentation of the check to
the drawee bank for encashment, the same was dishonored for the reason that the account of the
accused with the United Coconut Planters Bank, Tagbilaran Branch, had already been closed, to
the damage and prejudice of the said Alfredo Oculam in the aforestated amount.

Petitioner brought the case to the Court of Appeals, arguing that the RTC erred in finding her
criminally liable for conspiring with her husband as conspiracy is inapplicable to B.P. Blg. 22
which is a special law; moreover, she is not a signatory of the checks and had no participation in
the issuance thereof.

ISSUE:
Is the Revised Penal Code (RPC) suppletory to Special Penal Laws? In this case can Conspiracy
be supplemented to B.P.

RULING:
YES FOR BOTH.

ART. 10. Offenses not subject to the provisions of this Code. – Offenses which are or in the future
may be punishable under special laws are not subject to the provisions of this Code. This Code
shall be supplementary to such laws, unless the latter should specially provide the contrary.

B.P. Blg. 22 does not expressly proscribe the suppletory application of the provisions of the RPC.
Thus, in the absence of contrary provision in B.P. Blg. 22, the general provisions of the RPC
which, by their nature, are necessarily applicable, may be applied suppletorily.

Hence; in the case at bar, Conspiracy from the Revised Penal Code may be applied to B.P. 22.

28
CRIMINAL LAW 1 CASE DIGESTS
ATTY. LYAN DAVD JUANICO
BLOCK 1-M A.Y. 2020 - 2021

21. People vs. Simon


G.R. No. 93028; July 29, 1994
REGALADO, J.:

FACTS:
Accused Martin Simon was charged with a violation of Section 4, Article II of Republic Act No.
6425 (Dangerous Drugs Act of 1972). Simon sold four tea bags of marijuana to a Narcotics
Command (NARCOM) poseur-buyer in consideration of 40.00php. The same tea bags tested
positive for marijuana.
After a diligent review of the evidence produced by both parties, it was found that Simon was
caught in flagrante delicto in the illegal sale of prohibited drugs. According to the prosecution,
Martin Simon sold two tea bags of marijuana dried leaves to Sgt. Lopez. The latter himself
creditably testified as to how the sale took place and his testimony was likewise sufficiently
corroborated by his colleagues.
ISSUE:
Whether or not in determining the penalty to be imposed on Martin Simon which is here to be
taken from the penalty of prision correccional, the presence or absence of mitigating, aggravating
or other circumstances modifying criminal liability should be taken into account.
RULING:
As a general rule, the rules on the imposition of modifying circumstances under the Revised Penal
Code cannot and should not be applied. This is so because special laws usually involve and provide
their own specific penalties for the offenses punished therein and were not taken with reference to
the Revised Penal Code. Since the penalties provided by special laws do not provide for the
minimum, medium or maximum periods, it would consequently be impossible to consider
modifying circumstances in the imposition of penalties because such circumstances are meant to
determine the period of the penalty in accordance with the rules in Article 64 of the Revised Penal
Code.
However, when an offense is defined in and is punished under a special law and the penalty
therefor is taken from the Revised Penal Code in its technical nomenclature and, inevitably, with
its duration, correlation and legal effects under the system of penalties in the Revised Penal Code,
then the Code shall be considered supplementary to such special law.
Prision correccional has a duration of 6 months and 1 day to 6 years and, as a divisible penalty, it
consists of three periods as provided in Article 76 of the Revised Penal Code. In the case at bar,
the imposable penalty under Republic Act No. 6425 (as amended by Republic Act No. 7659) is

29
CRIMINAL LAW 1 CASE DIGESTS
ATTY. LYAN DAVD JUANICO
BLOCK 1-M A.Y. 2020 - 2021

prision correccional in its medium period; this is pursuant to Article 64 of the Revised Penal Code,
there being no modifying circumstances.
Republic Act No. 6425 (as amended by Republic Act No. 7659) makes use of the technical
nomenclature of penalties provided in the Revised Penal Code; it incorporates and prescribes the
technical penalties which come in three scales. Thus, the appreciation and effects of modifying
circumstances must be applied in the imposition of the penalty and the different classifications of
penalties and the rules for graduating such penalties should have supplementary effect on Republic
Act No. 6425 (as amended by Republic Act No. 7659).

30
CRIMINAL LAW 1 CASE DIGESTS
ATTY. LYAN DAVD JUANICO
BLOCK 1-M A.Y. 2020 - 2021

22. People vs. Siton


G.R. No. 169364; September 18, 2009
YNARES-SANTIAGO, J.:

FACTS:
Assailed in this petition for review on certiorari is the July 29, 2005 Order of Branch 11, Davao
City Regional Trial Court in Special Civil Case No. 30-500-2004 which granted respondent’s
Petition for Certiorari and declared Article 202, par. 2 of the Revised Penal Code unconstitutional.
Respondents Evangeline Siton and Krystel Kate Sagarano were charged with vagrancy pursuant
to Article 202, par. 2 of the Revised Penal Code in two separate Informations. It was alleged that
on or about November 14, 2003, in Davao City, Philippines, herein respondents wandered and
loitered around San Pedro and Legaspi Streets (both in Davao City), without visible means to
support themselves nor lawful and justifiable purpose.
Respondents then filed an original petition for certiorari and prohibition with the Regional Trial
Court of Davao City and directly challenged the constitutionality of the anti-vagrancy law.
Respondents argued that the of the crime of vagrancy under Article 202, par. 2, in addition to being
vague, also results in an arbitrary identification of violators. According to respondents, the
definition of the crime includes in its coverage persons who are merely performing ordinary
peaceful acts. They similarly claimed that Article 202, par. 2 violates the equal protection clause
enshrined in the Constitution because it discriminates against the poor and unemployed and is
likewise an arbitrary classification. Respondents argued that Article 202, par. 2 on its face violates
the constitutionally-guaranteed rights to due process and the equal protection of the laws and that
the presumption on constitutionally was overwhelmed.
Petitioner argued that every statute must be presumed valid and all reasonable doubts should be
resolved in favor of its constitutionality. Petitioner cited Romualdez v. Sandiganbayan and claimed
that the overbreadth and vagueness doctrines may only be applied to free-speech cases and may
not be used to challenge the validity of penal statutes. According to petitioner, respondents failed
to overcome the presumption on the validity of Article 202, par. 2 and that such provision is a valid
regulation of individual conduct for the promotion of public welfare in the exercise of its police
power.
ISSUE:
Whether or not the RTC committed a reversible error in declaring unconstitutional Art. 202, par.
2 of the Revised Penal Code.

31
CRIMINAL LAW 1 CASE DIGESTS
ATTY. LYAN DAVD JUANICO
BLOCK 1-M A.Y. 2020 - 2021

RULING: The Court ruled in favor of petitioner.


Article 202, par. 2 does not violate the equal protection clause enshrined in the Constitution. The
provision in question does not discriminate against the poor and unemployed because the offenders
of such provision are punished not for their status (of being poor or unemployed) but for
conduction themselves under circumstances which endanger public peace or cause alarm and
unease to the community. The status of being poor or unemployed does not excuse immoral
conduct and indecent acts.
Article 202, par. 2 is a public order law. Public order laws were made to maintain minimum
standards of decency, morality, and civility in human society. Civilly, these are covered by the
"abuse of rights" doctrine embodied in the preliminary articles of the Civil Code concerning
Human Relations. Criminally, public order laws include a range of acts – from public indecencies
and immoralities, to public nuisances, to disorderly conduct. The acts punished are made illegal
due to their offensiveness to society’s basic sensibilities and their adverse effect on the quality of
life of the people of society.
Vagrancy is a public order crime which punishes persons for conducting themselves, at a specific
place and time which orderly society deems unfamiliar, under such conditions that are distasteful
and disgraceful to the public standards of decency and morality in a just, civilized and ordered
society, as would produce a reasonable apprehension for the safety and well-being of the
community.

32
CRIMINAL LAW 1 CASE DIGESTS
ATTY. LYAN DAVD JUANICO
BLOCK 1-M A.Y. 2020 - 2021

MODULE 2: GENERAL PRINCIPLES OF CRIMINAL LIABILITY

1. People vs. Silvestre and Atienza


G.R. No. 35748; December 14, 1931
VILLA-REAL, J.:

FACTS:
Appellant Martin Atienza was convicted as principal by direct participation and appellant Romana
Silvestre as accomplice of the crime of arson by the Court of First Instance.
On the night of November 25, 1930, while Nicolas de la Cruz and his wife were gathered together
with the appellants herein after supper, Atienza told said couple to take their furniture out of the
house because he was going to set it on fire. After being asked by Nicolas and his wife why he
wanted to set fire to the house, he answered that it was the only way he could be take revenge on
the people of Masocol, who, he said, had initiated the charge of adultery against him and Romana
Silvestre. Since Atienza armed with a pistol, no one dared to go against his wishes, not even
Romana Silvestre, who was about a meter away from Atienza. Worried at what Martin Atienza
had said, the Nicolas and his wife left the house at once to tell the barrio lieutenant, Buenaventura
Ania, as to what they had just hear Atienza say. However, they had hardly gone a hundred arms’
length when they heard cries of “Fire! Fire!” then they saw their home in flames. The fire destroyed
about 48 other houses.
Romana listened to Atienza’s threat without raising a protest and did not give the alarm when the
latter set fire to the house.
ISSUE:
Whether or not Romana Sylvestre may be held liable as an accomplice.
RULING:
No, Romana Sylvestre may not be held liable as an accomplice.
Mere passive presence at the scene of another’s crime mere silence and failure to give the alarm,
without evidence of agreement or conspiracy, is not punishable by law. Romana Sylvestre was
thus, acquitted.

33
CRIMINAL LAW 1 CASE DIGESTS
ATTY. LYAN DAVD JUANICO
BLOCK 1-M A.Y. 2020 - 2021

2. People vs. Talingdan


G.R. No. L-32126; July 6, 1978
CALLEJO, SR., J.:

FACTS:
One Friday morning, Corazon, Teresa Domogma’s daughter, was in a creek to wash clothes.
Corazon saw her mother Teresa in a meeting with Nemesio Talingdan and their co-appellants
Magellan Tobias, Augusto Berras, and Pedro Bides in a small hut owned by Bernardo Bagabag.
Corazon heard one of them say "Could he elude a bullet." When Teresa noticed Corazon, she
jostled her away saying "You tell your father that we will kill him."
The Saturday thereafter, Corazon was cooking food for supper when she saw her mother go down
the house to go to the yard where the latter again met with the other appellants. Corazon noted the
long guns that the appellants were carrying during the meeting. Teresa then came back to the house
and proceeded to her room.
Corazon informed her father, Bernardo, who was then working on a plow, regarding the presence
of the armed persons downstairs, but Bernardo paid no attention. Bernardo proceeded to the
kitchen and sat himself on the floor near the door. Bernardo was suddenly fired upon from below
the stairs of the batalan.
The accused climbed the stairs of the batalan and after seeing that Bernardo was still alive,
Talingdan and Tobias shot him again. Bides and Berras did not fire at that exact same time but
when Corazon tried to call for help, Bides threatened that he would kill her if she calls for help.
After the aforesaid events, Teresa came out of her room and upon being confronted by Corazon
that the latter recognized the perpetrators, the former threatened to kill the latter if she revealed
their identities to anyone.
ISSUE:
Whether or not Teresa Domogma is liable as an accessory to Bernardo’s murder.
RULING:
Yes, Teresa Domogma may be held liable as an accessory to Bernardo’s murder.
The proof of her direct participation in the conspiracy is not beyond reasonable doubt; however,
she cannot have the same liability as her co-accused. Teresa Domogma did not participate in the
actual shooting of her husband. The prosecution also failed to prove that she participated in the
planning and preparation of the shooting of her husband although she knew what was about to be

34
CRIMINAL LAW 1 CASE DIGESTS
ATTY. LYAN DAVD JUANICO
BLOCK 1-M A.Y. 2020 - 2021

done and did not object. There was no proof as to Teresa Domogma being the mastermind of the
crime or if she masterminded the crime alongside Talingdan.
It was, however, proven that Teresa Domogma is at the very least an accessory to the crime
committed by her co-accused. After coming out of her room after her husband was shot, she
enjoined her daughter from revealing the identities of those who killed her own husband to the
extent of saying “Don’t tell it to anyone. I will kill you if you tell this to somebody.” Moreover,
when the peace officers went to Teresa Domogma’s house to investigate, instead of helping the
authorities with the information given to her by her daughter regarding the identities of the
perpetrators, she claimed that she had no idea who the perpetrators may be.
In summary, before the actual shooting of Bernardo, Teresa Domogma was merely passive in her
attitude regarding the conspiracy of her co-appellants. Nevertheless, her acts subsequent to the
killing of Bernardo must be deemed as active cooperation with her co-appellants. Her acts
constitute "concealing or assisting in the escape of the principal in the crime" which makes her
liable as an accessory pursuant to Article 19, par. 3 of the Revised Penal Code.

35
CRIMINAL LAW 1 CASE DIGESTS
ATTY. LYAN DAVD JUANICO
BLOCK 1-M A.Y. 2020 - 2021

3. Manuel vs. People


G.R. No. 165842; November 29, 2005
CALLEJO, SR., J.:

FACTS:
On July 28, 1975, Eduardo Manuel got married to Rubylus Gaña. Sometime in January 1996, he
met private complainant Tina B. Gandalera in Dagupan City; the latter stayed in Bonuan, Dagupan
City for two days to look for a friend.
Manuel went to Baguio City to visit Gandalera. Eventually, they went to a motel where, despite
Gandalera’s resistance, Eduardo succeeded in copulating with her. Manuel proposed to her several
times and assured her that he was single. Manuel likewise brought his parents to Baguio City to
meet Gandalera’s parents and the former’s parents also assured the latter’s parents that their son
was still single. Manuel and Gandalera got married in April 1996 before Judge Antonio C. Reyes.
It appeared in their marriage contract that Manuel was single.
Through their joint efforts, the couple was able to build their home in Baguio City. However, in
1999, Manuel would only go to their house only twice or thrice a year. Gandalera was jobless and
whenever she would ask money from Manuel, the latter would slap her. In January 2001, Manuel
took all his clothes, left their home, and did not return. He also stopped giving financial support.
In August 2001, Gandalera made inquiries from the NSO in Manila and discovered that Manuel
had been previously married and such marriage was already existing when they exchanged their
vows. She secured an NSO-certified copy of the marriage contract.
Manuel argued that he declared he was single in his marriage contract with Gandalera because he
believed in good faith that his first marriage was invalid. According to him, he was unaware that
he had to go seek the nullification of his first marriage before marrying Gandalera. Manuel insisted
that pursuant to Article 3 of the Revised Penal Code, there must be malice for one to be criminally
liable for bigamy.
ISSUE:
Whether or not Manuel is liable for bigamy.
RULING:
Yes, Manuel is liable for bigamy.
Article 3, par. 2 of the Revised Penal Code provides that there is deceit when the act is performed
with deliberate intent. Thus, a felony cannot exist without intent. Since a felony by dolo is
classified as an intentional felony, it is considered voluntary. Although the words "with malice"

36
CRIMINAL LAW 1 CASE DIGESTS
ATTY. LYAN DAVD JUANICO
BLOCK 1-M A.Y. 2020 - 2021

do not appear in Article 3 of the Revised Penal Code, such phrase is included in the word
"voluntary." For one to be criminally liable for a felony by dolo, there must be a confluence of
both an evil act and an evil intent (Actus non facit reum, nisi mens sit rea).
In the case at bar, it was proved that the petitioner was married to Gaña in 1975, and such marriage
was not judicially declared a nullity; therefore, the marriage is presumed to subsist. The
prosecution also proved that the petitioner married the private complainant in 1996, which was
after the effectivity of the Family Code. Manuel is presumed to have acted with malice or evil
intent when he married the Gandalera. As a general rule, mistake of fact or good faith of the
accused is a valid defense in a prosecution for a felony by dolo; such defense negates malice or
criminal intent. However, ignorance of the law is not an excuse because everyone is presumed to
know the law (Ignorantia legis neminem excusat).
It was burden of Manuel to prove that when he married Gandalera in 1996, he was of the well-
grounded belief that his first wife was already dead as he had no news of her for more than 20
years since 1975. He had the burden of producing in evidence a decision of a competent court
declaring the presumptive death of his first wife as mandated by Article 349 of the Revised Penal
Code (in relation to Article 41 of the Family Code). Such judicial declaration of the presumptive
death of his first wife would have constituted proof that he acted in good faith and would have
proven his defense. However, Manuel failed to discharge such burden and must therefore be held
liable for bigamy.

37
CRIMINAL LAW 1 CASE DIGESTS
ATTY. LYAN DAVD JUANICO
BLOCK 1-M A.Y. 2020 - 2021

4. People vs. Puno


G.R. No. 97471
February 17, 1993
REGALADO, J.:

FACTS:
Mrs. Maria Socorro Mutuc-Sarmiento owned a bakeshop in Araneta Avenue, Quezon City called
Nika Cakes and Pastries. She had a driver of her own just as her husband did. In the afternoon of
January 13, 1988, accused Isabelo Puno, who was the personal driver of Mrs. Sarmiento's husband
arrived at the bakeshop. Puno informed Mrs. Sarmiento that her own driver Fred had to go to
Pampanga to attend to an emergency and that he would cover for Fred instead.
Mrs. Sarmiento then had to go home to her house in Valle Verde in Pasig City so she got into the
Mercedes Benz of her husband with Puno as the driver. After the car turned right in a corner of
Araneta Avenue, it stopped. A certain Enrique Amurao, boarded the car beside Puno. Once
Amurao got inside the car, he seated himself next to Mrs. Sarmiento and poked a gun at her.
Puno informed Mrs. Sarmiento that Amurao was his nephew and said, “Ma’am, you know, I want
to get money from you." Mrs. Sarmiento replied by telling them she had 7,000 php in her bag and
that they may take such amount in exchange for her freedom. However, the two told her they
wanted P100,000 more. Mrs. Sarmiento agreed to give them such amount but told them that they
would have to drop her at a certain gas station where the money was.
The car went toward North superhighway. There, Puno asked Mrs. Sarmiento to issue a check
amounting to 100,000php. Mrs. Sarmiento complied and drafted 3 checks in denominations of two
for 30,000php and one for 40,000php. Amurao then ordered Mrs. Sarmiento to swallow a pill but
the latter rebuffed.
Puno turned the car around and headed toward Metro Manila. Far along, he changed his mind and
turned the car toward Pampanga. Mrs. Sarmiento jumped out of the car then crossed to the other
side of the superhighway and was then able to flag down a fish vendors van. Her dress had blood
because, according to Mrs. Sarmiento, she fell down on the ground and was injured when she
escaped from the car.
Puno and Amurao were arrested the day after the incident while the latter was trying to encase
Mrs. Sarmiento’s 40,000php check at PCI Bank in Makati.
ISSUE:
Whether or not Puno may be convicted of kidnapping for ransom.

38
CRIMINAL LAW 1 CASE DIGESTS
ATTY. LYAN DAVD JUANICO
BLOCK 1-M A.Y. 2020 - 2021

RULING:
No, Puno may not be convicted of kidnapping for ransom.
There was no proof adducing that the accused had any motive conceived prior to or at the time of
the commission of the crime other than the extortion of money from Mrs. Sarmiento by means of
threats or intimidation.
In order to establish the specific intent of the accused, there must be indubitable proof that the
actual intent of the accused was to deprive Mrs. Sarmiento of her liberty and that the restraint of
her freedom was not merely a means of committing another crime intended by the accused.
In United States vs. Ancheta it was ruled that “…the detention and/or forcible taking away of the
victims by the accused, even for an appreciable period of time but for the primary and ultimate
purpose of killing them, holds the offenders liable for taking their lives or such other offenses they
committed in relation thereto, but the incidental deprivation of the victims' liberty does not
constitute kidnapping or serious illegal detention.”

39
CRIMINAL LAW 1 CASE DIGESTS
ATTY. LYAN DAVD JUANICO
BLOCK 1-M A.Y. 2020 - 2021

5. People vs. Delim


G.R. No. 142773; January 28, 2003
CALLEJO, SR., J.:

FACTS:
Marlon, Ronald, and Leon together with Manuel alias Bong, and Robert, all surnamed Delim were
indicted for murder of Modesto Manalo Bantas, who was adopted by the father of the accused.
On January 23, 1999, Modesto was forcibly taken fro his home by defendants who were armed.
Marlon poked his gun at Modesto while Robert and Ronald concurrently grabbed and restrained
Modesto; Rita and Randy (the victim’s wife and son) were warned not to leave the house.
Modesto’s body was found four days later by Randy and his relatives. The accused were found
guilty for murder.
The Information read: “That on or about January 23, 1999, in the evening at Brgy. Bila, Sison,
Pangasinan, and within the jurisdiction of this Honorable Court, the above-named accused, armed
with short firearms barged-in and entered the house of Modesto Delim and once inside with intent
to kill, treachery, evident premedidation (sic), conspiring with one another, did then and there,
wilfully, unlawfully and feloniously grab, hold, hogtie, gag with a piece of cloth, brought out and
abduct Modesto Delim, accused Leon Delim and Manuel Delim stayed in the house guarded and
prevented the wife and son of Modesto Delim from helping the latter, thereafter with abuse of
superior strength stabbed and killed said Modesto Delim, to the damage and prejudice of his heirs.”
The Regional Trial Court found the accused guilty of aggravated murder and was sentenced to
death.
ISSUE:
Whether or not accused should be held liable for murder or kidnapping.
RULING:
Accused should be held liable for murder.
In determining what crime is charged in an information, the material inculpatory facts stated
therein describing the crime committed in relation to a penal provision must be controlling. The
specific intent of the accused determines the crime and such specific intent must be alleged in the
information and likewise proved by the prosecution. In the case at bar, kidnapping was merely
used as a means to consummate the crime of murder. As ruled in the case of People vs. Puno, “For
kidnapping to exist, there must be indubitable proof that the actual specific intent of the malefactor
is to deprive the offended party of his liberty and not where such restraint of his freedom of action
is merely incident in the commission of another offense primarily intended by the malefactor.”
40
CRIMINAL LAW 1 CASE DIGESTS
ATTY. LYAN DAVD JUANICO
BLOCK 1-M A.Y. 2020 - 2021

Specific intent is defined as “a state of mind which exists where circumstances indicate that an
offender actively desired certain criminal consequences or objectively desired a specific result to
follow his act or failure to act.” Specific intent is the particular purpose or specific intention in
doing the forbidden act. Kidnapping and murder are specific intent crimes. Specific intent may be
verified by direct evidence or circumstantial evidence. It may similarly be inferred from the
circumstances of the actions of the accused as established by the evidence on record.

41
CRIMINAL LAW 1 CASE DIGESTS
ATTY. LYAN DAVD JUANICO
BLOCK 1-M A.Y. 2020 - 2021

6. U.S. vs Ah Chong
G.R. No. L-5272; March 19, 1910
Carson, J.

FACTS:

Ah Chong was employed as a cook at Officer’s quarters, No.27 located at Fort McKinley, Rizal
Province where the deceased, Pascual Gualberto was also employed as a house boy. Officer’s
quarters No 27 is a detached house situated some 40 meters from the nearest building. In August
1908, no one slept in the house except Ah Chong and Pascual, who jointly occupied a small room
towards the rear of the building, the door of which opened upon a narrow porch running along the
side of the building. This porch was covered by a heavy growth of vines, while the door of the
room was not furnished with a permanent lock, hence, the occupants, as a measure of security, had
attached a small hook on the inside of the door, and were in the habit of reinforcing this somewhat
insecure means of fastening the door by placing against it a chair. Inside the room, there was only
a small window which also opened in the porch; aside from the door and the window, no other
openings of any kind is in the room.

Prior to the day of the incident, there had been several cases of robbery in Fort McKinley, hence,
for personal protection Ah Chong kept a knife under his pillow at night. Further, Ah Chong and
Pascual both agreed that when either returned at night, he should knock at the door and acquaint
his roommate of his identity. Furthermore, Ah Chong and Pascual appears to be in friendly terms
with each other before the incident.

At about 10:00 pm of August 14, 1908, Ah Chong who was already sleeping was suddenly
awakened by someone trying to force open the door of the room. He sat up in bed and called out
twice: “Who was there?” however there was no answer. Ah Chong was convinced by the noise at
the door that it was being pushed open by someone who wants to force his way into the room. Due
to the heavy growth of vines along the front of the porch, the room was very dark. Ah Chong,
fearing that the intruder was a thief, leaped to his feet and called out: “If you enter this room, I will
kill you.” At that moment he was struck just above the knee by the edge of the chair which had
been placed against the door. In the darkness and confusion, Ah Chong thought that the blow had
been inflicted by the person who forced the door open, whom he thought to be a thief. Seizing a
kitchen knife which he kept under his pillow, Ah Chong struck his intruder who turns out to be
Pascual, his roommate. Pascual ran out to the porch and fell down in a desperately wounded
condition followed by Ah Chong, whom upon recognizing his roommate called to his employers
who slept in the next house, No.28, then ran back to his room to secure bandages for Pascual’s
wounds.

42
CRIMINAL LAW 1 CASE DIGESTS
ATTY. LYAN DAVD JUANICO
BLOCK 1-M A.Y. 2020 - 2021

Argument of the Petitioner:

Ah Chong admitted that he had stabbed his roommate but only because he thought Pascual was a
thief who was forcing his way inside the room despite the warnings given by the former. He
insisted that he struck the fatal blow without any intent to do a wrongful act, in a lawful exercise
of self-defense.

Ruling of the Trial Court:

The defendant was charged with the crime of assassination. He was found guilty of simple
homicide with extenuating circumstances and was sentenced to six years and one day presidio
mayor, the minimum penalty prescribed by law.

Ruling of the Supreme Court:

Whether or not self-defense is applicable in this case:

According to Article 8 of the Penal Code, there were three requisites for self defense to be availed:
illegal aggression, reasonable necessity of the means employed to prevent or repel it, and lack of
sufficient provocation on the part of the person defending himself. However, such defense cannot
be applied in this case because the evidence clearly states that the intruder was not a thief, that
neither the defendant nor his property was in real danger at the time he struck the fatal blow, hence,
there was no “unlawful aggression” on the part of the deceased. Furthermore, there was no “real
necessity” for the defendant to use a knife to defend his person or his property.

Whether in this jurisdiction one can be held criminally responsible who, by reason of a mistake of
facts, does an act for which he would be exempt from criminal liability if the facts were as he
supposed them to be, but which would constitute the crime of homicide or assassination if the actor
had known the true state of the facts at the time when he committed the act.

Under such circumstances there is no criminal liability, provided always that the alleged ignorance
or mistake of fact was not due to negligence or bad faith.

Mistake of fact, if such ignorance or mistake of fact is sufficient to negative a particular intent
which under the law is a necessary ingredient of the offense charged cancels the presumption of
intent, and works an acquittal; except in those cases where the circumstances demand a conviction
under the penal provisions touching criminal negligence; and in cases where under the provisions
of Article 1 of the Penal Code one voluntarily committing a crime or misdemeanour incurs criminal

43
CRIMINAL LAW 1 CASE DIGESTS
ATTY. LYAN DAVD JUANICO
BLOCK 1-M A.Y. 2020 - 2021

liability for any wrongful act committed by him, even though it be different from that which he
intended to commit.

Since evil intent is in general an inseparable element in every crime, any mistake of fact which
shows the act committed to have proceeded from no sort of evil in the mind necessarily relieves
the actor from criminal liability provided there is no fault or negligence on his part. As laid down
by Baron Parke, “The guilt of the accused must depend on the circumstances as they appear to
him” that is to say, the question as to whether he honestly, in good faith, and without fault or
negligence fell into the mistake is to be determined by the circumstances as they appeared to him
at the time when the mistake was made, and the effect which the surrounding circumstances might
reasonably be expected to have on his mind, in forming the intent criminal or otherwise, upon
which he acted.

A careful examination of the facts shows that the defendant struck the fatal blow in the firm belief
that the intruder who forced open the door of his sleeping room was a thief, from whose assault he
was in imminent peril, both of his life and his property; that in view of all the circumstances, as
they must have presented themselves to the defendant at that time, he acted in good faith, without
malice or criminal intent, in the belief that he was doing no more than exercising his legitimate
right of self-defense; that had the facts been as he believed them to be he would have been wholly
exempt from criminal liability on account of his act; and that he cannot be said to have been guilty
of negligence or recklessness or even carelessness in falling into his mistake as to the facts, or in
the means adopted by him to defend himself from the imminent danger which he believe threatened
his person and his property.

The judgement of conviction and the sentence imposed by the trial court should be reversed,
and the defendant acquitted of the crime with which he is charged and his bail bond
exonerated, with the cost of both instances de officio.

44
CRIMINAL LAW 1 CASE DIGESTS
ATTY. LYAN DAVD JUANICO
BLOCK 1-M A.Y. 2020 - 2021

7. People vs. Oanis


G.R No. L-47722; July 27, 1943
Moran, J.

FACTS:

On December 24, 1983, Captain Godofredo Monsod, Constabulary Provincial Inspector at


Cabanatuan, Nueva Ecija, received from Major Guido a telegram stating: “Information received
escaped convict Anselmo Balagtas with bailarina and Irene in Cabanatuan get him dead or alive.”
Capt. Monsod accordingly called for his first sergeant and asked that he be given four men; these
four men were Corporal Alberto Galanta, and Privates Nicomedes Oralo, Venancio Serna and D.
Fernandez. They were instructed to arrest Balagtas and, if overpowered, to follow the instruction
contained in the telegram. The same instruction was given to Chief of Police Oanis who was
likewise called by Capt. Monsod. When Oanis was asked whether he knew one Irene, a bailarina,
he answered that he did know one of loose morals. Upon the request of Capt. Monsod, Oanis tried
to locate his men to guide the constabulary soldiers however, failing to see anyone, he volunteered
to go instead. Capt. Monsod divided the party into two groups with Oanis and Galanta, and Private
Fernandez in the same group.

When their group arrived at Irene’s house, Oanis approached one Brigada Mallare who was then
stripping banana stalks, and asked her where Irene’s room was. Brigada indicated the place and
upon further inquiry also said that Irene was sleeping with her paramour. Trembling, Brigada
hurriedly returned to her room which was very near to the one Irene was occupying. Oanis and
Galanta then went to the room of Irene, and upon seeing a man sleeping with his back towards the
door where the two were, simultaneously fired at him with their .32 and.45 calibre revolvers.
Awakened by the gunshots, Irene saw her paramour already wounded, and looking at the door
where the shots came from, she saw the defendants still firing at him. Shocked by the entire scene,
Irene fainted; it turned out that the person the defendants killed was not Anselmo Balagtas but an
innocent citizen named Serapio Tecson who was Irene’s paramour.

Capt. Monsod, upon knowing of the killing, went to the scene and when he asked as to who killed
Tecson, Galanta answered “We two, sir” referring to himself and Oanis. The corpse was thereafter
brought to the provincial hospital where it was found that multiple gunshot wounds inflicted by a
.32 and .45 caliber revolvers were the cause of Tecson’s death.

Argument Of The Petitioner:

45
CRIMINAL LAW 1 CASE DIGESTS
ATTY. LYAN DAVD JUANICO
BLOCK 1-M A.Y. 2020 - 2021

The appellants contended that they acted in innocent mistake of fact in the honest performance of
their official duties, both of them believing that Tecson was Balagtas, hence, they incur no criminal
liability.

Ruling of The Trial Court:

After due trial, the defendants were found guilty of homicide through reckless imprudence and
were sentenced each to an indeterminate penalty from one year and six months to two years and
two months of prision correccional and to indemnify jointly and severally the heirs of the deceased
in the amount of 1,000php.

Ruling of the Supreme Court:

Whether or not the defendants correctly argued mistake of fact as their defence:

Ignorantia facti excusat applies only when the mistake is committed without fault or carelessness.
There is an innocent mistake of fact committed without any fault or carelessness when the accused
having no time or opportunity to make a further inquiry, and being pressed by circumstances to
act immediately, had no other alternative but to take the facts as they then appeared to him, hence,
such facts justified his act of killing.

However, in the case, appellants found no circumstances whatsoever which would press them to
immediate action. The deceased was sleeping in the room when they found him; therefore
appellants had ample time and opportunity to ascertain his identity without hazard to themselves,
and could even effect a bloodless arrest if any reasonable effort to that end had been made, as the
victim was unarmed. This is the only legitimate course of action for appellants to follow even if
the victim was really Balagtas, as they were instructed not to kill Balagtas at sight but to arrest
him, and to get him dead or alive only if resistance was offered by him.

Whether or not the mitigating circumstance of incomplete justifying circumstance of fulfilment of


duty can be applied in the case:

Under article 11, paragraph 5 of the Revised Penal Code, there are two requisites for when one
acts in fulfilment of duty or in the lawful exercise of a right or office, namely: that the offender
acted in the performance of a duty or in the lawful exercise of a right or office; and that the injury
or offense committed be the necessary consequence of due performance of such duty or the lawful
exercise such right or office.

In the case, the first requisite is present but the second one is lacking for the crime done by them
is not the necessary consequence of a due performance of their duty. Their duty was to arrest
Balagtas and to get him dead or alive only when he offer resistance, but because of impatience and

46
CRIMINAL LAW 1 CASE DIGESTS
ATTY. LYAN DAVD JUANICO
BLOCK 1-M A.Y. 2020 - 2021

desire to take no chances, they exceeded in the fulfilment of such duty by killing the person whom
they believed to be Balagtas without any resistance from him and without making any inquiry to
his identity.

For all the foregoing, the judgment is modified and appellants are hereby declared guilty of
murder with the mitigating circumstance above mentioned, and accordingly sentenced to an
indeterminate penalty of from five (5) years of prision correctional to fifteen (15) years
of reclusion temporal, with the accessories of the law, and to pay the heirs of the deceased
Serapio Tecson jointly and severally an indemnity of P2,000, with costs.

47
CRIMINAL LAW 1 CASE DIGESTS
ATTY. LYAN DAVD JUANICO
BLOCK 1-M A.Y. 2020 - 2021

8. Padilla vs. Dizon


Administrative Case No. 3086; February 23, 1988
Per Curiam

FACTS:

This is an administrative case filed by the then Commissioner of Customs, Alexander Padilla,
against respondent RTC judge Baltazar R. Dizon for rendering a manifestly erroneous decision
due to gross incompetence and gross ignorance of the law in the criminal case entitled “People of
the Philippines vs. Lo Chi Fai”, acquitting the said accused of the offense of smuggling of foreign
currency out of the country.

People of the Philippines vs. Lo Chi Fai

On July 09, 1986, a Chinese tourist named Lo Chi Fai was apprehended by a customs guard and
two PAFSECOM officers while on board Flight PR 300 of the Philippine Air Lines bound for
Hongkong. At the time of his apprehension, Lo Chi Fai was found carrying with him 380 pieces
of foreign currency and foreign exchange instruments amounting to US$355,349.57 in various
denominations, namely: Japanese Yen, Swiss Franc, Australian Dollar, Singapore Dollar, HFL
Guilder, French Franc, U.S. Dollar English Pound, Malaysian Dollar, Deutsche Mark, Canadian
Dollar and Hongkong Dollar, without any authority as provided by law.

At the time Lo Chi Fai was apprehended, he was able to show only two currency declarations
which he had accomplished upon arrival in Manila during his previous trips, one dated May 04,
1986 entitled CB Currency Declaration NO. 05048 for US$39, 600.00, and another dated June 29,
1986 entitled CB Currency Declaration No. 06346 for Japanese Yen 8,600,000.00.

Afterwards, Lo Chi Fai was charged for violation of Section 6, Central Bank Circular No. 960.

At trial, Lo Chi Fai narrated that he was a businessman from Kowloon, Hongkong engaged in
garment business. He had a group of business associates who decided to invest in business with
him; according to him, they were Wakita Noboyuki, Kobayashi Nabuo, Lee Shiang Pin, Lee Chin
and Cze Kai Kwan, all of whom also had their own businesses in Japan and Hongkong. On April
12, 1986, he brought US$50,000.00 and Japanese Yen 8,500,000.00 which he tried to declare but
the Central Bank representative refused to accept his declaration until he could get a confirmation
as to the sources of the money, for which reason his bank in Hongkong which sent a telex to him
on April 03, 1986. He further testified that because of an agreement to invest in some business,
Lao Chi Fai and his friends decided to start putting their money in a common fund, hence, every
time one of them came to the Philippines, they would declare the money they were bringing in,
and all declarations were handed to and kept by him. Lao Chi Fai further testified that the reason

48
CRIMINAL LAW 1 CASE DIGESTS
ATTY. LYAN DAVD JUANICO
BLOCK 1-M A.Y. 2020 - 2021

he was going back to Hongkong bringing with him all the money was because he was urged by
his associates to bring the money out of the Philippines because the revolution taking place in
Manila might become widespread.

The respondent judge in the present case acquitted Lo Chi Fai stating that “the fact that the accused
had in his possession the foreign currencies when he was about to depart from the Philippines did
not by that act alone make him liable for violation of Section 6. What is imperative is the purpose
for which the act of bringing foreign currencies out of the country was done. It is that which
qualifies the act as criminal or not. There must be that clear intention to violate and benefit from
the act done”.

The Ruling of the Supreme Court:

Whether or not intent is material in prosecuting violators of Section 6 of Circular No. 960:

The respondent judge ought to know that proof of malice or deliberate intent is not essential in
offenses punished by special laws, which are mala prohibita.

Whether or not respondent Judge Baltazar R. Dizon had shown gross incompetence and gross
ignorance of the law in rendering his decision in the case People of the Philippines vs. Lo Chi Fai:

In requiring proof of malice, the respondent has, by his gross ignorance, allowed the accused to go
scot free.

It is a well-established rule that a judge cannot be held to account or answer criminally, civilly or
administratively, for an erroneous decision rendered by him in good faith. However in the present
case, various circumstances make the story concocted by Lao Chi Fai unbelievable as to render
the findings of the respondent judge obviously contrived to favour his acquittal , thereby negating
the respondent’s claim that he rendered the decision in good faith.

It did not matter to the respondent that the foreign currency and foreign currency instruments found
in the possession of Lo Chi Fai did not correspond to the foreign currency declarations presented
by the accused in the trial. The respondent closed his eyes to the fact that the very substantial
amounts of foreign exchange found in possession of the accused at the time of his apprehension
consisted of personal checks of other people, as well as cash in various currency denominations
which clearly belied the claim of Lo Chi Fai that they were part of the funds which he and his
supposed associates had brought in and kept in the Philippines for the purpose of investing in some
business ventures.

Accordingly, the Court finds the respondent Regional Trial Court Judge, Baltazar R. Dizon,
guilty of gross incompetence, gross ignorance of the law and grave and serious misconduct

49
CRIMINAL LAW 1 CASE DIGESTS
ATTY. LYAN DAVD JUANICO
BLOCK 1-M A.Y. 2020 - 2021

affecting his integrity and efficiency, and consistent with the responsibility of this Court for
the just and proper administration of justice and for the attainment of the objective of
maintaining the people's faith in the judiciary (People vs. Valenzuela, 135 SCRA 712), it is
hereby ordered that the Respondent Judge be DISMISSED from the service. All leave and
retirement benefits and privileges to which he may be entitled are hereby forfeited with
prejudice to his being reinstated in any branch of government service, including government-
owned and/or controlled agencies or corporations.

50
CRIMINAL LAW 1 CASE DIGESTS
ATTY. LYAN DAVD JUANICO
BLOCK 1-M A.Y. 2020 - 2021

9. Magno vs. C.A.


G.R. No. 96132; June 26, 1992
Paras, J.

FACTS:

In April 1983 Oriel Magno was in the process of putting up a car repair shop, however he faced
two problems, one is that he did not have complete equipment that would make his venture
workable, and two is that he lacked funds to purchase the necessary equipment to make his
business operational. Hence, Magno approached Corazon Teng, the Vice President of Mancor
Industries, for his needed car repair service equipment of which Mancor was a distributor.

Having been approached by Magno on his predicament who admitted that he had no funds to buy
the equipment needed, Teng referred him to LS Finance and Management Corporation advising
its Vice President, Joey Gomez, that Mancor was willing and able to supply the pieces of
equipment needed if LS Finance could accommodate Magno and provide him credit facilities.

The arrangement went through on condition that Magno has to put up a warranty deposit equivalent
to 30% of the total value of the equipment to be purchased amounting to P29, 790.00. Since Magno
could not come up with such amount, he requested Gomez on a personal level to look for a third
party who could lend him the equivalent amount of the warranty deposit, however, unknown to
Magno, Teng was the one who advanced the warranty deposit on the condition that the same would
be paid as a short term loan at 3% interest.

As part of the arrangement, Magno and LS Finance entered into a leasing agreement whereby LS
Finance would lease the garage equipment and Magno would pay the corresponding rent with the
option to buy the same. After the agreement had been completed, the equipment was delivered to
Magno who in turn issued a post-dated check and gave it to Gomez who then delivered the same
to Teng. When the check matured, Magno requested Gomez not to deposit the check as Magno
was no longer banking with Pacific Bank.

To replace the first check issued, Magno issued another six post-dated checks. Two of those were
dated July 29, 1983 and were deposited and cleared, however, the remaining four were held
momentarily by Teng on the request of Magno as they were not covered with sufficient funds.
These four post-dated checks are the one in question in the present case.

Meanwhile, Magno could not pay the monthly rentals to LS Finance, thus it pulled out the garage
equipment. It was then that Magno discovered that Teng was the one who advanced the warranty
deposit, thus Magno with his wife personally went to Teng to promise to pay the latter, however,

51
CRIMINAL LAW 1 CASE DIGESTS
ATTY. LYAN DAVD JUANICO
BLOCK 1-M A.Y. 2020 - 2021

the payment was never made and when the four post-dated checks were deposited they were
returned for the reason “account closed.”

The Ruling of the Regional Trial Court:

The accused-petitioner was convicted for violations of BP Blg. 22 and was sentenced one year
imprisonment in each Criminal Case Nos. Q-35693, Q-35695 and Q-35696, and to pay to the
complainant the respective amounts reflected in subject checks.

The Ruling of the Supreme Court:

Whether or not Magno is liable for the four unpaid post-dated checks under BP Blg.22:

This Court is intrigued about the outcome of the checks subject of the cases which were intended
by the parties to cover the warranty deposit requirement of the financing company.

By the nature of the warranty deposit amounting to P29, 790.00 corresponding to the 30% of the
purchase/lease value of the equipment subject of the agreement, it is obvious that the cash out
made by Mrs. Teng was not used by Magno. It would have been different if Magno opted to
purchase the equipment on or about the termination of the lease-purchase agreement in which case
he had to pay the additional amount of the warranty deposit which should have formed part of the
purchase price, however, the transaction did not ripen into a purchase. Hence, it is lawful and just
that the warranty deposit should not be charged against Magno. To charge Magno for the refund
of the warranty deposit which he did not withdraw as it was not his account is equivalent in making
him pay an unjust debt.

Additionally, the fact that Magno did not even know that the checks he issued were turned over by
Gomez to Teng, who requested that he be not informed, was enough to evoke suspicion that the
transaction is irregular and immoral per se. It is safe to assume that such transaction is a scheme
whereby Teng as the supplier of the equipment in the name of her corporation Mancor would be
able to sell/lease its goods, and at the same time privately financing those who desperately need
petty accommodation as the one in the present case.

Batasang Pambansa Blg. 22 was devised to safeguard the interest of the banking system and the
legitimate public checking account user, it did not intend to shelter or favour nor encourage users
of the system to enrich themselves through manipulations and circumvention of the noble purpose
and objective of the law. Under the utilitarian theory, the protective theory affirms that the primary
function of punishment is to protect the society against actual and potential wrongdoers. It may
not be clear whether Magno could be considered as actually having committed the wrong sought
to be punished in the offense charged but it is safe to say that the actuations of Teng amount to that

52
CRIMINAL LAW 1 CASE DIGESTS
ATTY. LYAN DAVD JUANICO
BLOCK 1-M A.Y. 2020 - 2021

of potential wrongdoers whose operations should also be clipped at some point in time on order
that the unwary public will not be falling prey to such vicious transaction.

In applying the punishment imposed upon Magno, the objective of retribution of a wronged society
should be directed against the actual and potential wrongdoers. In the present case, there is no
doubt the Magno’s four checks were used to collateralize an accommodation, and not to cover the
receipt of an “actual account or credit for value” as this was absent, therefore, Magno should not
be punished for mere issuance of the checks in question. Furthermore, the element of “knowing at
the time of issue that he does not have sufficient funds in or credit with the drawee bank for the
payment of such check in full upon its presentment, which check is subsequently dishonoured by
the drawee bank for insufficiency of funds or credit or would have been dishonoured for the same
reason” is inversely applied. From the very beginning, Magno never hid the fact that he did not
have sufficient funds to put up the warranty deposit.

Wherefore, the appealed decision is REVERSED and the accused-petitioner is hereby


ACQUITTED of the crime charged.

53
CRIMINAL LAW 1 CASE DIGESTS
ATTY. LYAN DAVD JUANICO
BLOCK 1-M A.Y. 2020 - 2021

10. Garcia vs. C.A.


G.R. No. 157171; March 14, 2006
Quisumbing, J.

FACTS:

The present case is based on the complaint-affidavit of Aquilino Q. Pimentel, Jr., who ran in the
1995 senatorial elections. On March 30, 1998, an Information was filed in the RTC charging
Herminio R. Romero, Renato R. Viray, Rachel Palisoc and Francisca de Vera and Aresnia B.
Garcia with violation of Republic Act 6646 Section 27(b).

According to the information, on May 11, 1995, within the canvassing period of the 1995 elections,
Election Officer Aresenia B. Garcia, Municipal Treasurer Herminio R. Romero, Public School
District Supervisor Renato R. Viray, Chairman, Vice-Chairman and Member-Secretary
respectively of the Municipal Board of Canvassers of Pangasinan, along with tabulators Rachel
Palisoc and Francisca e Vera decreased the votes received by senatorial candidate Aquilino Q.
Pimentel, Jr. from 6, 998 votes, as clearly disclosed in the total number if votes in the 159 precincts,
to 1, 921 votes, with a difference of 5, 077 votes.

The Ruling of the Regional Trial Court:

On September 11, 2000, the RTC acquitted all the accused for insufficiency of evidence, except
Arsenia B. Garcia.

The Ruling of the Court of Appeals:

The petitioner appealed before the Court of Appeals which affirmed with modification the decision
made by the RTC.

The Argument of the Petitioner:

According to Garcia, there was no motive on her part to reduce the votes of Pimentel, Jr.

The Argument of the Respondent:

According to the respondent, good faith is not a defence in the violations of an election law, which
falls under the class of mala prohibita.

The Ruling of the Supreme Court:

Whether or not a violation of Section 27(b) of RA No. 6646 is classified under mala prohibita:

54
CRIMINAL LAW 1 CASE DIGESTS
ATTY. LYAN DAVD JUANICO
BLOCK 1-M A.Y. 2020 - 2021

Generally, mala in se felonies are defined and penalized under Revised Penal Code, however,
when the acts complained of are inherently immoral, they are deemed mala in se, even if they are
punished by a special law. Accordingly, in mala in se felonies, criminal intent must be clearly
established with the other elements of the crime, otherwise there is no crime committed.

On the other hand, in mala prohibita felonies, the criminal acts are not inherently immoral but
become punishable only because the law says they are forbidden. With these crimes, the sole issue
is whether the law has been violated, therefore, criminal intent is not necessary where the acts are
prohibited for reasons of public policy.

Clearly, the acts prohibited in Section 27(b) are mala in se. For otherwise, even errors and mistakes
committed due to overwork and fatigue would be punishable. Given the volume of votes to be
counted and canvassed within a limited amount of time, errors and miscalculations are bound to
happen, and it could not be the intent of the law to punish unintentional election canvass errors.
However, intentionally increasing or decreasing the number of votes received by a candidate is
inherently immoral, since it is done with malice and intent to injure another.

Criminal intent is presumed to exist on the part of the person who executes an act which the law
punishes, unless the contrary shall appear, thus, whoever invokes good faith as a defence has the
burden of proving its existence.

During the trial of the case, Garcia admitted that she was indeed the one who announced the figure
of 1, 291, which was then entered by Viray who was the secretary of the board that time. Garcia
further admitted that she was also the one who prepared the certificate of canvas even though it
was not her duty to do so. Hence, in our mind, preparing the certificate of canvas even if it was not
her task manifests an intention to perpetuate the erroneous entry in the said certificate.

As chairman of the Municipal Board of Canvassers, Garcia’s concern was to assure accurate,
correct and authentic entry of the votes. Her failure to exercise maximum efficiency and fidelity
to her trust deserves not only censure but also the concomitant sanctions as a matter of criminal
responsibility pursuant to the dictates of the law.

Furthermore, upon our review, the votes should total to 6, 998 and not 6, 921. As between the
alleged votes of 6, 921 and the actual votes received of 6, 998, the difference is only 77 votes.
Such discrepancy may be validly attributed to mistake or error due to fatigue. However, a decrease
of 5,000 votes such as the one made by Garcia is substantial. It cannot be allowed to remain in
record unchallenged, especially when the error results from the mere transfer of totals from one
document to another.

55
CRIMINAL LAW 1 CASE DIGESTS
ATTY. LYAN DAVD JUANICO
BLOCK 1-M A.Y. 2020 - 2021

11. People vs. Pugay


G.R. No. L-74324; November 17, 1988
Medialdea, J.

FACTS:

Bayani Miranda, a 25-year old retardate, and Fernando Pugay were friends. Miranda used to run
errands for Pugay and there are times when they slept together.

On the evening of May 19, 1982, a town fiesta fair was being held in the public plaza of Rosario,
Cavite. There were different kinds of rides, one of which is a Ferris wheel. After midnight of the
same date, Eduardo Gabion was sitting in the Ferris wheel and reading a comic book with his
friend Henry when Pugay and Samson with several companions arrived. As the group saw Miranda
walking nearby, they started making fun of him. They made Miranda dance by tickling him with
a piece of wood. Not content with what they were doing, Pugay suddenly took a can of gasoline
from under the engine of the Ferris wheel and poured it in the body of Miranda. Gabion told Pugay
not to do so while the latter was already pouring the contents. Then Samson set Miranda on fire.
The Ferris wheel operator later arrived and doused with water the burning body of Miranda and
others wrapped the victim with rags to extinguish the flame.

The body of Miranda was still aflame when police officer Rolando Silangcruz and other police
officers arrived at the scene of the incident. Upon inquiring as to who was responsible for the act,
the people around spontaneously pointed to Pugay and Samson. The victim was later rushed to
Grace Hospital for treatment.

The Argument of the Defense:

Although Pugay admitted that he poured the can of gasoline on Miranda, he argued that what he
believed to be the contents of the can was water and not gasoline when he did such act. He further
testified that he saw Samson set the victim on fire.

Samson on the other hand denied that he was the one who set Miranda on fire. He admitted that
he saw Pugay pour gasoline on Miranda but failed to see who set the victim in fire.

The Ruling of the Regional Trial Court:

The trial court rendered a decision finding both accused guilty on the crime of murder but crediting
in favour of Pugay the mitigating circumstance of lack of intention to commit s grave a wrong.

The Ruling of the Supreme Court:

56
CRIMINAL LAW 1 CASE DIGESTS
ATTY. LYAN DAVD JUANICO
BLOCK 1-M A.Y. 2020 - 2021

Whether or not the accused Pugay is guilty on the crime of murder:

On the criminal responsibility of Pugay, the Court believes that the accused knew that the contents
of the can was gasoline because the stinging smell of the flammable liquid could not have escaped
his notice. Clearly, he failed to exercise all the diligence necessary to avoid every undesirable
consequence arising from any act that may be committed by his companions, who at that time were
making fun of Miranda. In US vs. Maleza , the Court had ruled that a man must use common sense
and exercise due reflection in all his acts; it is his duty to be cautious, careful and prudent, if not
from instinct, then through fear of incurring punishment. Hence, the accused is only guilty of
homicide through reckless imprudence defined in Article 365 of the Revised Penal Code, as
amended.

Whether or not the accused Samson is guilty on the crime of murder:

Meanwhile, on the criminal responsibility of Samson, there is an entire absence of proof in the
record that the accused Samson had some reason to kill Miranda before the incident. However,
there can be no doubt that Samson knew very well that the liquid poured on Miranda was
gasoline and that such is flammable for he would not have committed the act of setting Miranda
on fire if it were otherwise. Giving Samson the benefit of doubt, it can be said that he merely
intended to set Miranda’s clothes on fire; however, his act does not relieve him of criminal
responsibility. Burning the clothes of the victim would cause at the very least some kind of physical
injuries which in itself is a felony. If his act resulted into a graver offense, he must be held
responsible thereof. Article 4 of the RPC provides that criminal responsibility shall be incurred by
any person committing a felony although the wrongful act done be different from that which he
intended. Therefore, as there is no sufficient evidence appearing in the record establishing any
qualifying circumstances, Samson is only guilty of the crime of homicide.

The proper penalty that the accused Samson must suffer is an indeterminate one ranging
from eight (8) years of prision mayor, as minimum, to fourteen (14) years of reclusion
temporal, as maximum. The lower court held the accused solidarily liable for P13,940.00, the
amount spent by Miranda's parents for his hospitalization, wake and interment. The
indemnity for death is P30,000.00. Hence, the indemnity to the heirs of the deceased Miranda
is increased to P43,940.00. Both accused shall be jointly and severally liable for the aforesaid
amount plus the P10,000.00 as moral damages and P5,000.00 as exemplary damages as found
by the court a quo.
Accordingly, the judgment is affirmed with the modifications above-indicated. Costs against
the accused-appellants.

57
CRIMINAL LAW 1 CASE DIGESTS
ATTY. LYAN DAVD JUANICO
BLOCK 1-M A.Y. 2020 - 2021

12. Ivler vs. San Pedro


G.R. No. 172716 ; November 17, 2010
Carpio, J.

FACTS:
On August 2004, following a vehicular collision, Jason Ivler was charged with two separate
offenses, one reckless imprudence resulting in slight physical injuries for injuries sustained by
Evangeline L. Ponce, and one reckless imprudence resulting in homicide and damage to property
for the death of Evangeline’s husband Nestor C. Ponce and damage to the spouses’ vehicle.
On September 7, 2004, Ivler pleaded guilty to the charge in Criminal Case No. 82367 where he
was charged with reckless imprudence resulting in slight physical injuries. He was given the
penalty of public censure. Invoking this conviction, petitioner moved to quash the Information in
Criminal Case No. 82366, where he was charged with reckless imprudence resulting in homicide
and damage to property, for placing him in jeopardy of second punishment for the same offense
of reckless imprudence.

The Ruling of the Supreme Court:


Whether or not the protection afforded by the Constitution shielding petitioner from prosecutions
placing him in jeopardy of second punishment for the same offense bars further proceedings in
Criminal Case No. 82366:
An accused has a constitutional right not to be “twice put in jeopardy of punishment for the same
offense” which protects him from post-conviction prosecution for the same offense, with the prior
verdict rendered by a court of competent jurisdiction upon a valid information. Therefore, if the
contention of Ivler that criminal case nos. 82367 and 82366 involve the same offense is of merit
then the above mentioned right would be applicable to him.
The two charges against petitioner arise from the same facts and were prosecuted under the same
provision of the Revised Penal Code, Article 365 which defines and penalizes quasi-offenses.
Article 365 defines the two kinds of imprudence, reckless and simple imprudence. Reckless
imprudence consists in voluntary, but without malice, doing or failing to do an act from which
material damage results by reason of inexcusable lack of precaution on the part of the person
performing or failing to perform such act, taking into consideration his employment or occupation,
degree of intelligence, physical condition and other circumstances regarding persons, time and
place.
Meanwhile, simple imprudence consists in the lack of precaution displayed in those cases in which
the damage impending to be caused is not immediate nor the danger clearly manifest.

58
CRIMINAL LAW 1 CASE DIGESTS
ATTY. LYAN DAVD JUANICO
BLOCK 1-M A.Y. 2020 - 2021

Conceptually, quasi-offenses penalize the mental attitude or condition behind the act, the
dangerous recklessness, lack of care or foresight, the imprudencia punible , unlike wilful offenses
which punishes the intentional criminal act.
Reckless imprudence under Article 365 is a single quasi-offense by itself and not merely a means
to commit other crimes such that conviction or acquittal of such quasi-offense bars subsequent
prosecution for the same quasi-offense, regardless of its various resulting acts. For the essence of
the quasi offense of criminal negligence under said Article lies in the execution of an imprudent
or negligent act that, if intentionally done, would be punishable as a felony. The law penalizes thus
the negligent or careless act, not the result thereof. The gravity of the consequence is only taken
into account to determine the penalty, it does not qualify the substance of the offense. And, as the
careless act is single, whether the injurious result should affect one person or several persons, the
offense remains one and the same, and cannot be split into different crimes and prosecutions.
WHEREFORE, we GRANT the petition. We REVERSE the Orders dated 2 February 2006
and 2 May 2006 of the Regional Trial Court of Pasig City, Branch 157. We DISMISS the
Information in Criminal Case No. 82366 against petitioner Jason Ivler y Aguilar pending
with the Metropolitan Trial Court of Pasig City, Branch 71 on the ground of double
jeopardy.

59
CRIMINAL LAW 1 CASE DIGESTS
ATTY. LYAN DAVD JUANICO
BLOCK 1-M A.Y. 2020 - 2021

THE PEOPLE OF THE PHILIPPINES vs. JULIO GUILLEN


G.R. No. L-1477; January 18, 1950
PER CURIAM, J.:

FACTS:

Julio Guillen voted for the defeated candidate in the Presidential Elections held in 1946. Manuel
Roxas won that election and assumed the office of the President of the Republic of the Philippines.
According to the accused, he became disappointed in former President Roxas when he failed to
fulfill his promises he made during the presidential elections. Guillen further state that his
disappointment was aggravated when President Roxas sponsored and campaigned for the approval
of the so-called "parity" measure. Hence he determined to assassinate the President.

On the meeting held by the Liberal Party in Plaza Miranda, Guillen was carrying two hand
grenades and he decided to carry out his evil purpose. He stood on the chair on which he had been
sitting and he threw the grenade at the President when the latter had just closed his speech.

Guillen was arrested by members of the Police Department about two hours after the occurrence.

ISSUE:
Whether or not appellant is guilty of the complex crime of murder and multiple frustrated murder.

RULING:
Yes. Appellant is guilty of the crimes charged against him.

In throwing hand grenade at the President with the intention of killing him, the appellant acted
with malice. He is therefore liable for all the consequences of his wrongful act. In accordance with
article 4 of the Revised Penal Code, criminal liability is incurred by any person committing felony
(delito) although the wrongful act done be different from that which he intended. In criminal
negligence, the injury caused to another should be unintentional, it being simply the incident of
another act performed without malice.

The Court however ruled that he is only liable for attempted murder. There can be no question that
the accused attempted to kill President Roxas by throwing a hand grenade at him with the intention
to kill him, thereby commencing the commission of a felony by over acts, but he did not succeed
in assassinating him “by reason of some cause or accident other than his own spontaneous
desistance.”

60
CRIMINAL LAW 1 CASE DIGESTS
ATTY. LYAN DAVD JUANICO
BLOCK 1-M A.Y. 2020 - 2021

14. PEOPLE OF THE PHILIPPINES vs. ROLLY ADRIANO y SAMSON, et al.


G.R. No. 205228; July 15, 2015
PEREZ, J.:

FACTS:
On the morning of 13 March 2007, POI Garabiles and P02 Santos wearing civilian clothes, were
on their way to Camp Olivas, Pampanga, riding a motorcycle. While they were at Barangay
Malapit San Isidro, Nueva Ecija, a speeding blue Toyota Corolla (Corolla) with plate no. WHK
635, heading towards the same direction, overtook them and the car in front of them, a maroon
Honda CRV (CRY) with plate no. CTL 957.

When the Corolla reached alongside the CRV, the passenger on the front seat of the Corolla shot
the CRV and caused the CRV to swerve and fall in the canal in the road embankment. Four (4)
armed men then suddenly alighted the Corolla and started shooting at the driver of the CRV, who
was later identified as Cabiedes. During the shooting, a bystander, Bulanan, who was standing
near the road embankment, was hit by a stray bullet.

The police officers were able to track down the car and was found out that the owner of the Corolla
was registered to a certain Rivera. Rivera admitted that it was his car however he further states that
he is in the rental business and leased the car to Adriano. When Adriano arrived at Rivera’s shop
with the Corolla, the police officers were able to identify him and was immediately arrested.
ISSUE:
Whether or not Adriano is responsible for the death of the bystander, Ofelia Bulanan.
RULING:
Yes. Appellant is liable for the death of Bulanan.
Adriano is guilty of the death of Bulanan under Article 4 of the Revised Penal Code, pursuant to
the doctrine of aberratio ictus, which imposes criminal liability for the acts committed in violation
of law and for all the natural and logical consequences resulting therefrom. While it may not have
been Adriano's intention to shoot Bulanan, this fact will not exculpate him. Bulanan' s death caused
by the bullet fired by Adriano was the natural and direct consequence of Adriano's felonious deadly
assault against Cabiedes.
Criminal liability is incurred by any person committing a felony although the wrongful act be
different from that which is intended. One who commits an intentional felony is responsible for all
the consequences which may naturally or logically result therefrom, whether foreseen or intended
or not. The rationale of the rule is found in the doctrine, “el que es causa de la causa es causa del
mal causado”, or he who is the cause of the cause is the cause of the evil caused.

Thus, Adriano is responsible for the consequences of his act of shooting Cabiedes.

61
CRIMINAL LAW 1 CASE DIGESTS
ATTY. LYAN DAVD JUANICO
BLOCK 1-M A.Y. 2020 - 2021

15. THE PEOPLE OF THE PHILIPPINE ISLANDS vs. GINES ALBURQUERQUE Y


SANCHEZ
G.R. No. L-38773; December 19, 1933
AVANCEÑA, C.J.:

FACTS:
Appellant Gines Albuquerque was a 55-year-old widower and father to nine children. One of his
daughters is Maria who was married and there was another who was a nun. Other daughters and
appellant lived with Maria with whom they depended for support.

One of them named Pilar had intimate relations with deceased Manuel Osma in 1928 and bore a
child together. It was through this that Albuquerque got to know Osma although he did not know
the level of their intimacy. The appellant did not know that his daughter's relations with the
deceased had gone to such extremes, that he had to be deceived with the information that she had
gone to her godfather's house in Singalong, when in fact she had been taken to the Chinese Hospital
for delivery. The appellant learned the truth only when Pilar returned home with her child.
Albuquerque only learned the truth when Pilar returned home with her child. He was deeply
affected by this incident as it brought dishonor to the family and it also meant added burden to
Maria.

Manuel Osma promised to provide financial support for the child but didn’t keep his promise. One
day, appellant went to Osma’s workplace to propose that Osma marry Pilar. Upon his refusal,
appellant whipped out his penknife. He only intended to scar his face but due to lack of control of
movement and Osma’s resistance, the penknife landed on the base of Osma’s neck.

ISSUE:
Whether or not Appellant intentionally killed the deceased.

RULING:
No. The appellant in this case did not intentionally kill the deceased.
The Supreme Court ruled that appellant only wanted to inflict a wound that would leave a
permanent scar on the face of the deceased, or one that would compel him to remain in the hospital
for a week or two but never intended to kill him, because then it would frustrate his plan of
compelling him to marry or, at least, support his daughter. This was supported by evidence
presented by the defense, saying that the appellant have no control of his right are on account of
paralysis. That the resulting injury is greater that what he has intended. The appellant had stated
this intention in some of his letters to the deceased by way of a threat to induce him to accept his
proposal for the benefit of his daughter.

62
CRIMINAL LAW 1 CASE DIGESTS
ATTY. LYAN DAVD JUANICO
BLOCK 1-M A.Y. 2020 - 2021

16. SALUD VILLANUEVA VDA. DE BATACLAN vs. MARIANO MEDINA


G.R. No. L-10126; October 22, 1957
EN BANC

FACTS:
On September 13, 1952, bus no. 30 of the Medina Transportation, operated by its owner defendant
Mariano Medina, left the town of Amadeo, Cavite, on its way to Pasay City, driven by its regular
chauffeur, Conrado Saylon. There were about eighteen passengers, including the driver and
conductor. Among the passengers were Juan Bataclan. At about 2:00 o'clock that same morning,
while the bus was running within the jurisdiction of Imus, Cavite, one of the front tires burst and
the vehicle began to zig-zag until it fell into a canal or ditch on the right side of the road and turned
turtle. Some of the passengers managed to leave the bus the best way they could, others had to be
helped or pulled out, while the three passengers were not able to get out of the bus.
Among the burned bodies that were found was Juan Bataclan. This prompted his widow, Salud
Villanueva, in her name and in behalf of her five minor children, brought the present suit to recover
from Mariano Medina damages.

ISSUE:
Whether or not the common carrier is liable for the death of the deceased.

RULING:
Yes. The common carrier is liable for the death of the deceased.

The Supreme Court ruled that there was negligence on the part of the defendant, through the driver,
Saylon. There is evidence to show that at the time of the blow out, the bus was speeding, as testified
to by one of the passengers, and as shown by the fact that according to the testimony of the
witnesses. The Court held that the proximate cause was the overturning of the bus because when
the vehicle turned not only on its side but completely on its back, the leaking of the gasoline from
the tank was not unnatural or unexpected. The coming of the men with a lighted torch was in
response to the call for help, made not only by the passengers, but most probably, by the driver
and the conductor themselves, and that because it was dark (about 2:30 in the morning), the
rescuers had to carry a light with them, and coming as they did from a rural area where lanterns
and flashlights were not available.

In other words, the coming of the men with a torch was to be expected and was a natural sequence
of the overturning of the bus, the trapping of some of its passengers and the call for outside help.

63
CRIMINAL LAW 1 CASE DIGESTS
ATTY. LYAN DAVD JUANICO
BLOCK 1-M A.Y. 2020 - 2021

17. PEOPLE OF THE PHILIPPINES v. FERNANDO ILIGAN y JAMITO, EDMUNDO


ASIS y ILIGAN and JUAN MACANDOG (at large), Defendants, FERNANDO ILIGAN y
JAMITO and EDMUNDO ASIS y ILIGAN
G.R. No. 75369; November 26, 1990
FERNAN, J.:

FACTS:
Esmeraldo Quiñnoes, Zaldy Asis, and Felix Lukban were walking home from Brgy. Sto. Domingo,
Vinzons, CamNor after attending a barrio fiesta dance. They met accused Fernando Iligan,
Edmundo Asis, and Juan Macandog. Edmunso Asis pushed Quiñones et al aside prompting Asis
to box him. Lukban said they had no desire to fight. Iligan, upon seeing his nephew fall, drew from
his back a bolo and hacked Zaldy Asis but missed. Terrified, Quiñones et al ran pursued by the 3
accused for half an hour. After resting for a while, Quiñones invited Asis and Lukban to
accompany him to his house so he could change to his working clothes. While they were walking,
the 3 accused emerged and Iligan hacked Quiñnoes with his bolo hitting him on the forehead and
causing him to fall down. Horrified, Asis and Lukban fled but returned when they heard shouts of
people. “May nadale na.” On the spot where Quiñones was hacked, Asis and Lukban saw him
already dead with his head busted. Autopsy revealed he died of “shock and massive cerebral
hemorrhages due to a vehicular accident.” After Quiñones, Jr. had fallen from the bolo-hacking
perpetrated by Iligan, he was run over by a vehicle.

ISSUE:
Whether or not the appellant is liable for the death of the deceased.

RULING:
Yes. Appellant in this case is still liable for the death of the deceased.

The intentional felony committed was the hacking of the head of Quiñones, Jr. by Iligan. That it
was considered as superficial by the physician who autopsied Quiñones is beside the point. What
is material is that by the instrument used in hacking Quiñones, Jr. and the location of the wound,
the assault was meant not only to immobilize the victim but to do away with him as it was directed
at a vital and delicate part of the body, the head

While Iligan’s hacking of Quiñones, Jr.’s head might not have been the direct cause, it was the
proximate cause of the latter’s death. The sequence of events from Iligan’s assault on him to the
time Quiñones, Jr. was run over by a vehicle is one unbroken chain of events. Having triggered
such events, Iligan cannot escape liability.

64
CRIMINAL LAW 1 CASE DIGESTS
ATTY. LYAN DAVD JUANICO
BLOCK 1-M A.Y. 2020 - 2021

18. FILOMENO URBANO vs. HON. INTERMEDIATE APPELLATE COURT AND


PEOPLE OF THE PHILIPPINES
G.R. No. 72964; January 7, 1988
GUTIERREZ, JR., J.:

FACTS:
Petitioner was on his way to his ricefield in Dagupan. He found the place where he stored palay
flooded with water coming from the irrigation canal. Urbano went to the elevated portion to see
what happened, and there he saw Marcelino Javier and Emilio Efre cutting grass. Javier admitted
that he was the one who opened the canal. A quarrel ensued, and Urbano hit Javier on the right
palm with his bolo, and again on the leg with the back of the bolo. Urbano and Javier had an
amicable settlement. Javier was rushed to the hospital where he had lockjaw and convulsions. The
doctor found the condition to be caused by tetanus toxin which infected the healing wound in his
palm. He died the following day. Urbano was charged with homicide and was found guilty both
by the trial court and on appeal by the Court of Appeals. The lower courts ruled that Javier’s death
was the natural and logical consequence of Urbano’s unlawful act.

ISSUE:
Whether or not petitioner is liable for the death of the deceased.

RULING:
No. The petitioner in this case cannot be held liable for the death of the deceased.

The petitioner's criminal liability in this respect was wiped out by the victim's own act.
The rule is that the death of the victim must be the direct, natural, and logical consequence of the
wounds inflicted upon him by the accuse and since we are dealing with a criminal conviction, the
proof that the accused caused the victim's death must convince a rational mind beyond reasonable
doubt. The medical findings, however, leads the Court to a distinct possibility that the infection of
the wound by tetanus was an efficient intervening cause later or between the time Javier was
wounded to the time of his death. The infection was, therefore, distinct and foreign to the crime.

65
CRIMINAL LAW 1 CASE DIGESTS
ATTY. LYAN DAVD JUANICO
BLOCK 1-M A.Y. 2020 - 2021

19. SULPICIO INTOD vs. HON. COURT OF APPEALS AND PEOPLE OF THE
PHILIPPINES
G.R. No. 103119 Octoboer 21, 1992
CAMPOS, JR., J.:

FACTS:
Petitioner gathered four other men to kill Bernardina Palangpangan because of a land dispute
between the two. When they reached her house, Intod and his companions fired at the bedroom of
Bernardina. It turned out that Bernardina was in another city and her son-in-law and his family
were the only ones occupying her house. Fortunately, no one was injured.
Petitioner and his companions were positively identified by witnesses. The RTC convicted Intod
and his companions of attempted murder which was affirmed by the Court of Appeals. Hence this
case.

ISSUE:
Whether or not Intod is guilty of attempted murder.

RULING:
No. The Supreme Court ruled that appellant is only guilty of an impossible crime.

Legal impossibility would apply to those circumstances where (1) the motive, desire and
expectation is to perform an act in violation of the law; (2) there is intention to perform the physical
act; (3) there is a performance of the intended physical act; and (4) the consequence resulting from
the intended act does not amount to a crime.
On the other hand, factual impossibility occurs when extraneous circumstances unknown to the
actor or beyond his control prevent the consummation of the intended crime. The case at bar
belongs to this category. Petitioner shoots the place where he thought his victim would be, although
in reality, the victim was not present in said place and thus, the petitioner failed to accomplish his
end.

Further, factual impossibility of the commission of the crime is not a defense. If the crime could
have been committed had the circumstances been as the defendant believed them to be, it is no
defense that in reality the crime was impossible of commission.

66
CRIMINAL LAW 1 CASE DIGESTS
ATTY. LYAN DAVD JUANICO
BLOCK 1-M A.Y. 2020 - 2021

20. PEOPLE V. SALADINO


G.R. No. L-3634; 30 May 1951
J. Bengzon

FACTS:
In the night of June 23, 1948 Corporal Bartolo Saladino and Private Anastacia Alejo of the
Philippine Constabulary were resting in the house of Celso Abucay in Paoay, Ilocos Norte,
together with policemen Melchor Quevedo, Wilfredo Osman and George Plan of that municipality.
They had gone on patrol duty to the barrio for the purpose of apprehending those who on a previous
night had fired upon the dwelling. About midnight they were suddenly awakened by cries for help.
They went down and were approached by one Felix Pasion who reported he had been robbed, one
of the robbers being Luis Bernabe. The next morning, Saladino and Alejo, accompanied by the
policemen proceeded to the house of Luis Bernabe in Barrio Samac of San Nicolas same province.
Having found the suspect, they brought him, for questioning, to the residence of Felix Pasion in
Barrio Singao same municipality. It was about ten in the morning. As Pasion reiterated his
imputation, Saladino led Bernabe up the house for further investigation. He was followed by
Anastacio Alejo and the policemen. Bernabe denied the charge. To extract a confession, Saladino
repeatedly boxed and kicked him in different parts of the body. Bernabe continued denying his
guilt. Saladino got a piece of wood, two inched thick and one yard long, and clubbed him several
times on the chest, abdomen and the back. Then he called on Alejo to take his turn. Alejo
reluctantly whipped Bernabe four times with the branch of a tree, and then retired to the kitchen.
Saladino again questioned his prisoner and as the latter would not admit his culpability, he repeated
the severe beating, and tying Bernabe’s wrists together with a rubber strap, made him stand on a
chair, tied the strap to a beam in the ceiling and then pushed the chair from under Bernabe with
the result that the latter was left hanging in the air. While in that position Bernabe was cudgeled
by Saladino, with the wooden club, on the sides, armpits, stomach, hips and back. It was at this
juncture that policeman Plan interceded for the victim saying, Stop now corporal. Better bring him
to your headquarters and there you will investigate him”. But Saladino ignored plea, and resumed
the maltreatment, loudly predicting that Bernabe would confess before noon. After Bernabe had
remained suspended for five minutes, Saladino untied him, made him sit on a chair and urged him
to acknowledge his offense. As Bernabe persisted in his refusal, Saladino kicked the chair and
Bernabe fell on the floor, even as Saladino pouncing on his captive booted him several times until
the latter lay motionless on the floor. “It seems he is dead,” Policeman Oaman exclaimed. Saladino
replied “No, he is only feigning death” and presently stepped on Bernabe’s throat and chest. Then
Saladino let him alone for fifteen minutes, during which time Bernabe did not stir nor breathe. An
old man approaching Bernabe and taking his pulse said that the man was dead. Suddenly realizing
his predicament, Saladino ordered two civilians to carry Bernabe down and told Alejo: “shoot him
67
CRIMINAL LAW 1 CASE DIGESTS
ATTY. LYAN DAVD JUANICO
BLOCK 1-M A.Y. 2020 - 2021

now and we will say that he ran away”. Complying with the corporal’s order Alejo shot Bernabe
four times with his carbine, after the latter had been laid down flat on
his stomach about thirty meters away from the house. Three days afterwards Bernabe was intered.

ISSUE:
Whether or not Alejo is guilty as a conspirator to the murder of Bernabe

RULING:
From the foregoing it is plain that Bernabe having died as a consequence of the violent mauling
by Saladino, the latter must be declared guilty of assassination. Anastacio Alejo does not appear
to have conspired with him, and is not liable either as principal or as accomplice of the murder.
But he is guilty as accessory after the fact for having performed acts tending to conceal Saladino’s
crime by making it appear that Bernabe had run away. As accessory after the fact, Alejo is liable
to a penalty lower by two degrees than that prescribed by law for the consummated felony of
murder, namely, prision correctional in its maximum period to prision mayor in its medium period.
(Art. 53 in connection with Art. 248 of the Revised Penal Code.)

68
CRIMINAL LAW 1 CASE DIGESTS
ATTY. LYAN DAVD JUANICO
BLOCK 1-M A.Y. 2020 - 2021

21. JACINTO V. PEOPLE


G.R. No. 162540; 13 July 2009
J. Peralta

FACTS:
In the month of June 1997, Isabelita Aquino Milabo, also known as Baby Aquino, handed
petitioner Banco De Oro (BDO) Check Number 0132649 postdated July 14, 1997 in the amount
of ₱10,000.00. The check was payment for Baby Aquino's purchases from Mega Foam Int'l., Inc.,
and petitioner was then the collector of Mega Foam. Somehow, the check was deposited in the
Land Bank account of Generoso Capitle, the husband of Jacqueline Capitle; the latter is the sister
of petitioner and the former pricing, merchandising and inventory clerk of Mega Foam.
Meanwhile, Rowena Ricablanca, another employee of Mega Foam, received a phone call
sometime in the middle of July from one of their customers, Jennifer Sanalila. The customer
wanted to know if she could issue checks payable to the account of Mega Foam, instead of issuing
the checks payable to CASH. Said customer had apparently been instructed by Jacqueline Capitle
to make check payments to Mega Foam payable to CASH. Around that time, Ricablanca also
received a phone call from an employee of Land Bank, Valenzuela Branch, who was looking for
Generoso Capitle. The reason for the call was to inform Capitle that the subject BDO check
deposited in his account had been dishonored.
Ricablanca then phoned accused Anita Valencia, a former employee/collector of Mega Foam,
asking the latter to inform Jacqueline Capitle about the phone call from Land Bank regarding the
bounced check. Ricablanca explained that she had to call and relay the message through Valencia,
because the Capitles did not have a phone; but they could be reached through Valencia, a neighbor
and former co-employee of Jacqueline Capitle at Mega Foam.
Valencia then told Ricablanca that the check came from Baby Aquino, and instructed Ricablanca
to ask Baby Aquino to replace the check with cash. Valencia also told Ricablanca of a plan to take
the cash and divide it equally into four: for herself, Ricablanca, petitioner Jacinto and Jacqueline
Capitle. Ricablanca, upon the advise of Mega Foam's accountant, reported the matter to the owner
of Mega Foam, Joseph Dyhengco.
Thereafter, Joseph Dyhengco talked to Baby Aquino and was able to confirm that the latter indeed
handed petitioner a BDO check for ₱10,000.00 sometime in June 1997 as payment for her
purchases from Mega Foam. Baby Aquino further testified that, sometime in July 1997, petitioner
also called her on the phone to tell her that the BDO check bounced. Verification from company
records showed that petitioner never remitted the subject check to Mega Foam. However, Baby

69
CRIMINAL LAW 1 CASE DIGESTS
ATTY. LYAN DAVD JUANICO
BLOCK 1-M A.Y. 2020 - 2021

Aquino said that she had already paid Mega Foam ₱10,000.00 cash in August 1997 as replacement
for the dishonored check.
Generoso Capitle, presented as a hostile witness, admitted depositing the subject BDO check in
his bank account, but explained that the check came into his possession when some unknown
woman arrived at his house around the first week of July 1997 to have the check rediscounted. He
parted with his cash in exchange for the check without even bothering to inquire into the identity
of the woman or her address. When he was informed by the bank that the check bounced, he merely
disregarded it as he didn’t know where to find the woman who rediscounted the check.
Meanwhile, Dyhengco filed a Complaint with the National Bureau of Investigation (NBI) and
worked out an entrapment operation with its agents. Ten pieces of ₱1,000.00 bills provided by
Dyhengco were marked and dusted with fluorescent powder by the NBI. Thereafter, the bills were
given to Ricablanca, who was tasked to pretend that she was going along with Valencia's plan.
On August 15, 2007, Ricablanca and petitioner met at the latter's house. Petitioner, who was then
holding the bounced BDO check, handed over said check to Ricablanca. They originally intended
to proceed to Baby Aquino's place to have the check replaced with cash, but the plan did not push
through. However, they agreed to meet again on August 21, 2007.
On the agreed date, Ricablanca again went to petitioner’s house, where she met petitioner and
Jacqueline Capitle. Petitioner, her husband, and Ricablanca went to the house of Anita Valencia;
Jacqueline Capitle decided not to go with the group because she decided to go shopping. It was
only petitioner, her husband, Ricablanca and Valencia who then boarded petitioner's jeep and went
on to Baby Aquino's factory. Only Ricablanca alighted from the jeep and entered the premises of
Baby Aquino, pretending that she was getting cash from Baby Aquino. However, the cash she
actually brought out from the premises was the ₱10,000.00 marked money previously given to her
by Dyhengco. Ricablanca divided the money and upon returning to the jeep, gave ₱5,000.00 each
to Valencia and petitioner. Thereafter, petitioner and Valencia were arrested by NBI agents, who
had been watching the whole time.
Petitioner and Valencia were brought to the NBI office where the Forensic Chemist found
fluorescent powder on the palmar and dorsal aspects of both of their hands. This showed that
petitioner and Valencia handled the marked money. The NBI filed a criminal case for qualified
theft against the two and one Jane Doe who was later identified as Jacqueline Capitle, the wife of
Generoso Capitle.
The defense, on the other hand, denied having taken the subject check and presented the following
scenario.

70
CRIMINAL LAW 1 CASE DIGESTS
ATTY. LYAN DAVD JUANICO
BLOCK 1-M A.Y. 2020 - 2021

Petitioner admitted that she was a collector for Mega Foam until she resigned on June 30, 1997,
but claimed that she had stopped collecting payments from Baby Aquino for quite some time
before her resignation from the company. She further testified that, on the day of the arrest,
Ricablanca came to her mother’s house, where she was staying at that time, and asked that she
accompany her (Ricablanca) to Baby Aquino's house. Since petitioner was going for a pre-natal
check-up at the Chinese General Hospital, Ricablanca decided to hitch a ride with the former and
her husband in their jeep going to Baby Aquino's place in Caloocan City. She allegedly had no
idea why Ricablanca asked them to wait in their jeep, which they parked outside the house of Baby
Aquino, and was very surprised when Ricablanca placed the money on her lap and the NBI agents
arrested them.
Anita Valencia also admitted that she was the cashier of Mega Foam until she resigned on June
30, 1997. It was never part of her job to collect payments from customers. According to her, on
the morning of August 21, 1997, Ricablanca called her up on the phone, asking if she (Valencia)
could accompany her (Ricablanca) to the house of Baby Aquino. Valencia claims that she agreed
to do so, despite her admission during cross-examination that she did not know where Baby
Aquino resided, as she had never been to said house. They then met at the house of
petitioner's mother, rode the jeep of petitioner and her husband, and proceeded to Baby Aquino's
place. When they arrived at said place, Ricablanca alighted, but requested them to wait for her in
the jeep. After ten minutes, Ricablanca came out and, to her surprise, Ricablanca gave her money
and so she even asked, "What is this?" Then, the NBI agents arrested them.
ISSUE:
Whether the crime of qualified theft was actually produced

RULING:
No. The requisites of an impossible crime are: (1) that the act performed would be an offense
against persons or property; (2) that the act was done with evil intent; and (3) that its
accomplishment was inherently impossible, or the means employed was either inadequate or
ineffectual. The aspect of the inherent impossibility of accomplishing the intended crime under
Article 4(2) of the Revised Penal Code was further explained by the Court in Intod in this wise:
Under this article, the act performed by the offender cannot produce an offense against persons or
property because: (1) the commission of the offense is inherently impossible of accomplishment;
or (2) the means employed is either (a) inadequate or (b) ineffectual.

71
CRIMINAL LAW 1 CASE DIGESTS
ATTY. LYAN DAVD JUANICO
BLOCK 1-M A.Y. 2020 - 2021

That the offense cannot be produced because the commission of the offense is inherently
impossible of accomplishment is the focus of this petition. To be impossible under this clause, the
act intended by the offender must be by its nature one impossible of accomplishment. There must
be either (1) legal impossibility, or (2) physical impossibility of accomplishing the intended act in
order to qualify the act as an impossible crime.
Legal impossibility occurs where the intended acts, even if completed, would not amount to a
crime.
There can be no question that as of the time that petitioner took possession of the check meant for
Mega Foam, she had performed all the acts to consummate the crime of theft, had it not been
impossible of accomplishment in this case. The circumstance of petitioner receiving the ₱5,000.00
cash as supposed replacement for the dishonored check was no longer necessary for the
consummation of the crime of qualified theft. Obviously, the plan to convince Baby Aquino to
give cash as replacement for the check was hatched only after the check had been dishonored by
the drawee bank. Since the crime of theft is not a continuing offense, petitioner's act of receiving
the cash replacement should not be considered as a continuation of the theft. At most, the fact that
petitioner was caught receiving the marked money was merely corroborating evidence to
strengthen proof of her intent to gain.
Moreover, the fact that petitioner further planned to have the dishonored check replaced with cash
by its issuer is a different and separate fraudulent scheme. Unfortunately, since said scheme was
not included or covered by the allegations in the Information, the Court cannot pronounce
judgment on the accused; otherwise, it would violate the due process clause of the Constitution. If
at all, that fraudulent scheme could have been another possible source of criminal liability.

72
CRIMINAL LAW 1 CASE DIGESTS
ATTY. LYAN DAVD JUANICO
BLOCK 1-M A.Y. 2020 - 2021

MODULE 3: STAGES OF EXECUTION


1. US V. EDUAVE
G.R. No. 12155; 02 February 1917
J. Moreland

FACTS:
The accused rushed upon the girl suddenly and struck her from behind, in part at least, with a
sharp bolo, producing a frightful gash in the lumbar region and slightly to the side eight and one-
half inches long and two inches deep, severing all of the muscles and tissues of that part.
The motive of the crime was that the accused was incensed at the girl for the reason that she had
theretofore charged him criminally before the local officials with having raped her and with being
the cause of her pregnancy. He was her mother’s querido and was living with her as such at the
time the crime here charged was committed.

ISSUE:
Whether he is guilty of frustrated or attempted murder

RULING:
The Supreme Court ruled that the defendant-appellant is guilty of frustrated murder. Article 3 of
the Penal Code defines a frustrated felony as follows:
"A felony is frustrated when the offender performs all the acts of execution which should produce
the felony as a consequence, but which, nevertheless, do not produce it by reason of causes
independent of the will of the perpetrator."
An attempted felony is defined thus:
"There is an attempt when the offender commences the commission of the felony directly by overt
acts, and does not perform all the acts of execution which constitute the felony by reason of some
cause or accident other than his own voluntarily desistance."
The crime cannot be attempted murder. This is clear from the fact that the defendant performed all
of the acts which should have resulted in the consummated crime and voluntarily desisted from
further acts. A crime cannot be held to be attempted unless the offender, after beginning the
commission of the crime by overt acts, is prevented, against his will, by some outside cause from

73
CRIMINAL LAW 1 CASE DIGESTS
ATTY. LYAN DAVD JUANICO
BLOCK 1-M A.Y. 2020 - 2021

performing all of the acts which should produce the crime. In other words, to be an attempted
crime the purpose of the offender must be thwarted by a foreign force or agency which intervenes
and compels him to stop prior to the moment when he has performed all of the acts which should
produce the crime as a consequence, which acts it is his intention to perform. If he has performed
all of the acts which should result in the consummation of the crime and voluntarily desists from
proceeding further, it cannot be an attempt. The essential element which distinguishes attempted
from frustrated felony is that, in the latter, there is no intervention of a foreign or extraneous cause
or agency between the beginning of the commission of the crime and the moment when all of the
acts have been performed which should result in the consummated crime; while in the former there
is such intervention and the offender does not arrive at the point of performing all of the acts which
should produce the crime. He is stopped short of that point by same cause apart from his from his
voluntary desistance.
To put it in another way, in case of an attempt the offender never passes the subjective phase of
the offense. he is interrupted and compelled to desist by the intervention of outside causes before
the subjective phase is passed.
On the other hand, in case of frustrated crimes the subjective phase is completely passed.
Subjectively the crime is complete. Nothing interrupted the offender while he was passing through
the subjective phase. The crime, however, is not consummated by reason of the intervention of
causes independent of the will of the offender. he did all that was necessary to commit the crime.
If the crime did not result as a consequence it was due to something beyond his control.
The subjective phase is that portion of the acts constituting the crime included between the act
which begins the commission of the crime and the last act performed by the offender which, with
the prior acts, should result in the consummated crime. From that time forward the phase is
objective. It may also be said to be that period occupied by the acts of the offender over which he
has control — that period between the point where he begins and the point where he voluntarily
desists. If between these two points the offender is stopped by reason of any cause outside of his
own voluntary desistance, the subjective phase has not been passed and it is an attempt. If he is
not so stopped but continues until he performs the last act, it is frustrated.
Then the case before us is frustrated is clear.

74
CRIMINAL LAW 1 CASE DIGESTS
ATTY. LYAN DAVD JUANICO
BLOCK 1-M A.Y. 2020 - 2021

2. RIVERA V. PEOPLE
G.R. No. 166326; 25 January 2006
J. Callejo, Sr.

FACTS:
At noon of May 2, 1998, Ruben went to a nearby store to buy food. Edgardo mocked him for being
jobless and dependent on his wife for support. Ruben resented the rebuke and hurled invectives at
Edgardo. A heated exchange of words ensued.
At about 7:30 p.m. the next day, a Sunday, Ruben went to the store to buy food and to look for his
wife. His three-year-old daughter was with him. Momentarily, Esmeraldo and his two brothers,
Ismael and Edgardo, emerged from their house and ganged up on Ruben. Esmeraldo and Ismael
mauled Ruben with fist blows and he fell to the ground. In that helpless position, Edgardo hit
Ruben three times with a hollow block on the parietal area. Esmeraldo and Ismael continued
mauling Ruben. People who saw the incident shouted: "Awatin sila! Awatin sila!" Ruben felt dizzy
but managed to stand up. Ismael threw a stone at him, hitting him at the back. When policemen on
board a mobile car arrived, Esmeraldo, Ismael and Edgardo fled to their house.
ISSUE:
Whether or not they should be held criminally liable for physical injuries only
RULING:
No. Petitioners are guilty of attempted murder. An essential element of murder and homicide,
whether in their consummated, frustrated or attempted stage, is intent of the offenders to kill the
victim immediately before or simultaneously with the infliction of injuries. Intent to kill is a
specific intent which the prosecution must prove by direct or circumstantial evidence, while
general criminal intent is presumed from the commission of a felony by dolo.
In People v. Delim, the Court declared that evidence to prove intent to kill in crimes against persons
may consist, inter alia, in the means used by the malefactors, the nature, location and number of
wounds sustained by the victim, the conduct of the malefactors before, at the time, or immediately
after the killing of the victim, the circumstances under which the crime was committed and the
motives of the accused. If the victim dies as a result of a deliberate act of the malefactors, intent to
kill is presumed.
The last paragraph of Article 6 of the Revised Penal Code defines an attempt to commit a felony,
thus:

75
CRIMINAL LAW 1 CASE DIGESTS
ATTY. LYAN DAVD JUANICO
BLOCK 1-M A.Y. 2020 - 2021

There is an attempt when the offender commences the commission of a felony directly by overt
acts, and does not perform all the acts of execution which should produce the felony by reason of
some cause or accident other than his own spontaneous desistance.
The essential elements of an attempted felony are as follows:
1. The offender commences the commission of the felony directly by overt acts;
2. He does not perform all the acts of execution which should produce the felony;
3. The offender’s act be not stopped by his own spontaneous desistance;
4. The non-performance of all acts of execution was due to cause or accident other than his
spontaneous desistance.
The first requisite of an attempted felony consists of two elements, namely:
(1) That there be external acts;
(2) Such external acts have direct connection with the crime intended to be committed.

76
CRIMINAL LAW 1 CASE DIGESTS
ATTY. LYAN DAVD JUANICO
BLOCK 1-M A.Y. 2020 - 2021

3. BALEROS V. PEOPLE
G.R. no. 138033; 22 February 2006
J. Garcia

FACTS:
Like most of the tenants of the Celestial Marie Building (hereafter "Building") along A.H. Lacson
Street, Sampaloc, Manila, Malou, occupying Room 307 with her maid, Marvilou Bebania
(Marvilou), was a medical student of the University of Sto. Tomas [UST] in 1991.
In the evening of December 12, inside Unit 307, Malou retired at around 10:30. Outside, right in
front of her bedroom door, her maid, Marvilou, slept on a folding bed.
Early morning of the following day, Malou was awakened by the smell of chemical on a piece of
cloth pressed on her face. She struggled but could not move. Somebody was pinning her down on
the bed, holding her tightly. She wanted to scream for help but the hands covering her mouth with
cloth wet with chemicals were very tight. Still, Malou continued fighting off her attacker by
kicking him until at last her right hand got free. With this, the opportunity presented itself when
she was able to grab hold of his sex organ which she then squeezed.
The man let her go and Malou went straight to the bedroom door and roused Marvilou. Over the
intercom, Malou told S/G Ferolin that: "may pumasok sa kuarto ko pinagtangkaan ako". Who it
was she did not, however, know. The only thing she had made out during their struggle was the
feel of her attacker’s clothes and weight. His upper garment was of cotton material while that at
the lower portion felt smooth and satin-like. He was wearing a t-shirt and shorts.
To Room 310 of the Building where her classmates Christian Alcala, Bernard Baptista, Lutgardo
Acosta and Rommel Montes were staying, MALOU then proceeded to seek help.
It was then when MALOU saw her bed topsy-turvy. Her nightdress was stained with blue. Aside
from the window with grills which she had originally left opened, another window inside her
bedroom was now open. Her attacker had fled from her room going through the left bedroom
window, the one without iron grills which leads to Room 306 of the Building.
ISSUE:
Whether or not the act of the petitioner, i.e., the pressing of a chemical-soaked cloth while on top
of Malou, constitutes an overt act of rape

77
CRIMINAL LAW 1 CASE DIGESTS
ATTY. LYAN DAVD JUANICO
BLOCK 1-M A.Y. 2020 - 2021

RULING:
No. Under Article 335 of the Revised Penal Code, rape is committed by a man who has carnal
knowledge or intercourse with a woman under any of the following circumstances: (1) By using
force or intimidation; (2) When the woman is deprived of reason or otherwise unconscious; and
(3) When the woman is under twelve years of age or is demented. Under Article 6, in relation to
the aforementioned article of the same code, rape is attempted when the offender commences the
commission of rape directly by overt acts and does not perform all the acts of execution which
should produce the crime of rape by reason of some cause or accident other than his own
spontaneous desistance.
It would be too strained to construe petitioner's act of pressing a chemical-soaked cloth in the
mouth of Malou which would induce her to sleep as an overt act that will logically and necessarily
ripen into rape. As it were, petitioner did not commence at all the performance of any act indicative
of an intent or attempt to rape Malou. It cannot be overemphasized that petitioner was fully clothed
and that there was no attempt on his part to undress Malou, let alone touch her private part. For
what reason petitioner wanted the complainant unconscious, if that was really his immediate
intention, is anybody’s guess. The CA maintained that if the petitioner had no intention to rape, he
would not have lain on top of the complainant. Plodding on, the appellate court even anticipated
the next step that the petitioner would have taken if the victim had been rendered unconscious.
Lest it be misunderstood, the Court is not saying that petitioner is innocent, under the premises, of
any wrongdoing whatsoever. The information filed against petitioner contained an allegation that
he forcefully covered the face of Malou with a piece of cloth soaked in chemical. And during the
trial, Malou testified about the pressing against her face of the chemical-soaked cloth and having
struggled after petitioner held her tightly and pinned her down. Verily, while the series of acts
committed by the petitioner do not determine attempted rape, as earlier discussed, they constitute
unjust vexation punishable as light coercion under the second paragraph of Article 287 of the
Revised Penal Code.

78
CRIMINAL LAW 1 CASE DIGESTS
ATTY. LYAN DAVD JUANICO
BLOCK 1-M A.Y. 2020 - 2021

4. VALENZUELA V. PEOPLE
G.R. No. 160188; 21 June 2007
J. Tinga

FACTS:
On 19 May 1994, at around 4:30 p.m., petitioner and Calderon were sighted outside the Super Sale
Club, a supermarket within the ShoeMart (SM) complex along North EDSA, by Lorenzo Lago
(Lago), a security guard who was then manning his post at the open parking area of the
supermarket. Lago saw petitioner, who was wearing an identification card with the mark
"Receiving Dispatching Unit (RDU)," hauling a push cart with cases of detergent of the well-
known "Tide" brand. Petitioner unloaded these cases in an open parking space, where Calderon
was waiting. Petitioner then returned inside the supermarket, and after five (5) minutes, emerged
with more cartons of Tide Ultramatic and again unloaded these boxes to the same area in the open
parking space.
Thereafter, petitioner left the parking area and haled a taxi. He boarded the cab and directed it
towards the parking space where Calderon was waiting. Calderon loaded the cartons of Tide
Ultramatic inside the taxi, then boarded the vehicle. All these acts were eyed by Lago, who
proceeded to stop the taxi as it was leaving the open parking area. When Lago asked petitioner for
a receipt of the merchandise, petitioner and Calderon reacted by fleeing on foot, but Lago fired a
warning shot to alert his fellow security guards of the incident. Petitioner and Calderon were
apprehended at the scene, and the stolen merchandise recovered. The filched items seized from the
duo were four (4) cases of Tide Ultramatic, one (1) case of Ultra 25 grams, and three (3) additional
cases of detergent, the goods with an aggregate value of ₱12,090.00.
ISSUE:
Whether or not he should only be convicted of frustrated theft since at the time he was
apprehended, he was never placed in a position to freely dispose of the articles stolen
RULING:
No. Article 6 defines those three stages, namely the consummated, frustrated and attempted
felonies. A felony is consummated "when all the elements necessary for its execution and
accomplishment are present." It is frustrated "when the offender performs all the acts of execution
which would produce the felony as a consequence but which, nevertheless, do not produce it by
reason of causes independent of the will of the perpetrator." Finally, it is attempted "when the
offender commences the commission of a felony directly by overt acts, and does not perform all

79
CRIMINAL LAW 1 CASE DIGESTS
ATTY. LYAN DAVD JUANICO
BLOCK 1-M A.Y. 2020 - 2021

the acts of execution which should produce the felony by reason of some cause or accident other
than his own spontaneous desistance."
Each felony under the Revised Penal Code has a "subjective phase," or that portion of the acts
constituting the crime included between the act which begins the commission of the crime and the
last act performed by the offender which, with prior acts, should result in the consummated crime.
After that point has been breached, the subjective phase ends and the objective phase begins. It has
been held that if the offender never passes the subjective phase of the offense, the crime is merely
attempted. On the other hand, the subjective phase is completely passed in case of frustrated
crimes, for in such instances, "subjectively the crime is complete."
Truly, an easy distinction lies between consummated and frustrated felonies on one hand, and
attempted felonies on the other. So long as the offender fails to complete all the acts of execution
despite commencing the commission of a felony, the crime is undoubtedly in the attempted stage.
Since the specific acts of execution that define each crime under the Revised Penal Code are
generally enumerated in the code itself, the task of ascertaining whether a crime is attempted only
would need to compare the acts actually performed by the accused as against the acts that constitute
the felony under the Revised Penal Code.
In contrast, the determination of whether a crime is frustrated or consummated necessitates an
initial concession that all of the acts of execution have been performed by the offender. The critical
distinction instead is whether the felony itself was actually produced by the acts of execution. The
determination of whether the felony was "produced" after all the acts of execution had been
performed hinges on the particular statutory definition of the felony. It is the statutory definition
that generally furnishes the elements of each crime under the Revised Penal Code, while the
elements in turn unravel the particular requisite acts of execution and accompanying criminal
intent.
We next turn to the statutory definition of theft. Under Article 308 of the Revised Penal Code, its
elements are spelled out as follows:
Art. 308. Who are liable for theft.— Theft is committed by any person who, with intent to gain but
without violence against or intimidation of persons nor force upon things, shall take personal
property of another without the latter’s consent.
Theft is likewise committed by:
1. Any person who, having found lost property, shall fail to deliver the same to the local authorities
or to its owner;

80
CRIMINAL LAW 1 CASE DIGESTS
ATTY. LYAN DAVD JUANICO
BLOCK 1-M A.Y. 2020 - 2021

2. Any person who, after having maliciously damaged the property of another, shall remove or
make use of the fruits or object of the damage caused by him; and
3. Any person who shall enter an inclosed estate or a field where trespass is forbidden or which
belongs to another and without the consent of its owner, shall hunt or fish upon the same or shall
gather cereals, or other forest or farm products.
Article 308 provides for a general definition of theft, and three alternative and highly idiosyncratic
means by which theft may be committed. In the present discussion, we need to concern ourselves
only with the general definition since it was under it that the prosecution of the accused was
undertaken and sustained. On the face of the definition, there is only one operative act of execution
by the actor involved in theft ─ the taking of personal property of another. It is also clear from the
provision that in order that such taking may be qualified as theft, there must further be present the
descriptive circumstances that the taking was with intent to gain; without force upon things or
violence against or intimidation of persons; and it was without the consent of the owner of the
property.
Indeed, we have long recognized the following elements of theft as provided for in Article 308 of
the Revised Penal Code, namely: (1) that there be taking of personal property; (2) that said property
belongs to another; (3) that the taking be done with intent to gain; (4) that the taking be done
without the consent of the owner; and (5) that the taking be accomplished without the use of
violence against or intimidation of persons or force upon things.
With that in mind, a problem clearly emerges with the Diño/Flores dictum. The ability of the
offender to freely dispose of the property stolen is not a constitutive element of the crime of theft.
It finds no support or extension in Article 308, whether as a descriptive or operative element of
theft or as the mens rea or actus reus of the felony. To restate what this Court has repeatedly held:
the elements of the crime of theft as provided for in Article 308 of the Revised Penal Code are: (1)
that there be taking of personal property; (2) that said property belongs to another; (3) that the
taking be done with intent to gain; (4) that the taking be done without the consent of the owner;
and (5) that the taking be accomplished without the use of violence against or intimidation of
persons or force upon things.
There would be all but certain unanimity in the position that theft is produced when there is
deprivation of personal property due to its taking by one with intent to gain. Viewed from that
perspective, it is immaterial to the product of the felony that the offender, once having committed
all the acts of execution for theft, is able or unable to freely dispose of the property stolen since
the deprivation from the owner alone has already ensued from such acts of execution.

81
CRIMINAL LAW 1 CASE DIGESTS
ATTY. LYAN DAVD JUANICO
BLOCK 1-M A.Y. 2020 - 2021

5. PEOPLE V. DIO
G.R. No. L-36461; 29 June 1984
J. Abad Santos

FACTS:
At about noontime on July 24, 1971, Crispulo Alega, a civil engineer by profession working at the
Sugar Construction Company, with a salary of more than P500.00 a month went to the
Southeastern College, Pasay City to fetch his girlfriend, Remedios Maniti, a third year high school
student thereat. They proceeded to the Pasay City Public Market. As they were going up the stairs
leading to the Teresa and Sons Restaurant, Remedios, who was was about an arms-length ahead
of Crispulo suddenly heard the dropping of her folders and other things, being carried by Crispulo.
When she looked back, she saw a man — later Identified as Danilo Tobias but still at large —
twisting the neck of Crispulo, while the appellant was holding his (Crispulo's) two hands. The
appellant and his companion tried to divest Crispulo of his "Seiko" wrist watch, but Crispulo
resisted their attempt and fought the robbers. At this juncture, the man who was twisting the neck
of Crispulo stabbed the latter on the left side of his chest. Crispulo ran down the stairs followed by
Remedies who shouted for help. When he reached the front of the Pasay Commercial Bank he fell
down and expired. At the time of his death, the "Seiko" watch was strapped to his wrist.
ISSUE:
Whether or not the appellant should not have been convicted of the special complex crime of
robbery with homicide because the robbery was not consummated
RULING:
Yes. The evidence adduced show that the appellant and his companion were unsuccessful in their
criminal venture of divesting the victim of his wrist watch so as to constitute the consummated
crime of robbery. Indeed, as adverted to earlier, when the victim expired, the 'Seiko' watch was
still securely strapped to his wrist. The killing of Crispulo Alega may be considered as merely
incidental to and an offshoot of the plan to carry out the robbery, which however was not
consummated because of the resistance offered by the deceased. Consequently, this case would
properly come under the provision of Art. 297 of the Revised Penal Code which states that —
“When by reason or on occasion of an attempted or frustrated robbery a homicide is committed,
the person guilty of such offenses shall be punished by reclusion temporal in its maximum period
to reclusion perpetua, unless the homicide committed shall deserve a higher penalty under the
provisions of this Code.”

82
CRIMINAL LAW 1 CASE DIGESTS
ATTY. LYAN DAVD JUANICO
BLOCK 1-M A.Y. 2020 - 2021

The crime committed by the appellant is attempted robbery with homicide and the penalty
prescribed by law is reclusion temporal in its maximum period to reclusion perpetua.

83
CRIMINAL LAW 1 CASE DIGESTS
ATTY. LYAN DAVD JUANICO
BLOCK 1-M A.Y. 2020 - 2021

6. People v. Salvilla
G.R. No. 86163, 26 April 1990
MELENCIO-HERRERA, J.:

FACTS:

On April 12 1986, around noon, 4 accused staged a robbery at the New Iloilo lumber yard. They
used homemade weapons and a live grenade. The accused Salvilla entered the facility and pointed
a gun at the owner and asked for money. The owner gave 20,000 pesos placed in a paper bag and
gave to the accused. A little bit later accused Canasares took the watch and the wallet of the owner.
In the afternoon, the accused asked for more money but since it was a Saturday they could not do
so. The police, mayor later negotiated with the accused to release the hostages. The mayor gave
50,000 in order to appease the hostage takers. Later the police launched the assault and captured
the accused.

ISSUE:

Whether or not that the crime charged was consummated robbery?

RULING:

Yes, The court held that robbery was consummated. The factual circumstances are: first on the
instance when the employee gave the paperbag with money to the appellant Salvilla, second on
the instance of the accused taking the wallet and wristwatch of the owner.

Severance of the goods from the possession of the owner and absolute control of the property by
the taker, even for an instant, constitutes asportation.

It is no defense either that appellant and his co-accused had no opportunity to dispose of the
personalities taken. That fact does not affect the nature of the crime, From the moment the offender
gained possession of the thing, even if the culprit had no opportunity to dispose of the same, the
unlawful taking is complete.

84
CRIMINAL LAW 1 CASE DIGESTS
ATTY. LYAN DAVD JUANICO
BLOCK 1-M A.Y. 2020 - 2021

7. People v. Lamahang
G.R. No. L-43530; 3 August 1935
RECTO, J.:

FACTS:

At early dawn on March 2, 1935, policeman Jose Tomambing, who was patrolling on Delgado and
C.R. Fuentes streets of the City of Iloilo, caught the accused in the act of making an opening with
an iron bar on the wall of a store. At that time the owner of the store, Tan Yu, was sleeping inside
with another Chinaman. The accused had only succeeded in breaking one board and in unfastening
another from the wall, when the policeman showed up, who instantly arrested him and placed him
under custody.

ISSUE:

Whether or not the accused was guilty of attempted robbery?

RULING:

No, The court held that the overt acts show that the crime was attempted trespass to dwelling. In
this instance the accused was caught in the act with an iron bar forced open the wall of the store
by breaking a board before he was arrested. There were no overt acts showing that the accused
attempted to rob or take possession of some property belonging to another.

85
CRIMINAL LAW 1 CASE DIGESTS
ATTY. LYAN DAVD JUANICO
BLOCK 1-M A.Y. 2020 - 2021

8. People v. Borinaga
G.R. No. 33463; 18 December 1930
MALCOM, J.:

FACTS:

Prior to March 4, 1929, Harry Mooney a resident of Calubian Leyte contracted with Juan Lawaan
for the construction of a fish corral. On that morning Lawaan tried to collect money from Mooney
not withstanding that only 2/3 of the fish corral has been completed. On the evening, Mooney was
in the store of the neighbor by the name of Perpetua Najarro. He had taken a seat on a chair in
front of the Perpetua and his back being toward the window. Then Perpetua saw Boringa strike
Mooney from the window but fortunately the knife was lodged at the back of the chair and Mooney
was not injured. Boringa tried again but failed since Mooney was now on guard.

ISSUE:

Whether or not the crime is frustrated murder?

RULING:

Yes, the court held that the essential condition of a frustrated murder, that the author perform all
the acts of execution, attended the attack. Nothing remained to be done to accomplish the work of
the assailant completely. The cause resulting in the failure of the attack arose by reason of forces
independent of the will of the perpetrator.

86
CRIMINAL LAW 1 CASE DIGESTS
ATTY. LYAN DAVD JUANICO
BLOCK 1-M A.Y. 2020 - 2021

9. People v. Kalalo
G.R. No. 39303-05; 17 March 1934
DIAZ, J.:

FACTS:

On October 1, 1932, Isabela Holgado and her brother Arcadio Holgado decided to order the land
plowed and employed several laborers. After lunch Marcelo Kalalo approached Arcadio, while the
appellants Felipe Kalalo, Juan Kalalo and Gregorio Ramos, approached Marcelino Panaligan. At
a remark from Fausta Abrenica they all simultaneously struck with their bolos, the appellant
Marcelo Kalalo slashing Arcadio Holgado, while the appellants Felipe Kalalo, Juan Kalalo and
Gregorio Ramos slashed Marcelino Panaligan, inflicting upon them the wounds. Arcadio Holgado
and Marcelino Panaligan died instantly from the wounds received by them in the presence of
Isabela Holgado and Maria Gutierrez. After Arcadio Holgado and Marcelino Panaligan had died,
the appellant Marcelo Kalalo took from its holster on the belt of Panaligan body, the revolver
which the deceased carried, and fired four shots at Hilarion Holgado who was then fleeing from
the scene in order to save his own life.

ISSUE:

Whether or not the crime charged was frustrated murder?

RULING:

No, as to case No. 6860 (G.R. No. 39305), the evidence shows that Marcelo Kalalo fired four
successive shots at Hilarion Holgado while the latter was fleeing from the scene of the crime in
order to be out of reach of the appellants and their companions and save his own life. The fact that
the said appellant, fired successive shots at Hilarion Holgado, added to the circumstance that
immediately before doing so he and his co-appellants had already killed Arcadio Holgado and
Marcelino Panaligan, shows that he was then bent on killing Hilarion Holgado. He performed
everything necessary to commit the crime that he determined to commit but he failed by reason of
causes independent of his will, either because of his poor aim or because his intended victim
succeeded in dodging the shots. The acts thus committed by the said appellant Marcelo Kalalo
constitute attempted homicide with no modifying circumstance to be taken into consideration,
because none has been established.

87
CRIMINAL LAW 1 CASE DIGESTS
ATTY. LYAN DAVD JUANICO
BLOCK 1-M A.Y. 2020 - 2021

10. People v. Trinidad


G.R. No. 79123-25; 9 January 1989
MELENCIO-HERRERA, J.:

FACTS:

On Jan 19, 1983 Lolito Soriano was a fish dealer in Davao. His helpers were a driver, Tan and
Laroa. The rode on a ford fiera and arrived at Butuan to sell fish. Tan was left behind to sell fish
and they continued to Buenavista and meet there. There accused Trinidad a member of integrated
policed, armed with carbine and a 38. revolver hitched a ride with Soriano to Bayugan which was
on the way to Davao. On the way, Tan heard 2 gunshots wherein Soriano and Laroa died. Tan
quickly ran towards the direction of Butuan city and hid himself in the bushes. After waiting for a
jeep pass by, hailed and got on and noticed Trinidad at the back. Tan ran around while Trinidad
chased after him and shot him at the leg. Eventually, Tan got help from another PC on the way to
Butuan.

ISSUE:

Whether the crime charged was frustrated murder?

RULING:

No, The court held that the accused can be only convicted of attempted murder. Trinidad had
commenced the commission of the felony directly by overt acts but was unable to perform all the
acts of execution which would have produced it by reason of causes other than his spontaneous
desistance, such as, that the jeep to which Tan was clinging was in motion, and there was a spare
tire which shielded the other parts of his body. The wound on his thigh was not fatal and the
doctrinal rule is that where the wound inflicted on the victim is not sufficient to cause his death,
the crime is only attempted murder, the accused not having performed all the acts of execution that
would have brought about death.

88
CRIMINAL LAW 1 CASE DIGESTS
ATTY. LYAN DAVD JUANICO
BLOCK 1-M A.Y. 2020 - 2021

11. Martinez v. CA
G.R. No. 168827; 13 April 2007
CALLEJO, SR., J.:

FACTS:

On Oct 28 Dean and wife filed a complaint for damages against spouses Martinez for peddling
false reports of the Dean and Elvisa having illicit relationship. Dean requested Lilibeth to stop her
husband Martinez from spreading lies, and found out that Elvisa was Martinez mistress. Elvisa
also filed for damages against Martinez for accusing her for having illicit affair with Dean. On Feb
3 1999, while Dean went to the Tubao Credit Cooperative to pick up the dividend certificate he
was suddenly attacked by Martinez when he was going back to the car. Dean was struck at the left
breast and tried to run away. Dean entered the bank while Martinez chased after him. Dean parried
the bolo attack with his right hand. When Dean managed to run, Martinez was caught by the police.

ISSUE:

Whether or not the crime charged was frustrated murder?

RULING:

Yes, When Dean was attacked he was unarmed. He had just exited the cooperative building and
had no inkling that he would be waylaid as he made his way towards his car. Upon the other hand,
petitioner was armed with a deadly 14½-inch bolo.

A crime is frustrated when the offender has performed all the acts of execution which should result
in the consummation of the crime. The offender has passed the subjective phase in the commission
of the crime. Nothing interrupted the offender while passing through the subjective phase. He did
all that is necessary to consummate the crime. However, the crime was not consummated by reason
of the intervention of causes independent of the will of the offender. In homicide cases, the
offender is said to have performed all the acts of execution if the wound inflicted on the victim is
mortal and could cause the death of the victim barring medical intervention or attendance.

89
CRIMINAL LAW 1 CASE DIGESTS
ATTY. LYAN DAVD JUANICO
BLOCK 1-M A.Y. 2020 - 2021

12. Mondragon v. People


G.R. No. L-17666; 30 June 1966
ZALDIVAR, J.:

FACTS:

On July 11, 1954 5pm in the afternoon, Nacionales was opening the dike of his rice field to drain
the water. He heard Mondragon shouting not to open the dike. Nacionales informed Mondragon
that he was preparing his rice field. Suddenly Mondragon tried to hit Nacionales and dodged it.
Both of them fought with bolos, then after both sustaining injuries they parted ways.

ISSUE:

Whether or not the crime charged was attempted homicide?

RULING:

No, The court ruled that there was no intent to kill. the petitioner started the assault on the offended
party by just giving him fist blows; the wounds inflicted on the offended party were of slight
nature, indicating no homicidal urge on the part of the petitioner; the petitioner retreated and went
away when the offended party started hitting him with a bolo, thereby indicating that if the
petitioner had intended to kill the offended party he would have held his ground and kept on hitting
the offended party with his bolo to kill him. The injuries suffered by the offended party were not
necessarily fatal and could be healed in less than 30 days.

90
CRIMINAL LAW 1 CASE DIGESTS
ATTY. LYAN DAVD JUANICO
BLOCK 1-M A.Y. 2020 - 2021

13. People v. Sy Pio


G.R. No. L-5848; 30 April 1954
LABRADOR, J.:

FACTS:
Early in the morning of September 3, 1949, the defendant-appellant entered the store at 511
Misericordia, Sta Cruz, Manila. Once inside he started firing a .45 caliber pistol that he had in his
hand. The first one shot was Jose Sy. Tan Siong Kiap, who was in the store and saw the accused
enter and afterwards fire a shot at Jose Sy, asked the defendant-appellant, "What is the idea?"
Thereupon defendant-appellant turned around and fired at him also. The bullet fired from
defendant-appellant's pistol entered the right shoulder of Tan Siong Kiap immediately ran to a
room behind the store to hide. From there he still heard gunshot fired from defendant-appellant's
pistol, but afterwards defendant-appellant ran away.

Tan Siong Kiap was brought to the Chinese General Hospital, where his wound was treated. He
stayed there from September 3 to September 12, 1949, when he was released upon his request and
against the physician's advice. He was asked to return to the hospital for further treatment, and he
did so five times for a period of more than ten days. Thereafter his wound was completely healed.
He spent the sum of P300 for hospital and doctor's fees.

The defendant-appellant shot two other persons in the morning of September 3, 1949, before
shooting and wounding Tan Siong Kiap; one was Ong Pian and the other Jose Sy. On September
5 information was received by the Manila Police Department that defendant-appellant was in
custody of the Constabulary in Tarlac, so a captain of the Manila police by the name of Daniel V.
Lomotan proceeded to Tarlac. There he saw the defendant-appellant and had a conversation with
him. On this occasion defendant-appellant and had a conversation with him. On this occasion
defendant-appellant admitted to Lomotan that his victims were Tan Siong Kiap, Ong Pian, and
Jose Sy. The Constabulary in Tarlac also delivered to Lomotan the pistol used by the defendant-
appellant, marked Exhibit C, and its magazine, Exhibit C-1, both of which the Constabulary had
confiscated from the defendant-appellant. The defendant-appellant was thereupon delivered to the
custody of Lomotan, and the latter brought him to Manila, where his statement was taken down in
writing. This declaration was submitted at the time of the trial as Exhibit D, and it contains all the
details of the assaults that defendant-appellant 3 against the persons of Tan Siong Kiap, Ong Pian,
and Jose Sy.

ISSUE:
Whether or not the defendant-appellant should be found guilty only of crime of frustrated murder.

91
CRIMINAL LAW 1 CASE DIGESTS
ATTY. LYAN DAVD JUANICO
BLOCK 1-M A.Y. 2020 - 2021

RULING:
No, the defendant-appellant fired at his victim, and the latter was hit, but he was able to escape
and hide in another room. The fact that he was able to escape, which appellant must have seen,
must have produced in the mind of the defendant-appellant that he was not able to his his victim
at a vital part of the body. In other words, the defendant-appellant knew that he had not actually
all the acts of execution necessary to kill his victim. Under these circumstances, it can not be said
that the subjective phase of the acts of execution had been completed. And as it does not appear
that the defendant-appellant continued in the pursuit, and as a matter of fact, he ran away
afterwards a reasonable doubt exist in our mind that the defendant-appellant had actually believed
that he has committed all the acts of execution or passed the subjective phase of the said acts. This
doubt must be resolved in favor of the defendant-appellant.
Thus, the defendant-appelant is guilty of attempted murder, because he did not perform all the acts
of execution, actual and subjective, in order that the purpose and intention that he had to kill his
victim might be carried out.

92
CRIMINAL LAW 1 CASE DIGESTS
ATTY. LYAN DAVD JUANICO
BLOCK 1-M A.Y. 2020 - 2021

14. People v. Orita


G.R. No. 88724; 3 April 1990
MEDIALDEA, J.:

FACTS:
In the early morning of March 20, 1983, complainant arrived at her boarding house. Her classmates
had just brought her home from a party. Shortly after her classmates had left, she knocked at the
door of her boarding house. All of a sudden, somebody held her and poked a knife to her neck.
She then recognized appellant who was a frequent visitor of another boarder.

She pleaded with him to release her, but he ordered her to go upstairs with him. Since the door
which led to the first floor was locked from the inside, appellant forced complainant to use the
back door leading to the second floor. With his left arm wrapped around her neck and his right
hand poking a "balisong" to her neck, appellant dragged complainant up the stairs. When they
reached the second floor, he commanded her to look for a room. With the Batangas knife still
poked to her neck, they entered complainant's room.

Upon entering the room, appellant pushed complainant who hit her head on the wall. With one
hand holding the knife, appellant undressed himself. He then ordered complainant to take off her
clothes. Scared, she took off her T-shirt. Then he pulled off her bra, pants and panty.
He ordered her to lie down on the floor and then mounted her. He made her hold his penis and
insert it in her vagina. She followed his order as he continued to poke the knife to her. At said
position, however, appellant could not fully penetrate her. Only a portion of his penis entered her
as she kept on moving.

Appellant then lay down on his back and commanded her to mount him. In this position, only a
small part again of his penis was inserted into her vagina. At this stage, appellant had both his
hands flat on the floor. Complainant thought of escaping.
She dashed out to the next room and locked herself in. Appellant pursued her and climbed the
partition. When she saw him inside the room, she ran to another room. Appellant again chased her.
She fled to another room and jumped out through a window.

Still naked, she darted to the municipal building, which was about eighteen meters in front of the
boarding house, and knocked on the door. When there was no answer, she ran around the building
and knocked on the back door. When the policemen who were inside the building opened the door,
they found complainant naked sitting on the stairs crying. Pat. Donceras, the first policeman to see
her, took off his jacket and wrapped it around her. When they discovered what happened, Pat.

93
CRIMINAL LAW 1 CASE DIGESTS
ATTY. LYAN DAVD JUANICO
BLOCK 1-M A.Y. 2020 - 2021

Donceras and two other policemen rushed to the boarding house. They heard a sound at the second
floor and saw somebody running away. Due to darkness, they failed to apprehend appellant.

ISSUE:
Whether or not the crime committed by the accused is frustrated rape.

RULING:
No, the requisites of a frustrated felony are: (1) that the offender has performed all the acts of
execution which would produce the felony and (2) that the felony is not produced due to causes
independent of the perpetrator's will.

Clearly, in the crime of rape, from the moment the offender has carnal knowledge of his victim he
actually attains his purpose and, from that moment also all the essential elements of the offense
have been accomplished. Nothing more is left to be done by the offender, because he has
performed the last act necessary to produce the crime. Thus, the felony is consummated. In a long
line of cases.

Any penetration of the female organ by the male organ is sufficient. Entry of the labia or lips of
the female organ, without rupture of the hymen or laceration of the vagina is sufficient to warrant
conviction. Necessarily, rape is attempted if there is no penetration of the female organ because
not all acts of execution was performed. The offender merely commenced the commission of a
felony directly by overt acts. Taking into account the nature, elements and manner of execution of
the crime of rape and jurisprudence on the matter, it is hardly conceivable how the frustrated stage
in rape can ever be committed.

Thus, the accused the crime of consummated rape.

94
CRIMINAL LAW 1 CASE DIGESTS
ATTY. LYAN DAVD JUANICO
BLOCK 1-M A.Y. 2020 - 2021

15. People v. Campuhan


G.R. No. 129433; 30 March 2000
BELLOSILLO, J.:

FACTS:
On 25 April 1996, at around 4 o'clock in the afternoon, Ma. Corazon P. Pamintuan, mother of four
(4)-year old Crysthel Pamintuan, went down from the second floor of their house to prepare Milo
chocolate drinks for her two (2) children. At the ground floor she met Primo Campuhan who was
then busy filling small plastic bags with water to be frozen into ice in the freezer located at the
second floor. Primo was a helper of Conrado Plata Jr., brother of Corazon. As Corazon was busy
preparing the drinks, she heard one of her daughters cry, "Ayo'ko, ayo'ko!" prompting Corazon to
rush upstairs. Thereupon, she saw Primo Campuhan inside her children's room kneeling before
Crysthel whose pajamas or "jogging pants" and panty were already removed, while his short pants
were down to his knees.

According to Corazon, Primo was forcing his penis into Crysthel's vagina. Horrified, she cursed
the accused, "P - t - ng ina mo, anak ko iyan!" and boxed him several times. He evaded her blows
and pulled up his pants. He pushed Corazon aside when she tried to block his path. Corazon then
ran out and shouted for help thus prompting her brother, a cousin and an uncle who were living
within their compound, to chase the accused. Seconds later, Primo was apprehended by those who
answered Corazon's call for help. They held the accused at the back of their compound until they
were advised by their neighbors to call the barangay officials instead of detaining him for his
misdeed. Physical examination of the victim yielded negative results. No evident sign of extra-
genital physical injury was noted by the medico-legal officer on Crysthel's body as her hymen was
intact and its orifice was only 0.5 cm. in diameter.

Primo Campuhan had only himself for a witness in his defense. He maintained his innocence and
assailed the charge as a mere scheme of Crysthel's mother who allegedly harbored ill will against
him for his refusal to run an errand for her. He asserted that in truth Crysthel was in a playing
mood and wanted to ride on his back when she suddenly pulled him down causing both of them to
fall down on the floor. It was in this fallen position that Corazon chanced upon them and became
hysterical. Corazon slapped him and accused him of raping her child. He got mad but restrained
himself from hitting back when he realized she was a woman. Corazon called for help from her
brothers to stop him as he ran down from the second floor.

Vicente, Corazon's brother, timely responded to her call for help and accosted Primo. Vicente
punched him and threatened to kill him. Upon hearing the threat, Primo immediately ran towards
the house of Conrado Plata but Vicente followed him there. Primo pleaded for a chance to explain

95
CRIMINAL LAW 1 CASE DIGESTS
ATTY. LYAN DAVD JUANICO
BLOCK 1-M A.Y. 2020 - 2021

as he reasoned out that the accusation was not true. But Vicente kicked him instead. When Primo
saw Vicente holding a piece of lead pipe, Primo raised his hands and turned his back to avoid the
blow. At this moment, the relatives and neighbors of Vicente prevailed upon him to take Primo to
the barangay hall instead, and not to maul or possibly kill him.

ISSUE:
Whether or not the accused is guilty of Attempted Rape.

RULING:
Yes, the possibility of Primo's penis having breached Crysthel's vagina is belied by the child's own
assertion that she resisted Primo's advances by putting her legs close together; consequently, she
did not feel any intense pain but just felt "not happy" about what Primo did to her. Thus, she only
shouted "Ayo'ko, ayo'ko!" not "Aray ko, aray ko!" In cases where penetration was not fully
established, the Court had anchored its conclusion that rape nevertheless was consummated on the
victim's testimony that she felt pain, or the medico-legal finding of discoloration in the inner lips
of the vagina, or the labia minora was already gaping with redness, or the hymenal tags were no
longer visible. None was shown in this case.

Under Art. 6, in relation to Art. 335, of the Revised Penal Code, rape is attempted when the
offender commences the commission of rape directly by overt acts, and does not perform all the
acts of execution which should produce the crime of rape by reason of some cause or accident
other than his own spontaneous desistance.

Thus, all the elements of attempted rape and only of attempted rape are present in the instant case,
hence, the accused should be punished only for it.

96
CRIMINAL LAW 1 CASE DIGESTS
ATTY. LYAN DAVD JUANICO
BLOCK 1-M A.Y. 2020 - 2021

16. US v. Valdes
G.R. No. L-14128; 10 December 1918
TORRES, J.:

FACTS:
Between 8 and 9 o'clock in the morning of April 28th of this year, when M. D. Lewin was absent
from the house in which he was living his family, at No. 328, San Rafael Street, San Miguel, Mrs.
Auckback, who appears to have been a resident of the neighborhood, called Mrs. Lewin and told
her that much smoke was issuing from the lower floor of the latter's house, for until then Mrs.
Lewin had not noticed it, and as soon as her attention was brought to the fact she ordered the
servant Paulino Banal to look for the fire, as he did and he found, so asked with kerosene oil and
placed between a post of the house and a partition of the entresol, a piece of a jute sack and a rag
which were burning. At that moment the defendant Valdes was in the entresol, engaged in his work
of cleaning, while, the other defendant Hugo Labarro was cleaning the horses kept at the place

ISSUE:
Whether or not the crime committed by the accused is Frustrated Arson.

RULING:
Yes, the fact of setting fire to a jute sack and a rag, soaked with kerosene oil and placed beside an
upright of the house and a partition of the entresol of the building, thus endangering the burning
of the latter, constitutes the crime of frustrated arson of an inhabited house, on an occasion when
some of its inmates were inside of it.

The crime is classified only as frustrated arson, inasmuch as the defendant performed all the acts
conceive to the burning of said house, but nevertheless., owing to causes independent of his will,
the criminal act which he intended was not produced. The offense committed cannot be classified
as consummated arson by the burning of said inhabited house, for the reason that no part of the
building had yet commenced to burn, although, as the piece of sack and the rag, soaked in kerosene
oil, had been placed near partition of the entresol, the partition might have started to burn, had the
fire not been put out on time.

97
CRIMINAL LAW 1 CASE DIGESTS
ATTY. LYAN DAVD JUANICO
BLOCK 1-M A.Y. 2020 - 2021

MODULE 4: CONSPIRACY AND PROPOSAL TO COMMIT A FELONY &


PENALTIES AS TO SEVERITY

1. Dungo v. People
G.R. No. 209464; 1 July 2015
MENDOZA, J.:

FACTS:
Susan Ignacio (Ignacio) was the owner of the sari-sari store located at Purok 5, Pansol, Calamba
City, in front of Villa Novaliches Resort, which was barely ten steps away. On January 13, 2006,
at around 8:30 to 9:00 o'clock in the evening, she was tending her store when she saw a jeepney
with more than twenty (20) persons arrive at the resort. Ignacio identified Dungo as the person
seated beside the driver of the jeepney. She estimated the ages of these persons in the group to be
between 20 to 30 years old. They were in civilian clothes, while the other men wore white long-
sleeved shirts. Before entering the resort, the men and women shook hands and embraced each
other. Three (3) persons, riding on a single motorcycle, also arrived at the resort.

Ignacio saw about fifteen (15) persons gather on top of the terrace of the resort who looked like
they were praying, and then the lights of the resort were turned off. Later that evening, at least
three (3) of these persons went to her store to buy some items. During her testimony, she was
shown photographs and she identified Christopher Braseros and Sibal as two of those who went to
her store. It was only on the morning of January 14, 2006 that she learned from the policemen
visiting the resort that the deceased person was Villanueva.

Donato Magat (Magat), a tricycle driver plying the route of Pansol, Calamba City, testified that at
around 3:00 o'clock in the morning of January 14, 2006, he was waiting for passengers at the comer
of Villa Novaliches Resort. A man approached him and told him that someone inside the resort
needed a ride. Magat went to the resort and asked the two (2) men at the gate who needed a ride.
Afterwards, he saw three (3) men in their 20's carrying another man, who looked very weak, like
a vegetable, towards his tricycle. Magat touched the body of the man being carried and sensed it
was cold.

Magat asked the men what happened to their companion. They replied that he had too much to
drink. Then they instructed Magat to go to the nearest hospital. He drove the tricycle to JP Rizal
Hospital. Upon their arrival, two of his passengers brought their unconscious companion inside
the emergency room, while their other companion paid the tricycle fare. Magat then left to go
home. Several days after, he learned that the person brought to the hospital had died.

98
CRIMINAL LAW 1 CASE DIGESTS
ATTY. LYAN DAVD JUANICO
BLOCK 1-M A.Y. 2020 - 2021

ISSUE:

Whether or not the participation of the offenders in the criminal conspiracy can be proven by the
prima facie evidence due to their presence during the hazing

RULING:
Yes, conspiracy exists when two or more persons come to an agreement concerning the
commission of a felony and decide to commit it. To determine conspiracy, there must be a common
design to commit a felony. The overt act or acts of the accused may consist of active participation
in the actual commission of the crime itself or may consist of moral assistance to his co-
conspirators by moving them to execute or implement the criminal plan.

In conspiracy, it need not be shown that the parties actually came together and agreed in express
terms to enter into and pursue a common design. The assent of the minds may be and, from the
secrecy of the crime, usually inferred from proof of facts and circumstances which, taken together,
indicate that they are parts of some complete whole. Responsibility of a conspirator is not confined
to the accomplishment of a particular purpose of conspiracy but extends to collateral acts and
offenses incident to and growing out of the purpose intended.

Furthermore, R.A. No. 8049, nevertheless, presents a novel provision that introduces a disputable
presumption of actual participation; and which modifies the concept of conspiracy. Section 4,
paragraph 6 thereof provides that the presence of any person during the hazing is prima facie
evidence of participation as principal, unless he prevented the commission of the punishable acts.
This provision is unique because a disputable presumption arises from the mere presence of the
offender during the hazing, which can be rebutted by proving that the accused took steps to prevent
the commission of the hazing.

Hence, generally, mere presence at the scene of the crime does not in itself amount to conspiracy.
Exceptionally, under R.A. No. 8049, the participation of the offenders in the criminal conspiracy
can be proven by the prima facie evidence due to their presence during the hazing, unless they
prevented the commission of the acts therein.

99
CRIMINAL LAW 1 CASE DIGESTS
ATTY. LYAN DAVD JUANICO
BLOCK 1-M A.Y. 2020 - 2021

2. People v. Aguilos
G.R. No. 121828; 27 June 2003
CALLEJO, SR., J.:

FACTS:
On February 5, 1988, at around 11:30 p.m., Elisa Rolan was inside their store at 613 Nueve de
Pebrero Street, Mandaluyong City, waiting for her husband to arrive. Joselito Capa and Julian
Azul, Jr. were drinking beer. Edmar Aguilos and Odilon Lagliba arrived at the store. Joselito and
Julian invited them to join their drinking spree, and although already inebriated, the two
newcomers obliged. In the course of their drinking, the conversation turned into a heated argument.
Edmar nettled Julian, and the latter was peeved. An altercation between the two ensued. Elisa
pacified the protagonists and advised them to go home as she was already going to close up. Edmar
and Odilon left the store.

Joselito and Julian were also about to leave, when Edmar and Odilon returned, blocking their way.
Edmar took off his eyeglasses and punched Julian in the face. Elisa shouted: "Tama na. Tama na."
Edmar and Julian ignored her and traded fist blows until they reached Aling Sotera’s store at the
end of the street, about twelve to fifteen meters away from Elisa’s store. For his part, Odilon
positioned himself on top of a pile of hollow blocks and watched as Edmar and Julian swapped
punches. Joselito tried to placate the protagonists to no avail. Joselito’s intervention apparently did
not sit well with Odilon. He pulled out his knife with his right hand and stepped down from his
perch. He placed his left arm around Joselito’s neck, and stabbed the latter. Ronnie and the
appellant, who were across the street, saw their gangmate Odilon stabbing the victim and decided
to join the fray. They pulled out their knives, rushed to the scene and stabbed Joselito. Elisa could
not tell how many times the victim was stabbed or what parts of his body were hit by whom. The
victim fell in the canal.

Odilon and the appellant fled, while Ronnie went after Julian and tried to stab him. Julian ran for
dear life. When he noticed that Ronnie was no longer running after him, Julian stopped at E.
Rodriguez Road and looked back. He saw Ronnie pick up a piece of hollow block and with it
bashed Joselito’s head. Not content, Ronnie got a piece of broken bottle and struck Joselito once
more. Ronnie then fled from the scene. Joselito died on the spot. Elisa rushed to Joselito’s house
and informed his wife and brother of the incident.

ISSUE:
Whether or not the appellant conspired with Ronnie and Odilon in stabbing the victim to death.

100
CRIMINAL LAW 1 CASE DIGESTS
ATTY. LYAN DAVD JUANICO
BLOCK 1-M A.Y. 2020 - 2021

RULING:
Yes there is conspiracy when two or more persons agree to commit a felony and decide to commit
it. Conspiracy as a mode of incurring criminal liability must be proved separately from and with
the same quantum of proof as the crime itself. Conspiracy need not be proven by direct evidence.

To hold an accused guilty as a co-principal by reason of conspiracy, he must be shown to have


performed an overt act in pursuance or furtherance of the conspiracy. The mere presence of an
accused at the situs of the crime will not suffice; mere knowledge, acquiescence or approval of the
act without cooperation or agreement to cooperate on the part of the accused is not enough to make
him a party to a conspiracy. There must be intentional participation in the transaction with a view
to the furtherance of the common design and purpose.

In this case, Odilon all by himself initially decided to stab the victim. The appellant and Ronnie
were on the side of the street. However, while Odilon was stabbing the victim, the appellant and
Ronnie agreed to join in; they rushed to the scene and also stabbed the victim with their respective
knives. The three men simultaneously stabbed the hapless victim. Odilon and the appellant fled
from the scene together, while Ronnie went after Julian. When he failed to overtake and collar
Julian, Ronnie returned to where Joselito fell and hit him with a hollow block and a broken bottle.
Ronnie then hurriedly left. All the overt acts of Odilon, Ronnie and the appellant before, during,
and after the stabbing incident indubitably show that they conspired to kill the victim.

Thus, all things considered, it was ruled that Ronnie and the appellant conspired with Odilon to
kill the victim; hence, all of them are criminally liable for the latter’s death. The appellant is not
merely an accomplice but is a principal by direct participation.

101
CRIMINAL LAW 1 CASE DIGESTS
ATTY. LYAN DAVD JUANICO
BLOCK 1-M A.Y. 2020 - 2021

3.People v. Larrañaga
421 SCRA 530; 3 February 2004
PER CURIAM:

FACTS:
On July 15, 1997, while Rusia was loafing around at the Cebu Plaza Hotel, Cebu City, Rowen
approached him and arranged that they meet the following day at around 2:00 o'clock in the
afternoon. When they saw each other the next day, Rowen told him to stay put at the Ayala Mall
because they would have a "big happening" in the evening. All the while, he thought that Rowen's
"big happening" meant group partying or scrounging. He thus lingered at the Ayala Mall until the
appointed time came.

At 10:30 in the evening, Rowen returned with Josman. They met Rusia at the back exit of the
Ayala Mall and told him to ride with them in a white car. Rusia noticed that a red car was following
them. Upon reaching Archbishop Reyes Avenue, same city, he saw two women standing at the
waiting shed. Rusia did not know yet that their names were Marijoy and Jacqueline.

Josman stopped the white car in front of the waiting shed and he and Rowen approached and
invited Marijoy and Jacqueline to join them. But the sisters declined. Irked by the rejection, Rowen
grabbed Marijoy while Josman held Jacqueline and forced both girls to ride in the car. Marijoy
was the first one to get inside, followed by Rowen. Meanwhile, Josman pushed Jacqueline inside
and immediately drove the white car. Rusia sat on the front seat beside Josman.

Fourteen (14) meters from the waiting shed, Jacqueline managed to get out of the car. Josman
chased her and brought her back into the car. Not taking anymore chances, Rowen elbowed
Jacqueline on the chest and punched Marijoy on the stomach, causing both girls to faint. Rowen
asked Rusia for the packaging tape under the latter's seat and placed it on the girls' mouths. Rowen
also handcuffed them jointly. The white and red cars then proceeded to Fuente Osmeña, Cebu City.

At Fuente Osmeña, Josman parked the car near a Mercury Drug Store and urged Rusia to inquire
if a van that was parked nearby was for hire. A man who was around replied "no" so the group
immediately left. The two cars stopped again near Park Place Hotel where Rusia negotiated to hire
a van. But no van was available. Thus, the cars sped to a house in Guadalupe, Cebu City known
as the safehouse of the "Jozman Aznar Group" Thereupon, Larrañaga, James Anthony and James
Andrew got out of the red car.

Larrañaga, James Anthony and Rowen brought Marijoy to one of the rooms, while Rusia and
Josman led Jacqueline to another room. Josman then told Rusia to step out so Rusia stayed at the

102
CRIMINAL LAW 1 CASE DIGESTS
ATTY. LYAN DAVD JUANICO
BLOCK 1-M A.Y. 2020 - 2021

living room with James Andrew. They remained in the house for fifteen (15) to twenty (20)
minutes. At that time, Rusia could hear Larrañaga, James Anthony, and Rowen giggling inside the
room.

Thereafter, the group brought Marijoy and Jacqueline back to the white car. Then the two cars
headed to the South Bus Terminal where they were able to hire a white van driven by Alberto.
Ariel was the conductor. James Andrew drove the white car, while the rest of the group boarded
the van. They traveled towards south of Cebu City, leaving the red car at the South Bus Terminal.

Inside the van, Marijoy and Jacqueline were slowly gaining strength. James Anthony taped their
mouths anew and Rowen handcuffed them-together. Along the way, the van and the white car
stopped by a barbeque store. Rowen got off the van and bought barbeque and Tanduay rhum.
They proceeded to Tan-awan. Then they parked their vehicles near a precipice where they drank
and had a pot session. Later, they pulled Jacqueline out of the van and told her to dance as they
encircled her. She was pushed from one end of the circle to the other, ripping her clothes in the
process. Meanwhile, Josman told Larrañaga to start raping Marijoy who was left inside the van.
The latter did as told and after fifteen minutes emerged from the van saying, "who wants next?”
Rowen went in, followed by James Anthony, Alberto, the driver, and Ariel, the conductor. Each
spent a few minutes inside the van and afterwards came out smiling.

Then they carried Marijoy out of the van, after which Josman brought Jacqueline inside the vehicle.
Josman came out from the van after ten minutes, saying, "whoever wants next go ahead and hurry
up." Rusia went inside the van and raped Jacqueline, followed by James Andrew. At this instance,
Marijoy was to breathe her last for upon Josman's instruction, Rowen and Ariel led her to the cliff
and mercilessly pushed her into the ravine which was almost 150 meters deep.

As for Jacqueline, she was pulled out of the van and thrown to the ground. Able to gather a bit of
strength, she tried to run towards the road. The group boarded the van, followed her and made fun
of her by screaming, "run some more" There was a tricycle passing by. The group brought
Jacqueline inside the van. Rowen beat her until she passed out. The group then headed back to
Cebu City with James Andrew driving the white car. Rusia got off from the van somewhere near
the Ayala Center.

ISSUE:
Whether or not all the appellants conspired in the commission of the crimes charged.

103
CRIMINAL LAW 1 CASE DIGESTS
ATTY. LYAN DAVD JUANICO
BLOCK 1-M A.Y. 2020 - 2021

RULING:
Yes, from the evidence of the prosecution, there is no doubt that all the appellants conspired in the
commission of the crimes charged. Their concerted actions point to their joint purpose and
community of intent. Well settled is the rule that in conspiracy, direct proof of a previous
agreement to commit a crime is not necessary. It may be deduced from the mode and manner by
which the offense was perpetrated, or inferred from the acts of the accused themselves when such
point to a joint design and community of interest. Otherwise stated, it may be shown by the conduct
of the accused before, during, and after the commission of the crime.

Appellants' actions showed that they have the same objective to kidnap and detain the Chiong
sisters. Rowen and Josman grabbed Marijoy and Jacqueline from the vicinity of Ayala Center.
Larrañaga, James Andrew and James Anthony who were riding a red car served as back-up of
Rowen and Josman. Together in a convoy, they proceeded to Fuente Osmeña to hire a van, and
thereafter, to the safehouse of the "Jozman Aznar Group" in Guadalupe, Cebu where they initially
molested Marijoy and Jacqueline. They headed to the South Bus Terminal where they hired the
white van driven by Alberto, with Ariel as the conductor. Except for James Andrew who drove the
white car, all appellants boarded the white van where they held Marijoy and Jacqueline captive. In
the van, James Anthony taped their mouths and Rowen handcuffed them together. They drank and
had a pot session at Tan-awan. They encircled Jacqueline and ordered her to dance, pushing her
and ripping her clothes in the process. Meanwhile, Larrañaga raped Marijoy, followed by Rowen,
James Anthony, Alberto, and Ariel. On other hand, Josman and James Andrew raped Jacqueline.
Upon Josman's order, Rowen and Ariel led Marijoy to the cliff and pushed her. After leaving Tan-
awan, they taunted Jacqueline to run for her life. And when Rusia got off from the van near Ayala
Center, the appellants jointly headed back to Cebu City.

104
CRIMINAL LAW 1 CASE DIGESTS
ATTY. LYAN DAVD JUANICO
BLOCK 1-M A.Y. 2020 - 2021

4. People v. Garchitorena
597 SCRA 420, August 28, 2009
Leonardo-de Castro, J

FACTS:

This case is a petition for review of the decision of the Court of Appeals which affirmed the
decision of the Regional Trial Court of Binan City finding the accused Arnold Garchitorena y
Gamba, Joey Pamplona, and Jessie Garcia y Adorino guilty of murder, qualified by abuse of
superior strength, for the death of Mauro Biay y Almarinez.

According to the witness Dulce Borero, who was the sister of the victim, during the evening of
September 22, 1995 the victim, a balut vendor, was peddling his product near Sta. Inez, Almeda
Subdivision. The accused Jessie Garcia called for him, and as Mauro approached the group near
the artesian well of the subdivision, the former suddenly twisted his hand and Jessie’s companions
Garchitorena and Pamplona took turns stabbing the victim with a shiny bladed instrument while
strangling him from behind in order to prevent him from escaping. After the incident it was also
seen by witnesses that Garchitorena ordered his companions to flee from the place.

The accused appellants denied the charge against them, raising different defenses. Arnold
Garchitorena invokes the exempting circumstance of insanity, Jessie Garcia raised the defense of
alibi, and claims that conspiracy is not present in the case at bar claiming that it was only Arnold
Garchitorena who stabbed the victim.

ISSUE:

Is Conspiracy present in the case at bar?

RULING:

Yes. Conspiracy exists when two or more persons come to an agreement concerning the
commission of a felony and decide to commit it. Direct proof is not essential, for conspiracy may
be inferred from the acts of the accused prior to, during, or subsequent to the incident. Such acts
must point to a joint purpose, concert of action, or community of interest.

In the case at bar the following acts showed the common objective of the accused. First the three
accused-appellants’ prior act of waiting for the victim in the place. Second the act of Jessie Garcia
calling the victim which prompted him to approach the group. Third, when the victim was near
enough it was also the former who with the help of his co accused Joey Pamplona who restrained

105
CRIMINAL LAW 1 CASE DIGESTS
ATTY. LYAN DAVD JUANICO
BLOCK 1-M A.Y. 2020 - 2021

the victim. Furthermore the witness’ accounts that Garchitorena instructed his fellow accused to
flee from the scene further solidifies their actions’ community of interest.

The accused Garcia’s contention that he did not participate in the stabbing is also of no
merit as, in a conspiracy “the act of one is the act of all” it does not matter who landed the fatal
blow to the victim, with conspiracy established all of conspirators are liable as co-principals

Additionally, the defenses of insanity and alibi by the defense cannot be given any merit,
first, it is a well-settled rule that alibi cannot prevail over positive identification of the accused.
Second, it was stated by the medical expert involved in the case that the accused Garchitorena was
aware of his murder case and it was established that during the commission of the crime he is not
totally deprived of his reason and freedom of will.

106
CRIMINAL LAW 1 CASE DIGESTS
ATTY. LYAN DAVD JUANICO
BLOCK 1-M A.Y. 2020 - 2021

5. People v. Carandang
G.R. No. 175926, July 6, 2011
Leonardo-de Castro, J

FACTS:

This case is an appeal of the decision of the Court of Appeals which affirmed the decision of the
Regional Trial Court finding the accused Restituto Carandang, Henry Milan, and Jackman Chua
guilty of two counts of murder for the death of PO2 Dionisio Salgo, and SPO2 Wilfredo y Pilar
and one count of frustrated murder for SPO1 Montecalvo y Dalio.

On the afternoon of April 5, 2001 information about a suspected drug deal was received by the
drug enforcement unit of the La Loma Police Station which would take place involving the accused
Henry Milan and others in a house located in Barangay Salvacion, Quezon City. Acting on this
information a unit was formed to raid the place, among the members of the said unit were the
deceased and other policemen from the same police station.

Arriving at the place they split up in two groups and scouted the premises of the house, eventually
meeting at the back of the house and saw that the door to the room of Henry Milan was open and
they were able to see his fellow accused, Carandang and Chua inside. When the officers introduced
themselves as policemen Milan immediately shut the door. Alonzo and Red then pushed the door
while shouting “walang gagalaw”, gunshots rang out and the 2 policemen dropped to the floor
killed instantly. Montecalvo meanwhile was shot in the neck by Carandang and when he fell to the
ground he hear Chua instruct Milan to attack him but the former was able to fire his gun and hit
Milan. Another policeman was able to pull Montecalvo out of the house and into safety.

After this more reinforcements arrived, Milan who was shot stepped out of the house and was
rushed to the hospital. And after a lengthy negotiation Chua and Carandang surrendered in the
presence of Colonel Reyes and Ramon Tulfo.

The lower courts convicted the three in conspiracy with one another guilty of murder, qualified by
treachery. The two accused Chua and Milan assailed the decision of the lower courts claiming that
there was no conspiracy in the case at bar as it was only Carandang who was proven to have fired
at the policemen. And assuming that there was conspiracy that the lower court erred in convicting
them of murder as the qualifying circumstance of treachery was absent.

ISSUE:

Is conspiracy present in the case at bar?

107
CRIMINAL LAW 1 CASE DIGESTS
ATTY. LYAN DAVD JUANICO
BLOCK 1-M A.Y. 2020 - 2021

RULING:

Yes. In order for the presence of conspiracy to be proved evidence need not establish the actual
agreement among the conspirators showing a preconceived plan or motive for the commission of
the crime. Proof of concerted action before, during and after the crime, which demonstrates their
unity of design and objective, is sufficient. When conspiracy is established the act of one is the act
of all regardless of the degree of participation of each.

In the case at bar, the conspiracy between the three was established by the following acts. First,
prior to the shooting was the act of Milan closing the door which allowed Carandang to wait in
ambush of the officers. Second, Chua’s instruction to Milan ordering him to attack the then shot
Montecalvo and Milan following Chua’s orders. Additionally conspiracy does not need a sufficient
period of time in order for it to exist, it could arise on the very moment the plotters agree, expressly
or impliedly to commit the felony.

On a final note it was also established that the qualifying circumstance of treachery was present as
it was shown that the intention of the accused was to prevent any chance for the policemen to
defend themselves, this was shown by the way Carandang set up to ambush the policemen when
Milan closed the door.

WHEREFORE: The decision of the Court of Appeals is affirmed.

108
CRIMINAL LAW 1 CASE DIGESTS
ATTY. LYAN DAVD JUANICO
BLOCK 1-M A.Y. 2020 - 2021

6. People v. Dadao
G.R. No. 201860, January 22, 2014
Leonardo-de Castro, J

FACTS:

This case is an appeal of the decision of the Court of Appeals which affirmed the decision of the
Regional Trial Court of Bukidnon which convicted the accused Marcelino Dadao, Antonio
Sulindao, Eddie Malogsi and Alfemio Malogsi guilty of murder qualified by treachery, for the
death of Pionio Yacapin.

On the evening of July 11, 1993 the four accused were seen by multiple witnesses including the
stepchildren of the victim, Eddie and Alfemio Malogsi were seen holding rifles while Marcelino
Dadao and Antonio Sulindao held bolos. Together the four accused shot and killed the victim
Pionio Yacapin while the latter was inside his house.

The lower courts convicted.

The defense assails the decision of the lower court arguing that, the witnesses’ credibility are
questionable, all of the accused tested negative for gunpowder according to their paraffin tests,
and lastly that the lower court erred in appreciating the qualifying circumstance of abuse of
superior strength as it was not allegend in the criminal information filed against them.

ISSUE:

Is conspiracy present in the case at bar?

RULING:

Yes. Conspiracy is present when two or more persons come to an agreement concerning the
commission of the crime and then decide to commit it. It arises the very instant the plotters agree,
either expressly or impliedly to commit the felony and forthwith decide to pursue it. In the case at
bar it was established based on the witnesses’ testimonies that all four accused share a community
of criminal design showing that they have in their common intention to kill Pionio Yacapin.

It has also been established in jurisprudence that a paraffin test is not conclusive proof that a person
has not fired a gun. Additionally it was only held that only Eddie and Alfemio Malogsi held
firearms while the other two accused held bolos. The paraffin test results hold no merit in the case
at bar as once conspiracy has been established “the act of one is the act of all” and each of the

109
CRIMINAL LAW 1 CASE DIGESTS
ATTY. LYAN DAVD JUANICO
BLOCK 1-M A.Y. 2020 - 2021

accused should suffer the same criminal liability attached to the criminal act regardless of who
fired the weapon which delivered the fatal wound to the victim.

Lastly in the case at bar the contention that abuse of superior strength was wrongly appreciated by
the lower court as a qualifying circumstance is unmeritorious as the murder was qualified by
treachery and not abuse of superior strength.

WHEREFORE: the decision of the Court of Appeals is affirmed.

110
CRIMINAL LAW 1 CASE DIGESTS
ATTY. LYAN DAVD JUANICO
BLOCK 1-M A.Y. 2020 - 2021

7. People v. Octa
G.R. No. 195196, July 13, 2015
Sereno, C.J.

FACTS:

This case is an appeal of the decision of the Court of appeals which affirmed the decision of the
Regional Trial Court of Manila convicting the accused Estanly Octa y Bas guilty of the crime of
kidnapping for ransom.

On the morning of September 25, 2003 Johnny Corpuz and Mike Batuigas were riding a silver
Honda Civic travelling along Sampaloc Manila when they were suddenly blocked by an orange
Mitsubishi Lancer. Four armed men alighted from the orange car and forced their way inside the
Honda Civic ordering Corpuz to transfer to the back where he was blindfolded, handcuffed, and
boxed. The men then brought johnny to their safehouse.

The kidnappers contacted the wife of Johnny and negotiated the amount of ransom starting from
20 million but was ultimately reduced to five hundred thirty eight thousand which the kidnappers
accepted. The kidnappers then decided that the ransom money be delivered to a person wearing a
red cap at a Caltex Auto Supply in Quezon City, Ana Marie went to the said place and saw the
said man, but made sure to confirm that he was indeed part of the kidnappers by making him talk
to the other kidnappers on the phone. A few days after the ransom money was delivered Johnny
and Mike were released.

The said man as described by her as good looking with dimples was the accused Estanly Octa.
Octa was arrested by the police in connection with another kidnap for ransom incident but was
identified by Ana Marie from a police line-up as the person who received the ransom money from
her.

The lower courts then convicted him guilty of the crime of kidnapping with ransom deeming his
acts of receiving the money as enough to establish his conspiracy with the kidnappers, as during
that time the crime of kidnapping was still continuing as Johnny and Mike were still illegally
detained by the kidnappers.

The defense assails the decision of the lower courts claiming that it erred in considering him as a
conspirator claiming that his receipt of the money happened only after the victims were already
taken which consummated the crime of kidnapping and his act was not an essential element of the
crime.

111
CRIMINAL LAW 1 CASE DIGESTS
ATTY. LYAN DAVD JUANICO
BLOCK 1-M A.Y. 2020 - 2021

ISSUE:

Did the lower court erred in considering accused appellant as a co-conspirator in the crime of
kidnapping with ransom?

RULING:

No. Where all the accused acted in concert at the time of the commission of the offense, and it is
shown by such acts that they had the same purpose or common design and were united in its
execution, conspiracy is sufficiently established. It must be shown that all participants performed
specific acts with such closeness and coordination as to indicate a common purpose or design to
commit the felony. To hold an accused guilty as a co-principal by reason of conspiracy, he must
be shown to have performed an overt act in pursuance or furtherance of the complicity. There must
be intentional participation with a view to the furtherance of the common design and purpose.

In the case at bar the Court of Appeals correctly observed that the crime of kidnapping was still
continuing at the time the ransom money was received by accused-appellant since both of the
victims were still being illegally detained by the kidnappers and although his receipt of the money
was not an element of the crime, it was part of the grand plan and was in fact the main purpose of
the kidnapping of the victims. That is because without the ransom money the freedom of the
detained victims cannot be achieved.

WHEREFORE: the decision of the Court of Appeals is affirmed.

112
CRIMINAL LAW 1 CASE DIGESTS
ATTY. LYAN DAVD JUANICO
BLOCK 1-M A.Y. 2020 - 2021

8. People v. Feliciano
G.R. No. 196735, May 5, 2014
LEONEN, J.:

FACTS:

This case is an appeal of the decision of the Court of Appeals which affirmed the decision of the
Regional trial court of Quezon City which found respondents Danilo Feliciano Jr., et al. guilty of
the crimes of Murder, attempted murder, and slight physical injuries.

On the afternoon of December 8, 1994 at the Beach House canteen near the main library of UP
Diliman. Seven members of the Sigma Rho Fraternity were eating lunch, when Dennis Venturina,
one of the fraternity member, shouted and alerted his fraternity brothers that masked men holding
baseball bats and pipes are approaching them. About ten men charged towards the Sigma Rho
members and started hitting them with their weapons, most of them were wearing handkerchiefs
and shirts as masks in order to cover their faces.

As the attack continued some of the attackers’ mask fell off and the victims were able to recognize
some of them as Alvir, Zingapan, and Medalla members of a rival fraternity. As the Sigma Rho
members tried to escape their attackers they saw Dennis Venturina lying on the ground while
respondent Danilo Feliciano Jr. was beating him up with a lead pipe.

The attack resulted in the death of Dennis Venturina, and the hospitalization of the other Sigma
Rho members, with some injuries taking a period of ten days to thirty days from the date of
infliction to heal.

The Court of Appeals modified the earlier charge of attempted murder for the attack on the Sigma
Rho members Mangrobang, Lachica, and Gaston to slight physical injuries as the Court of Appeals
reasoned that they were no longer chased by the attackers which showed that the respondents
“voluntarily desisted from pursuing them and from inflicting harm to them, which shows that they
did not have the intent to do more than to make them suffer pain by slightly injuring them.”

ISSUE:

Did the Court of Appeals erred in its decision that the Mangrobang, Lachica, and Gaston

RULING:

Yes. The charges of slight physical injuries should be modified to attempted murder.

113
CRIMINAL LAW 1 CASE DIGESTS
ATTY. LYAN DAVD JUANICO
BLOCK 1-M A.Y. 2020 - 2021

Conspiracy once proven has the effect of attaching liability to all of the accused, regardless of their
degree of participation, thus once an express or implied conspiracy is proved, all of the conspirators
are liable as co-principals regardless of the extent and character of their respective active
participation in the commission of the crime or crimes perpetrated in furtherance of the conspiracy
because in contemplation of law the “act of one is the act of all”

In the case at bar since the qualifying circumstance of treachery was proven in order for murder to
be established. The conspiracy of the accused-appellants can be established as having originated
from the moment that they, armed with baseball bats and lead pipes, agreed to inflict injury and
the highest amount of damage possible to the victims. The intent to kill was already present at the
moment of attack and being in conspiracy with each other it is therefore immaterial to distinguish
between the seriousness of the injuries inflicted by each of the attackers to determine their
individual liability.

WHEREFORE: the assailed judgment of the Court of Appeals is modified

114
CRIMINAL LAW 1 CASE DIGESTS
ATTY. LYAN DAVD JUANICO
BLOCK 1-M A.Y. 2020 - 2021

10. People v. Morilla


G.R. No. 189833, February 5, 2014
Perez, J

FACTS:

This case is an appeal of the decision of the Court of Appeals which affirmed the decision of the
Regional Trial Court of Quezon convicting the accused Mayor Mitra, Javier Morilla y Avellano,
Willie y Yang, and Ruel Dequilla y Regodan guilty of violating Section 15 of RA 6425 for
transporting illegal drugs.

On the evening of October 13, 2001 two vehicles, a Starex van driven by Mayor Mitra and the
municipal ambulance of Panukulan Quezon which was driven by Morilla left Infanta Quezon, en
route to Manila. The municipal ambulance was then stopped in a routine checkpoint near Real,
Quezon; noticing several sacks inside the ambulance which prompted the police to order Morilla
to open the car for inspection. Morilla then claimed that he was with Mayor Mitra who was able
to pass the checkpoint in his Starex, and when the police discovered that the said sacks contained
methamphetamine hydrochloride “shabu” they chased down the Starex and also found
methamphetamine in the said vehicle.

The court found Mayor Mitra and Morilla guilty while two of the accused Yang and Dequilla were
acquitted for lack of proof beyond reasonable doubt, with the lower courts ruling that their mere
presence in the vehicle as passengers was inadequate to prove that they were also conspirators of
Mayor Mitra and Morilla.

Respondent Mitra and Morilla now assail the decision of the lower court arguing that there was no
conspiracy in the case at bar, claiming that they were without knowledge of the contents of the
sacks and that they were merely requested to tranpost them to Manila; Mitra claiming that it was
due to the request of a certain Ben Tan, while Morilla claims that the sacks to his knowledge only
contained wooden tiles and electronic spare parts.

ISSUE:

Was conspiracy correctly established by the lower courts in the case at bar?

RULING:

Yes.

A conspiracy exists when two or more persons come to an agreement concerning the commission
of a crime and decide to commit it. To determine conspiracy, there must be a common design to

115
CRIMINAL LAW 1 CASE DIGESTS
ATTY. LYAN DAVD JUANICO
BLOCK 1-M A.Y. 2020 - 2021

commit a felony – it need not be shown that parties actually came together and agreed in express
terms that they intent to pursue a common design, it is enough that it is established from proof of
facts and circumstances which, when taken together, indicate that they are parts of some complete
whole.

In the case at bar the totality of the factual circumstances leads to a conclusion that Morilla is in
conspiracy with Mayor Mitra with both of them sharing a common desire of transporting the said
dangerous drugs from Quezon to Manila. As it was shown that both vehicles were in convoy from
Quezon to Manila, and furthermore when Morilla was flagged down in the checkpoint he told the
police that he was with Mayor Mitra who managed to get through the said checkpoint establishing
the connection between the two.

As to the defense of the two that they are without knowledge of what they are transporting, it is
without merit and cannot be appreciated. As criminal intent is not needed to establish guilt in this
case, as the very act of transporting methamphetamine hydrochloride is malum prohibitum since
it is punished under a special law.

WHEREFORE: the assailed decision of the Court of Appeals is affirmed

116
CRIMINAL LAW 1 CASE DIGESTS
ATTY. LYAN DAVD JUANICO
BLOCK 1-M A.Y. 2020 - 2021

11. People v. Bokingco


G.R. No. 187356, August 10, 2011
Perez, J

FACTS:
Review of the decision of the Court of Appeals which affirmed the decision of the Regional Trial
Court finding the accused Michael Bokingco and Reynante Col in conspiracy with one another
guilty of the crime of murder qualified by treachery and evident premeditation for the death of
Noli Pasion.

The victim, Noli Pasion and his wife Elsa resided in a house that is connected to a pawnshop, at
the back of the said house they also maintained two rows of apartment units, one of these rows
was under construction at the time and the accused were then employed as construction workers.

At around 1 am on February 29, 2000, Pasion’s brother in law who lived in one of the apartment
units was doing laundry when he heard a commotion in apartment number 3; checking the noise
he saw Bokingco hitting something on the floor with his claw hammer. Bokingco upon being
spotted suddenly attacked him with the said hammer, they struggled and he managed to bite
Bokingco in the neck which gave him opportunity to escape as Bokingco chased him but was
finally subdued by a fellow construction worker. After escaping he was told by his wife that Pasion
was found dead in apartment number 3.

Meanwhile Elsa who was in the master bedroom of the main house also heard the noises and her
husband’s moans so she decided to go and check, but upon reaching the kitchen she was blocked
by Col who sprayed tear gas on her eyes and poked a sharp object under her chin. Col then ordered
her to open the vault of the pawnshop but she claimed that she did not know the combination. Elsa
then saw Bokingco open the screen door and she heard him tell Col “tara, patay na siya”, which
led to Col letting her go and proceeding to run away with Bokingco.

When he was captured Bokingco admitted extrajudicially to killing Pasion, he also claimed that
he planned the attack with Col days before the incident due to them being tired of Pasion
mistreating them as workers. Meanwhile Col claims that he doesn’t know Bokingco very well.

The respondents now assaild the decision, claiming that; the qualifying circumstances were not
present, and whether or not appellant Col is guilty beyond reasonable doubt as a co-conspirator.

ISSUE:

Did the lower court erred in finding Reynante Col guilty as a co-conspirator?

117
CRIMINAL LAW 1 CASE DIGESTS
ATTY. LYAN DAVD JUANICO
BLOCK 1-M A.Y. 2020 - 2021

RULING:

Yes. As a rule conspiracy must be established with the same quantum of proof as the crime itself
and must be shown as clearly as the commission of the crime.

In the case at bar the finding that Col was in conspiracy with Bokingco rests on Elsa’s testimony
of the events and on the extrajudicial confession of Bokingco. The extrajudicial confession of
Bokingco is inadmissible as it was given during preliminary investigation without assistance of
counsel in violation of Section 12, Article III of the 1987 Constitution, this confession is not only
inadmissible to him but also to his co-accused.

Elsa’s version of events meanwhile points out that Col was in the house, trying to rob the pawnshop
at the time the attack on Pasion in apartment number 3 was happening, the fact that she heard
Bokingco call to Col that Pasion had been killed does not prove that they acted in concert towards
the consummation of the crime. It only proves, at best, that there were two crimes being committed
simultaneously and they were united in their efforts to escape from the crimes they separately
committed.

Lastly, because of the inadmissibility of the confession of Bokingco the qualifying circumstances
was not successfully proven lowering the conviction of Bokingco from murder to homicide.

WHEREFORE: The appeal is granted Reynante Col is acquitted on ground of reasonable doubt.
Appellant Michael Bokingco is found guilty beyond reasonable doubt of the crime of Homicide.

118
CRIMINAL LAW 1 CASE DIGESTS
ATTY. LYAN DAVD JUANICO
BLOCK 1-M A.Y. 2020 - 2021

12. Fernan v. People


G.R. No. 14592; 24 August 2007
VELASCO, JR., J.:

FACTS:
Sometime in February 1977, accused Rolando Mangubat (Chief Accountant), Delia Preagido
(Accountant III), Jose Sayson (Budget Examiner), and Edgardo Cruz (Clerk II), all of MPH Region
VII, met at the Town and Country Restaurant in Cebu City and hatched an ingenious plan to siphon
off large sums of money from government coffers. Mangubat had found a way to withdraw
government money through the use of fake Letters of Advice of Allotments (LAAs), vouchers and
other documents and to conceal traces thereof with the connivance of other government officials
and employees. In fine, the fraudulent scheme involved the splitting of LAAs and RSEs so that the
amount covered by each general voucher is less than P50,000.00 to do away with the approval of
the Regional Auditor; the charging of disbursements to unliquidated obligations due the previous
year to provide the supposed source of funds; and the manipulation of the books of account by
negation or adjustment, i.e., the cancellation of checks through journal vouchers to conceal
disbursements in excess of the cash disbursement ceiling (CDC), so as not to reflect such
disbursements in the trial balances submitted to the Regional Office.

Mangubat enticed Preagido, Cruz and Sayson to join him. All three agreed to help him carry out
his plan. They typed the fake LAAs during Saturdays. Cruz and Sayson also took charge of
negotiating or selling the fake LAAs to contractors at 26% of the gross amount. Preagido on her
part manipulated the General Ledger, Journal Vouchers and General Journal thru negative entries
to conceal the illegal disbursements. Thus, in the initial report of the auditors (Exhibit “D”), it was
discovered that the doubtful allotments and other anomalies escaped notice.

The four formed the nucleus of the nefarious conspiracy. Other government employees, tempted
by the prospect of earning big money, allowed their names to be used and signed spurious
documents.

Rolando Mangubat, Jose Sayson, and Edgardo Cruz, all of MPH Region VII, were found guilty in
all 119 counts and were accordingly sentenced by the SB. The other conniver, Delia Preagido,
after being found guilty in some of the cases, became a state witness in the remainder. On the basis
of her testimony and pertinent documents, Informations were filed, convictions were obtained, and
criminal penalties were imposed on the rest of the accused. On the other hand, petitioners were
both Civil Engineers of the MPH assigned to the Cebu First Highway Engineering District.
Petitioner Fernan, Jr. was included allegedly for having signed six (6) tally sheets or statements of
deliveries of materials, used as bases for the preparation of the corresponding number of general

119
CRIMINAL LAW 1 CASE DIGESTS
ATTY. LYAN DAVD JUANICO
BLOCK 1-M A.Y. 2020 - 2021

vouchers. Fund releases were made to the suppliers, contractors, and payees based on these general
vouchers.

ISSUE:
Whether or not Fernan and Torevillas were included in the conspiracy and are criminally liable.

Ruling:
Yes. The Supreme Court held that what is present is a wheel or circle conspiracy. 4 persons headed
by the chief accountant (Mangubat) acted as the hub. They enticed all other 36 employees of the
DPWH to be one with them in committing fraud against the government. They falsified LAA's
and would negotiate it at a certain percentage, then one of them would compute the general
voucher, funds then will be issued as if materials will be delivered for the construction. Fernan and
Torevillas were civil engineers of the DPWH, they signed tally sheets, saying that there were
deliveries when in fact these were ghost deliveries. No actual deliveries of the materials. Fernan,
Jr. signed tally sheets on the ghost deliveries of Juliana de los Angeles and Ismael Sabio, Jr. On
the part of petitioner Torrevillas, he signed false tally sheets and delivery receipts on supplies
allegedly delivered by Rufino V. Nuñez, Juliana de los Angeles, Ismael Sabio, Jr., and Manuel
Mascardo. Lastly, the checks issued to these suppliers based on general vouchers supported by the
false tally sheets and general vouchers signed by petitioners cannot be traced to any genuine LAAs,
resulting in the inescapable conclusion that these LAAs were unauthorized; hence, fake or
fabricated. These are undisputed tell-tale signs of the complicity by petitioners with the Mangubat
syndicate.

120
CRIMINAL LAW 1 CASE DIGESTS
ATTY. LYAN DAVD JUANICO
BLOCK 1-M A.Y. 2020 - 2021

13. Gloria Macapagal-Arroyo v. People


G.R. No. 220598; 19 July 2016
BERSAMIN,, J.:

FACTS:
On July 10, 2012, the Ombudsman charged in the Sandiganbayan former President Gloria
Macapagal-Arroyo (GMA); Philippine Charity Sweepstakes Office (PCSO) Budget and Accounts
Officer Benigno Aguas; PCSO General Manager and Vice Chairman Rosario C. Uriarte; PCSO
Chairman of the Board of Directors Sergio O. Valencia; Members of the PCSO Board of Directors,
namely: Manuel L. Morato, Jose R. Taruc V, Raymundo T. Roquero, and Ma. Fatima A.S. Valdes;
Commission on Audit (COA) Chairman Reynaldo A. Villar; and COA Head of
Intelligence/Confidential Fund Fraud Audit Unit Nilda B. Plaras with plunder.

During the period from January 2008 to June 2010 or sometime prior or subsequent thereto, the
abovementioned, all public officers committing the offense in relation to their respective offices
and taking undue advantage of their respective official positions, authority, relationships,
connections or influence, conniving, conspiring and confederating with one another, did then and
there willfully, unlawfully and criminally amass, accumulate and/or acquire. Directly or indirectly,
ill-gotten wealth in the aggregate amount or total value of PHP365,997,915.00 more or less,
through any or a combination or a series of overt or criminal acts, or similar schemes or means,
described as diverting in several instances, funds from the operating budget of PCSO to its
Confidential/Intelligence Fund that could be accessed and withdrawn at any time with minimal
restrictions, and converting, misusing, and/or illegally conveying or transferring the proceeds
drawn from said fund in the aforementioned sum, also in several instances, to themselves, in the
guise of fictitious expenditures, for their personal gain and benefit; and raiding the public treasury
by withdrawing and receiving, in several instances, the above-mentioned amount from the
Confidential/Intelligence Fund from PCSO's accounts, and or unlawfully transferring or conveying
the same into their possession and control through irregularly issued disbursement vouchers and
fictitious expenditures; and taking advantage of their respective official positions, authority,
relationships, connections or influence, in several instances, to unjustly enrich themselves in the
aforementioned sum, at the expense of, and the damage and prejudice of the Filipino people and
the Republic of the Philippines. When Arroyo approved the requests, the PCSO was operating on
a deficit. This situation means that it is irregular to authorize additional CIF when the fund source
is negative. It is tantamount to authorizing the use of other PCSO funds - that of the Prize Fund
and Charity Fund - for purposes other than those allowed by law.

121
CRIMINAL LAW 1 CASE DIGESTS
ATTY. LYAN DAVD JUANICO
BLOCK 1-M A.Y. 2020 - 2021

ISSUE:

Whether or not the State sufficiently established the existence of conspiracy among Gloria
Macapagal-Arroyo, Aguas, and Uriarte.

RULING:
No. The Prosecution did not properly allege and prove the Existence of conspiracy among GMA,
Aguas and Uriarte Conspiracy exists when two or more persons come to an agreement concerning
the commission of a felony, and decide to commit it. In this jurisdiction, conspiracy is either a
crime in itself or a mere means to commit a crime. As a rule, conspiracy is not a crime unless the
law considers it a crime, and prescribes a penalty for it. The exception is exemplified in Article
115, Article 136, and Article 141 of the Revised Penal Code. When conspiracy is a means to
commit a crime, it is indispensable that the agreement to commit the crime among all the
conspirators, or their community of criminal design must be alleged and competently shown.

The court also stressed that the community of design to commit an offense must be a conscious
one. Conspiracy transcends mere companionship, and mere presence at the scene of the crime does
not in itself amount to conspiracy. Even knowledge of, or acquiescence in, or agreement to
cooperate is not enough to constitute one a party to a conspiracy, absent any active participation
in the commission of the crime with a view to the furtherance of the common design and purpose.
Hence, conspiracy must be established, not by conjecture, but by positive and conclusive evidence.

To be considered a part of the conspiracy, each of the accused must be shown to have performed
at least an overt act in pursuance or in furtherance of the conspiracy, for without being shown to
do so none of them will be liable as a co-conspirator, and each may only be held responsible for
the results of his own acts.

In her case, GMA points out that all that the State showed was her having affixed her unqualified
“OK” on the requests for the additional CIFs by Uriarte. She argues that such act was not even an
overt act of plunder because it had no immediate and necessary relation to plunder by virtue of her
approval not being per se illegal or irregular. The treatment by the Sandiganbayan of her
handwritten unqualified “OK” as an overt act of plunder was absolutely unwarranted considering
that such act was a common legal and valid practice of signifying approval of a fund release by
the President.

122
CRIMINAL LAW 1 CASE DIGESTS
ATTY. LYAN DAVD JUANICO
BLOCK 1-M A.Y. 2020 - 2021

14. Go-Tan v. Tan


G.R. No. 168852; September 30, 2008
AUSTRIA-MARTINEZ, J.:

FACTS:
On April 18, 1999, Sharica Mari L. Go-Tan (petitioner) and Steven L. Tan were married. Out of
this union, two female children were born, Kyra Danielle and Kristen Denise. On January 12,
2005, barely six years into the marriage, petitioner filed a Petition with Prayer for the Issuance of
a Temporary Protective Order (TPO) against Steven and her parents-in-law, Spouses Perfecto C.
Tan and Juanita L. Tan (respondents) before the RTC. She alleged that Steven, in conspiracy with
respondents, were causing verbal, psychological and economic abuses upon her in violation of
Section 5, paragraphs (e)(2)(3)(4), (h)(5), and (i) of Republic Act No. 9262, otherwise known as
the “Anti-Violence against Women and Their Children Act of 2004.” On January 25, 2005, the
RTC issued an Order/Notice granting petitioner’s prayer for a TPO. On February 7, 2005,
respondents filed a Motion to Dismiss with Opposition to the Issuance of Permanent Protection.

ISSUE:
Whether or not respondents-spouses, Perfecto and Juanita, parents-in-law of Sharica, may be
included in the petition for the issuance of a protective order, in accordance with RA 9262.

RULING:

Yes, the Court ruled in favor of the petitioner. While the provisions of RA 9262 provides that the
offender be related or connected to the victim by marriage, former marriage, or a sexual or dating
relationship, it does not preclude the application of the principle of conspiracy under the RPC. In
Section 47 of RA 9262, it has expressly provided for the suppletory application of the RPC. Hence,
legal principles developed from the Penal Code may be applied in a supplementary capacity to
crimes punished under special laws, such as RA 9262 in which the special law is silent on a
particular matter. The court stated that if the principle of conspiracy under Article 8 of the Revised
Penal Code (RPC) is applied to B.P. 22 in the absence of a contrary provision therein, with more
reason could the same principle be applied suppletorily to R.A. No. 9262 because of the express
provision of Section 47 that the Revised Penal Code (RPC) shall be supplementary to said law.
Thus, the principle of conspiracy may be applied to R.A. No. 9262. For once conspiracy or action
in concert to achieve a criminal design is shown, the act of one is the act of all the conspirators,
and the precise extent or modality of participation of each of them becomes secondary, since all
the conspirators are principals.

123
CRIMINAL LAW 1 CASE DIGESTS
ATTY. LYAN DAVD JUANICO
BLOCK 1-M A.Y. 2020 - 2021

MODULE 5: JUSTIFYING CIRCUMSTANCES AND ABSOLUTORY CAUSES

1. Manaban v. Court of Appeals


G.R. No. 150723; 11 July 2006
CARPIO, J.:

FACTS:
On October 11, 1996, at around 1:25 o’clock in the morning, Joselito Bautista, a father and a
member of the UP Police Force, took his daughter, Frinzi, who complained of difficulty in
breathing, to the UP Health Center. There, the doctors prescribed certain medicines to be
purchased. Needing money therefore, Joselito Bautista, who had taken alcoholic drinks earlier,
proceeded to the BPI Kalayaan Branch to withdraw some money from its Automated Teller
Machine (ATM).

Upon arrival at the bank, Bautista proceeded to the ATM booth but because he could not
effectively withdraw money, he started kicking and pounding on the machine. For said reason, the
bank security guard, Ramonito Manaban, approached and asked him what the problem was.
Bautista complained that his ATM was retrieved by the machine and that no money came out of
it. After Manaban had checked the receipt, he informed Bautista that the Personal Identification
Number (PIN) entered was wrong and advised him to just return the next morning. This angered
Bautista all the more and resumed pounding on the machine. Manaban then urged him to calm
down and referred him to their customer service over the phone. Still not mollified, Bautista
continued raging and striking the machine. When Manaban could no longer pacify him, he fired a
warning shot. That diverted the attention of Bautista. Instead of venting his ire against the machine,
he confronted Manaban. After some exchange of words, a shot rang out fatally hitting Bautista at
the back of his body.

Manaban allegedly admitted shooting Bautista. SPO1 Salvador and his team investigated the crime
scene. According to SPO1 Salvador, he saw Bautista lying on his back near the Automated Teller
Machine (“ATM”). A .38 caliber revolver inside a locked holster was tucked in Bautista’s right
waist. SPO1 Salvador noticed that Bautista, who was still breathing, had been shot in the back.
They brought Bautista to the East Avenue Medical Center where Bautista later died.

ISSUE:
Whether or not Manaban is justified in killing Bautista by virtue of self-defense.

124
CRIMINAL LAW 1 CASE DIGESTS
ATTY. LYAN DAVD JUANICO
BLOCK 1-M A.Y. 2020 - 2021

RULING:
No. Under paragraph 1, Article 11 of the Revised Penal Code, the three requisites to prove self-
defense as a justifying circumstance which may exempt an accused from criminal liability are: (1)
unlawful aggression on the part of the victim; (2) reasonable necessity of the means employed to
prevent or repel the aggression; and (3) lack of sufficient provocation on the part of the accused or
the person defending himself. Unlawful aggression is an indispensable requisite of self-defense.
Self-defense is founded on the necessity on the part of the person being attacked to prevent or repel
the unlawful aggression. Thus, without prior unlawful and unprovoked attack by the victim, there
can be no complete or incomplete self-defense.

Unlawful aggression is an actual physical assault or at least a threat to attack or inflict physical
injury upon a person. A mere threatening or intimidating attitude is not considered unlawful
aggression, unless the threat is offensive and menacing, manifestly showing the wrongful intent to
cause injury. There must be an actual, sudden, unexpected attack or imminent danger thereof,
which puts the defendant’s life in real peril.

In this case, there was no unlawful aggression on the part of the victim. First, Bautista was shot at
the back as evidenced by the point of entry of the bullet. Second, when Bautista was shot, his gun
was still inside a locked holster and tucked in his right waist. Third, when Bautista turned his back
at Manaban, the latter was already pointing his service firearm at Bautista. These circumstances
clearly belie Manaban’s claim of unlawful aggression on Bautista’s part.

125
CRIMINAL LAW 1 CASE DIGESTS
ATTY. LYAN DAVD JUANICO
BLOCK 1-M A.Y. 2020 - 2021

2. Senoja v. People
G.R. No. 160341; 19 October 2004
CALLEJO, SR., J.:

FACTS:
On April 16, 1997, petitioner Exequiel Senoja, Fidel Senoja, Jose Calica, and Miguel Lumasac
were drinking gin in the hut of Crisanto Reguyal in Barangay Zarah, San Luis, Aurora. An angry
Leon Lumasac suddenly arrived at the said place, holding a bolo in his right hand and looking for
his brother Miguel. Petitioner and Jose tried to pacify Leon. But when petitioner approached Leon,
the latter tried to hack him so he embraced Leon and Jose took Leon’s bolo. Then, Leon and
petitioner talked things out and later reconciled.

Subsequently, Leon walked out of Crisanto’s hut followed by petitioner. Suddenly, about ten
meters from the hut, petitioner stabbed Leon at the back. When Leon turned around, petitioner
continued stabbing him until he fell to the ground. Then, petitioner ran towards the barangay road
and threw away the “kolonial” knife he used in stabbing Leon. The latter died on the spot.
Municipal Health Officer, Dr. Uy, examined the cadaver of Leon and found multiple lesions on
his body and five fatal wounds on his chest. Dr. Uy issued a medico-legal report and death
certificate

ISSUE:
Whether or not Senoja acted in complete self-defense and is therefore free from criminal liability.
RULING:
No. Unlawful aggression presupposes an actual, sudden, unexpected attack or imminent danger
thereof, not merely a threatening or intimidating attitude. Hence, when an inceptual/unlawful
aggression ceases to exist, the one making a defense has no right to kill or injure the former
aggressor. After the danger has passed, one is not justified in following up his adversary to take
his life. The conflict for blood should be avoided if possible. An assault on his person, he cannot
punish when the danger or peril is over. When the danger is over, the right of self-defense ceases.
His right is defense, not retribution.

In this case, Leon Lumasac had ceased being the aggressor after he left the hut to go home, accused
Exequiel Senoja was now the unlawful aggressor in this second phase of their confrontation.
Absent such an actual or imminent peril to one’s life or limb, there is nothing to repel; there is no
necessity to take the life or inflict injuries on another.

Therefore, the court affirms that Exequiel Senoja is guilty beyond reasonable doubt of the crime
of homicide.

126
CRIMINAL LAW 1 CASE DIGESTS
ATTY. LYAN DAVD JUANICO
BLOCK 1-M A.Y. 2020 - 2021

3. People v. Decena
G.R. No. 107874; 4 August 1994
REGALADO, J.:

FACTS:
The case for the prosecution, anchored mainly on the testimony of Luzviminda Ballesteros, the
14-year old daughter of the victim, is to the effect that on Christmas Day of 1990, at around 4:00
P.M., said Luzviminda was playing with her siblings at home. She recalled being asked by her
mother, Teresita Ballesteros, to fetch her father, Jaime Ballesteros, who was then watching a game
in the basketball court. On her way to the hardcourt, Luzviminda met her father walking home in
an intoxicated state. Suddenly, she saw appellant rushing towards her father with a long bladed
weapon, prompting Luzviminda to warn her father to run for safety by shouting in the vernacular
“Batik kila, Tatay!” Instead, Jaime simply raised his hand, thus allowing appellant to stab him on
the right chest just below the nipple. Appellant then fled from the crime scene, while the victim
also managed to run but stumbled and fell to the ground.
It was claimed that before such incident, appellant was watching a basketball game. The victim,
Jaime Ballesteros, went around the basketball court, walking in a wobbly manner due to
drunkenness. Jaime stopped near the place where appellant was sitting and, for no apparent reason,
held the latter by the neck with one arm and, at the same time, poking a fork against it with the
other arm. Barangay Tanod Romeo Decena who was also watching the basketball game,
intervened. He took the fork from Jaime and advised appellant to go home. The latter left and was
followed later by Jaime.

ISSUE:
Whether or not Decena is absolved from criminal liability.

RULING:
No. The basic requirement for self-defense, as a justifying circumstance, is that there was an
unlawful aggression against the person defending himself. It must be positively shown that there
was a previous unlawful and unprovoked attack that placed the life of the accused in danger and
forced him to inflict more or less severe wounds. the elementary rule that when the aggressor
leaves, the unlawful aggression ceases, it follows that when appellant and Jaime heeded the advice
of the barangay tanod for them to go home, the unlawful aggression had ended. Consequently,
since unlawful aggression no longer existed, appellant had no right whatsoever to kill or even
wound the former aggressor. The supposed continuation of the unlawful aggression which could
have justified self-defense would have been the circumstance that Jaime persisted in his design to
attack appellant while the latter was already in front of his house. This fact, however, the defense
ruefully failed to establish. Decena is guilty of homicide.

127
CRIMINAL LAW 1 CASE DIGESTS
ATTY. LYAN DAVD JUANICO
BLOCK 1-M A.Y. 2020 - 2021

4. People v. Dela Cruz


G.R. No. 128359; 6 December 2000
AVANCEÑA, C.J.:

FACTS:

The victim Daniel Macapagal, a married man, had been a live-in partner of prosecution witness
Ma. Luz Perla San Antonio for about two to three years before San Antonio took appellant Roberto
dela Cruz, widower, as lover and live-in partner. At the time of the incident on May 27, 1996,
appellant and San Antonio were living in a house being rented by San Antonio at 094 Valino
District, Magsaysay Norte, Cabanatuan City.

At around 6:00 o’clock in the evening on May 27, 1996, San Antonio and appellant were resting
in their bedroom when they heard a car stop in front of their house and later knocks on their door.
San Antonio opened the front door and she was confronted by Macapagal who made his way inside
the house holding a gun in his hand, despite San Antonio’s refusal to let him in. He seemed to be
looking for something or somebody as Macapagal walked passed San Antonio and inspected the
two opened bedrooms of the house. He then went to the closed bedroom where appellant was and
banged at the door with his gun while yelling ‘Come out. Come out’. Appellant then opened the
door but he was greeted by Macapagal’s gun which was pointed at him. Appellant immediately
closed the door while Macapagal continued banging at it. When appellant again opened the door
moments later, he was himself armed with a .38 caliber revolver. The two at that instant
immediately grappled for each other’s firearm. A few moments later shots were heard. Macapagal
fell dead on the floor. Appellant told San Antonio to call the police on the phone.

Macapagal sustained four (4) gunshot wounds. Three of the wounds were non-penetrating or those
that did not penetrate a vital organ of the human body. They were found in the upper jaw of the
left side of the face, below the left shoulder and the right side of the waist. Another gunshot wound
on the left side of the chest penetrated the heart and killed Macapagal instantly.

ISSUE:
Whether or not Dela Cruz is correct in invoking self-defense.

RULING:
No. Self-defense requires that there be (1) an unlawful aggression by the person injured or killed
by the offender, (2) reasonable necessity of the means employed to prevent or repel that unlawful
aggression, and (3) lack of sufficient provocation on the part of the person defending himself. All
these conditions must concur.

128
CRIMINAL LAW 1 CASE DIGESTS
ATTY. LYAN DAVD JUANICO
BLOCK 1-M A.Y. 2020 - 2021

True, the victim barged into the house of accused-appellant and his live-in partner and, banging at
the master bedroom door with his firearm, he yelled, “come out.” Accused-appellant, however,
upon opening the door and seeing the victim pointing a gun at him, was able to prevent at this
stage harm to himself by promptly closing the door. He could have stopped there. Instead, accused-
appellant, taking his .38 caliber revolver, again opened the bedroom door and, brandishing his own
firearm, forthwith confronted the victim. At this encounter, accused-appellant would be quite hard
put to still claim self-defense.

The second element of self-defense would demand that the means employed to quell the unlawful
aggression were reasonable and necessary. The number of the wounds sustained by the deceased
in this case would negate the existence of this indispensable component of self-defense. The
autopsy report would show that the victim sustained four gunshot wounds which would, in fact,
indicate a determined effort to kill.

129
CRIMINAL LAW 1 CASE DIGESTS
ATTY. LYAN DAVD JUANICO
BLOCK 1-M A.Y. 2020 - 2021

5. People v. Jaurigue
G.R. No. 384; February 21, 1946
DE JOYA, J.:

FACTS:
Appellant Avelina Jaurigue and the deceased Amado Capina lived in the barrio of Sta. Isabel, San
Pablo City, Laguna. Sometime prior to the stabbing of the deceased, Amando Capina had been
courting Jaurigue in vain. On September 13, 1942, while Jaurigue was feeding a dog under her
house, Amado Capina approached her and spoke to her of his love, however the appellant flatly
refused, and he thereupon suddenly embraced and kissed her and touched her breasts. Avelina then
slapped Amado Capina, gave him fist blows and kicked him. Since then, she armed herself with a
long fan knife, whenever she went out, evidently for self-protection.

On September 15, 1942, about midnight, Amado climbed up the house of Jaurigue and entered the
room where she was sleeping. She immediately screamed for help, which awakened her parents
and brought them to her side. Nicolas Jaurigue sent for the barrio lieutenant and for Amado's
parents, the following morning. In the morning of September 20, 1942, Avelina received
information that Amado had been falsely boasting in the neighborhood of having taken liberties
with her person and that she had even asked him to elope with her and that if he should not marry
her, she would take poison; and that Avelina again received information of Amado's bragging at
about 5 o'clock in the afternoon of that same day.

At about 8 o'clock in the evening of September 20, 1942, Nicolas Jaurigue went to the chapel of
the Seventh Day Adventists of which he was the treasurer to attend religious services and sat on
the front bench facing the altar with the other officials of the organization and the barrio lieutenant.
Inside the chapel it was quite bright as there were electric lights. Shortly after the arrival of her
father, Avelina Jaurigue entered the chapel, also for the purpose of attending religious services,
and sat on the bench next to the last one nearest the door. Amado Capina was seated on the other
side of the chapel. Upon observing the presence of Avelina Jaurigue, Amado Capina went to the
bench on which Avelina was sitting and sat by her right side, and, without saying a word, Amado,
with the greatest of impudence, placed his hand on the upper part of her right thigh. Then, Avelina
Jaurigue pulled out with her right hand the fan knife which she had in a pocket of her dress, with
the intention of punishing Amado's offending hand. She stabbed Amado once at the base of the
left side of the neck, inflicting upon him a wound about 4 1/2 inches deep, which was necessarily
mortal. Amado Capina died from the wound a few minutes later.

The RTC found Avelina Jaurigue guilty of homicide for the death of Amando Capina.

130
CRIMINAL LAW 1 CASE DIGESTS
ATTY. LYAN DAVD JUANICO
BLOCK 1-M A.Y. 2020 - 2021

ISSUES:
1. Whether or not the appellant had acted in legitimate defense of her honor?
2. Whether or not there are mitigating circumstances that should be appreciated in the case?
3. Whether or not the aggravating circumstance that the crime be committed in a place dedicated to
religious worship should be appreciated?

RULINGS:
1. No, the appellant Avelina Jaurigue did not act in self-defense. As long as there is actual danger of
being raped, a woman is justified in killing her aggressor, in the defense of her honor. However,
in the instant case, when the deceased sat by the side of defendant and appellant on the same bench,
near the door of the barrio chapel and placed his hand on the upper portion of her right thigh,
without her consent, the said chapel was lighted with electric lights, and there were already several
people inside the chapel, including her own father and the barrio lieutenant. Under the
circumstances, there was and there could be no possibility of her being raped. And when she gave
Amado Capina a thrust at the base of the left side of his neck, inflicting upon him a mortal wound
4 1/2 inches deep, causing his death a few moments later, the means employed by her in the defense
of her honor was evidently excessive, thus she cannot be legally declared completely exempt from
criminal liability.

2. Yes, there are mitigating circumstances present in the crime. The appellant immediately
and voluntarily surrendered to the barrio lieutenant in said chapel, admitting having stabbed the
deceased, immediately after the incident, and agreed to go to her house shortly thereafter and to
remain there subject to the order of the said barrio lieutenant, an agent of the authorities and the
further fact that she had acted in the immediate vindication of a grave offense committed against
her a few moments before, and upon such provocation as to produce passion and obfuscation, or
temporary loss of reason and self-control, should be considered as mitigating circumstances in her
favor. Appellant further claims that she had not intended to kill the deceased but merely wanted
to punish his offending hand with her knife, as shown by the fact that she inflicted upon him only
one single wound.

3. No, the aggravating circumstance that the killing was done in a place dedicated to religious
worship should not be appreciated. There is no evidence to show that the defendant and appellant
had murder in her heart when she entered the chapel that fatal night. She only happened to kill the
victim under the greatest provocation against her honor. Appellant is a God-fearing young woman,
typical of our country girls, who still possess the consolation of religious hope in a world where
so many others have hopelessly lost the faith of their elders and now drifting away, they know not
where.

131
CRIMINAL LAW 1 CASE DIGESTS
ATTY. LYAN DAVD JUANICO
BLOCK 1-M A.Y. 2020 - 2021

6. People v. Narvaez
G.R. No. L-33466-67, April 20, 1983
MAKASIAR, J.:

FACTS:

In the afternoon of August 22, 1968, Graciano Juan, Jesus Verano and Cesar Ibanez together with
the two deceased Davis Fleischer and Flaviano Rubia, were fencing the land of George Fleischer,
father of deceased Davis Fleischer. At that time, appellant was taking his rest, but when he heard
that the walls of his house were being chiseled, he arose and there he saw the fencing going on. At
the place of the fencing is the house and rice drier of appellant Mamerto Narvaez. Hence, if the
fencing would go on, appellant would be prevented from getting into his house and the bodega of
his rice mill. So, he then addressed the group, saying 'Pare, if possible, you stop destroying my
house and if possible, we will talk it over what is good,' addressing the deceased Rubia, who is
appellant's compadre. The deceased Fleischer, however, answered: 'No, gademit, proceed, go
ahead.' Appellant apparently lost his equilibrium, and he got his gun and shot Fleischer, hitting
him. As Fleischer fell down, Rubia ran towards the jeep, and knowing there was a gun on the jeep,
appellant fired at Rubia, likewise hitting him. Both Fleischer and Rubia died as a result of the
shooting incident.

Prior to the shooting, Fleischer and Co. (the company of Fleischer's family) was involved in a legal
battle with the defendant and other land settlers of Cotabato over certain pieces of property.
Defendant had leased his property from Fleisher to avoid trouble since the civil case was still
pending and the ownership of the said property is still uncertain. On June 25, the defendant
received a letter from Fleischer and Co., terminating the contract because he allegedly did not pay
his rent and was given 6 months to remove his house from the land.

The RTC convicted the appellant with the crime of murder qualified by treachery with the
aggravating circumstance of evident premeditation offset by the mitigating circumstance of
voluntary surrender. On appeal, the defendant claims he killed in defense of his person and
property.

ISSUES:
1. Whether or not the appellant acted in defense of his person or rights under Art. 11(1).
2. Whether or not there are mitigating and aggravating circumstances that should be appreciated in
the case.

132
CRIMINAL LAW 1 CASE DIGESTS
ATTY. LYAN DAVD JUANICO
BLOCK 1-M A.Y. 2020 - 2021

RULINGS:
1. No. In order to appreciate the justifying circumstance under Art. 11(1) of the Revised Penal Code,
the following requisites must occur: First. Unlawful aggression; Second. Reasonable necessity of
the means employed to prevent or repel it; Third. Lack of sufficient provocation on the part of the
person defending himself. In the case at the bar, the reasonableness of the resistance was
disproportionate to the attack when the appellant fired his shotgun from his window, killing his
two victims. Thus, the appellant's act in killing the deceased was not justifiable, since not all the
elements for justification are present. He should therefore be held responsible for the death of his
victims.

2. Yes, he could be credited with the special mitigating circumstance of incomplete defense
and the ordinary mitigating circumstance of passion and obfuscation. The appellant awoke to find
his house being damaged and its accessibility to the highway as well as of his rice mill bodega
being closed. Not only was his house being unlawfully violated; his business was also in danger
of closing down for lack of access to the highway.

Consequently, there are no aggravating circumstances to be appreciated in the case at the bar.
Thus, the appellant is guilty of two crimes of homicide only since the killing not being attended
by any qualifying nor aggravating circumstance. The qualifying circumstance of treachery cannot
be appreciated in this case because of the presence of provocation on the part of the deceased and
in order to appreciate treachery, "it must clearly appear that the method of assault adopted by the
aggressor was deliberately chosen with a special view to the accomplishment of the act without
risk to the assailant from any defense that the party assailed might have made. Moreover, the
qualifying circumstance of evident premeditation is not sufficiently established. This single
evidence is not sufficient to warrant appreciation of the aggravating circumstance of evident
premeditation. As the Court consistently held, there must be a "showing" that the accused
premeditated the killing; that the culprit clung to their (his) premeditated act; and that there was
sufficient interval between the premeditation and the execution of the crime to allow them (him)
to reflect upon the consequences of the act.

133
CRIMINAL LAW 1 CASE DIGESTS
ATTY. LYAN DAVD JUANICO
BLOCK 1-M A.Y. 2020 - 2021

7. Sabang v. People
G.R. No. 168818, March 9, 2007
TINGA, J.:

FACTS:
On January 17, 1997, petitioner and Butad were having drinks together with spouses Cruz and
Andresa Villamor outside the store of Melania Sombilon in Sitio Landing, Barangay Liloan,
Ormoc City. Butad, a civilian agent with the Philippine National Police, was then armed with a
.38-caliber revolver which was tucked in his holster. In the midst of the drinking spree, Randy
Sabang suddenly and unexpectedly appeared before the group. His appearance triggered a negative
reaction from Butad, who then uttered the words ``I will shoot you" to Randy Sabang. Certain
circumstances attaching to this evident threat are disputed, as are the events that consequently
followed. What is certain is that shortly afterwards, Butad lay dead, having sustained four (4)
gunshot wounds from his own revolver. Petitioner appears to have fled but voluntarily surrendered
thereafter, turning over the revolver as he surrendered.

During arraignment, the petitioner pleaded innocence, but during the presentation of the evidence
for the defense, he claimed to have acted in defense of a relative. Petitioner testified that by the
time Butad had joined what was to be his last drinking spree, he was already in a belligerent mood.
Earlier that afternoon, he had been chasing after Ramil Perez when the latter demanded payment
for a bet Butad had lost over a cockfight. Witness Sombilon further testified that when Butad told
Randy Sabang, "I will shoot you," the deceased already had his revolver aimed at Randy. Petitioner
claimed that he then grabbed the arm of Butad, attempting to twist it toward his body and away
from his son. As they were grappling and the revolver was pointed towards the body of Butad, the
petitioner claimed he heard gunshots, and only after the shots were fired was he able to "take the
gun" from Butad.

The trial court convicted petitioner because of the absence of powder burns that indicates that the
gunshots were fired at a distance of more than 10 inches from the victim’s body and not close
range as claimed by petitioner.

ISSUE:
Whether or not the petitioner acted in defense of his relatives under Art. 11 of the Revised Penal
Code.

134
CRIMINAL LAW 1 CASE DIGESTS
ATTY. LYAN DAVD JUANICO
BLOCK 1-M A.Y. 2020 - 2021

RULING:

No. In order to successfully claim that he acted in defense of a relative, the accused must
prove the concurrence of the following requisites: (1) unlawful aggression on the part of the person
killed or injured; (2) reasonable necessity of the means employed to prevent or repel the unlawful
aggression; and (3) the person defending the relative had no part in provoking the assailant, should
any provocation have been given by the relative attacked.

Unlawful aggression is a primary and indispensable requisite without which defense of


relative, whether complete or otherwise, cannot be validly invoked. It is well-settled in this
jurisdiction that once an accused has admitted that he inflicted the fatal injuries on the deceased,
it is incumbent upon him in order to avoid criminal liability, to prove the justifying circumstance
claimed by him with clear, satisfactory and convincing evidence. Unlawful aggression must be
clearly established by the evidence. In this case, there is a divergence in the testimonies of the
prosecution and defense witnesses as to whether Butad aimed a gun at the petitioner's son as he
uttered the words "I will shoot you." With this conflict emerges the question of whether the
petitioner sensed an imminent threat to his son’s life. Payud unequivocally testified that the
petitioner even dismissed Butad’s utterance saying, "Just try to shoot my child because I’ll never
fight for him because he is a spoiled brat." This indicates to us that the petitioner did not consider
Butad’s words a threat at all. These circumstances clearly show there was no unlawful aggression
on the part of Butad which could have precipitated the petitioner's actions.

135
CRIMINAL LAW 1 CASE DIGESTS
ATTY. LYAN DAVD JUANICO
BLOCK 1-M A.Y. 2020 - 2021

8. People v. Dagani
G.R. No. 153875, August 16, 2006
AUSTRIA-MARTINEZ, J.:

FACTS:
On or about September 11, 1989, a group composed of Ernesto Javier, Lincoln Miran, and two
other individuals had been drinking at the canteen located inside the compound of the Philippine
National Railways (PNR) along C.M. Recto Avenue, Tondo, Manila. All of a sudden, appellants,
who were security officers of the PNR and covered by the Civil Service Rules and Regulations,
entered the canteen and approached the group. Appellant Dagani shoved Miran, causing the latter
to fall from his chair. Dagani then held Javier while Santiano shot Javier twice at his left side,
killing the latter.

Appellants invoked the justifying circumstances of self-defense and lawful performance of official
duty as PNR security officers. They testified that they were ordered by their desk officer to
investigate a commotion at the canteen. Upon reaching the place, Santiano ordered his co-accused,
Dagani, to enter, while the former waited outside. Dagani approached Javier who had been striking
a bottle of beer on the table. Javier then pulled out a .22 caliber revolver and attempted to fire at
Dagani, but the gun failed to go off. Then suddenly, while outside the canteen, Santiano heard
gunfire and, from his vantage point, he saw Javier and Dagani grappling for a .22 caliber gun which
belonged to Javier. During the course of the struggle, the gun went off, forcing Santiano to fire a
warning shot. He heard Javier’s gun fire again, so he decided to rush into the canteen. Santiano
then shot Javier from a distance of less than four meters.

The RTC convicted both accused of the crime of murder qualified by treachery. They also believe
that there is conspiracy in the case at the bar. The CA affirmed the decision of the RTC.

ISSUES:
1. Whether or not there was unlawful aggression on the part of the victim that would justify his
criminal liability.
2. Whether or not the appellants act in the fulfillment of a duty or in a lawful exercise of a right or
office.

RULING:
1. No. When self-defense is invoked, the burden of evidence shifts to the accused to show that the
killing was legally justified. Having owned the killing of the victim, the accused should be able to
prove to the satisfaction of the Court the elements of self-defense in order to avail of this
extenuating circumstance. He must discharge this burden by clear and convincing evidence. To

136
CRIMINAL LAW 1 CASE DIGESTS
ATTY. LYAN DAVD JUANICO
BLOCK 1-M A.Y. 2020 - 2021

2. invoke the justifying circumstance of self-defense, it requires that there be (1) an unlawful
aggression by the person injured or killed by the offender, (2) reasonable necessity of the means
employed to prevent or repel that unlawful aggression, and (3) lack of sufficient provocation on
the part of the person defending himself.

Unlawful aggression, a primordial element of self-defense, would presuppose an actual,


sudden and unexpected attack or imminent danger on the life and limb of a person – not a mere
threatening or intimidating attitude but most importantly, at the time the defensive action was taken
against the aggressor. To invoke self-defense successfully, there must have been an unlawful and
unprovoked attack that endangered the life of the accused, who was then forced to inflict severe
wounds upon the assailant by employing reasonable means to resist the attack. In the instant case,
the assertions that it was "quite probable" that Javier, during the course of the struggle for the
firearm, "could have easily killed" the appellants are uncertain and speculative. There is aggression
in contemplation of the law only when the one attacked face real and immediate threat to one’s
life. The peril sought to be avoided must be imminent and actual, not just speculative.

2. No. Article 11 of the Revised Penal Code provides that a person who acts in the fulfillment
of a duty or in the lawful exercise of a right or office does not incur any criminal liability. To
invoke this justifying circumstance, two requisites must concur: (1) the accused must have acted
in the performance of a duty or in the lawful exercise of a right or office; and (2) the injury caused,
or the offense committed should have been the necessary consequence of such lawful exercise.
However, in the instant case, these requisites are absent.

The defense failed to prove that the security officers were in fact on duty at the time they were
at the canteen. The trial court gave weight to the fact that the appellants were unable to submit
their daily time records to show that they were on duty at the time. Appellants’ assertion that they
were ordered to go on 24-hour duty was belied by PNR Security Investigator Rolando Marinay’s
testimony that PNR security officers work in two 12-hour shifts, from 7:00 a.m. to 7:00 p.m. and
from 7:00 p.m. to 7:00 a.m. As stated, considering that the imminent or actual danger to the life of
the appellants had been neutralized when Dagani grappled with Javier and restrained his hands;
that Javier had been thrown off-balance; that Dagani had been specially trained for these purposes;
and that Javier had been drinking immediately prior to the scuffle, this Court holds that the fatal
injuries that appellant Santiano inflicted on the victim cannot be deemed to be necessary
consequences of the performance of his duty as a PNR security officer.

137
CRIMINAL LAW 1 CASE DIGESTS
ATTY. LYAN DAVD JUANICO
BLOCK 1-M A.Y. 2020 - 2021

9. Palaganas v. People
G.R. No. 165483, September 12, 2006
CHICO-NAZARIO, J.:

FACTS:
On January 16, 1998, around 8:00 in the evening, the Ferrer brothers were having a drinking spree
in their house because Melton Ferrer, who was already living in San Fernando, La Union, visited
his three brothers and mother at their house in Sitio Baloking, Poblacion, Manaoag, Pangasinan.

On that same evening, the three brothers decided to proceed to Tidbits Videoke bar to continue
their drinking spree and to sing. Inside the karaoke bar, they were having a good time, singing and
drinking beer. Thereafter, Jaime Palaganas arrived together with Ferdinand Palaganas and Virgilio
Bautista. At that time, only the Ferrer brothers were the customers in the bar. The two groups
occupied separate tables. Later, when Jaime Palaganas was singing, Melton Ferrer sang along with
him as he was familiar with the song My Way. Jaime however, resented this and went near the
table of the Ferrer brothers and said in Pangasinan dialect "As if you are tough guys." Jaime further
said, "You are already insulting me in that way." Then, Jaime struck Servillano Ferrer with the
microphone, hitting the back of his head. A rumble ensued between the Ferrer brothers on the one
hand, and the Palaganases, on the other hand. During the rumble, Ferdinand went out of the bar.
He was however pursued by Michael. When Servillano saw Michael, he also went out and told the
latter not to follow Ferdinand. Servillano and Michael then went back inside the bar and continued
their fight with Jaime.

After they were pacified by the owner of the bar, Servillano noticed that his wristwatch was
missing. Unable to locate the watch inside the bar, the Ferrer brothers went outside. They saw
Ferdinand about eight (8) meters away standing at Rizal Street. Ferdinand was pointing at them
and said to his companion, the herein petitioner Rujjeric Palaganas, "Oraratan paltog mo lara",
meaning "They are the ones, shoot them." Petitioner then shot them hitting Servillano Ferrer first
causing him to fall on the ground and followed by Melton Ferrer who also fell to the ground. When
Servillano noticed that Melton was no longer moving, he told Michael "Bato, bato." Michael
picked up some stones and threw them at petitioner and Ferdinand. The latter then left the place.
Servillano later discovered that [Melton] was fatally hit in the head while Michael was hit in the
right shoulder.

The trial court rendered its Decision finding petitioner guilty only of the crime of Homicide and
two (2) counts of Frustrated Homicide. It also stated that petitioner cannot successfully invoke
self-defense since there was no actual or imminent danger to his life at the time, he and Ferdinand
saw the Ferrer brothers outside the videoke bar.

138
CRIMINAL LAW 1 CASE DIGESTS
ATTY. LYAN DAVD JUANICO
BLOCK 1-M A.Y. 2020 - 2021

ISSUE:
Whether or not the Court of Appeals erred in not appreciating the justifying circumstance of self-
defense.

RULING:
No. Article 11, paragraph 1 of the Revised Penal Code provides for the elements and/or requisites
in order that a plea of self-defense may be validly considered in absolving a person from criminal
liability: (1) Unlawful aggression; (2) Reasonable necessity of the means employed to prevent or
repel it; (3) Lack of sufficient provocation on the part of the person defending himself.

As an element of self-defense, unlawful aggression refers to an assault or attack, or a threat thereof


in an imminent and immediate manner, which places the defendant's life in actual peril. It is an act
positively strong showing the wrongful intent of the aggressor and not merely a threatening or
intimidating attitude. It is also described as a sudden and unprovoked attack of immediate and
imminent kind to the life, safety or rights of the person attacked. There must be actual physical
force or actual use of a weapon. In order to constitute unlawful aggression, the person attacked
must be confronted by a real threat on his life and limb; and the peril sought to be avoided is
imminent and actual, not merely imaginary.

In the case at the bar, it is clear that there was no unlawful aggression on the part of the Ferrer
brothers that justified the act of petitioner in shooting them. There was no actual or imminent
danger to the lives of petitioner and Ferdinand when they proceeded and arrived at the videoke bar
and saw there at the Ferrer brothers. It appears that the Ferrer brothers then were merely standing
outside the videoke bar and were not carrying any weapon when the petitioner arrived with his
brother Ferdinand and started firing his gun. Unlawful aggression is a primordial element in self-
defense, for without unlawful aggression on the part of the victim, there can be no complete or
incomplete self-defense.

139
CRIMINAL LAW 1 CASE DIGESTS
ATTY. LYAN DAVD JUANICO
BLOCK 1-M A.Y. 2020 - 2021

10. People v. Ricohermoso


G.R. No. L-30527-28, 29 March 1974
AQUINO, J.:

FACTS:
In the morning of January 30, 1965, Geminiano de Leon, together with his common-law wife
Fabiana Rosales, his son Marianito de Leon and one Rizal Rosales, encountered Pio Ricohermoso
in Barrio Tagbacan Silangan, Catanauan, Quezon. Geminiano owned a parcel of land in that barrio
which Ricohermoso cultivated as kaingin. Geminiano asked Ricohermoso about his share of the
palay harvest. Ricohermoso answered that Geminiano could go to his house anytime and he would
give the latter palay. Geminiano rejoined that he could not get the palay that morning because he
was on his way to Barrio Bagobasin but, on his return, he would stop at Ricohermoso's house and
get the palay.

At about two o'clock in the afternoon of the same day, when Geminiano returned to Barrio
Tagbacan Silangan, he stopped at Ricohermoso's place. Geminiano asked Ricohermoso about the
palay. However, Ricohermoso answered in a defiant tone said that whatever happens, he will not
give palay to Germiniano. Geminiano answered back, asking why did Ricohermoso tell them to
pass by his house if he was not willing to give the palay. In that moment, Ricohermoso unsheathed
his bolo and approached Geminiano and stabbed him on the neck with his bolo. Geminiano fell
face downward on the ground. While in that helpless position, he was hacked on the back with an
axe by Severo Padernal. At that same place and time, while Severo Padernal and Ricohermoso
were assaulting Geminiano de Leon, Juan Padernal suddenly embraced Marianito de Leon from
behind, with his right arm locked around Marianito's neck and his left hand pressing Marianito's
left forearm. They grappled and rolled downhill towards a camote patch. Marianito passed out.
When he regained consciousness, his rifle was gone. He walked uphill, saw his mortally wounded
father Geminiano in his death throes, and embraced him. He carried Geminiano for a short
distance. The 55-year-old Geminiano died at two o'clock on that same day.

Appellants' contend is that in the afternoon of January 30, 1965, when Ricohermoso refused to
give any palay to Geminiano de Leon, because the land tilled by the former was allegedly a public
land, Geminiano approached Ricohermoso. When Geminiano unsheathed his bolo, Ricohermoso
met him, drew his bolo and struck Geminiano on the left side of the neck. The latter tried to parry
the blow and was wounded in the wrist. As Geminiano turned right to flee, Ricohermoso struck
him again on the left side of his body, causing him to fall on the ground. Geminiano died on the
spot due to the bleeding from the wound on his neck. While Geminiano was being assaulted, his
son Marianito tried to shoot with his rifle, but Juan Padernal disabled him and wrested the gun.
Marianito suffered abrasions on the neck and other parts of the body.

140
CRIMINAL LAW 1 CASE DIGESTS
ATTY. LYAN DAVD JUANICO
BLOCK 1-M A.Y. 2020 - 2021

The Circuit Criminal Court at Lucena City convicted the accused of murder. On appeal, appellant
Juan Padernal invokes the justifying circumstance of avoidance of a greater evil or injury in
explaining his act of preventing Marianito de Leon from shooting Ricohermoso and Severo
Padernal. His reliance on that justifying circumstance is erroneous. The act of Juan Padernal in
preventing Marianito de Leon from shooting Ricohermoso and Severo Padernal, who were the
aggressors, was designed to insure the killing of Geminiano de Leon without any risk to his
assailants.

ISSUE:
Whether or not the appellant Juan Padernal can invoke the justifying circumstance of avoidance
of greater evil or injury under Article 11(4).

RULING:
No, Juan Padernal was not avoiding any evil when he sought to disable Marianito. Padernal's
malicious intention was to forestall any interference in the felonious assault made by his father and
brother-in-law on Geminiano. Considering the trio's behavior and appellant Juan Padernal's close
relationship to Ricohermoso and Severo Padernal, the ineluctable conclusion is that he acted in
conspiracy with them. It should be recalled that, in the morning, Geminiano had an understanding
with Ricohermoso that Germiniano would return in the afternoon to get his share of the palay
harvest. Ricohermoso gave Geminiano the impression that he (Ricohermoso) was amenable to
giving Geminiano his share of the harvest. However, during the interval, Ricohermoso changed
his mind. Instead of remaining steadfast to his original intention to give Geminiano palay,
Ricohermoso planned with his father-in-law, Severo Padernal, and his brother-in-law, appellant
Juan Padernal, the manner of liquidating Geminiano as to stop him from pestering Ricohermoso
with demands for a share in the harvest. So, when Geminiano reappeared at Ricohermoso's place
in the afternoon, Severo Padernal, Ricohermoso Juan Padernal, like actors in a well-rehearsed play,
performed their assigned roles with dramatic precision. Severo Padernal and Ricohermoso, one
armed with an axe and the other with a bolo, in a pincer movement, confronted Geminiano de
Leon. Simultaneously with that maneuver, the thirty-five-year-old Juan Padernal embraced
Marianito de Leon and prevented him from firing at Severo Padernal and Ricohermoso or from
helping his father. He coordinated and timed his seizure of Marianito with the assault of
Ricohermoso and Severo Padernal on Geminiano. It is doubtful if the assailants could have
consummated the killing of Geminiano, without their suffering any injury, if Marianito had not
been rendered helpless by appellant Juan Padernal.

141
CRIMINAL LAW 1 CASE DIGESTS
ATTY. LYAN DAVD JUANICO
BLOCK 1-M A.Y. 2020 - 2021

11. People v. Norma Hernandez


CA-G.R. No. 22553-R, April 14, 1959
CORONA, J.:

FACTS:
Sometime in August 1954, complainant Vivencio Lascano started courting appellant Maria Norma
Hernandez. In January 1955, appellant finally accepted Vivencio. On the same date, they talked
about their marriage. Consequently, in February 1955, the complainant's parents together with his
twelve aunts, bringing along about thirty chickens and three goats, went to the appellant's house
to ask for her hand in marriage. The parents of both parties agreed to the marriage of appellant and
complainant Vivencio and set March 19, 1955 as the day of the wedding. They also agreed that
Vivencio would buy the wedding dress, two vestidos, a pair of shoes for the bride, to advance P20
for fetching the sponsors in the wedding and to repay the roof of one Feliciano Martinez’ house,
an uncle of the appellant.

As the date of the wedding was approaching, appellant and Vivencio, together with their parents
filed their application for marriage and were issued their corresponding marriage license. They
also went to the parish priest to arrange the proclamation and to Isidora Lascano to order the
appellant's wedding gown. On March 16, 1955, the gown was delivered by Isidora to the
appellant’s house, however since no one was there and the house was closed, the gown was just
left in the balcony. On March 18 and 19, the parents of Vivencio cleaned the yard of the appellant’s
house and constructed a temporary shed where the wedding feast was to be held.

On the morning of March 19, the appellant was nowhere to be found. Vivencio and his parents
waited for her until twelve midnight of March 19, but the appellant never showed up thus causing
them great shame and humiliation.

Appellant testified that Vivencio was really courting her however she was not in love with him.
Her parents, however, tried to persuade her to accept Vivencio’s proposal of marriage. As an
obedient child, she accepted Vivencio’s love although she felt no love for him. On the day of the
wedding, she felt a sense of torture because she was not honestly in love with Vicencio, so she
decided to leave her home as a last recourse in order to prevent the marriage believing that if
anyone would be humiliated by the failure of the marriage, it would be she being the girl and not
Vivencio. So on March 11, 1955, she left for Mindoro and stayed there with her cousin and
remained there until April 1955 when she was already under arrest on account of this present case.

The Court of First Instance of Batangas convicted appellant Maria Norma Hernandez with the
crime of serious slander by deed.

142
CRIMINAL LAW 1 CASE DIGESTS
ATTY. LYAN DAVD JUANICO
BLOCK 1-M A.Y. 2020 - 2021

ISSUE:
Whether or not the appellant should be acquitted pursuant to Article 11(4) of the Revised Penal
Code.

RULING:
Yes. Under Article 11(4) of the Revised Penal Code, any person who, in order to avoid an evil or
injury, does not act which causes damage to another, provided that the following requisites are
present: First. That the evil sought to be avoided actually exists; Second. That the injury feared be
greater than that done to avoid it; Third. That there be no other practical and less harmful means
of preventing it. In the case at the bar, the appellant had the right to avoid to herself the evil of
going through a loveless marriage.

A party to an agreement to marry who backs out cannot be held liable for the crime of slander by
deed for then, that would be an inherent way of compelling said party to enter into a marriage
without his or her consent, and this would contravene the principle of law that what could not be
done directly could not be done indirectly. Furthermore, one of the essential requisites of slander,
which is malice, has not been proven. The act of the appellant in changing her mind does not have
malice. She was merely exercising her right not to give consent to the marriage after mature
consideration.

143
CRIMINAL LAW 1 CASE DIGESTS
ATTY. LYAN DAVD JUANICO
BLOCK 1-M A.Y. 2020 - 2021

12. PEOPLE VS. FELIPE DELIMA


G.R. No. L-18660; December 22, 1922
ROMUALDEZ, J.:

FACTS:
Lorenzo Napilon had escaped from the jail where he was serving sentence. Some days afterwards,
the policeman Felipe Delima, who was looking for him, found him armed with a pointed piece of
bamboo in the shape of a lance, and demanded his surrender. Napilon answered with a stroke of
his lance. The policeman dodged, it, and to impose his authority fired his revolver, but the bullet
did not hit him. The criminal ran away, without parting with his weapon. These peace officer went
after him and fired again his revolver, this time hitting and killing him. Delima was convicted with
Homicide. He appeals that the conviction must be reversed because the killing was done in
performance of a duty.

ISSUE:
Whether or not the act of Delima was done in performance of a duty which is a justifying
circumstance.

RULING:
YES. The act of Delima was done in performance of a duty which is a justifying circumstance.
Under Article 11, par. 5 of the Revised Penal Code, any person who acts in the fulfillment of a
duty or in the lawful exercise of a right or office, do not incur any criminal liability.
In this case, the deceased was under the obligation to surrender, and had no right, after evading
service of his sentence, to commit assault and disobedience with a weapon in the hand, which
compelled the policeman to resort to such an extreme means, which, although it proved to be fatal,
was justified by the circumstances. Hence, Delima do not incur any criminal liability.

144
CRIMINAL LAW 1 CASE DIGESTS
ATTY. LYAN DAVD JUANICO
BLOCK 1-M A.Y. 2020 - 2021

13. PEOPLE VS. IGNACIO LAGATA


G.R. Nos. L-1940-42; March 24, 1949
PERFECTO, J.:
FACTS:
Ignacio Lagata was a provincial guard in which six prisoners were then assigned to work in the
capitol's plaza namely Jesus, Tipace, Eusebio, Mariano, Eustaquio and Labong. Lagata ordered
them to go to the nursery to pick up gabi. Not long afterwards they were called to assemble where
they discovered that Labong was missing. Lagata ordered the five prisoners to call Labong. As
Labong did not answer their call Lagata ordered the five to look farther for him. The five prisoners
went towards the mountain. While the five prisoners were on the camote plantation, Lagata
suddenly shot Eusebio in the arm which caused the other prisoners to panic and run. Lagata
subsequently shot Tipace which killed the latter immediately. Lagata was convicted for murder,
serious physical injuries and evasion through negligence. For Lagata’s defense, he stated that when
he and the five prisoners went to the camote plantation, he ordered them to stop because they were
already far from him. When the prisoners did not heed the order, Lagata fired in the air as a warning
shot. When he saw that the prisoners were running, he shot Tipace, because he claims that the
latter is running towards him, and Eusebrio, who was only 4 meters away, so that they won’t
escape. He argues that he acted in the fulfillment of a duty which is a justifying circumstance.
ISSUE:
Whether or not the acts of Lagata were done in the performance of a duty which is a justifying
circumstance.
RULING:
NO. The acts of Lagata were not done in the performance of a duty which is a justifying
circumstance.
Under Article 11, par. 5 of the Revised Penal Code, any person who acts in the fulfillment of a
duty or in the lawful exercise of a right or office does not incur any criminal liability. The requisites
for this justifying circumstance are a) that the offender acted in the performance of a duty; and b)
that the injury caused is the necessary consequence of the due performance.
In this case, Lagata acted in the performance of his duty as a provincial guard however, there was
no necessity for him to fire directly against the prisoners so as seriously wound one of them and
kill instantaneously another. Since only one requisite of Art. 11, par. 5 is present, which is the
performance of a duty, Lagata was granted the mitigating circumstance of incomplete justifying
circumstance of fulfillment of a duty.

145
CRIMINAL LAW 1 CASE DIGESTS
ATTY. LYAN DAVD JUANICO
BLOCK 1-M A.Y. 2020 - 2021

14. PEOPLE VS. ROLANDO DAGANI AND OTELLO SANTIANO


G.R. No. 153875; August 16, 2006
AUSTRIA-MARTINEZ, J.:

FACTS:
The appellants, who were security officers of the PNR, was ordered by their desk officer
to investigate a commotion at the canteen located inside the compound of the Philippine National
Railways. Upon reaching the place, Santiano ordered his co-accused, Dagani, to enter, while the
former waited outside. Dagani approached Javier who had been striking a bottle of beer on the
table. Javier then pulled out a revolver and attempted to fire at Dagani, but the gun failed to go off.
Then suddenly, while outside the canteen, Santiano heard gunfire and, from his vantage point, he
saw Javier and Dagani grappling for a gun which belonged to Javier. During the course of the
struggle, the gun went off, forcing Santiano to fire a warning shot. He heard Javier’s gun fire again,
so he decided to rush into the canteen. Santiano then shot Javier from a distance of less than four
meters. The appellants were convicted of Murder by the trial court which was affirmed by the
Court of Appeal. The appellants now claim that the lower courts failed to appreciate the justifying
circumstances of self-defense and fulfillment of a duty.
ISSUE:
Whether or not the justifying circumstance of Self-Defense is present in this case.
Whether or not the justifying circumstance of Fulfillment of a Duty is present in this case.
RULING:
Self-Defense
NO. The justifying circumstance of Self-Defense is not present in this case. To invoke self-defense
successfully, there must have been an unlawful aggression or an unprovoked attack that
endangered the life of the accused, who was then forced to inflict severe wounds upon the assailant
by employing reasonable means to resist the attack. In this case, the imminence of the danger to
their lives had already ceased the moment Dagani held down the victim and grappled for the gun
with the latter. After the victim had been thrown off-balance, there was no longer any unlawful
aggression that would have necessitated the act of killing. Hence, there is no complete self-
defense.

146
CRIMINAL LAW 1 CASE DIGESTS
ATTY. LYAN DAVD JUANICO
BLOCK 1-M A.Y. 2020 - 2021

Fulfillment of a Duty

NO. The justifying circumstance of Fulfillment of a Duty is not present in this case. Article
11 of the Revised Penal Code provides that a person who acts in the fulfillment of a duty or in
the lawful exercise of a right or office does not incur any criminal liability. Two requisites must
concur before this defense can prosper: 1) the accused must have acted in the performance of a
duty or in the lawful exercise of a right or office; and 2) the injury caused or the offense committed
should have been the necessary consequence of such lawful exercise. These requisites are absent
in the instant case. It was proved that Dagani and Santiano were not on duty when the incident
occurred. Second, the fatal injuries that appellant Santiano inflicted on the victim cannot be
deemed to be necessary consequences of the performance of his duty as a PNR security officer.
Hence, there is no justifying circumstance of fulfillment of a duty.

147
CRIMINAL LAW 1 CASE DIGESTS
ATTY. LYAN DAVD JUANICO
BLOCK 1-M A.Y. 2020 - 2021

15. RUFINO MAMANGUN VS. PEOPLE


G.R. No. 149152; February 2, 2007
GARCIA, J.:
FACTS:
On July 31, 1992, at about 8:00 in the evening, a certain Liberty Contreras was heard shouting,
"Magnanakaw Magnanakaw." Several residents responded and thereupon chased the suspect who
entered the yard of Antonio Abacan and proceeded to the rooftop of Abacan's house. At about 9:00
o'clock that same evening, the desk officer of the Meycauayan PNP Police Station, upon receiving
a telephone call that a robbery-holdup was in progress, immediately contacted and dispatched to
the scene the petitioner together with other policemen. With the permission of Abacan, petitioner
Mamangun went to the rooftop of the house whereat the suspect was allegedly taking refuge. They
saw a man whom they thought was the robbery suspect. At that instance, petitioner Mamangun,
who was walking ahead of the group, fired his handgun once, killing the man. The man turned out
to be Gener Contreras who was not the robbery suspect. According to Ayson, the eye witness of
the incident, Mamangun pointed his pistol at the man, who instantly exclaimed, "Hindi ako, hindi
ako!," to which Mamangun replied, "Anong hindi ako?" Before he could say anything, Mamangun
fired his gun, hitting the man who turned out to be Contreras. Mamangun was convicted of
Homicide. On appeal, he claims that the trial court failed to apply Art. 11, par. 5 of the Revised
Penal Code or the justifying circumstance of fulfillment of a duty.
ISSUE:
Whether or not the act of Mamangun was done in the fulfillment of a duty which is a justifying
circumstance.
RULING:
NO. The act of Mamangun was not done in the fulfillment of a duty which is a justifying
circumstance.
The justifying circumstance of fulfillment of duty under paragraph 5, Article II, of the Revised
Penal Code may be invoked only after the defense successfully proves that: (1) the accused acted
in the performance of a duty; and (2) the injury inflicted or offense committed is the necessary
consequence of the due performance or lawful exercise of such duty.
In this case, only the first requisite is present. Mamangun, a police officer, was responding to a
robbery-holdup incident. His presence at the situs of the crime was in accordance with the
performance of his duty. However, proof that the shooting and ultimate death of Contreras was a
necessary consequence of the due performance of his duty as a policeman is essential to exempt

148
CRIMINAL LAW 1 CASE DIGESTS
ATTY. LYAN DAVD JUANICO
BLOCK 1-M A.Y. 2020 - 2021

him from criminal liability. In the absence of the equally necessary justifying circumstance that
the injury or offense committed be the necessary consequence of the due performance of such
duty, there can only be incomplete justification, a privileged mitigating circumstance under
Articles 13 and 69 of the Revised Penal Code.

149
CRIMINAL LAW 1 CASE DIGESTS
ATTY. LYAN DAVD JUANICO
BLOCK 1-M A.Y. 2020 - 2021

16. PEOPLE VS. MANUEL BERONILLA


G.R. No. L-4445; February 28, 1955
REYES, J.B.L., J.:
FACTS:
On 1944, appellant Manuel Beronilla was appointed Military Mayor of La Paz by Lt. Col. R. H.
Arnold, regimental commander of the 15th Infantry, Philippine Army. On 1945, while the
operations for the liberation of the province of Abra were in progress, Arsenio Borjal returned to
La Paz with his family in order to escape the bombing of Bangued. Beronilla, pursuant to his
instructions fromm Arnold, placed Borjal under custody and asked the residents of La Paz to file
complaints against him. In no time, charges of espionage, aiding the enemy, and abuse of authority
were filed against Borjal. The trial was held in which the jury found Borjal guilty on all accounts
and imposed upon him instruction from his superiors. Mayor Beronilla forwarded the records of
the case to Lt. Col. R. H. Arnold. On the same day, Beronilla ordered the execution of Borjal. Two
years after, Beronilla was indicted in the Court of First Instance for murder, for allegedly
conspiring and confederating in the execution of Arsenio Borjal. The prosecution posted that Col.
Volckmann, the overall area commander, called the attention of Lt. Arnold, through a telegram,
stating that the Jury System made for the case of Borjal is Illegal. However, for some unknown
reason, the said telegram was not sent by Lt. Arnold to Mayor Beronilla. Beronilla now claims that
his act of ordering the execution of Borjal were done pursuant to express the orders of the 15th
Infantry Headquarters which is an instructions of a superior military authorities.
ISSUE:
Whether or not Beronilla acted in obedience to a lawful order of a superior which is a justifying
circumstance.
RULING:
YES. Beronilla acted in obedience to a lawful order of a superior which is a justifying
circumstance.
Under Art. 11, par. 6 of the Revised Penal Code, any person who acts in obedience to an order
issued by a superior for some lawful purpose.
It was clear that Beronilla did not received the Volkman Telegram when he ordered the execution
of Borjal. In this case Beronilla, acted upon orders, of a superior officer that they, as military
subordinates, could not question, and obeyed in good faith, without being aware of their illegality,
without any fault or negligence on their part, therefore, the criminal intent argued by the
prosecution was not established.

150
CRIMINAL LAW 1 CASE DIGESTS
ATTY. LYAN DAVD JUANICO
BLOCK 1-M A.Y. 2020 - 2021

17. LUIS TABUENA VS. SANDIGANBAYAN


G.R. Nos. 103501-03; February 17, 1997
FRANCISCO, J.:
FACTS:
Tabuena is the General Manager of the Manila International Airport Authority (MIAA). He
was instructed by President Marcos through a Memorandum (Marcos Memorandum) to pay
directly to the President’s Office what the MIAA owes the Philippine National Construction
Corporation (PNCC) in the amount of 55 million pesos. Tabuena complied with the said order of
President Marcos in which he delivered the 55 million pesos in cash to the President’s Secretary
with no receipt of the transaction. Afterwards, the record showed that the PNCC did not received
such payment. Tabuena was charged of the crime Malversation of Public Funds in the
Sandiganbayan. Tabuena claims that he only acted in good faith by following the order of
President Marco through the Marcos Memorandum.
ISSUE:
Whether or not Tabuena acted in obedience to a lawful order.
RULING:
YES. Tabuena acted in obedience to a lawful order.
Article 11, paragraph 6 of the Revised Penal Code provides that “any person who acts in
obedience to an order issued by a superior for some lawful purpose is a justifying circumstance”
In this case, the sole reason for the said disbursement of the 55 million is because of the said
Marcos Memorandum. Tabuena had no other choice but to make the withdrawals, for that was
what the MARCOS Memorandum required him to do. Marcos was undeniably Tabuena's superior
the former being then the President of the Republic who unquestionably exercised control over
government agencies such as the MIAA and PNCC.
Hence, Tabuena is entitled to the justifying circumstance of "Any person who acts in obedience to
an order issued by a superior for some lawful purpose."

151
CRIMINAL LAW 1 CASE DIGESTS
ATTY. LYAN DAVD JUANICO
BLOCK 1-M A.Y. 2020 - 2021

18. INTESTATE ESTATE OF MANOLITA GONZALES VS. PEOPLE


G.R. No. 181409; February 11, 2010
CORONA, J.:

FACTS:
Mediatrix G. Carungcong, in her capacity as the duly appointed administratrix of petitioner
intestate estate of her deceased mother Manolita Gonzales vda. de Carungcong, filed a complaint
for estafa against her brother-in-law, William Sato, a Japanese national. She claims that Sato,
through fraudulent misrepresentations, was able to secure the signature and thumbmark of my
mother on a Special Power of Attorney, giving the latter the power to dispose 4 pieces of land in
Tagaytay. Sato claims that that under Article 332 of the Revised Penal Code, his relationship to
the person allegedly defrauded, the deceased Manolita who was his mother-in-law, was an
exempting circumstance.

ISSUE:
Whether or not Article 332 of the Revised Penal Code is applicable in cases of Estafa through
Falsification.
RULING:
NO. Article 332 of the Revised Penal Code is not applicable in cases of Estafa through
Falsification.
Article 332 of the Revised Penal Code provides that no criminal, but only civil liability shall
result from the commission of the crime of theft, swindling, or malicious mischief committed or
caused mutually by the following persons: a) Spouses, ascendants and descendants, or relatives by
affinity in the same line; b) The widowed spouse with respect to the property which belonged to
the deceased spouse before the same shall have passed into the possession of another; and c)
Brothers and sisters and brothers-in-law and sisters-in-law, if living together.
In this case, Art. 332 is not applicable because the crimes mentioned under Article 332 is
complexed with another crime, such as theft through falsification or estafa through falsification.
Since the crime with which respondent was charged was not simple estafa but the complex crime
of estafa through falsification of public documents. Hence, Sato cannot avail himself of the
absolutory cause provided under Article 332 of the Revised Penal Code in his favor.

152
CRIMINAL LAW 1 CASE DIGESTS
ATTY. LYAN DAVD JUANICO
BLOCK 1-M A.Y. 2020 - 2021

19. People vs. Lua Chu and Uy Se Tieng


G.R. No. 34917; September 7, 1931
VILLA-REAL, J.:

FACTS:

On November 1929, Uy Se Tieng was the consignee of the shipments of opium coming from
Hongkong. He collaborated with the chief of customs secret service of Cebu, Juan Samson and the
then collector of customs of Port of Cebu, Joaquin Natividad by paying them P6,000 for the opium
to be safely released from customs.

Upon arrival of the Shipment of Opium in the ports of Cebu, Uy Se Tieng went to Samson’s house
and was told that he must pay first before taking the opium out of the custom house. Uy Se Tieng
left to tell the owner, Lua Chu. The following day, Samson informed Colonel Francisco of the
Constabulary, of all that had taken place, and said colonel instructed the provincial commander
Captain Buencosejo to discuss the capture of the opium owners with Samson. Samson also went
to the office of Provincial Fiscal requesting a stenographer to take down the conversation between
him and Uy Se Tieng in the presence of Captain Buenconsejo.

On the night of Dec. 17, 1929, Captain Buencosejo, Lieutenant Fernando, and the stenographer
went to Samson’s house and hid themselves behind the curtains. Uy Se Tieng went to Samson’s
house accompanied by Lua Chu.

Captain Buencosejo and the stenographer noted the following:

1. Uy Se Tieng informed Samson that Lua Chu was one of the owners of the Opium.

2. Lua Chu informed Samson that aside from him, there were co-owners named Tan and another
in Amoy.

3. Lua Chu promised to pay the P6,000 upon delivery of the opium from the warehouse of Uy Se
Tieng.

4. A Customs Collector had a conversation before when Samson was on vacation in Europe, with
Lua Chu and agreed on the business of shipping the Opium.

The following morning, Uy Se Tieng and companion, Uy Ay presented papers to Samson. Captain
Buencosejo who had been hiding, appeared and arrested the two Chinamen. The Constabulary then
arrested Lua Chu and confiscated 3,252 opium tins amounting to P50K. Court of First Instance of
Cebu convicted defendants of illegal importation of opium.

153
CRIMINAL LAW 1 CASE DIGESTS
ATTY. LYAN DAVD JUANICO
BLOCK 1-M A.Y. 2020 - 2021

Defendants’ principal defense is that they were induced by Juan Samson to import the opium in
question.

ISSUE:

Whether or not Juan Samson, a public official, should be convicted for inducing appellants in the
importation of opium.

RULING:

No, a public official who induces a person to commit a crime for purposes of gain, does not take
the necessary steps to seize the instrument of the crime and to arrest the offender, before he
obtained the profits in mind.

Even though Juan Samson smoothed the way for the introduction of the prohibited drugs, the
accused have already planned and actually ordered the importation of opium without the consent
or participation of Juan Samson. Samson did not help the accused to successfully implement their
plan rather, to assure the seizure of the imported drug and the arrest of the smugglers.

Mere fact that the chief of the customs secret service pretended to agree to a plan for smuggling
illegally imported opium through the customhouse, in order the better to assure the seizure of said
opium and the arrest of its importers is no bar to the prosecution and conviction of the latter.

154
CRIMINAL LAW 1 CASE DIGESTS
ATTY. LYAN DAVD JUANICO
BLOCK 1-M A.Y. 2020 - 2021

20. People vs. Doria


G.R. No. 125299; January 22, 1999
PUNO, J.:

FACTS:

Members of the PNP Narcotics Command received information that one “ Jun” (Doria) was
engaged in illegal drug activities, so they decided to entrap and arrest him in a buy-bust operation.
He was arrested. They frisked him but did not find the marked bills on him, and upon inquiry, he
revealed that he left it at the house of his associate “ Neneth ” (Gaddao), so he led the police team
to her house.

The team found the door open and a woman inside the house. “Jun” identified her
as “Neneth, ” and she was asked by SPO1 Badua about the marked money. PO3 Manlangit looked
over her house from the outdside. Standing by the door, PO3 Manlangit noticed a carton box under
the dining table. One of the box’ s flaps was open, and inside it was something wrapped in plastic,
and it appeared similar to the marijuana earlier sold to him by “Jun. ” His suspicion aroused, so he
entered the house and took hold of the box. He peeked inside the box and saw 10 bricks of what
appeared to be dried marijuana leaves. SPO1 Badua recovered the marked bills
from “Neneth ” and they arrested her. The bricks were examined and they were found to be dried
marijuana leaves.

The Regional Trial Court convicted Florencio Doria and Violeta Gaddao with violation of RA
6425 (Dangerous Drugs Act of 1972), Section 4 (Sale, Administration, Delivery, Distribution and
Transportation of Prohibited Drugs) in relation to Section 21 (Attempt and Conspiracy).

Accused Doria contend that the trial court erred in admitting as evidence the marijuana fruiting
found inside the carton box because they were obtained through a warrantless search and does not
come within the plain view doctrine. Accused Gaddao assailed the validity of warrantless search
leading to the seizure of the marijuana inside her house.

ISSUES:

1. Whether or not the warrantless arrest of Doria and Gadao is lawful.

2. Whether or not the RTC correctly found that the box of marijuana was in plain view, making its
warrantless seizure valid.

155
CRIMINAL LAW 1 CASE DIGESTS
ATTY. LYAN DAVD JUANICO
BLOCK 1-M A.Y. 2020 - 2021

RULING:

1. As to Doria, Yes. Under section 5 (a), a person may be arrested without warrant if he “has
committed, is actually committing, or is attempting to commit an offense.” Doria was caught in
the act of committing an offense. When accused is apprehended in flagrante delicto as a result of
a buy-bust operation, the police are not only authorized but duty bound to arrest him even without
warrant.

As to Gadao, No. The warrantless arrest of Gadao, the search of her person and residence, and the
seizure of the box of marijuana and marked bills are different matters. Accused Gadao was not
caught red-handed during the buy bust operation to give ground for her arrest under section 5 (a)
of Rule 113. She was not committing any crime. Accused Gadao was arrested solely on the basis
of the alleged identification made by her co-accused. Since the warrantless arrest of accused Gadao
was illegal, it follows that the search of her person and home and the subsequent seizure of the
marked bills and marijuana cannot be deemed legal as an incident to her arrest,

2. No, An object is in plain view if the object itself is plainly exposed to sight. Where the object
seized was inside a closed package, the object itself is not in plain view and therefore cannot be
seized without a warrant. if the package is such that an experienced observer could infer from its
appearance that it contains the prohibited article, then the article is deemed in plain view and may
be seized. It must be immediately apparent to the police that the items that they observe may be
evidence of a crime, contraband or otherwise subject to seizure.

Records revealed that the plastic wrapper was not colorless and transparent as to clearly manifest
its contents to a viewer. PO3 Manlangit himself admitted on cross-examination that the contents
of the box could be items other than marijuana. He did not know exactly what the box contained
that he had to ask appellant Gadao about its contents. It was not immediately apparent to PO3
Manlangit that the content of the box was marijuana. The marijuana was not in plain view and its
seizure without the requisite search warrant was in violation of the law and the Constitution. It
was fruit of the poisonous tree and should have been excluded and never considered by the trial
court.

156
CRIMINAL LAW 1 CASE DIGESTS
ATTY. LYAN DAVD JUANICO
BLOCK 1-M A.Y. 2020 - 2021

21. People vs. Miranda


G.R. No. 231989; September 4, 2018
PERALTA, J.:

FACTS:

On October 21, 2010, Romy Lim y Miranda (Lim) was charged with illegal possession of 0.02
gram of methamphetamine hydrochloride (shabu). On even date, Lim, together with his stepson
Eldie Gorres (Gorres) was also charged with illegal sale of 0.02 gram shabu.

A report from a confidential informant (CI) stated that a certain Lim was engaged in the sale of
drugs in Bonbon, Cagayan de Oro City. Police Officer Orellan and his teammates were directed to
conduct a buy-bust operation. Police Officer Carin was assigned to be the poseur-buyer. The team
proceeded to the target area and upon arrival, Police Officer Carin and the CI went to the house of
Lim. The CI knocked and called for Lim but it was Gorres who came out and invited them to enter.
The CI introduced Police Officer Carin as a shabu buyer. Gorres handed a small medicine box to
Lim and the latter took out one piece of heat-sealed transparent plastic of shabu and gave it to
Police Officer Carin who in turn paid using the Php500.00 bill buy-bust money.

Thereafter, Police Officer Carin made a missed call to Police Officer Orellan, which is the pre-
arranged signal. Police Officer Orellan and the other members of the team rushed to Lim’s house
and introduced themselves as PDEA agents. Police Officer Orellan frisked Lim and found in his
pocket the buy-bust money and a transparent plastic sachet containing a white substance.

While in the house, Police Officer Orellan marked the two plastic sachets. The inventory was
conducted by Police Officer Orellan in their office. Despite efforts to secure the attendance of a
media representative and barangay official, none arrived to witness the conduct of the inventory.
After laboratory examination, the Forensic Chemist found Lim’s urine sample positive for the
presence of shabu. The two plastic sachets were found both positive of shabu.

The trial court found Lim guilty of illegal possession and sale of shabu. Gorres was acquitted for
lack of evidence linking him as a conspirator. On appeal, the CA affirmed the RTC Decision. Lim
prayed for acquittal, that the case records are bereft of evidence showing that the buy-bust team
followed the procedure mandated in Section 21 (1), Article II of RA 9165.

ISSUE:

Whether or not Lim should be acquitted for the non-compliance of the police officers of the
procedure mandated in Section 21, Article II of R.A. No. 9165.

157
CRIMINAL LAW 1 CASE DIGESTS
ATTY. LYAN DAVD JUANICO
BLOCK 1-M A.Y. 2020 - 2021

RULING:

Yes, Lim should be acquitted on reasonable doubt. The Police Officers did not follow the
procedures laid down in Section 21, Article II of R.A No. 9165,

Section 21, Article II of R.A. No. 9165 provides:

“SEC. 21. Custody and Disposition of Confiscated, Seized, and/or Surrendered Dangerous Drugs,
Plant Sources of Dangerous Drugs, Controlled Precursors and Essential Chemicals,
Instruments/Paraphernalia and/or Laboratory Equipment. ̶ The PDEA shall take charge and have
custody of all dangerous drugs, plant sources of dangerous drugs, controlled precursors and
essential chemicals, as well as instruments/paraphernalia and/or laboratory equipment so
confiscated, seized and/or surrendered, for proper disposition in the following manner:

(1) The apprehending team having initial custody and control of the drugs shall, immediately after
seizure and confiscation, physically inventory and photograph the same in the presence of the
accused or the person/s from whom such items were confiscated and/or seized, or his/her
representative or counsel, a representative from the media and the Department of Justice (DOJ),
and any elected public official who shall be required to sign the copies of the inventory and be
given a copy thereof;”

Section 21, Article II of R.A No. 916 enumerates three witnesses that should be present during the
conduct of the inventory and photographing of the seized evidence, namely: (1) a media
representative, (2) an elected public official and (3) a DOJ representative.

In the present case, the three witnesses required under Section 21, Article II of R.A. No. 9165 were
not present during the conduct of the inventory and the photographing of the seized items. Further,
the police officers failed to justify the absence of the three witnesses. The police officers’ reasoning
that it was already late at night, it was raining and therefore unsafe for them to wait for the arrival
of the witnesses at Lim’s house, was not accepted by the Court as valid justification. The Court
found that there was no genuine and sufficient attempt to comply with the law. The police officers
failed to establish the details of an earnest effort to coordinate with and secure the presence of the
required witnesses. Thus, the police officers’ unjustified non-compliance with Section 21 of R.A.
9165 entitles Lim to a verdict of acquittal.

158
CRIMINAL LAW 1 CASE DIGESTS
ATTY. LYAN DAVD JUANICO
BLOCK 1-M A.Y. 2020 - 2021

22. Velasquez vs. People


G.R. No. 195021; March 15, 2017
GONZAGA-REYES, J.:

FACTS:

On May 24, 2003 in the evening, Jesus and Ana Del Mundo left their home to sleep in their nipa
hut. Arriving at the nipa hut, they saw Ampong and Nora in the midst of having sex. Jesus decided
to pursue Ampong, but to no avail, he began making his way back home when he was blocked by
Ampong and his fellow accused. Velasquez armed with stones, hit Jesus in the head while another
accused struck Jesus with a bamboo, and Ampong punched Jesus. Accused left Jesus on the
ground, bloodied. Fearing that the accused might return, Jesus got up and staggered his way back
to their house. Jesus was found to have sustained a crack in his skull.Accused insisted on their
version of events and claimed that they merely acted in response to Jesus’ aggressive behavior and
they acted in defense of themselves and their relatives leading to Jesus’ injuries. Trial Court found
accused guilty of attempted murder, which the Court of Appeals affirmed. Accused filed petition
for Review the resolution of CA to be reversed and set aside.

ISSUE:

Whether or not there was sufficient evidence to prove that justifying circumstances existed.

RULING:

No, accused failed to satisfy the requisite of self-defense. It is settled that when an accused admits
harming the victim but invokes self-defense to escape criminal liability, the accused assumes the
burden to establish his plea by credible, clear and convincing evidence. Otherwise, conviction
would follow from his admission that he harmed the victim. Self-defense cannot be justifiably
appreciated when uncorroborated by independent and competent evidence or when it is extremely
doubtful by itself. Indeed, in invoking self-defense, the burden of evidence is shifted and the
accused claiming self-defense must rely on the strength of his own evidence and not on the
weakness of the prosecution.

Even if it were to be granted that Jesus was the initial aggressor, the beating dealt to him by the
petitioners and their co-accused was in excess of what would have sufficed to neutralize him. It
was far from a reasonably necessary means to repel his supposed aggression. Accused failed to
satisfy the second requisite of self-defense and of defense of a relative which is the “reasonable
necessity of the means employed to prevent or repel the aggression”.

159
CRIMINAL LAW 1 CASE DIGESTS
ATTY. LYAN DAVD JUANICO
BLOCK 1-M A.Y. 2020 - 2021

23. People vs. Regalario


G.R. No. 174483; March 31, 2009
LEONARDO-DE CASTRO, J.:

FACTS:

The accused all surnamed Regalario, are barangay officials of Natasan, Libon, Albay and related
to one another by consanguinity. On one night, a dance and singing contest was being held.
Rolando Sevilla and Armando Poblete were enjoying the festivities when accused Sotero
Regalario approached them. To avoid trouble, the two distanced themselves from Sotero.
Nevertheless, a commotion ensued.

Accused Sotero and Bienvenido Regalario were seen striking Rolando Sevilla several times with
their respective nightsticks, locally known as bahi. The blows caused Sevilla to fall down in a
sitting position but after a short while he was able to get up. He ran away in the direction of the
house of accused Mariano Regalario, the barangay captain. Bienvenido and Sotero chased Sevilla.
When Sevilla was already near Marciano's house, he was waylaid by accused Ramon Regalario
and at this point, Marciano and his son Noel Regalario came out of their house. Noel was carrying
a seven-inch knife. The five appellants caught the victim in front of Marciano's house. Armed with
their nightsticks, they took turns in hitting the victim until he slumped to the ground face down. In
that position, Sevilla was boxed by Marciano in the jaw. After a while, when Sevilla was no longer
moving, Marciano first ordered the others to kill the victim and to tie him up. Upon hearing the
order, Bienvenido, with the help of Sotero, tied the neck, hands and feet of the victim with a nylon
rope used by farmers for tying carabao. The rest of the group just stood by watching.

Cynthia Sevilla, the victim's widow, after she was informed of her husband's death, went to the
poblacion of Libon to report the incident at the town's police station. However, her statements were
not entered in the police blotter because accused Marciano had earlier reported to them a different
version of the incident, that it was the victim Sevilla who shot Marciano's brother Ramon and that
Sevilla, allegedly still alive, was placed under the custody of the barangay tanods. In the same
morning, the policemen took the victim's cadaver to the police station in the poblacion where
pictures were taken showing the victim's hands and legs tied behind him.

The Court of Appeals affirmed the Trial court’s decision finding accused Ramon, Marciano,
Sotero, Bienvenido and Noel, all surnamed Regalario guilty of Murder.

All raised the defense of denial except for Ramon who admitted the act charged but claimed self-
defense. Accused Ramon contends that the victim Rolando Sevilla committed an act of unlawful
aggression with no provocation on his part.

160
CRIMINAL LAW 1 CASE DIGESTS
ATTY. LYAN DAVD JUANICO
BLOCK 1-M A.Y. 2020 - 2021

ISSUE:

Whether or not Ramon Regalario acted in self-defense.

RULING:

No, Ramon did not act in self-defense. When self-defense is invoked by an accused charged with
murder or homicide he necessarily owns up to the killing but may escape criminal liability by
proving that it was justified and that he incurred no criminal liability therefore. Hence, the three
(3) elements of self-defense, namely: (a) unlawful aggression on the part of the victim; (b)
reasonable necessity of the means employed to prevent or repel the aggression; and (c) lack of
sufficient provocation on the part of the person defending himself, must be proved by clear and
convincing evidence. However, without unlawful aggression, there can be no self-defense, either
complete or incomplete.

By Ramon's own account, after he was shot, he hit the victim at the back of the latter's head and
he continued hitting the victim who retreated backward. From that moment, the inceptive unlawful
aggression on the part of the victim ceased to exist and the continuation of the offensive stance of
Ramon put him in the place of an aggressor. There was clearly no longer any danger, but still
Ramon went beyond the call of self-preservation.

Ramon’s claim of self-defense is further belied by the presence of two stab wounds on the neck,
four lacerated wounds on the head, as well as multiple abrasions and contusions on different parts
of the victim’s body. And even if it were true that the victim acted out, the number, nature and
severity of the injuries suffered by the victim indicated that the force used against him by Ramon
and his co-accused was not only to disarm the victim or prevent him from doing harm to others.

161
CRIMINAL LAW 1 CASE DIGESTS
ATTY. LYAN DAVD JUANICO
BLOCK 1-M A.Y. 2020 - 2021

24. People vs. Dulin


G.R. No. 171284; June 29, 2015
BERSAMIN, J.

FACTS:

In the evening of August 22, 1990, Dulin was in his house with some guests to see the fighting
cocks being sold. After the transaction, Dulin accompanied his guests to the highway to get a
tricycle ride. Dulin asked the others to go ahead, and that he would just catch up with them.

Thereafter, Batulan, the cousin of his mother, stabbed him on the right side of his body and in the
left hand. Dulin complained to Batulan, but Batulan replied “I will really kill you.” Dulin ran to
the upper level of the house while being pursued by Batulan who stabbed him again several times.
They grappled for the weapon until Dulin was able to wrest it from Batulan. Dulin stabbed Batulan
with the weapon and they struggled until Dulin felt weak, eventually falling to the ground. Batulan
died and had sustained 12 stab wounds.

In convicting Dulin of murder, the RTC appreciated the privileged mitigating circumstance of
incomplete self-defense.

In Dulin’s appeal to the CA, he contended that he should be guilty only of homicide instead of
murder, considering the RTC’s appreciation of incomplete self-defense and that it was the victim
who had first attacked by stabbing him. However, CA affirmed with modification his conviction
for the murder of Batulan.

Dulin posits that the totality of circumstances indicated that his acts constituted incomplete self-
defense, and must be appreciated as a privileged mitigating circumstances.

ISSUE:

Whether or not Dulin acted with self-defense.

RULING:

No, Dulin did not act in order to defend himself. Unless the victim committed unlawful aggression
against the accused, self-defense, whether complete or incomplete, should not be appreciated, for
the two other essential elements of self-defense would have no factual and legal bases without any
unlawful aggression to prevent or repel.

In rejecting Dulin’s argument, The CA observed that although Batulan had initiated the attack
against Dulin, the unlawful aggression from Batulan effectively ceased once Dulin had wrested

162
CRIMINAL LAW 1 CASE DIGESTS
ATTY. LYAN DAVD JUANICO
BLOCK 1-M A.Y. 2020 - 2021

the weapon from the latter. Batulan, albeit the initial aggressor against Dulin, ceased to be the
aggressor as soon as Dulin had dispossessed him of the weapon. Whatever Dulin did thereafter
like stabbing Batulan with the weapon, constituted retaliation which is not the same as self-
defense. In retaliation, the victim started already ceased when the accused attacked him, but in
self-defense, the aggression was still continuing when the accused injured the aggressor. As such,
there was no unlawful aggression on the part of Batulan to justify his fatal stabbing by Dulin.

Dulin should prove the elements of incomplete sef-defense by first credibly establishing that the
victim had committed unlawful aggression against him. With Batulan’s aggression having already
ceased from the moment that Dulin divested Batulan of the weapon, there would not be any
incomplete self-defense. Dulin did not act in order to defend himself or to repel any attack, but
instead to inflict injury on Batulan.

163
CRIMINAL LAW 1 CASE DIGESTS
ATTY. LYAN DAVD JUANICO
BLOCK 1-M A.Y. 2020 - 2021

25. People vs. Fontanilla


G.R. No. 177743; January 25, 2012
BERSAMIN, J.:

FACTS:

Jose Olais was walking along the provincial road in Butubut Oeste, Balaoan, La Union when
Alfonso Fontanilla suddenly struck him in the head with a piece of long coconut night stick called
bellang. Olais fell face down to the ground, but Fontanilla hit him again in the head with a stone.
Fontanilla desisted from hitting Olais a third time only because the son-sin-law of Olais shouted
at him, causing him to ran away. Olais was pronounced dead on arrival, Fontanilla was charged
with murder, consequently.

At the trial, Fontanilla claimed self-defense, that he was forced to defend himself against Olais
who was drunk. However, RTC rejected Fontanilla’s plea of self-defense.

Accused prayed for review and reversal of the decision promulgated by CA which affirmed his
conviction for murder by RTC.

ISSUES:

1. Whether or not Fontailla acted in self defense.

2. Whether or not treachery was present.

RULINGS:

A. No, Fontanilla did not act in self-defense. An indispensable requisite of self-defense is that the
victim must have mounted an unlawful aggression against the accused. Without such unlawful
aggression, the accused cannot invoke self-defense as a justifying circumstance.

In order for self-defense to be appreciated, accused had to prove by clear and convincing evidence
the following elements: (a) unlawful aggression on the part of the victim; (b) reasonable necessity
of the means employed to prevent or repel it; and (c) lack of sufficient provocation on the part of
the person defending himself.

Fontanilla did not discharge his burden. Olais did not commit unlawful aggression against
Fontanilla, and Fontanilla’s act of hitting the victim’s head with a stone, causing a mortal injury,
was not proportional to, and constituted an unreasonable response to the victim’s first attack. The
plea of self defense was thus belied by the weapons used by Fontanilla and the location and number

164
CRIMINAL LAW 1 CASE DIGESTS
ATTY. LYAN DAVD JUANICO
BLOCK 1-M A.Y. 2020 - 2021

of wounds he inflicted on Olais which revealed his intent to kill and not merely an effort to prevent
or repel an attack.

B. Yes, treachery was present. Fontanilla had appeared out of nowhere to strike Olais on the head.
The suddenness and unexpectedness of the attack effectively denied to Olais the ability to defend
himself or to retaliate against Fontanilla.

165
CRIMINAL LAW 1 CASE DIGESTS
ATTY. LYAN DAVD JUANICO
BLOCK 1-M A.Y. 2020 - 2021

26. RAMON JOSUE y GONZALES VS. PEOPLE OF THE PHILIPPINES


G.R. No. 199579; December 10, 2012
REYES, J.:

FACTS:
Macario, a barangay tanod, was buying medicine from a store near Ramon’s residence when he
saw Ramon going towards him while shouting to ask him why he had painted Ramon’s vehicle.
Macario denied but Ramon still pointed and shot his gun at Macario, Macario was hit in his elbow
and fingers. As the unarmed Macario tried to flee from his assailant, Ramon still fired his gun at
his back. Ramon declared to have merely acted in self-defense. He claimed that he, together with
his son Rafael, heard a sound indicating that the hood of his jeepney was being opened. He then
went to the place where his jeepney was parked armed with a pistol on his waist. There he saw
Macario, together with 2 people, in the act of removing the locks of his vehicle’s battery. When
Ramon sought the attention of Macario’s group, Macario pointed his gun and pulled its trigger,
but the gun jammed and failed to fire. Ramon then got his gun and used it to fire at Macario, who
was hit in the upper arm. Macario again tried to use his gun, but it still jammed then fell on the
ground. As Macario reached down for the gun, Ramon fired at him once more, hitting him at the
back. When Macario still tried to fire his gun, Ramon fired at him for the third time, hitting his
hand and causing Macario to drop his gun. Ramon got Macario’s gun and kept it in his residence.
The petitioner’s son testified in court to corroborate his father’s testimony. The Regional Trial
Court ruled against Ramon. And the Court of Appeals affirmed its decision.

ISSUE:
Whether or not there was a self-defense on the part of Ramon

RULING:
No. There was no valid self-defense. Ramon invoked self-defense, after he had admitted that he
caused the victim’s wounds when he shot Macario several times using a deadly weapon. In order
to be exonerated from the charge, the accused then assumed the burden of proving, beyond
reasonable doubt, that he merely acted in self-defense. Through the review, the Court established
that Ramon failed to establish self-defense in this regard. While the three requisites must be
adhered to, self-defense, first and foremost relies on proof of unlawful aggression on the part of
the victim. If no unlawful aggression is proved, then no self-defense may be successfully pleaded.
If no unlawful aggression is proved, then no self-defense may be successfully pleaded. In this
present case, particularly significant to this element of “unlawful aggression” is the trial court’s
finding that Macario was unarmed at the time of the shooting, while Ramon was carrying a pistol.
According to the prosecution’s witness, it was even Ramon who confronted the victim who was
only buying medicine from the sari-sari store. Granting that the victim tried to steal Ramon’s car

166
CRIMINAL LAW 1 CASE DIGESTS
ATTY. LYAN DAVD JUANICO
BLOCK 1-M A.Y. 2020 - 2021

battery, it did not equate to a danger in his life or personal safety. The weapon used and the number
of gunshots fired by Ramon further negate the claim of self-defense. Therefore, there was no valid
self-defense.

167
CRIMINAL LAW 1 CASE DIGESTS
ATTY. LYAN DAVD JUANICO
BLOCK 1-M A.Y. 2020 - 2021

27. NOE TOLEDO y TAMBOONG -versus- PEOPLE OF THE PHILIPPINES


G.R. No. 158057. September 24, 2004
FACTS:
Toledo went home late in the afternoon when he saw a group of people with the victim, Ricky
Guarte. Toledo requested the group of Ricky to refrain from making any noise and proceeded
inside his house and went to sleep. Around 9pm, Ricky came back to his house together with his
companions and went to sleep at Guarte’s home. They had not laid down for long when they heard
stones being hurled at the roof of the house. The stoning was made three (3) times. Ricky rose
from his bed and peeped through the window. He saw Toledo, his uncle, stoning their house. Ricky
went out of the house and proceeded to Toledo’s house. Ricky asked Toledo why he was stoning
their house. Toledo did not answer but met Ricky at the doorstep of the house and, without any
warning, stabbed Ricky on the abdomen with a bolo. Ricky’s mom followed him and upon seeing
that Ricky was stabbed, she shouted for help. The people who rushed to help Ricky asked who
stabbed him and he said it was Toledo. Ricky died while on the operation. Toledo insisted that
when he killed the victim, it was purely accidental. Toledo claimed that the victim was so drunk
that the same charge was at the door of his house. This prompted Toledo to get his bolo and when
he tried to prevent Ricky from entering, he accidentally hit Ricky. The Regional Trial Court found
Toledo guilty. And the Court of Appeals affirmed its decision.
ISSUE:
Whether or not there was an accidental self-defense
HELD:
No. There was not an accidental self-defense. The Court ruled that there is no such thing as
accidental self-defense. Toledo cannot claim the death purely accidental and the findings of the
lower courts were unfavorable, later on changing his defense by alleging that what happened was
purely self-defense. The two defenses perpetuated by Toledo are totally inconsistent with each
other. Although in the justifying circumstance of self-defense, an accused is excused because of
deliberately trying to repel an unlawful aggression which could have killed or injured him. Such
acts are not in tune with accident which presupposes an act which was not even contemplated or
planned but purely accidental. Therefore, there was no self-defense or an accidental self-defense.

168
CRIMINAL LAW 1 CASE DIGESTS
ATTY. LYAN DAVD JUANICO
BLOCK 1-M A.Y. 2020 - 2021

28. SHERWIN DELA CRUZ VS. PEOPLE OF THE PHILIPPINES


G.R. No. 189405; November 19, 2014
PERALTA, J.:

FACTS:
Sherwin de la Cruz with the use of an unlicensed firearm feloniously shot Jeffrey Gonzales on the
head which directly caused his death.
Based on the prosecution, Sherwin went to the office where his wife and Jeffrey worked. Sherwin
went to the work station of the deceased Jeffrey who was facing his back towards the aisle. As
Sherwin was approaching the victim, he was already holding a gun pointed at the back of Jeffrey’s
head. Jeffrey deflected the hand of the petitioner holding the gun. There was a short struggle of
the gun that happened where Sherwin won and remained in the possession of the gun. He pointed
the gun to Jeffrey’s head and pulled the trigger four (4) times. The fourth hit Jeffrey in the forehead,
which resulted in his death.
According to defense, Sherwin went to the building to fetch his wife. Sherwin underwent the
regular security check-up procedures, frisked twice and no firearms were found in his possession.
He did not find his wife at the table. Sherwin approached a man and asked where his wife was.
Sherwin was asked why he was looking for Darlene however Jeffrey’s reply shocked Sherwin.
Sherwin asked who he was and cursed at him. Jeffrey picked up the gun on his chair and pointed
it at Sherwin’s face. The gun did not fire. They grappled for the possession of the gun and it was
clicked multiple times but it still did not fire. Sherwin was able to wrest away the gun from Jefferey
and run away. Jeffrey then blocked the path and shouted for guards, he then took a fire extinguisher
and was ready to smash the Sherwin’s head. Acting instinctively, Sherwin parried the attack while
still holding the gun. The gun accidentally fired and the single bullet hit Jeffrey’s forehead. After
the incident, Darlene left Sherwin and brought with her their children. Sherwin found out that
Darlene had an illicit relationship with Jeffrey.
Sherwin was found guilty beyond reasonable doubt by the Regional Trial Court. And his motion
for reconsideration was denied. Hence, the present petition.

ISSUE:
Whether or not there was an unlawful aggression, which is a requisite for self-defense, that was
established in this case

RULING:
No. There was not an unlawful aggression. Unlawful aggression means that there must be an
actual, sudden, unexpected or imminent danger. There is an aggression when the one attacked
faces real and immediate threat to his life. The peril sought to be avoided must be imminent and
actual not merely speculative. The defense did not show enough evidence that Jeffrey responded

169
CRIMINAL LAW 1 CASE DIGESTS
ATTY. LYAN DAVD JUANICO
BLOCK 1-M A.Y. 2020 - 2021

to Sherwin’s questions or initiated the confrontation before the shooting incident. And that Jeffrey
pulled a gun from his chair and tried to shoot Sherwin but failed which could possibly lead to an
assault that may have cause Sherwin to fear for his life. Even if the gun originated from Jeffrey

and an altercation transpired and danger may have existed, the imminence of the danger had
already ceased the moment Sherwin disarmed Jeffrey by wrestling the gun from him. After
Sherwin seized it, there was no longer any unlawful aggression to speak of that would have
necessitated the need to kill Jeffrey. Therefore, the unlawful aggression was not present in this
case.

170
CRIMINAL LAW 1 CASE DIGESTS
ATTY. LYAN DAVD JUANICO
BLOCK 1-M A.Y. 2020 - 2021

29. RODOLFO GUEVARRA VS. PEOPLE OF THE PHILIPPINES


G.R. No. 170462. February 5, 2014
BRION, J.:

FACTS:
Rodolfo and his son, Joey were charged with the crimes of frustrated homicide and homicide.
To prove the petitioners’ claim of self-defense, the defense presented the testimonies of Rodolfo,
Joey and the petitioner’s neighbor, Balibino Agustin.

Testimony of Rodolfo
Rodolfo’s testimony narrated that brothers Erwin and David, together with their companion,
Philip, forced their way into his compound and threw stones at his house and tricycle. Through the
back door of his house, Rodolfo went down to the basement and told them to stop. David saw him
and threatened to kill and struck him with a “panabas'' hitting him on the palm of his left hand.
Rodolfo responded by reaching for the bolo tucked in the “solera” of his house and hacked and
stabbed Erwin and David until the two brothers fell to the ground. Upon seeing Erwin and David
lying on the ground, Rodolfo called someone to bring them to the hospital and waited in his house
until the officials arrived.

Testimony of Joey
Joey narrated that he was awakened by the sound of stones being thrown at their house. Through
the window, he saw Erwin, David and Philip breaking into their gate. He then heard his father
Rodolfo confronting the three and David retorted in their dialect.

Testimony of Balbino
Balbino narrated that from inside his house he heard a person from the outside shouting about
guarding the place. He opened his door and saw the three throwing stones at the house of his
neighbor Crisanto Briones. Briones got mad and scolded the three men. The three men aimed their
stones at Guevarra's house. Balbino heard David telling Joey to get away but no one came out of
Rodolfo’s house. Afterwards, Balbino saw the three destroy Rodolfo’s gate, entered the house and
damaged Rodolfo’s tricycle. He heard Rodolfo trying to compromise with David however, David
approached Rodolfo and hacked him. Radolfo parried the blow with the back of his hand, and
David and Rodolfot struggled for the possession of the “panabas.” Balibino also saw Erwin hit
Rodolfo on the face with a stone and Joey was hit on his right foot, causing Rodolfo and Joey to
retreat to the “silung” of their house from where Rodolfo got “something shiny” and with it stabbed
David and Erwin.

171
CRIMINAL LAW 1 CASE DIGESTS
ATTY. LYAN DAVD JUANICO
BLOCK 1-M A.Y. 2020 - 2021

Prosecution’s version
Erwin, who survived the hacking, narrated that him and his brother David and Philip went to a
birthday party and passed in front of Rodolfo’s compound. He was walking ahead of his
companions when, suddenly, Philip ran up to him saying that David was being stabbed by Joey
with a bolo. While approaching the scene of the stabbing, Erwin was met by Rodolfo who then
hacked him, hitting his arm and back. Thereafter, Rodolfo and Joey dragged Erwin inside
Rodolfo’s compound and kept on hacking him. He was hacked and stabbed thirteen times. Erwin
denied that he and David threw stones at Rodolfo’s house and damaged his property. The gate was
only damaged when his brother David clung on to it while he was being pulled by Rodolfo and
Erwin into their compound. While they were being hacked and stabbed by Rodolfo and Erwin,
stones actually rained on them and people outside Rodolfo’s gate were telling them to stop.
The Regional Trial Court gave credence to Erwin's version of the incident. And the Court of
Appeals affirmed said judgment.

ISSUE:
Whether or not the justifying circumstance of self-defense was established

RULING:
No. There was no self-defense. By invoking the justifying circumstance of self-defense, Rodolfo
and Joey, in effect, admitted to the commission of the acts for which they were charged. With this
said admission, the burden of proof is now shifted to the accused to show that the crime was
attended by the requisites of self-defense. The requisites are (a) unlawful aggression on the part of
the victims (b) reasonable necessity of the means employed to prevent or repel such aggression
and (3) lack of sufficient provocation on the part of the persons resorting to self-defense. The most
important element is unlawful aggression. It must be established first before self-defense can be
invoked. The Court found the absence of the element of unlawful aggression of the part of the
victims. Rodolfo and Joey did not act in self-defense in killing David and wounding Erwin.
Rodolfo and Joey were the aggressors. Therefore, the justifying circumstance of self-defense was
not established.

172
CRIMINAL LAW 1 CASE DIGESTS
ATTY. LYAN DAVD JUANICO
BLOCK 1-M A.Y. 2020 - 2021

30. PEOPLE OF THE PHILIPPINES VS. OSCAR SEVILLANO y RETANAL


G.R. No. 200800; February 9, 2015
PEREZ, J.:

FACTS:
Pablo was having their usual chit-chat with two other people at a vacant lot. While conversing,
they saw Sevillano coming towards their direction. Sevillano could not walk straight and appeared
to be drunk. Without warning, Sevillano pulled out a knife from his waist and stabbed Pablo on
his chest. Pablo’s companions tried to restrain Sevillano from attacking the victim, but one of them
experienced leg cramps and lost his hold on Sevillano. Sevillano turned again on Pablo and
continued to stab him several times more. Pablo was heard asking Sevillano why and Pablo’s wife
came to the scene and embraced Sevillano as she wrestled for the knife. The victim died.

Pablo denied the accusations against him. He invoked self-defense to exempt him from criminal
liability. He averred that he went to the vacant lot where Pablo and his friends usually hang-out to
feed his chicken. While Pablo, whom he described to have bloodshot eyes, walked towards him
and stepped on his injured foot. While he was on his knees because of the pain, he saw Pablo draw
a knife. Pablo thereafter stabbed at him but missed his target. As Sevillano and Pablo grappled for
the knife, Pablo was accidentally stabbed. When he saw blood oozing out of the victim, Sevillano
became apprehensive of the victim's relative to such an extent that he fled the scene.

The Regional Trial Court (RTC) found Sevillano guilty. The Court of Appeal affirmed the RTC’s
decision. Hence, this appeal.

ISSUE:
Whether or not Sevillano acted in self-defense.

RULING:
No. Sevillano did not act in self-defense. In order for self-defense may be invoked under Article
11 (1) of the Revised Penal Code the following requisites must be established: (a) unlawful
aggression (b) reasonable necessity of the means employed to prevent or repel it (c) lack of
sufficient provocation on the part of the person defending himself. Sevillano’s claim that it was
Pablo who was armed with a knife and threatened to stab him was found by the RTC to be
untenable. Assuming that there was indeed unlawful aggression on the part of the victim, the
imminence of that danger had already ceased the moment Sevillano was able to wrestle the knife
from Pablo. Thus, there was no longer any unlawful aggression to speak of that would justify the
need for him to kill the victim or the former aggressor. This Court has ruled that if an accused still
persists in attacking his adversary, he can no longer invoke the justifying circumstance of self-

173
CRIMINAL LAW 1 CASE DIGESTS
ATTY. LYAN DAVD JUANICO
BLOCK 1-M A.Y. 2020 - 2021

defense. The fact that the victim suffered many stab wounds in the body that caused his demise,
and the nature and location of the wound also belies and negates the claim of self-defense. It
demonstrates a criminal mind resolved to end the life of the victim. Therefore, Sevillano did not
act in self-defense and will be held liable for the crime that he committed.

174
CRIMINAL LAW 1 CASE DIGESTS
ATTY. LYAN DAVD JUANICO
BLOCK 1-M A.Y. 2020 - 2021

MODULE 6: EXEMPTING CIRCUMSTANCES

1. THE PEOPLE OF THE PHILIPPINE ISLANDS VS. POTENCIANO TANEO


G.R. No. 37673; March 31, 1933
AVANCEÑA, C.J.:

FACTS:
Potenciano Taneo lived with his wife in his parent’s house. A fiesta was being celebrated and
visitors were entertained including Fred Tanner and Luis Malinao. Early that afternoon, Taneo
went to sleep and while sleeping he suddenly got up and left the room with bolo in hand and upon
meeting his wife who tried to stop him, he wounded her in the abdomen. Taneo attacked Tanner
and Malinao and tried to attack his father after which he wounded himself. Potenciano’s wife, who
was pregnant, died five days after as a result of her wound. Taneo was charged with parricide.
From the evidence the day before the commission of the crime, Taneo had a quarrel over a glass
of “tuba” with Collantes and Abadilla. On the day of the commission of the crime, it was noted
that Taneo was sad and weak and early in the afternoon he had a severe stomachache. Taneo states
that when he fell asleep, he dreamed that Collantes was trying to stab him with a bolo while
Abadilla held his feet, by reason of which he got up; and as it seemed to him that his enemies were
inviting him to come down, he armed himself with a bolo and left the room. At the door, he met
his wife who seemed to say to him that she was wounded. Then he fancied seeing his wife really
wounded and in desperation wounded himself. As his enemies seemed to multiply around him, he
attacked everybody that came his way. The evidence shows that the defendant not only did not
have any trouble with his wife, but that he loved her dearly. Neither did he have any dispute with
Tanner and Malinao, or had any motive for assaulting them.
ISSUE:
Whether or not Taneo is criminally liable.

RULING:
No. Taneo is not criminally liable. The Court concluded that the defendant acted while in a dream
and his acts, with which he is charged, were not voluntary in the sense of entailing criminal
liability. The Court took the special circumstances of the case, in which the victim was Taneo’s
own wife whom he dearly loved, and taking into consideration the fact that the Taneo tried to
attack also his father, in whose house and under whose protection he lived, besides attacking
Tanner and Malinao, his guests, whom he himself invited as may be inferred from the evidence
presented, the Court found not only a lack of motive for the defendant to voluntarily commit the
acts complained of, but also motives for not committing said acts. It was ordered that he be
confined in the Government insane asylum since his acts were not voluntary to warrant him a
criminal liability. Therefore, Taneo is not criminally liable.

175
CRIMINAL LAW 1 CASE DIGESTS
ATTY. LYAN DAVD JUANICO
BLOCK 1-M A.Y. 2020 - 2021

2. THE PEOPLE OF THE PHILIPPINES VS. CELESTINO BONOAN Y CRUZ


G.R. No. 45130; February 17, 1937
LAUREL, J.:

FACTS:
Celestino Bonoan met with Carlos Guison. A certain witness heard Bonoan said he will kill
Guison. Guison said he will pay Bonoan but Bonoan denied and proceeded to stab Guison with a
knife thrice on his left side which caused him death two days afterwards. The defense counsel
objected at the time of the arraignment on the ground that the defendant was mentally deranged
saying Bonoan was confined in the Psychopathic hospital. Because of this circumstance, the court
issued orders requiring the director of the hospital and hospital’s doctor to render a report on the
mental condition of Bonoan. The professionals reported that Bonoan was indeed suffering from a
disease diagnosed as dementia praecox which is similar to manic depressive psychosis. Two years
later, Bonoan’s mental health recovered and he was discharged from the hospital and can now
appear

ISSUE:
Whether or not Bonoan can invoke the exempting circumstance of insanity on the ground that he
is suffering from dementia praecox when he committed the crime

RULING:
Yes. Bonoan is exempted from criminal liability. Insanity as a defense must be proved beyond
reasonable doubt when the commission of the crime is established. In the Philippines, when a
defendant in a criminal case interposes the defense of mental incapacity, the burden of establishing
the fact rests upon him. In this case, the defense claimed that Bonoan was insane at the time he
killed the Guison. Prior to the commission of the offense, he was confined in the insane department
of a hospital suffering from dementia praecox. Persons with dementia praecox are disqualified
from legal responsibility because they have no control of their acts; dementia praecox symptoms
similar to manic depression psychosis. It is similar to those of manic depressive psychosis where
the mind appears “deteriorated” because when a person becomes affected by this kind of disease,
during the time of excitement, he has no control whatever he acts. Bonoan had an insomnia attack,
a symptom leading to dementia praecox, four days prior to act according to the professionals.
Accused was sent to the psychopathic hospital on the same day when the crime happened which
indicates the police’s doubt of his mental normalcy. Therefore, Bonoan is exempted from criminal
liability.

176
CRIMINAL LAW 1 CASE DIGESTS
ATTY. LYAN DAVD JUANICO
BLOCK 1-M A.Y. 2020 - 2021

3. People vs. Formigones


G.R. No. L-3246; November 29, 1950
MONTEMAYOR, J.:

FACTS:
Defendant Abelardo Formigones, along with his wife, Julia Agricola, and his five children went
to live in the house of his half-brother to find employment as harvesters of palay. After about a
month's stay, the accused, without any previous quarrel or provocation whatsoever, took his bolo
from the wall of the house and stabbed his wife, Julia, in the back, resulting in her death. After
Julia fell to the ground, Abelardo took her up into his arms, carried her up the house, laid her on
the floor of the living room and then laid down beside her.

During the investigation, Abelardo admitted that he killed his wife. The motive was admittedly
jealousy because according to his statement he used to have quarrels with his wife for the reason
that he suspected that she and his brother were maintaining illicit relations. At the trial of the case,
Abelardo entered a plea of not guilty, but did not testify. Evidence was presented showing that his
conduct while in confinement was strange and that he behaved like an insane person, exhibiting a
range of eccentric behavior. Abelardo was found guilty of parricide and was sentenced to reclusion
perpetua. The Defendant then appealed, claiming that the appellant is an imbecile and therefore
exempt from criminal liability under article 12 of the Revised Penal Code.

ISSUE:
Whether or not the appellant is an imbecile and therefore exempt from criminal liability under
article 12 of the Revised Penal Code.

RULING:
After careful study, the Court was convinced that the appellant is not an imbecile. According to an
expert witness who examined the defendant, Abelardo was suffering only from feeblemindedness
and not imbecility and that he could distinguish right from wrong. In order that a person could be
regarded as an imbecile within the meaning of article 12 of the Revised Penal Code so as to be
exempt from criminal liability, he must be deprived completely of reason or discernment and
freedom of the will at the time of committing the crime, mere abnormality of his mental faculties
does not exclude imputability.

The allegation of insanity or imbecility must be clearly proved. Without positive evidence that the
defendant had previously lost his reason or was demented, a few moments prior to or during the
perpetration of the crime, it will be presumed that he was in a normal condition. Acts penalized by
law are always reputed to be voluntary, and it is improper to conclude that a person acted
unconsciously, in order to relieve him from liability, on the basis of his mental condition, unless
his insanity and absence of will are proved. According to the evidence, during his marriage of
about 16 years, he has not done anything or conducted himself in any way so as to warrant an
opinion that he was or is an imbecile. The judgment of the lower court was affirmed with
modification as to the period of imprisonment.

177
CRIMINAL LAW 1 CASE DIGESTS
ATTY. LYAN DAVD JUANICO
BLOCK 1-M A.Y. 2020 - 2021

4. People vs. Puno


G.R. No. L-33211; February 17, 1993
REGALADO, J.:

FACTS:

The defendant Ernesto Puno, entered the home of Francisca Col (Aling Kikay) and accused her of
witchcraft. Then, he assaulted her and maimed her with a hammer until she was dead. Puno
threatened the only two witnesses not to inform the authorities about the killing and fled
afterwards. The witnessess informed the authorities once Puno had left.

Puno was eventually taken into custody and was brought to the National Mental Hospital. Puno
pretended that he did not remember killing Aling Kikay. He was convinced that he should kill
those who he believed to be practicioners of witchcraft in order to ensure his safety. It was testified
that prior to and on the night of the killing, Puno was exhibiting concerning signs such as reddish
eyes, a headache and unusual behavior such as attempting to restrain his dog with a religious
garment, and assaulting said dog, refusing to change his wet clothes and wearing the clothes of
deceased individuals. Puno had a history of mental illness and was a patient at the National Mental
Hospital many years ago, it was observed that Puno was suffering from “schizophrenic reaction”
but he could act with discernment.

Expert witnesses testified that Puno acted with discernment when committed the killing and that
Puno could distinguish between right and wrong and that Puno was not mentally deficient. The
lower court concluded that Puno was sane or knew that the killing of Francisca Col was wrong and
that he would be punished for it. Puno was convicted of murder, and was sentenced him to death.

ISSUE:
Whether or not the defense of insanity should be sustained.

RULING:
The Court concluded that Puno was not legally insane when he killed the victim. When insanity is
alleged as a ground for exemption from responsibility, the evidence must refer to the time
preceding the act under prosecution or to the very moment of its execution. Insanity should be
proven by clear and positive evidence. Insanity under article 12 of the Revised Penal Code means
that the accused must be deprived completely of reason or discernment and freedom of the will at
the time of committing the crime. Insanity exists when there is complete deprivation of intelligence
in committing the act. The accused is deprived of reason, he acts without the least discernment
because there is complete absence of the power to discern, or that there is total deprivation of
freedom of the will. Mere abnormality of the mental faculties will not exclude imputability.

The facts and the findings of the psychiatrists reveal that he was not completely deprived of reason
and freedom of will during the killing. Therefore, the defense of insanity cannot be sustained. The
Court held that the lower court was correct in characterizing the killing as murder but set aside the
death penalty in favor of reclusion perpetua.

178
CRIMINAL LAW 1 CASE DIGESTS
ATTY. LYAN DAVD JUANICO
BLOCK 1-M A.Y. 2020 - 2021

5. People vs. Dungo


G.R. No. 89420; July 31, 1991
PARAS, J.:

FACTS:
The accused Rosalino Dungo, went to the place where the victim, Mrs. Belen Sigua was holding
office at the Department of Agrarian Reform. After a brief talk, the accused drew a knife from the
envelope he was carrying and stabbed Mrs. Sigua several times, inflicting fatal wounds which
directly caused her death. The accused raised the defense of insanity at the time of the commission
of the offense. Testimony of the defense showed unusual behavior of the accused such as always
being in deep thought, uncharacteristically maltreating their children, demanding unjust payments
from customers, chasing after children he perceived as hostile, and delusions of being immoliated.
The accused was confined in a mental institution and it was concluded that the accused was
psychotic or insane long before, during and after the commission of the alleged crime and that his
insanity was classified under organic mental disorder secondary to cerebro-vascular accident or
stroke. The accused claimed to have undergone treatment for his condition and that he has no
knowledge of the victim or of his crime. Testimony of the prosecution showed that the accused
was rehabilitated after a series of treatments and that the accused is functioning at a low level of
intelligence. The lower court found Dungo guilty of murder and was sentenced to reclusion
perpetua and to indemnify the family of the victim.

ISSUE:
Whether or not the accused was insane at the time of the commission of the crime.

RULING:
The Court is convinced that the accused at the time that he perpetrated the act was sane. There are
three major criteria in determining the existence of insanity, namely: delusion test, irresistible
impulse test, and the right and wrong test. Insane delusion is manifested by a false belief for which
there is no reasonable basis and which would be incredible under the given circumstances to the
same person if he is of compos mentis. Under the delusion test, an insane person believes in a state
of things, the existence of which no rational person would believe. A person acts under an
irresistible impulse when, by reason of duress or mental disease, he has lost the power to choose
between right and wrong, to avoid the act in question, his free agency being at the time destroyed.
Under the right and wrong test, a person is insane when he suffers from such perverted condition
of the mental and moral faculties as to render him incapable of distinguishing between right and
wrong.
The accused concealed his weapon used for the crime, the suddeness of the attack classified the
killing as treacherous, afterwards the accused ran away from the scene. This to the mind of the
Court is another indiciation that he was conscious and knew the consequences of his acts in
stabbing the victim . The burden of proving insanity rests on the defense, sanity is presumed. The
quantum of evidence required to overthrow the presumption of sanity is proof beyond reasonable
doubt. The appellant has not successfully discharged the burden of overcoming the presumption
that he committed the crime as charged freely, knowingly, and intelligently. The decision of the
lower court was affirmed.

179
CRIMINAL LAW 1 CASE DIGESTS
ATTY. LYAN DAVD JUANICO
BLOCK 1-M A.Y. 2020 - 2021

6. People vs. Rafanan


G.R. No. L-54135; November 21, 1991
FELICIANO, J.:

FACTS:
The accused Policarpio Rafanan had sexual intercourse with a 14-year-old named Estelita Ronaya
under threat of violence, against her will and inspite of her resistance and struggle. After the sexual
intercourse, the accused cautioned Estelita not to report the matter to her mother or anybody in the
house, otherwise he would kill her. The incident was eventually uncovered and the accused was
taken into custody. The appellant claimed is that he was suffering from a metal aberration
characterized as schizophrenia when he inflicted his violent intentions upon Estelita. The trial court
suspended the trial and ordered appellant confined at the National Mental Hospital for observation
and treatment. Medical reports concluded that the appellant is suffering from a mental disorder
called schizophrenia and is psychotic or insane, hence cannot stand court trial. He needs further
hospitalization and treatment. After some time in confinement at the National Mental Hospital, the
mental conditon of the appellant improved enough to allow him so stand trial. The case then
proceeded to trial and the appellant was convicted but later appealed.

ISSUE:
Whether or not the court erred in convicting appellant who at the time of the alleged rape was
suffering from insanity.

RULING:
The court rejected the insanity defense of appellant Rafanan. Jurisprudence established two
distinguishable tests in proving insanity: (a) the test of cognition, where there is complete
deprivation of intelligence in committing the criminal act, and the test of volition, where there is a
total deprivation of freedom of the will." Jurisprudence relies more on the test of cognition. In
previous cases where schizophrenia was interposed as an exempting circumtance, it has mostly
been rejected by the court. In each of these cases, the evidence presented tended to show that if
there was impairment of the mental faculties, such impairment was not so complete as to deprive
the accused of intelligence or the consciousness of his acts.

The testimony of medical experts negated complete destruction of intelligence at the time of
commission of the act charged. The fact that the appellant threatened Estelita with death should
she reveal that she had been sexually assaulted by him, indicates that the appellant was aware of
the consequences of his actions. It is complete loss of intelligence which must be shown if the
exempting circumstance of insanity is to be found. Appellant failed to present clear and convincing
evidence regarding his state of mind immediately before and during the sexual assault on Estelita..
The decision of the lower court was affirmed and the amount of moral damages was increased.

180
CRIMINAL LAW 1 CASE DIGESTS
ATTY. LYAN DAVD JUANICO
BLOCK 1-M A.Y. 2020 - 2021

7. People vs. Madarang


G.R. No. 132319; May 12, 2000
PUNO, J.:

FACTS:
Accused Fernando Madarang was charged with parricide for killing his wife Lilia Madarang. At
the initial hearing of the case, the accused's counsel manifested that his client had been observed
behaving in an abnormal manner inside the provincial jail. The court called the accused to the
stand but he refused to answer any of the questions propounded by the court. Hence, the court
directed the transfer of the accused to the National Center for Mental Health for psychiatric
evaluation to determine his fitness to stand trial. Initial examination of the accused revealed that
he was suffering from schizophrenia. The accused was detained at the hospital and was
administered medication for his illness.

After more than two years of confinement, the accused was discharged as he was already found fit
to stand trial. The accused continued to sustain the defense of insanity and claimed that he has
absolutely no recollection as to the circumstances surrounding the death of his wife and as to his
trial and his confinement at the National Center for Mental Health. The trial court convicted the
accused as his evidence failed to refute the presumption of sanity at the time he committed the
offense and was given the penalty of reclusion perpetua.

ISSUE:
Whether or not the appellant was insane at the time of the commission of the criminal act.

RULING:
The court found the evidence presented by the defense insufficient to establish his claim of insanity
at the time he killed his wife. None of the witnesses presented by the appellant declared that he
exhibited any of the myriad symptoms associated with schizophrenia immediately before or
simultaneous with the stabbing incident. Records do not show even a single account of abnormal
or bizarre behavior on the part of the appellant prior to the killing. The evidence of insanity after
the commission of the offense may be accorded weight only if there is also proof of abnormal
behavior immediately before or simultaneous to the commission of the crime. Evidence on the
alleged insanity must refer to the time preceding the act under prosecution or to the very moment
of its execution. There is a lack of evidence on record to show that the appellant was completely
of unsound mind prior to or coetaneous with the commission of the crime. The arguments advanced
by the appellant to prove his insanity are speculative and non-sequitur.

An accused invoking the insanity defense admits committing the crime but claims that he is not
guilty because he was insane at the time of its commission. Hence, the accused is tried on the issue
of sanity alone and if found to be sane, a judgment of conviction is rendered without any trial on
the issue of guilt as he had already admitted committing the crime. The appellant failed to establish
by convincing evidence his alleged insanity at the time he killed his wife, the decision of the trial
court was affirmed in toto.

181
CRIMINAL LAW 1 CASE DIGESTS
ATTY. LYAN DAVD JUANICO
BLOCK 1-M A.Y. 2020 - 2021

8. People v. Robios
G.R. No. 138453; May 29, 2002
PANGANIBAN, J.:

FACTS:
Appellant Robios was charged with killing his pregnant wife and the fetus inside her. According
to the prosecution, the appellant was under the influence of alcohol when he came home and
argued/quarreled with his wife, until the suspect got irked, and drew a double knife and delivered
forty one (41) stab blows to his wife. The appeallant also stabbed his own body and was brought
to the Provincial Hospital. The appellant did not refute the factual allegations of the prosecution
that he indeed killed his wife, but raised the defense of insanity. The defense claimed that the
appellant exhibited unusual behavior while detained and that he had no recollection of the killing
of his wife. The trial court found Robios guilty beyond reasonable doubt of the complex crime of
parricide with unintentional abortion and was sentenced to death. In view of the penalty imposed
by the trial court, the case was automatically elevated for review.

Issue:
Whether or not the court erred in disregarding accused-appellants defense of insanity.

Ruling:
The court ruled that the appellant was not insane at the time of the commission of the crime. A
review of the records of the case reveals that appellants claim of insanity is unsubstantiated and
wanting in material proof. Testimonies from both prosecution and defense witnesses show no
substantial evidence that appellant was completely deprived of reason or discernment when he
perpetrated the brutal killing of his wife. An evaluation of the testimonies of the defense witnesses
hardly supports his claim of insanity. The bulk of the defense evidence points to his allegedly
unsound mental condition after the commission of the crime. Defense witnesses testified on the
supposed manifestations of his insanity after he had already been detained in prison. Insanity must
have existed at the time of the commission of the offense, or the accused must have been deranged
even prior thereto. Otherwise he would still be criminally responsible. His alleged insanity should
have pertained to the period prior to or at the precise moment when the criminal act was committed,
not at any time thereafter.

Medical experts did not conduct background examinations of the mental condition of the appellant
before the incident by interviewing persons who had the opportunity to associate with him. The
appellant who invoked insanity should have proven that he had already been completely deprived
of reason when he killed the victim. The evidence proffered by the defense did not indicate that he
had been completely deprived of intelligence or freedom of will when he stabbed his wife to death.
Insanity is a defense in the nature of a confession or avoidance and, as such, clear and convincing
proof is required to establish its existence. The defense failed to meet the quantum of proof
required to overthrow the presumption of sanity. The decision of the lower court was affirmed
with the modification that the penalty is reduced to reclusion perpetua.

182
CRIMINAL LAW 1 CASE DIGESTS
ATTY. LYAN DAVD JUANICO
BLOCK 1-M A.Y. 2020 - 2021

9. People v. Opuran
G.R. Nos. 147674- 75; 17 March 2004
DAVIDE, JR., C.J.:

FACTS:
Appellant Anacito Opuran was charged with two counts of murder. Allan Dacles was stabbed by
the appellant and died from the wounds a short while later. After about fiteen minutes, Demetrio
Patrimonio was stabbed by the appellant near the national highway, Demetrio died the following
day after being brought to the hospital. During the court hearings, the pyschiatric examination of
the appellant was ordered. Medical experts stated that the appellant had a normal mental status but
was susuffering from some degree of Mental Aberration, which required further psychiatric
evaluation. Medical experts found the appellant was psychotic before and during the commission
of the crime and even up to the present so that he could not stand trial and would need treatment.
The diagnosis was that the appellant was suffering from schizophrenia. The lower court found the
appellant guilty of murder for the death of Demetrio Patrimonio, Jr., and homicide for the death of
Allan Dacles and gave the penalty of reclusion perpetua.

ISSUE:
Whether or not the trial court erred in disregarding the exempting circumstance of insanity.

RULING:
The appellant failed to prove by clear and convincing evidence the defense of insanity therefore
the appellant is not exempted under Article 12 (1) of the Revised Penal Code. He who pleads the
exempting circumstance of insanity bears the burden of proving it, for insanity as a defense is in
the nature of confession and avoidance. An accused invoking insanity admits to have committed
the crime but claims that he is not guilty because he is insane. The testimony or proof of an
accused's insanity must, however, relate to the time immediately preceding or coetaneous with the
commission of the offense with which he is charged. Not every aberration of the mind or mental
deficiency constitutes insanity. A man may act insane, but it does not necessarily and conclusively
prove that he is legally so. Tested against the stringent criterion for insanity to be exempting, the
appellant’s unusual behavior are not sufficient proof that he was insane immediately before or at
the time he committed the crimes. Such unusual behavior may be considered as mere abnormality
of the mental faculties, which will not exclude imputability.

There is nothing that can be discerned from a medical expert’s short psychiatric evaluation report
and testimony that the appellant’s judgment and mental faculties were totally impaired as to
warrant a conclusion that his mental condition when he killed his victims was the same when he
was psychiatrically examined. An inquiry into the mental state of an accused should relate to the
period immediately before or at the precise moment of the commission of the act which is the
subject of the inquiry. His mental condition after that crucial period or during the trial is
inconsequential for purposes of determining his criminal liability. The appellant also failed to raise
insanity at the earliest opportunity. He invoked it only after he had already testified on his defenses
of alibi and denial. It has been held that the invocation of denial and alibi as defenses indicates that
the accused was in full control of his mental faculties. The appellant failed to establish by

183
CRIMINAL LAW 1 CASE DIGESTS
ATTY. LYAN DAVD JUANICO
BLOCK 1-M A.Y. 2020 - 2021

convincing evidence his alleged insanity at the time he killed Demetrio Jr. and Allan Dacles. He
is presumed sane. The decision of the lower court was affirmed with modificaiton as to the
damages.

184
CRIMINAL LAW 1 CASE DIGESTS
ATTY. LYAN DAVD JUANICO
BLOCK 1-M A.Y. 2020 - 2021

10. Verdadero vs. People


G.R. No. 216021, 2 March 2016
MENDOZA, J.:

FACTS:
In an Information, dated September 9, 2009, Verdadero was charged with the crime of murder for
killing Romeo B. Plata (Romeo)
On March 12, 2009, at around 3:00 o'clock in the afternoon, Maynard Plata (Maynard) and his
father Romeo were at the Baggao Police Station. Together with Ronnie Elaydo (Ronnie), they
went there to report that Verdadero had stolen the fan belt of their irrigation pump.
After a confrontation with Verdadero at the police station, the three men made their way home on
a tricycle but stopped at a drugstore as Maynard intended to buy some baby supplies. Romeo
proceeded towards a store near the drugstore while Ronnie stayed inside the tricycle. From the
drug store, Maynard saw Verdadero stabbing Romeo, after he was alerted by the shouts of Ronnie.
Verdadero stabbed Romeo on the left side of the latter's upper back with the use of a Rambo knife.
He again struck Romeo's upper back, just below the right shoulder. Maynard tried to help his father
but Verdadero attempted to attack him as well. He defended himself using a small stool, which he
used to hit Verdadero in the chest.
Meanwhile, Ronnie ran towards the police station to seek assistance. The responding police
officers arrested Verdadero, while Maynard and Ronnie brought Romeo to a clinic Romeo,
however, died upon arrival at the CVMC.
On June 3, 2011, Verdadero was arraigned and pleaded "Not Guilty." During the pre-trial, he
invoked the defense of insanity but did not consent to a reverse trial. Thereafter, trial ensued.
Since 1999, Verdadero had been an outpatient of CVMC's Psychiatric Department as he claimed
to hear strange voices and had difficulty in sleeping. Sometime in 2001, Miriam Verdadero
(Miriam), Verdadero's sister, again brought him to the Psychiatric Department of CVMC after he
became violent and started throwing stones at a tricycle with a child on board. Verdadero was
confined for two (2) months and was diagnosed to be suffering from mental depression.
On July 21, 2003, he was diagnosed with schizophrenia and was given medications to address his
mental illness. Verdadero would irregularly consult with his doctors as he had a lifelong chronic
disease. Then, in 2009, he was again confined for the fourth (4th) time at CVMC due to a relapse.
Acting on the January 4, 2011 Order of the RTC, Dr. Ethel Maureen Pagaddu (Dr. Pagaddu)
conducted a mental examination on Verdadero. She confirmed that as early as 1999, he was already

185
CRIMINAL LAW 1 CASE DIGESTS
ATTY. LYAN DAVD JUANICO
BLOCK 1-M A.Y. 2020 - 2021

brought to CVMC and that he was diagnosed with schizophrenia on July 21, 2003. Dr. Pagaddu
agreed with Dr. Andres-Juliana that Verdadero had suffered a relapse on the day of the stabbing
incident
The RTC Ruling
On May 30, 2013, the RTC rendered a decision finding Verdadero guilty for the crime of homicide.
The RTC ruled that the crime committed was only homicide, as the prosecution failed to establish
the presence of treachery and evident premeditation to qualify the killing to murder. The trial court,
however, opined that Verdadero failed to establish insanity as an exempting circumstance. The
trial court posited that Verdadero was unsuccessful in establishing that he was not in a lucid
interval at the time he stabbed Romeo or that he was completely of unsound mind prior to or
coetaneous with the commission of the crime.|
The CA Ruling
In its July 10, 2014 Decision, the CA upheld Verdadero's conviction of homicide. The appellate
court agreed that the defense was able to establish that Verdadero had a history of schizophrenic
attacks, but was unable to prove that he was not lucid at the time of the commission of the offense.
ISSUE:
Whether the court of appeals gravely erred in affirming the petitioner's conviction despite the fact
that his insanity at the time of the incident was established by clear and convincing evidence.|
RULING:
Yes, the Court of appeals erred in not finding that Verdadero sufficiently proved that he was insane
at the time of the stabbing.
The Court finds that Verdadero sufficiently proved that he was insane at the time of the stabbing.
Thus, the Court takes a view different from that of the CA as the latter concluded that Verdadero's
insanity was not clearly proven.
It is true that there is no direct evidence to show Verdadero's mental state at the exact moment the
crime was committed. This, however, is not fatal to the finding that he was insane. His insanity
may still be shown by circumstances immediately before and after the incident.
To completely evade culpability, Verdadero raises insanity as a defense claiming that he had
suffered a relapse of his schizophrenia. Under Article 12 of the RPC, an imbecile or an insane
person is exempt from criminal liability, unless the latter had acted during a lucid interval. The

186
CRIMINAL LAW 1 CASE DIGESTS
ATTY. LYAN DAVD JUANICO
BLOCK 1-M A.Y. 2020 - 2021

defense of insanity or imbecility must be clearly proved for there is a presumption that the acts
penalized by law are voluntary.
In the case at bench, it is undisputed that (1) as early as 1999, Verdadero was brought to the
Psychiatric Department of CVMC for treatment; (2) he was diagnosed with depression in 2001;
(3) he was diagnosed with schizophrenia on July 21, 2003; (4) he was confined in the psychiatric
ward sometime in 2009 due to a relapse; (5) he was in and out of psychiatric care from the time of
his first confinement in 1999 until the stabbing incident; and (6) he was diagnosed to have suffered
a relapse on March 20, 2009.
Thus, it is without question that he was suffering from schizophrenia and the only thing left to be
ascertained is whether he should be absolved from responsibility in killing Romeo because of his
mental state.
Schizophrenia is a chronic mental disorder characterized by inability to distinguish between
fantasy and reality, and often accompanied by hallucinations and delusions. A showing that an
accused is suffering from a mental disorder, however, does not automatically exonerate him from
the consequences of his act. Mere abnormality of the mental faculties will not exclude
imputability.
In People v. Florendo, the Court explained the standard in upholding insanity as an exempting
circumstance, to wit:
Insanity under Art. 12, par. 1, of The Revised Penal Code exists when there is a complete
deprivation of intelligence in committing the act, i.e., appellant is deprived of reason; he acts
without the least discernment because of complete absence of the power to discern; or, there is a
total deprivation of freedom of the will. The onus probandi rests upon him who invokes insanity
as an exempting circumstance, and he must prove it by clear and convincing evidence.
In People v. Isla, the Court elucidated that insanity must relate to the time immediately preceding
or simultaneous with the commission of the offense with which the accused is charged. Otherwise,
he must be adjudged guilty for the said offense. In short, in order for the accused to be exempted
from criminal liability under a plea of insanity, he must categorically demonstrate that: (1) he was
completely deprived of intelligence because of his mental condition or illness; and (2) such
complete deprivation of intelligence must be manifest at the time or immediately before the
commission of the offense.
In raising the defense of insanity, Verdadero admits to the commission of the crime because such
defense is in the nature of a confession or avoidance. As such, he is duty bound to establish with
certainty that he was completely deprived, not merely diminished, of intelligence at the time of the

187
CRIMINAL LAW 1 CASE DIGESTS
ATTY. LYAN DAVD JUANICO
BLOCK 1-M A.Y. 2020 - 2021

commission of the crime. Failing which, Verdadero should be criminally punished for impliedly
admitting to have stabbed Romeo to death.
Proving insanity is a tedious task for it requires an examination of the mental state of the accused.
In People v. Opuran, the Court explained how one's insanity may be established, to wit:
Since insanity is a condition of the mind, it is not susceptible of the usual means of proof. As no
man can know what is going on in the mind of another, the state or condition of a person's mind
can only be measured and judged by his behavior. Thus, the vagaries of the mind can only be
known by outward acts, by means of which we read the thoughts, motives, and emotions of a
person, and then determine whether the acts conform to the practice of people of sound mind.
Insanity is evinced by a deranged and perverted condition of the mental faculties which is
manifested in language and conduct.
Establishing the insanity of an accused often requires opinion testimony which may be given by a
witness who is intimately acquainted with the accused; has rational basis to conclude that the
accused was insane based on his own perception; or is qualified as an expert, such as a psychiatrist.
In the earlier case of People v. Austria, the Court elucidated that evidence of the mental condition
of the accused during a reasonable period before and after the commission of the offense is
material, to wit:
In order to ascertain a person's mental condition at the time of the act, it is permissible to receive
evidence of his mental condition during a reasonable period before and after. Direct testimony is
not required nor are specific acts of disagreement essential to establish insanity as a defense. A
person's mind can only be plumbed or fathomed by external acts. Thereby his thoughts, motives
and emotions may be evaluated to determine whether his external acts conform to those of people
of sound mind. To prove insanity, clear and convincing circumstantial evidence would suffice.
Dr. Paggadu, without any reservations, stated that Verdadero was suffering a relapse of his
schizophrenia at the time of the stabbing incident. In contrast, she was hesitant to opine that
Verdadero might have been in a lucid interval because of the medications taken. Thus, it is
reasonable to conclude, on the basis of the testimony of an expert witness, that Verdadero was of
unsound mind at the time he stabbed Romeo.
Further, the finding of Verdadero's insanity is supported by the observations made by Maynard, a
witness for the prosecution.

188
CRIMINAL LAW 1 CASE DIGESTS
ATTY. LYAN DAVD JUANICO
BLOCK 1-M A.Y. 2020 - 2021

Maynard was familiar with Verdadero as the latter was his neighbor for a long time. He had
observed that there were times that Verdadero appeared to be of unsound mind as he would
sometimes become violent. On the day of the stabbing incident, Maynard perceived that Verdadero
was again of unsound mind noting that he had reddish eyes and appeared to be drunk. Moreover,
he was immediately transferred to the psychiatry department because of his impaired sleep and to
control him from harming himself and others.
These circumstances are consistent with Dr. Paggadu's testimony that drinking wine, poor sleep
and violent behavior were among the symptoms of a relapse, the same testimony that was used
as basis for his previous diagnosis. The evidence on record supports the finding that Verdadero
exhibited symptoms of a relapse of schizophrenia at the time of the stabbing incident. Thus, Dr.
Pagaddu reiterated Dr. Andre-Juliana's conclusion that Verdadero was having a relapse of his
illness on that fateful day.
Further, on March 22, 2009, he was officially diagnosed to have suffered a relapse of
schizophrenia. Generally, evidence of insanity after the commission of the crime is immaterial.
It, however, may be appreciated and given weight if there is also proof of abnormal behavior
before or simultaneous to the crime.
In exonerating Verdadero on the ground of insanity, the Court does not totally free him from the
responsibilities and consequences of his acts. Article 12 (1) of the RPC expressly states that
"[w]hen an insane person has committed an act which the law defines as a felony, the court shall
order his confinement in one of the hospitals or asylums established for persons thus afflicted,
which he shall not be permitted to leave without first obtaining the permission of the same court."
Instead of incarceration, Verdadero is to be confined in an institution where his mental condition
may be addressed so that he may again function as a member of society. He shall remain confined
therein until his attending physicians give a favorable recommendation for his release.
In appreciating insanity in favor of Verdadero, the Court absolves him from criminal
responsibility. He is, nevertheless, responsible to indemnify the heirs of Romeo for the latter's
death. An exempting circumstance, by its nature, admits that criminal and civil liabilities exist, but
the accused is freed from the criminal liability.
WHEREFORE, the Court grants the petition and ACQUITS accused-appellant Solomon
Verdadero y Galera of Homicide by reason of insanity.

189
CRIMINAL LAW 1 CASE DIGESTS
ATTY. LYAN DAVD JUANICO
BLOCK 1-M A.Y. 2020 - 2021

11. PEOPLE vs. GENOSA


G.R 125981, January 15, 2004
PANGANIBAN, J.:

FACTS:
Admitting she killed her husband, appellant anchors her prayer for acquittal on a novel theory --
the "battered woman syndrome" (BWS), which allegedly constitutes self-defense. Under the
proven facts, however, she is not entitled to complete exoneration because there was no unlawful
aggression -- no immediate and unexpected attack on her by her batterer-husband at the time she
shot him.
For automatic review before this Court is the September 25, 1998 Decision of the Regional Trial
Court (RTC) of Ormoc City, finding Marivic Genosa guilty beyond reasonable doubt of parricide.
Appellant and Ben Genosa were united in marriage on November 19, 1983 in Ormoc City.
Thereafter, they lived with the parents of Ben in their house at Isabel, Leyte. For a time, Ben's
younger brother, Alex, and his wife lived with them too. Sometime in 1995, however, appellant
and Ben rented from Steban Matiga a house at Barangay Bilwang, Isabel, Leyte where they lived
with their two children, namely: John Marben and Earl Pierre.
"On November 15, 1995, Ben and Arturo Basobas went to a cockfight after receiving their salary.
They each had two (2) bottles of beer before heading home. Arturo would pass Ben's house before
reaching his.
Arturo did not see appellant arrive but on his way home passing the side of the Genosas' rented
house, he heard her say 'I won't hesitate to kill you' to which Ben replied 'Why kill me when I am
innocent?' That was the last time Arturo saw Ben alive. Arturo also noticed that since then, the
Genosas' rented house appeared uninhabited and was always closed.
On November 16, 1995, appellant asked Erlinda Paderog, her close friend and neighbor living
about fifty (50) meters from her house, to look after her pig because she was going to Cebu for a
pregnancy check-up. Appellant likewise asked Erlinda to sell her motorcycle to their neighbor
Ronnie Dayandayan who unfortunately had no money to buy it.
"That same day, about 12:15 in the afternoon, Joseph Valida was waiting for a bus going to Ormoc
when he saw appellant going out of their house with her two kids in tow, each one carrying a bag,
locking the gate and taking her children to the waiting area where he was. Joseph lived about fifty
(50) meters behind the Genosas' rented house. Joseph, appellant and her children rode the same
bus to Ormoc. They had no conversation as Joseph noticed that appellant did not want to talk to
him.

190
CRIMINAL LAW 1 CASE DIGESTS
ATTY. LYAN DAVD JUANICO
BLOCK 1-M A.Y. 2020 - 2021

"On November 18, 1995, the neighbors of Steban Matiga told him about the foul odor emanating
from his house being rented by Ben and appellant. Steban went there to find out the cause of the
stench but the house was locked from the inside. Since he did not have a duplicate key with him,
Steban destroyed the gate padlock with a borrowed steel saw. He was able to get inside through
the kitchen door but only after destroying a window to reach a hook that locked it. Alone, Steban
went inside the unlocked bedroom where the offensive smell was coming from. There, he saw the
lifeless body of Ben lying on his side on the bed covered with a blanket. He was only in his briefs
with injuries at the back of his head. Seeing this, Steban went out of the house and sent word to
the mother of Ben about his son's misfortune. Later that day, Iluminada Genosa, the mother of
Ben, identified the dead body as that of [her] son.
Appellant admitted killing Ben. She testified that going home after work on November 15, 1995,
she got worried that her husband who was not home yet might have gone gambling since it was a
payday. With her cousin Ecel Araño, appellant went to look for Ben at the marketplace and taverns
at Isabel, Leyte but did not find him there. They found Ben drunk upon their return at the Genosas'
house. Ecel went home despite appellant's request for her to sleep in their house.
"Then, Ben purportedly nagged appellant for following him, even challenging her to a fight. She
allegedly ignored him and instead attended to their children who were doing their homework.
Apparently disappointed with her reaction, Ben switched off the light and, with the use of a
chopping knife, cut the television antenna or wire to keep her from watching television. According
to appellant, Ben was about to attack her so she ran to the bedroom, but he got hold of her hands
and whirled her around. She fell on the side of the bed and screamed for help. Ben left. At this
point, appellant packed his clothes because she wanted him to leave. Seeing his packed clothes
upon his return home, Ben allegedly flew into a rage, dragged appellant outside of the bedroom
towards a drawer holding her by the neck, and told her 'You might as well be killed so nobody
would nag me.' Appellant testified that she was aware that there was a gun inside the drawer but
since Ben did not have the key to it, he got a three-inch long blade cutter from his wallet. She
however, 'smashed' the arm of Ben with a pipe, causing him to drop the blade and his wallet.
Appellant then 'smashed' Ben at his nape with the pipe as he was about to pick up the blade and
his wallet. She thereafter ran inside the bedroom.
"Appellant, however, insisted that she ended the life of her husband by shooting him. She
supposedly 'distorted' the drawer where the gun was and shot Ben. He did not die on the spot,
though, but in the bedroom.
Ruling of the Trial Court
Finding the proffered theory of self-defense untenable, the RTC gave credence to the prosecution
evidence that appellant had killed the deceased while he was in bed sleeping. Further, the trial

191
CRIMINAL LAW 1 CASE DIGESTS
ATTY. LYAN DAVD JUANICO
BLOCK 1-M A.Y. 2020 - 2021

court appreciated the generic aggravating circumstance of treachery, because Ben Genosa was
supposedly defenseless when he was killed -- lying in bed asleep when Marivic smashed him with
a pipe at the back of his head.
On February 19, 2000, appellant filed an Urgent Omnibus Motion praying that this Court allow
(1) the exhumation of Ben Genosa and the reexamination of the cause of his death; (2) the
examination of appellant by qualified psychologists and psychiatrists to determine her state of
mind at the time she had killed her spouse; and (3) the inclusion of the said experts' reports in the
records of the case for purposes of the automatic review or, in the alternative, a partial reopening
of the case for the lower court to admit the experts' testimonies.
On September 29, 2000, this Court issued a Resolution granting in part appellant's Motion,
remanding the case to the
Acting on the Court's Resolution, the trial judge authorized the examination of Marivic by two
clinical psychologists, Drs. Natividad Dayan10 and Alfredo Pajarillo,11 supposedly experts on
domestic violence. Their testimonies, along with their documentary evidence, were then presented
to and admitted by the lower court before finally being submitted to this Court to form part of the
records of the case.
ISSUE:
(1) whether appellant acted in self-defense and in defense of her fetus; and
(2) whether treachery attended the killing of Ben Genosa.
RULING:
The appeal is partly meritorious.
First Legal Issue:
Self-Defense and Defense of a Fetus
Appellant admits killing Ben Genosa but, to avoid criminal liability, invokes self-defense and/or
defense of her unborn child. When the accused admits killing the victim, it is incumbent upon her
to prove any claimed justifying circumstance by clear and convincing evidence.Well-settled is the
rule that in criminal cases, self-defense (and similarly, defense of a stranger or third person) shifts
the burden of proof from the prosecution to the defense.
The Battered Woman Syndrome
In claiming self-defense, appellant raises the novel theory of the battered woman syndrome. While
new in Philippine jurisprudence, the concept has been recognized in foreign jurisdictions as a form

192
CRIMINAL LAW 1 CASE DIGESTS
ATTY. LYAN DAVD JUANICO
BLOCK 1-M A.Y. 2020 - 2021

of self-defense or, at the least, incomplete self-defense. By appreciating evidence that a victim or
defendant is afflicted with the syndrome, foreign courts convey their "understanding of the
justifiably fearful state of mind of a person who has been cyclically abused and controlled over a
period of time."
A battered woman has been defined as a woman "who is repeatedly subjected to any forceful
physical or psychological behavior by a man in order to coerce her to do something he wants her
to do without concern for her rights. Battered women include wives or women in any form of
intimate relationship with men. Furthermore, in order to be classified as a battered woman, the
couple must go through the battering cycle at least twice. Any woman may find herself in an
abusive relationship with a man once. If it occurs a second time, and she remains in the situation,
she is defined as a battered woman."
Battered women exhibit common personality traits, such as low self-esteem, traditional beliefs
about the home, the family and the female sex role; emotional dependence upon the dominant
male; the tendency to accept responsibility for the batterer's actions; and false hopes that the
relationship will improve.
More graphically, the battered woman syndrome is characterized by the so-called "cycle of
violence," which has three phases: (1) the tension-building phase; (2) the acute battering incident;
and (3) the tranquil, loving (or, at least, nonviolent) phase.
During the tension-building phase, minor battering occurs -- it could be verbal or slight physical
abuse or another form of hostile behavior. The woman usually tries to pacify the batterer through
a show of kind, nurturing behavior; or by simply staying out of his way. What actually happens is
that she allows herself to be abused in ways that, to her, are comparatively minor. All she wants is
to prevent the escalation of the violence exhibited by the batterer. This wish, however, proves to
be double-edged, because her "placatory" and passive behavior legitimizes his belief that he has
the right to abuse her in the first place.
However, the techniques adopted by the woman in her effort to placate him are not usually
successful, and the verbal and/or physical abuse worsens. Each partner senses the imminent loss
of control and the growing tension and despair. Exhausted from the persistent stress, the battered
woman soon withdraws emotionally. But the more she becomes emotionally unavailable, the more
the batterer becomes angry, oppressive and abusive. Often, at some unpredictable point, the
violence "spirals out of control" and leads to an acute battering incident.
The acute battering incident is said to be characterized by brutality, destructiveness and,
sometimes, death. The battered woman deems this incident as unpredictable, yet also inevitable.
During this phase, she has no control; only the batterer may put an end to the violence. Its nature
can be as unpredictable as the time of its explosion, and so are his reasons for ending it. The

193
CRIMINAL LAW 1 CASE DIGESTS
ATTY. LYAN DAVD JUANICO
BLOCK 1-M A.Y. 2020 - 2021

battered woman usually realizes that she cannot reason with him, and that resistance would only
exacerbate her condition.
At this stage, she has a sense of detachment from the attack and the terrible pain, although she may
later clearly remember every detail. Her apparent passivity in the face of acute violence may be
rationalized thus: the batterer is almost always much stronger physically, and she knows from her
past painful experience that it is futile to fight back. Acute battering incidents are often very savage
and out of control, such that innocent bystanders or intervenors are likely to get hurt.
The final phase of the cycle of violence begins when the acute battering incident ends. During this
tranquil period, the couple experience profound relief. On the one hand, the batterer may show a
tender and nurturing behavior towards his partner. He knows that he has been viciously cruel and
tries to make up for it, begging for her forgiveness and promising never to beat her again. On the
other hand, the battered woman also tries to convince herself that the battery will never happen
again; that her partner will change for the better; and that this "good, gentle and caring man" is the
real person whom she loves.
A battered woman usually believes that she is the sole anchor of the emotional stability of the
batterer. Sensing his isolation and despair, she feels responsible for his well-being. The truth,
though, is that the chances of his reforming, or seeking or receiving professional help, are very
slim, especially if she remains with him. Generally, only after she leaves him does he seek
professional help as a way of getting her back. Yet, it is in this phase of remorseful reconciliation
that she is most thoroughly tormented psychologically.
The illusion of absolute interdependency is well-entrenched in a battered woman's psyche. In this
phase, she and her batterer are indeed emotionally dependent on each other -- she for his nurturant
behavior, he for her forgiveness. Underneath this miserable cycle of "tension, violence and
forgiveness," each partner may believe that it is better to die than to be separated. Neither one may
really feel independent, capable of functioning without the other
In the instant case, we meticulously scoured the records for specific evidence establishing that
appellant, due to the repeated abuse she had suffered from her spouse over a long period of time,
became afflicted with the battered woman syndrome. We, however, failed to find sufficient
evidence that would support such a conclusion. More specifically, we failed to find ample evidence
that would confirm the presence of the essential characteristics of BWS.
The defense fell short of proving all three phases of the "cycle of violence" supposedly
characterizing the relationship of Ben and Marivic Genosa. No doubt there were acute battering
incidents. In relating to the court a quo how the fatal incident that led to the death of Ben started,
Marivic perfectly described the tension-building phase of the cycle. She was able to explain in

194
CRIMINAL LAW 1 CASE DIGESTS
ATTY. LYAN DAVD JUANICO
BLOCK 1-M A.Y. 2020 - 2021

adequate detail the typical characteristics of this stage. However, that single incident does not
prove the existence of the syndrome. In other words, she failed to prove that in at least another
battering episode in the past, she had gone through a similar pattern.
Neither did appellant proffer sufficient evidence in regard to the third phase of the cycle. She
simply mentioned that she would usually run away to her mother's or father's house; that Ben
would seek her out, ask for her forgiveness and promise to change; and that believing his words,
she would return to their common abode.
In sum, the defense failed to elicit from appellant herself her factual experiences and thoughts
that would clearly and fully demonstrate the essential characteristics of the syndrome.
BWS as Self-Defense
In any event, the existence of the syndrome in a relationship does not in itself establish the legal
right of the woman to kill her abusive partner. Evidence must still be considered in the context of
self-defense.
From the expert opinions discussed earlier, the Court reckons further that crucial to the BWS
defense is the state of mind of the battered woman at the time of the offense60 -- she must have
actually feared imminent harm from her batterer and honestly believed in the need to kill him in
order to save her life.
Settled in our jurisprudence, however, is the rule that the one who resorts to self-defense must face
a real threat on one's life; and the peril sought to be avoided must be imminent and actual, not
merely imaginary. Thus, the Revised Penal Code provides the following requisites and effect of
self-defense:
"Art. 11. Justifying circumstances. -- The following do not incur any criminal liability:
"1. Anyone who acts in defense of his person or rights, provided that the following circumstances
concur;
First. Unlawful aggression;
Second. Reasonable necessity of the means employed to prevent or repel it;
Third. Lack of sufficient provocation on the part of the person defending himself."
Unlawful aggression is the most essential element of self-defense. It presupposes actual, sudden
and unexpected attack -- or an imminent danger thereof -- on the life or safety of a person. In the
present case, however, according to the testimony of Marivic herself, there was a sufficient time
interval between the unlawful aggression of Ben and her fatal attack upon him. She had already

195
CRIMINAL LAW 1 CASE DIGESTS
ATTY. LYAN DAVD JUANICO
BLOCK 1-M A.Y. 2020 - 2021

been able to withdraw from his violent behavior and escape to their children's bedroom. During
that time, he apparently ceased his attack and went to bed. The reality or even the imminence of
the danger he posed had ended altogether. He was no longer in a position that presented an actual
threat on her life or safety.
Had Ben still been awaiting Marivic when she came out of their children's bedroom -- and based
on past violent incidents, there was a great probability that he would still have pursued her and
inflicted graver harm -- then, the imminence of the real threat upon her life would not have ceased
yet. Where the brutalized person is already suffering from BWS, further evidence of actual
physical assault at the time of the killing is not required. Incidents of domestic battery usually have
a predictable pattern. To require the battered person to await an obvious, deadly attack before she
can defend her life "would amount to sentencing her to 'murder by installment.'" Still, impending
danger (based on the conduct of the victim in previous battering episodes) prior to the defendant's
use of deadly force must be shown. Threatening behavior or communication can satisfy the
required imminence of danger. Considering such circumstances and the existence of BWS, self-
defense may be appreciated.
We reiterate the principle that aggression, if not continuous, does not warrant self-defense.
In the absence of such aggression, there can be no self-defense -- complete or incomplete --
on the part of the victim. Thus, Marivic's killing of Ben was not completely justified under
the circumstances.
OTHER ISSUES:
Mitigating Circumstances Present
In any event, all is not lost for appellant. While she did not raise any other modifying circumstances
that would alter her penalty, we deem it proper to evaluate and appreciate in her favor
circumstances that mitigate her criminal liability.
"This is a classic case of a Battered Woman Syndrome. The repeated battering Marivic experienced
with her husband constitutes a form of cumulative provocation which broke down her
psychological resistance and natural self-control. It is very clear that she developed heightened
sensitivity to sight of impending danger her husband posed continuously. Marivic truly
experienced at the hands of her abuser husband a state of psychological paralysis which can only
be ended by an act of violence on her part.
In sum, the cyclical nature and the severity of the violence inflicted upon appellant resulted
in "cumulative provocation which broke down her psychological resistance and natural self-
control," "psychological paralysis," and "difficulty in concentrating or impairment of
memory."

196
CRIMINAL LAW 1 CASE DIGESTS
ATTY. LYAN DAVD JUANICO
BLOCK 1-M A.Y. 2020 - 2021

Based on the explanations of the expert witnesses, such manifestations were analogous to an
illness that diminished the exercise by appellant of her will power without, however, depriving
her of consciousness of her acts. There was, thus, a resulting diminution of her freedom of
action, intelligence or intent. Pursuant to paragraphs 9 and 10 of Article 13 of the Revised
Penal Code, this circumstance should be taken in her favor and considered as a mitigating
factor.
In addition, we also find in favor of appellant the extenuating circumstance of having acted
upon an impulse so powerful as to have naturally produced passion and obfuscation. It has
been held that this state of mind is present when a crime is committed as a result of an
uncontrollable burst of passion provoked by prior unjust or improper acts or by a legitimate
stimulus so powerful as to overcome reason.
To appreciate this circumstance, the following requisites should concur: (1) there is an act, both
unlawful and sufficient to produce such a condition of mind; and (2) this act is not far removed
from the commission of the crime by a considerable length of time, during which the accused
might recover her normal equanimity.
Here, an acute battering incident, wherein Ben Genosa was the unlawful aggressor, preceded his
being killed by Marivic. He had further threatened to kill her while dragging her by the neck
towards a cabinet in which he had kept a gun. It should also be recalled that she was eight months
pregnant at the time. The attempt on her life was likewise on that of her fetus. His abusive and
violent acts, an aggression which was directed at the lives of both Marivic and her unborn child,
naturally produced passion and obfuscation overcoming her reason. Even though she was able to
retreat to a separate room, her emotional and mental state continued. According to her, she felt her
blood pressure rise; she was filled with feelings of self-pity and of fear that she and her baby were
about to die. In a fit of indignation, she pried open the cabinet drawer where Ben kept a gun, then
she took the weapon and used it to shoot him.
The confluence of these events brings us to the conclusion that there was no considerable period
of time within which Marivic could have recovered her normal equanimity. Helpful is Dr.
Pajarillo's testimony that with "neurotic anxiety" -- a psychological effect on a victim of
"overwhelming brutality [or] trauma" -- the victim relives the beating or trauma as if it were real,
although she is not actually being beaten at the time. She cannot control "re-experiencing the whole
thing, the most vicious and the trauma that she suffered." She thinks "of nothing but the suffering."
Such reliving which is beyond the control of a person under similar circumstances, must have been
what Marivic experienced during the brief time interval and prevented her from recovering her
normal equanimity.

197
CRIMINAL LAW 1 CASE DIGESTS
ATTY. LYAN DAVD JUANICO
BLOCK 1-M A.Y. 2020 - 2021

Accordingly, she should further be credited with the mitigating circumstance of passion and
obfuscation.
Treachery Not Present
There is no showing of the victim's position relative to appellant's at the time of the shooting.
Besides, equally axiomatic is the rule that when a killing is preceded by an argument or a quarrel,
treachery cannot be appreciated as a qualifying circumstance, because the deceased may be said
to have been forewarned and to have anticipated aggression from the assailant.
Moreover, in order to appreciate alevosia, the method of assault adopted by the aggressor must
have been consciously and deliberately chosen for the specific purpose of accomplishing the
unlawful act without risk from any defense that might be put up by the party attacked . There is no
showing, though, that the present appellant intentionally chose a specific means of successfully
attacking her husband without any risk to herself from any retaliatory act that he might make. To
the contrary, it appears that the thought of using the gun occurred to her only at about the same
moment when she decided to kill her batterer-spouse. In the absence of any convincing proof that
she consciously and deliberately employed the method by which she committed the crime in order
to ensure its execution, this Court resolves the doubt in her favor.
There is treachery when one commits any of the crimes against persons by employing means,
methods or forms in the execution thereof without risk to oneself arising from the defense that the
offended party might make.In order to qualify an act as treacherous, the circumstances invoked
must be proven as indubitably as the killing itself; they cannot be deduced from mere inferences,
or conjectures, which have no place in the appreciation of evidence. Because of the gravity of the
resulting offense, treachery must be proved as conclusively as the killing itself.

198
CRIMINAL LAW 1 CASE DIGESTS
ATTY. LYAN DAVD JUANICO
BLOCK 1-M A.Y. 2020 - 2021

12. People vs. Doquena


G.R 46539; September 29, 1939
DIAZ, J.:

FACTS:
The accused-appellant, who is a minor, was prosecuted for homicide in the Court of First Instance
of Pangasinan, for having killed Juan Ragojos by stabbing him in the breast with a knife on
November 19, 1938, in the municipality of Sual, Pangasinan.
Between 1 and 2 o'clock in the afternoon of November 19, 1938, the now deceased Juan Ragojos
and one Epifanio Rarang were playing volleyball in the yard of the intermediate school of the
municipality of Sual, Province of Pangasinan. The herein accused, who was also in said yard,
intervened and, catching the ball, tossed it at Juan Ragojos, hitting him on the stomach. For this
act of the accused, Juan Ragojos chased him around the yard and, upon overtaking him, slapped
him on the nape. Said accused then turned against the deceased assuming a threatening attitude,
for which the reason said deceased struck him on the mouth with his fist, returning immediately to
the place where Epifanio Rarang was in order to continue playing with him. The accused, offended
by what he considered an abuse on the part of Juan Ragojos, who was taller and more robust than
he, looked around the yard for a stone with which to attack the now deceased Juan Ragojos, but
finding none, he approached a cousin of his named Romualdo Cocal, to ask the latter to lend him
his knife. Epifanio Rarang, who had heard what the accused had been asking his cousin, told the
latter not to give the accused his knife because he might attack Juan Ragojos with it. The accused,
however, succeeded in taking possession of the knife which was in a pocket of his cousin's pants.
Once in possession of the knife, Valentin Doqueña approached Juan Ragojos and challenged the
latter to give him another blow with his fist, to which the deceased answered that he did not want
to do so because he (Juan Ragojos) was bigger that the accused. Juan Ragojos, ignorant of the
intentions of the accused, continued playing and, while he was thus unprepared and in the act of
stopping the ball with his two hands, the accused stabbed him in the chest with the knife which he
carried.
ISSUE:
Whether or not the accused, a minor, acted with discernment or not in killing the victim
RULING:
Yes, the minor acted with discernment.

199
CRIMINAL LAW 1 CASE DIGESTS
ATTY. LYAN DAVD JUANICO
BLOCK 1-M A.Y. 2020 - 2021

The proven facts, as stated by the lower court in the appealed order, convinces us that the appeal
taken from said order is absolutely unfounded, because it is error to determine discernment by the
means resorted to by the attorney for the defense, as discussed by him in his brief. He claims that
to determine whether or not a minor acted with discernment, we must take into consideration not
only the facts and circumstances which gave rise to the act committed by the minor, but also his
state of mind at the time the crime was committed, the time he might have had at his disposal for
the purpose of meditating on the consequences of his act, and the degree of reasoning he could
have had at that moment.
It is clear that the attorney for the defense mistakes the discernment referred to in article 12,
subsection 3, of the Revised Penal Code, for premeditation, or at least for lack of intention which,
as a mitigating circumstance, is included among other mitigating circumstances in article 13 of
said Code. The discernment that constitutes an exception to the exemption from criminal liability
of a minor under fifteen years of age but over nine, who commits an act prohibited by law, is his
mental capacity to understand the difference between right and wrong, and such capacity may be
known and should be determined by taking into consideration all the facts and circumstances
afforded by the records in each case, the very appearance, the very attitude, the very comportment
and behaviour of said minor, not only before and during the commission of the act, but also after
and even during the trial (U.S. vs. Maralit, 36 Phil., 155). This was done by the trial court, and the
conclusion arrived at by it is correct.
Wherefore, the appealed order is affirmed, with the costs to the appellant.

200
CRIMINAL LAW 1 CASE DIGESTS
ATTY. LYAN DAVD JUANICO
BLOCK 1-M A.Y. 2020 - 2021

13. Ortega v People


G.R. No. 151085; August 20, 2008
NACHURA, J.:

FACTS:
Petitioner, then about 14 years old, was charged with the crime of Rape in two separate
informations both dated April 20, 1998, for allegedly raping AAA, then about eight (8) years of
age.
On February 27, 1990, AAA was born to spouses FFF and MMM. Among her siblings CCC, BBB,
DDD, EEE and GGG, AAA is the only girl in the family. Before these disturbing events, AAA's
family members were close friends of petitioner's family, aside from the fact that they were good
neighbors. However, BBB caught petitioner raping his younger sister AAA inside their own home.
BBB then informed their mother MMM who in turn asked AAA. There, AAA confessed that
petitioner raped her three (3) times on three (3) different occasions.
The first occasion happened sometime in August 1996. MMM left her daughter AAA, then 6 years
old and son BBB, then 10 years old, in the care of Luzviminda Ortega, mother of petitioner, for
two (2) nights because MMM had to stay in a hospital to attend to her other son who was sick.
During the first night at petitioner's residence, petitioner entered the room where AAA slept
together with Luzviminda and her daughter. Petitioner woke AAA up and led her to the sala. There
petitioner raped AAA. The second occasion occurred the following day, again at the petitioner's
residence. Observing that nobody was around, petitioner brought AAA to their comfort room and
raped her there. AAA testified that petitioner inserted his penis into her vagina and she felt pain.
In all of these instances, petitioner warned AAA not to tell her parents, otherwise, he would spank
her. AAA did not tell her parents about her ordeal.
The third and last occasion happened in the evening of December 1, 1996. Petitioner went to the
house of AAA and joined her and her siblings in watching a battery-powered television. At that
time, Luzviminda was conversing with MMM. While AAA's siblings were busy watching,
petitioner called AAA to come to the room of CCC and BBB. AAA obeyed. While inside the said
room which was lighted by a kerosene lamp, petitioner pulled AAA behind the door, removed his
pants and brief, removed AAA's shorts and panty, and in a standing position inserted his penis into
the vagina of AAA. AAA described petitioner's penis as about five (5) inches long and the size of
two (2) ballpens. She, likewise, narrated that she saw pubic hair on the base of his penis.
This last incident was corroborated by BBB in his testimony. When BBB was about to drink water
in their kitchen, as he was passing by his room, BBB was shocked to see petitioner and AAA both
naked from their waist down in the act of sexual intercourse. BBB saw petitioner holding AAA

201
CRIMINAL LAW 1 CASE DIGESTS
ATTY. LYAN DAVD JUANICO
BLOCK 1-M A.Y. 2020 - 2021

and making a pumping motion. Immediately, BBB told petitioner to stop; the latter, in turn,
hurriedly left.
Thereafter, BBB reported the incident to his mother, MMM.
MMM testified that when she asked AAA about what BBB saw, AAA told her that petitioner
inserted his fingers and his penis into her vagina. MMM learned that this was not the only incident
that petitioner molested AAA as there were two previous occasions. MMM also learned that AAA
did not report her ordeal to them out of fear that petitioner would spank her. MMM testified that
when BBB reported the matter to her, petitioner and Luzviminda already left her house. After
waiting for AAA's brothers to go to sleep, MMM, with a heavy heart, examined AAA's vagina and
she noticed that the same was reddish and a whitish fluid was coming out from it. Spouses FFF
and MMM were not able to sleep that night. The following morning, at about four o'clock, MMM
called Luzviminda and petitioner to come to their house. MMM confronted Luzviminda about
what petitioner did to her daughter, and consequently, she demanded that AAA should be brought
to a doctor for examination
The CA's Ruling
On October 26, 2000, the CA affirmed in toto the ruling of the RTC, holding that the petitioner's
defense of denial could not prevail over the positive identification of the petitioner by the victim
AAA and her brother BBB, which were categorical, consistent and without any showing of ill
motive. The CA also held that the respective medical examinations conducted by the two doctors
were irrelevant, as it is established that the slightest penetration of the lips of the female organ
consummates rape; thus, hymenal laceration is not an element of rape. Moreover, the CA opined
that petitioner acted with discernment as shown by his covert acts. Finally, the CA accorded great
weight and respect to the factual findings of the RTC, particularly in the evaluation of the
testimonies of witnesses.
ISSUE:
Whether the pertinent provisions of R.A. No. 9344 apply to petitioner's case, considering that at
the time he committed the alleged rape, he was merely 13 years old.
RULING:
Yes, R.A No. 9344 applies to petitioner’s case.
In sum, we are convinced that petitioner committed the crime of rape against AAA. In a
prosecution for rape, the complainant's candor is the single most important factor. If the
complainant's testimony meets the test of credibility, the accused can be convicted solely on that
basis.

202
CRIMINAL LAW 1 CASE DIGESTS
ATTY. LYAN DAVD JUANICO
BLOCK 1-M A.Y. 2020 - 2021

Section 6 of R.A. No. 9344 clearly and explicitly provides:


SECTION 6. Minimum Age of Criminal Responsibility. — A child fifteen (15) years of age or
under at the time of the commission of the offense shall be exempt from criminal liability.
However, the child shall be subjected to an intervention program pursuant to Section 20 of this
Act.
A child above fifteen (15) years but below eighteen (18) years of age shall likewise be exempt
from criminal liability and be subjected to an intervention program, unless he/she has acted with
discernment, in which case, such child shall be subjected to the appropriate proceedings in
accordance with this Act.
The exemption from criminal liability herein established does not include exemption from civil
liability, which shall be enforced in accordance with existing laws.
Section 64 of the law categorically provides that cases of children 15 years old and below, at the
time of the commission of the crime, shall immediately be dismissed and the child shall be referred
to the appropriate local social welfare and development officer (LSWDO). What is controlling,
therefore, with respect to the exemption from criminal liability of the CICL, is not the CICL's age
at the time of the promulgation of judgment but the CICL's age at the time of the commission of
the offense. In short, by virtue of R.A. No. 9344, the age of criminal irresponsibility has been
raised from 9 to 15 years old.
It is imperative that this Court accord retroactive application to the aforequoted provisions of R.A.
No. 9344 pursuant to the well-entrenched principle in criminal law - favorabilia sunt amplianda
adiosa restrigenda. Penal laws which are favorable to the accused are given retroactive effect.53
This principle is embodied in Article 22 of the Revised Penal Code
It bears stressing that the petitioner was only 13 years old at the time of the commission of
the alleged rape. This was duly proven by the certificate of live birth, by petitioner's own
testimony, and by the testimony of his mother. Furthermore, petitioner’s age was never
assailed in any of the proceedings before the RTC and the CA. Indubitably, petitioner, at the
time of the commission of the crime, was below 15 years of age. Under R.A. No. 9344, he is
exempted from criminal liability.
However, while the law exempts petitioner from criminal liability for the two (2) counts of rape
committed against AAA, Section 6 thereof expressly provides that there is no concomitant
exemption from civil liability.

203
CRIMINAL LAW 1 CASE DIGESTS
ATTY. LYAN DAVD JUANICO
BLOCK 1-M A.Y. 2020 - 2021

14. People v Mantalaba


G.R. No. 186227, July 20, 2011
PERALTA, J.:

FACTS:
The Task Force Regional Anti-Crime Emergency Response (RACER) in Butuan City received a
report from an informer that a certain Allen Mantalaba, who was seventeen (17) years old at the
time, was selling shabu at Purok 4, Barangay 3, Agao District, Butuan City. Thus, a buy-bust team
was organized, composed of PO1 Randy Pajo, PO1 Eric Simon and two (2) poseur-buyers who
were provided with two (2) pieces of ₱100 marked bills to be used in the purchase.
Around 7 o'clock in the evening of October 1, 2003, the team, armed with the marked money,
proceeded to Purok 4, Barangay 3, Agao District, Butuan City for the buy-bust operation. The two
poseur-buyers approached Allen who was sitting at a corner and said to be in the act of selling
shabu. PO1 Pajo saw the poseur-buyers and appellant talking to each other. Afterwards, the
appellant handed a sachet of shabu to one of the poseur-buyers and the latter gave the marked
money to the appellant. The poseur-buyers went back to the police officers and told them that the
transaction has been completed. Police officers Pajo and Simon rushed to the place and handcuffed
the appellant as he was leaving the place.
After the operation, and in the presence of the same barangay officials, the police officers made
an inventory of the items recovered from the appellant which are: (1) one big sachet of shabu
which they marked as RMP-1-10-01-03; (2) one small sachet of shabu which they marked as RMP
2-10-01-03; and (3) two (2) pieces of one hundred pesos marked money and a fifty peso (₱50) bill.
Thereafter, a letter-request was prepared by Inspector Ferdinand B. Dacillo for the laboratory
examination of the two (2) sachets containing a crystalline substance, ultra-violet examination on
the person of the appellant as well as the two (2) pieces of one hundred pesos marked money. The
request was brought by PO1 Pajo and personally received by Police Inspector Virginia Sison-
Gucor, Forensic Chemical Officer of the Regional Crime Laboratory Office XII Butuan City, who
immediately conducted the examination. The laboratory examination revealed that the appellant
tested positive for the presence of bright orange ultra-violet fluorescent powder; and the crystalline
substance contained in two sachets, separately marked as RMP-1-10-01-03 and RMP-2-10-01-03,
were positively identified as methamphetamine hydrochloride.
Thereafter, two separate Informations were filed before the RTC of Butuan City against appellant
for violation of Sections 5 and 11 of RA 9165.

204
CRIMINAL LAW 1 CASE DIGESTS
ATTY. LYAN DAVD JUANICO
BLOCK 1-M A.Y. 2020 - 2021

ISSUE:
Whether or not the provisions of RA 9344 or The Juvenile Act applies to the accused who was a
minor at the time of the commission of the offense
RULING:
Yes, RA 9344 still applies to the petitioner.
It may be argued that the appellant should have been entitled to a suspension of his sentence under
Sections 38 and 68 of RA 9344 which provide for its retroactive application, thus:
SEC. 38. Automatic Suspension of Sentence. - Once the child who is under eighteen (18) years of
age at the time of the commission of the offense is found guilty of the offense charged, the court
shall determine and ascertain any civil liability which may have resulted from the offense
committed. However, instead of pronouncing the judgment of conviction, the court shall place the
child in conflict with the law under suspended sentence, without need of application: Provided,
however, That suspension of sentence shall still be applied even if the juvenile is already eighteen
years (18) of age or more at the time of the pronouncement of his/her guilt.
Upon suspension of sentence and after considering the various circumstances of the child, the court
shall impose the appropriate disposition measures as provided in the Supreme Court [Rule] on
Juveniles in Conflict with the Law.
Sec. 68. Children Who Have Been Convicted and are Serving Sentence. - Persons who have been
convicted and are serving sentence at the time of the effectivity of this Act, and who were below
the age of eighteen (18) years at the time of the commission of the offense for which they were
convicted and are serving sentence, shall likewise benefit from the retroactive application of this
Act. x x x
However, this Court has already ruled in People v. Sarcia that while Section 38 of RA 9344
provides that suspension of sentence can still be applied even if the child in conflict with the law
is already eighteen (18) years of age or more at the time of the pronouncement of his/her guilt,
Section 40 of the same law limits the said suspension of sentence until the child reaches the
maximum age of 21. The provision states:
SEC. 40. Return of the Child in Conflict with the Law to Court. - If the court finds that the objective
of the disposition measures imposed upon the child in conflict with the law have not been fulfilled,
or if the child in conflict with the law has willfully failed to comply with the condition of his/her
disposition or rehabilitation program, the child in conflict with the law shall be brought before the
court for execution of judgment.

205
CRIMINAL LAW 1 CASE DIGESTS
ATTY. LYAN DAVD JUANICO
BLOCK 1-M A.Y. 2020 - 2021

If said child in conflict with the law has reached eighteen (18) years of age while under suspended
sentence, the court shall determine whether to discharge the child in accordance with this Act, to
order execution of sentence, or to extend the suspended sentence for a certain specified period
or until the child reaches the maximum age of twenty-one (21) years.
Hence, the appellant, who is now beyond the age of twenty-one (21) years can no longer avail of
the provisions of Sections 38 and 40 of RA 9344 as to his suspension of sentence, because such is
already moot and academic. It is highly noted that this would not have happened if the CA, when
this case was under its jurisdiction, suspended the sentence of the appellant. The records show that
the appellant filed his notice of appeal at the age of 19 (2005), hence, when RA 9344 became
effective in 2006, appellant was 20 years old, and the case having been elevated to the CA, the
latter should have suspended the sentence of the appellant because he was already entitled to the
provisions of Section 38 of the same law, which now allows the suspension of sentence of minors
regardless of the penalty imposed as opposed to the provisions of Article 192 of P.D. 603.
Nevertheless, the appellant shall be entitled to appropriate disposition under Section 51 of RA No.
9344, which provides for the confinement of convicted children as follows:
SEC. 51. Confinement of Convicted Children in Agricultural Camps and other Training Facilities.
- A child in conflict with the law may, after conviction and upon order of the court, be made to
serve his/her sentence, in lieu of confinement in a regular penal institution, in an agricultural camp
and other training facilities that may be established, maintained, supervised and controlled by the
BUCOR, in coordination with the DSWD.
the privileged mitigating circumstance of minority was applied as well in imposing the penalty of
the accused, which can now be appreciated in fixing the penalty that should be imposed.
OTHER ISSUES:
Another contention raised by the appellant is the failure of the prosecution to show the chain of
custody of the recovered dangerous drug. According to him, while it was Inspector Ferdinand B.
Dacillo who signed the request for laboratory examination, only police officers Pajo and Simon
were present in the buy-bust operation.
Section 21 of RA 9165 reads:
SEC. 21. Custody and Disposition of Confiscated, Seized, and/or Surrendered Dangerous Drugs,
Plant Sources of Dangerous Drugs, Controlled Precursors and Essential Chemicals,
Instruments/Paraphernalia and/or Laboratory Equipment. – The PDEA shall take charge and have
custody of all dangerous drugs, plant sources of dangerous drugs, controlled precursors and

206
CRIMINAL LAW 1 CASE DIGESTS
ATTY. LYAN DAVD JUANICO
BLOCK 1-M A.Y. 2020 - 2021

essential chemicals, as well as instruments/paraphernalia and/or laboratory equipment so


confiscated, seized and/or surrendered, for proper disposition in the following manner:
(1) The apprehending team having initial custody and control of the drugs shall, immediately after
seizure and confiscation, physically inventory and photograph the same in the presence of the
accused or the person/s from whom such items were confiscated and/or seized, or his/her
representative or counsel, a representative from the media and the Department of Justice (DOJ),
and any elected public official who shall be required to sign the copies of the inventory and be
given a copy thereof.
Non-compliance by the apprehending/buy-bust team with Section 21 is not fatal as long as there
is justifiable ground therefor, and as long as the integrity and the evidentiary value of the
confiscated/seized items are properly preserved by the apprehending officer/team. Its non-
compliance will not render an accused’s arrest illegal or the items seized/confiscated from him
inadmissible.What is of utmost importance is the preservation of the integrity and the evidentiary
value of the seized items, as the same would be utilized in the determination of the guilt or
innocence of the accused. In this particular case, it is undisputed that police officers Pajo and
Simon were members of the buy-bust operation team. The fact that it was Inspector Ferdinand B.
Dacillo who signed the letter-request for laboratory examination does not in any way affect the
integrity of the items confiscated. All the requirements for the proper chain of custody had been
observed.

207
CRIMINAL LAW 1 CASE DIGESTS
ATTY. LYAN DAVD JUANICO
BLOCK 1-M A.Y. 2020 - 2021

15. Samahan Ng Mga Progresibong Kabataan vs. Quezon City


Gr. 225442; August 8, 2017
PERLAS-BERNABE, J.:

FACTS:
Following the campaign of President Rodrigo Roa Duterte to implement a nationwide curfew for
minors, several local governments in Metro Manila started to strictly implement their curfew
ordinances on minors through police operations which were publicly known as part of "Oplan
Rody."
Among those local governments that implemented curfew ordinances were respondents: (a)
Navotas City, through Pambayang Ordinansa Blg. 99- 02, dated August 26, 1999, entitled
"Nagtatakdang 'Curfew' ng mga Kabataan na Wala Pang Labing Walong (18) Taong Gulang sa
Bayan ng Navotas, Kalakhang Maynila," as amended by Pambayang Ordinansa Blg. 2002-13,
dated June 6, 2002 (Navotas Ordinance); (b) City of Manila, through Ordinance No. 8046 entitled
"An Ordinance Declaring the Hours from 10:00 P.M. to 4:00 A.M. of the Following Day as
'Barangay Curfew Hours' for Children and Youths Below Eighteen (18) Years of Age; Prescribing
Penalties Therefor; and for Other Purposes" dated October 14, 2002 (Manila Ordinance); and (c)
Quezon City, through Ordinance No. SP- 2301, Series of 2014, entitled "An Ordinance Setting for
a [sic] Disciplinary Hours in Quezon City for Minors from 10:00 P.M. to 5:00 A.M., Providing
Penalties for Parent/Guardian, for Violation Thereof and for Other Purposes" dated July 31, 2014
(Quezon City Ordinance; collectively, Curfew Ordinances).
Petitioners, spearheaded by the Samahan ng mga Progresibong Kabataan (SPARK) - an
association of young adults and minors that aims to forward a free and just society, in particular
the protection of the rights and welfare of the youth and minors - filed this present petition, arguing
that the Curfew Ordinances are unconstitutional because they: (a) result in arbitrary and
discriminatory enforcement, and thus, fall under the void for vagueness doctrine; (b) suffer from
overbreadth by proscribing or impairing legitimate activities of minors during curfew hours; (c)
deprive minors of the right to liberty and the right to travel without substantive due process; and
(d) deprive parents of their natural and primary right in rearing the youth without substantive due
process. In addition, petitioners assert that the Manila Ordinance contravenes RA 9344, as
amended by RA 10630
More specifically, petitioners posit that the Curfew Ordinances encourage arbitrary and
discriminatory enforcement as there are no clear provisions or detailed standards on how law
enforcers should apprehend and properly determine the age of the alleged curfew violators. They
further argue that the law enforcer's apprehension depends only on his physical assessment, and,

208
CRIMINAL LAW 1 CASE DIGESTS
ATTY. LYAN DAVD JUANICO
BLOCK 1-M A.Y. 2020 - 2021

thus, subjective and based only on the law enforcer's visual assessment of the alleged curfew
violator.
Furthermore, petitioners claim that the Manila Ordinance, particularly Section 4 thereof,
contravenes Section 57-A of RA 9344, as amended, given that the cited curfew provision imposes
on minors the penalties of imprisonment, reprimand, and admonition. They contend that the
imposition of penalties contravenes RA 9344's express command that no penalty shall be imposed
on minors for curfew violations.
ISSUE:
Whether or not the ordinance contravenes Section 57-A of RA 9344
RULING:
No, it does not contravenes the provision of RA 9344
The provisions of RA 9344, as amended, should not be read to mean that all the actions of the
minor in violation of the regulations are without legal consequences. Section 57-A thereof
empowers local governments to adopt appropriate intervention programs, such as community-
based programs recognized under Section 54 of the same law.
In this regard, requiring the minor to perform community service is a valid form of intervention
program that a local government (such as Navotas City in this case) could appropriately adopt in
an ordinance to promote the welfare of minors. For one, the community service programs provide
minors an alternative mode of rehabilitation as they promote accountability for their delinquent
acts without the moral and social stigma caused by jail detention.
In the same light, these programs help inculcate discipline and compliance with the law and legal
orders. More importantly, they give them the opportunity to become productive members of
society and thereby promote their integration to and solidarity with their community.
The sanction of admonition imposed by the City of Manila is likewise consistent with Sections
57 and 57-A of RA 9344 as it is merely a formal way of giving warnings and expressing
disapproval to the minor's misdemeanor. Admonition is generally defined as a "gentle or friendly
reproof' or "counsel or warning against fault or oversight." The Black's Law Dictionary defines
admonition as "[a]n authoritatively issued warning or censure"; while the Philippine Law
Dictionary defines it as a "gentle or friendly reproof, a mild rebuke, warning or reminder,
[counseling], on a fault, error or oversight, an expression of authoritative advice or warning."
Notably, the Revised Rules on Administrative Cases in the Civil Service (RRACCS) and our
jurisprudence in administrative cases explicitly declare that "a warning or admonition shall not be
considered a penalty."

209
CRIMINAL LAW 1 CASE DIGESTS
ATTY. LYAN DAVD JUANICO
BLOCK 1-M A.Y. 2020 - 2021

In other words, the disciplinary measures of community-based programs and admonition are
clearly not penalties - as they are not punitive in nature - and are generally less intrusive on the
rights and conduct of the minor. To be clear, their objectives are to formally inform and educate
the minor, and for the latter to understand, what actions must be avoided so as to aid him in his
future conduct.
A different conclusion, however, is reached with regard to reprimand and fines and/or
imprisonment imposed by the City of Manila on the minor. Reprimand is generally defined as "a
severe or formal reproof." The Black's Law Dictionary defines it as "a mild form of lawyer
discipline that does not restrict the lawyer's ability to practice law"; while the Philippine Law
Dictionary defines it as a "public and formal censure or severe reproof, administered to a person
in fault by his superior officer or body to which he belongs. It is more than just a warning or
admonition." In other words, reprimand is a formal and public pronouncement made to denounce
the error or violation committed, to sharply criticize and rebuke the erring individual, and to sternly
warn the erring individual including the public against repeating or committing the same, and thus,
may unwittingly subject the erring individual or violator to unwarranted censure or sharp
disapproval from others. In fact, the RRACCS and our jurisprudence explicitly indicate that
reprimand is a penalty, hence, prohibited by Section 57-A of RA 9344, as amended.
Fines and/or imprisonment, on the other hand, undeniably constitute penalties - as provided in
our various criminal and administrative laws and jurisprudence - that Section 57-A of RA 9344,
as amended, evidently prohibits.
As worded, the prohibition in Section 57-A is clear, categorical, and unambiguous. It states that
"[n]o penalty shall be imposed on children for x x x violations [of] juvenile status offenses]."
Thus, for imposing the sanctions of reprimand, fine, and/or imprisonment on minors for curfew
violations, portions of Section 4 of the Manila Ordinance directly and irreconcilably conflict with
the clear language of Section 57-A of RA 9344, as amended, and hence, invalid. On the other hand,
the impositions of community service programs and admonition on the minors are allowed as they
do not constitute penalties.
In fine, the Manila and Navotas Ordinances are declared unconstitutional and thus, null and void,
while the Quezon City Ordinance is declared as constitutional and thus, valid in accordance with
this Decision.
For another, the Court has determined that the Manila Ordinance's penal provisions imposing
reprimand and fines/imprisonment on minors conflict with Section 57-A of RA 9344, as amended.
Hence, following the rule that ordinances should always conform with the law, these provisions
must be struck down as invalid.

210
CRIMINAL LAW 1 CASE DIGESTS
ATTY. LYAN DAVD JUANICO
BLOCK 1-M A.Y. 2020 - 2021

16. US v Tanedo
G.R No L-5318 , February 1910
MORELAND, J.:

FACTS:
The accused was a landowner. On the morning of the 26th of January, 1909, he, with Bernardino
Tagampa, Casimiro Pascual, Valeriano Paulillo, and Juan Arellano, went to work on a malecon or
dam on his land. The defendant took with him a shotgun and a few shells, with the intention to
hunt wild chickens after he had set his laborers at work. He remained with his laborers an hour or
so and then went a short distance away across a stream to see how the alteration which he had
made in the malecon affected the flow of water from the rice filed on the other side of the stream.
He carried his shotgun with him across the stream. On the other side of the stream he met the
deceased, who, with his mother and uncle, had been living in a small shack for a month or so
during the rice-harvesting season. The accused asked the uncle of the deceased where he could
find a good place in which to hunt wild chickens. The uncle was lying on the floor in the interior
of the shack sick of fever. The deceased, a young man about 20 years of age, was working at
something under a manga tree a short distance from the shack. Although the accused directed his
question to the uncle inside of the shack, the deceased answered the question and pointed out in a
general way a portion of the forest near the edge of which stood the shack. There is some
contradiction between the testimony of the accused and the Government witnesses just at this
point. The uncle of the deceased testified that the boy and the accused invited each other mutually
to hunt wild chickens and that the accused accepted the invitation. The accused, however, testified
that he did not invite the deceased to go hunting with him, neither did the deceased go with him,
but that he remained under the manga tree "trying something." At any rate the accused went into
the forest with his gun. What took place there is unknown to anybody except the accused.
Upon that subject he testified as follows:
And after Feliciano Sanchez pointed out that place to me, that place where the wild chickens were
to be found, I proceeded to hunt, because, in the first place, if I could kill some wild chickens we
would have something to eat on that day. So when I arrived at that place I saw a wild chickens and
I shot him. And after I shot that chicken I heard a human cry. I picked up the chicken and went
near the place where I heard the noise, and after I saw that I had wounded a man I went back
toward the malecon, where my companions were working, running back, and when I arrived there
I left my shotgun behind or by a tree not far from where my companions were working; and I
called Bernardino Tagampa to tell him about the occurrence, and to him I told of that occurence

211
CRIMINAL LAW 1 CASE DIGESTS
ATTY. LYAN DAVD JUANICO
BLOCK 1-M A.Y. 2020 - 2021

because he is my friend and besides that he was a relative of the deceased, and when Tagampa
heard of this he and myself went together to see the dead body.
Only one shot was heard that morning and a chicken was killed by gunshot wound. Chicken
feathers were found in considerable qualities at the point where the chicken was shot and where
the accident occurred. The defendant within a few minutes after the accident went out of the woods
to the malecon where he had left his laborers at work, carrying the dead chicken with him. The
accused called Bernardino Tagampa, on of the laborers, to go with him and they disappeared for
some time. Tagampa says that they went a little way toward the woods and came back. The accused
says that they went to the place where the body of the deceased lay and removed it to a place in
the cogon grass where it would not be easily observed. It is certain, however, that the body was
concealed in the cogon grass. During the afternoon Tagampa left the malecon, where his fellow
laborers were working, probably to hunt for a place in which to hide the body. The rest of the
laborers saw the witness Yumul take the chicken which had been killed by the accused. He
delivered it to the wife of the accused, who testified that she received the chicken from Yumul and
that it had been killed by a gunshot wound. That evening the accused and Tagampa went together
to dispose of the body finally. They took it from the cogon grass where it lay concealed and carried
it about seventeen or eighteen hundred meters from the place where it had originally fallen, and
buried it in an old well, covering it with straw and earth and burning straw on top of the well for
the purpose of concealing it. Tagampa said that he helped the accused dispose of the body because
he was afraid of him, although he admits that the accused in no way threatened or sought to compel
him to do so. The defendant prior to the trial denied all knowledge of the death of the deceased or
the whereabouts of the body. On the trial, however, he confessed his participation in the death of
the deceased and told the story substantially as above.
ISSUE:
Whether or not the accused is criminally liable?
RULING:
No, the accused must be acquitted.
In this case there is absolutely no evidence of negligence upon the part of the accused. Neither is
there any question that he was engaged in the commission of a lawful act when the accident
occurred. Neither is there any evidence of the intention of the accused to cause the death of the
deceased. The only thing in the case at all suspicious upon the part of the defendant are his
concealment and denial.

212
CRIMINAL LAW 1 CASE DIGESTS
ATTY. LYAN DAVD JUANICO
BLOCK 1-M A.Y. 2020 - 2021

The American doctrine is substantially the same. It is uniformly held that if life is taken by
misfortune or accident while in the performance of a lawful act executed with due care and without
intention of doing harm, there is no criminal liability.
We are of the opinion that the evidence is insufficient to support the judgment of conviction.
The judgment of conviction is, therefore, reversed, the defendant acquitted

213
CRIMINAL LAW 1 CASE DIGESTS
ATTY. LYAN DAVD JUANICO
BLOCK 1-M A.Y. 2020 - 2021

17. People v. Isaias Castillo


G.R. No. L-19238; July 6, 1966
Ynares-Santiago, J.:

FACTS:
The circumstances are that on Nov. 5, 1993, Consorcia Castillo(Consorcia), appellant’s wife, died
violently after having her neck wounded by a sling-and-arrow illegally possessed by her drunk
husband, appellant Isaias Castillo(Castillo). before her death, a quarrel ensued between Consorcia
and Castillo, which was heard by Consortcia’s father. After rushing Consorcia to the hospital,
Castillo escaped but was caught in a bathroom at the adjoining barangay.
Castillo’s version is that he was having a drinking spree with his friends who also own a sling-
and-arrow and were target shooting using such item when one accidentally hit Consorcia,
furthermore, Castillo argued that even if it was he who hit Consorcia, he did so without any intent
to kill.

Nevertheless, the RTC and CA found him guilty of parricide and sentenced him to Reclusion
Perpetua.

ISSUE:
Whether or not Castillo is guilty of Parricide?

RULING:
Yes, Castillo is guilty, for judgement to be based on circumstantial evidence, it must provide that
(1) there must be more than one circumstantial evidence; (2) the facts from which the interference
are derived are proven; and (3) the combination of such circumstances produced a conviction
beyond reasonable doubt. In this case, all circumstantial evidence were able to prove Castillo’s
guilt. These are:
Consorcia would confide to her sister Castillo’s violent tendencies;On the night of the incident,
Castillo was drunk and displaying violent behavior; He was last seen holding his sling-and-arrow;
A quarrel ensued between him and Consorcia; Castillo was later seen holding Consorcia’s bloody
body; After bringing his wife to the hospital, Castillo escaped but was later caught in a bathroom
in the adjoining barangay; Autopsy report shows that her death was caused by a puncture wound
to the neck; and While detained, Castillo wrote letters to Consorcia’s father and sister asking for
forgiveness.

These circumstances prove beyond reasonable doubt that Castillo is guilty for the death of
Consorcia.

214
CRIMINAL LAW 1 CASE DIGESTS
ATTY. LYAN DAVD JUANICO
BLOCK 1-M A.Y. 2020 - 2021

Furthermore, one of Castillo’s friends testified that the drinking spree were already done by the
time of the incident.
Lastly, on Castillo’s contention that such an incident was done with no intent to kill. Art. 12(4) of
the RPC provides for injury by mere accident, which has 4 requisites:
A person is performing a lawful act;
With due care;
He causes an injury by mere accident; and
Without fault or intention of causing it.
His usage of the deadly item is not a lawful act, moreover, mere possession of such item is
punishable by law, and that by raising such defense, he has the burden of proving it, which he was
unable to.

As such, his conviction was affirmed.

18. People v. Jesus Retubado alias “Jessie


G.R. No. 124058; Dec. 10, 2003
Callejo, Sr,., J.:

FACTS:
A joke on Edwin, Jessie’s special needs brother, was blamed on Emmanuel Cañon Jr.(Cañon Jr.),
which was later disproved. Jessie however, was bent on confronting Cañon Jr., as such, when he
saw Cañon Jr.’s father, the victim Emmanuel Cañon Sr.(Cañon Sr.)in his pedicab, he confronted
him about the incident, but Cañon Sr. Ignored him, which angered Jessie, who followed after, and
later overtook the pedicab. Jessie pushed the pedicab which nearly fell into a ditch, nevertheless,
Cañon Sr. Kept on pedaling until he reached his house where his wife was waiting. Jessie followed
later, opening his house door and asking for Cañon Jr. Who was already sleeping. The victim
confronted Jessie about his previous action of following Cañon Sr., his wife beside him, trying to
pacify him. Jessie then pulled out a gun and shot Cañon Sr. In the forehead then walked away from
the crime scene.
Jessie’s defense was that he was performing a lawful act of defending himself and that it was
Cañon Sr. Who tried to shoot him, but Jessie was able to grapple for the gun when it suddenly
fired at the time it was pointed at the forehead of Cañon Sr. Afterwards, he went home to change
his clothes, placing the subject firearm in the table. But while he was changing clothes, Edwin took
the gun and threw it into the sea.
Nevertheless, Jessie was found guilty of Murder and sentenced to Reclusion Perpetua.

ISSUE:
Whether or not Jessie is guilty of Murder?

215
CRIMINAL LAW 1 CASE DIGESTS
ATTY. LYAN DAVD JUANICO
BLOCK 1-M A.Y. 2020 - 2021

RULING:
No. He is guilty of Homicide.
First. Jessie’s contention of doing a lawful act of defending himself is inapplicable.
Art. 12(4), which is injury by mere accident, has four requisites:
A person is performing a lawful act;
With due care;
He causes an injury by mere accident; and
Without fault or intention of causing it.
In addition, Jessie must also prove that there was a state of necessity that made him commit
such crime, however, he was unable to prove it.
Furthermore, it was proven that Jessie himself was the aggressor. Cañon Sr’s wife who was beside
him saw Jessie pull out a handgun and shot the victim. Another fact was that he failed to surrender
the weapon used by reason that Edwin threw it out. Lastly, Jessie had the motive to shoot and kill
the victim, Cañon Sr. repeatedly ignored him while going home, later confronting the victim in his
home before shooting him.
On the other hand, there was no treachery, which has two requisites, (1) the employment of the
means of execution that give the person attacked no opportunity to defend himself or to retaliate;
and (2) the means of execution were deliberately or consciously adopted. The prosecution was
unable to provide evidence that the requisites above were present when Jessie shot Cañon Sr.
Therefore, Jessie is only guilty of Homicide.

216
CRIMINAL LAW 1 CASE DIGESTS
ATTY. LYAN DAVD JUANICO
BLOCK 1-M A.Y. 2020 - 2021

19. Roweno Pomoy v. People


G.R. No. 150647; Sept. 29, 2004
Panganiban, J.:

FACTS:
Pomoy was last seen escorting Balboa to be questioned when witnesses heard 2 gunshots, when
they went to see what happened, they saw that Balboa was lying in a pool of blood while Pomoy
was holding his .45 cal. handgun. Pomoy argued that the shooting was an accident since Balboa
tried to grab his service firearm.
Pomoy was later found guilty of Homicide, with the CA citing People v. Reyes, in which a revolver
does not accidentally discharge its ammunition, by reason that there must first be the act of cocking
the hammer then applying sufficient force in the trigger.

ISSUE:
Whether or not Accident is applicable.

RULING:
Yes. The discharge of the service firearm was accidental in nature. One of the defense witnesses
was able to prove that Balboa and Pomoy grappled with the possession of the firearm, stating that
Pomoy was using one hand to grapple for the firearm while using the other to ward off Balboa.
Art. 12(4), which is injury by mere accident, has four requisites:
A person is performing a lawful act;
With due care;
He causes an injury by mere accident; and
Without fault or intention of causing it.
As such the discharge of the firearm was accidental since during the scuffle. He was ordered to
escort the victim to be interrogated, and that while going towards the interrogation room, Balboa
suddenly tried to gain control of Pomoy’s service handgun, as such, Pomoy tried to defend his
possession of the said firearm, since it may be used by the victim in escaping. All of these show
that Pomoy performed a lawful act with due care by trying to grapple for the gun’s possession.
However, because the gun was pointed to Balboa at the time it was fired, an accident occurred
which resulted in Balboa's death.
The Court also rejected the CA’s application of People v. Reyes in this case, since what was used
in the Reyes case was a revolver, while here in Pomoy, what was used was a semi-automatic
handgun, which is prone to misfire.

217
CRIMINAL LAW 1 CASE DIGESTS
ATTY. LYAN DAVD JUANICO
BLOCK 1-M A.Y. 2020 - 2021

20. US v. Apolonio Caballeros


G.R. No. 1352; March 29, 1905
Mapa, J.:

FACTS:
4 American teachers were assassinated and their bodies buried to hide the crime from being
discovered. Baculi was arrested for burying those bodies, although he stated that he did so due to
being forced by the perpetrators, as well as Caballeros, who was stated to have confessed his
liability and guilt before an official of the division of information of the Constabulary.

ISSUE:
Whether or not Baculi and Caballeros are liable for the crime committed.

RULING:
They are not liable. Baculi, was compelled by the perpetrators to bury the bodies of the dead
teachers, this was supported by the only witness who is a prosecution witness, who testified that
Baculi was at his plantation gathering bananas when the crime happened, that Baculi tried to run
away but was caught by the leaders of the band and was hit with the butt of the rifle the perpetrators
had before forcing him to bury the bodies.
As for Caballeros, both Baculi and the lone witness testified that he was nowhere to be found in
the crime scene. With regard to his purported confession, it cannot be accepted since the officer
who stated that Caballeros confessed to him also stated that it was only Baculi who testified.
Furthermore, such confession was only made so that no harm would come to Caballeros, as such,
it cannot be accepted since confessions which do not appear to have been made freely and
voluntarily, without force, intimidation, or promise of pardon, can not be accepted as proof on a
trial.

As a result, both Baculi and Caballeros were acquitted.

218
CRIMINAL LAW 1 CASE DIGESTS
ATTY. LYAN DAVD JUANICO
BLOCK 1-M A.Y. 2020 - 2021

21. People v. Eustaquio Loreno and Jimmy Marantal


G.R. No. L-54414; July 9, 1984
Concepcion J., J.:

FACTS:
Loreno and Marantal were convicted of the crime of robbery with double rape committed against
Elias Monge and his family, and Fable, Elias’ farm helper. On the night of the crime, Elias Monge
was approached and later captured by people who introduce themselves as members of the NPA,
one was later identified as Loreno, who was ordered to bind Elias and his family, Fable, also
identified Marantal as one of the lookouts, who later kicked him repeatedly before being led back
inside by Loreno. After a while, a co-conspirator led Monica, one of Elias’ daughters, upstairs onto
a room where he raped her. While this was happening, Loreno led Beata, Elias’ wife, first to the
masters room and then to the teacher’s room, where Loreno took everything worth of value. The
co-conspirator, after satisfying his lust towards Monica, led her down and made her join the other
while he took interest in Christina, another of Elias’ daughters, and led her towards the teacher’s
room, where he raped her. After the ordeal, Loreno entered the room and tried to kiss and touch
Christina’s private parts but was called by another co-conspirator since a man was seen
approaching.

Loreno and Marantal were later caught. In their defense, they were only made to participate in the
crime because of an irresistible force/uncontrollable fear, by reason that those “NPA Members”
threatened to harm their families.

ISSUES:
Whether or not there was irresistible force\uncontrollable fear.

RULING:
Neither of the two were present. A person who acts under the compulsion of an irresistible force,
like one who acts under the impulse of uncontrollable fear of equal or greater injury is exempt
from criminal liability because he does not act with freedom. The force must be irresistible to
reduce him to a mere instrument who acts not only without will but against his will.

From the facts of the case, Loreno was not forced to help in the consummation of the crime, he
alone led Beata to different rooms to take the valuables stored in them, he also tried to kiss and
touch Christina’s private parts when Christina was left alone in the room.

As for Marantal, he acted as a lookout, from his position in front of the house, he could have heard
Monica or Christina’s cries for help, furthermore, his act of being a lookout encouraged the

219
CRIMINAL LAW 1 CASE DIGESTS
ATTY. LYAN DAVD JUANICO
BLOCK 1-M A.Y. 2020 - 2021

commission of the crime of robbery and rape. Lastly, Marantal did not voice any protest nor did
any act to prevent the commission of the crime.
These acts show voluntary participation and the conspiracy of the respondents.

220
CRIMINAL LAW 1 CASE DIGESTS
ATTY. LYAN DAVD JUANICO
BLOCK 1-M A.Y. 2020 - 2021

22. People v. Joselito Del Rosario


G.R. No. 127755; April 14, 1999
Bellosillo, J.:

FACTS:
Del Rosario was a tricycle driver found guilty as a co-conspirator in the crime of robbery with
homicide. His defense was he was compelled to stay in the scene by reason of an irresistible force,
explaining that while Marquez and alias “Dodong” were committing the crime, Santos was inside
the tricycle pointing a gun at him, therefore, preventing him from escaping and calling for help.

The trial court rejected his defense of compulsion of an irresistible force, stating that his fear was
mere speculative, and therefore was not uncontrollable, and that the gun pointed at him did not
constitute irresistible force because it fell short of the test required by law and jurisprudence.

ISSUE:
Whether or not irresistible force is present.

RULING:
The fact that the gun was pointed to del Rosario during the commission of the crime rendered
him immobile and subject to Santos’ will, therefore proving that irresistible force was present.
As provided in the maxim , “Actus me invito factus non est meus actus”, or “An act done by me
against my will is not my act”, the fact that del Rosario was prevented by Santos to leave the
crime scene and further help them by driving them out of the scene cannot be blamed on del
Rosario, as he was forced to do so against his will.

221
CRIMINAL LAW 1 CASE DIGESTS
ATTY. LYAN DAVD JUANICO
BLOCK 1-M A.Y. 2020 - 2021

23. People v. Josefina Bandian


R. No. 45186; Sept. 30, 1936
Diaz, J.:

FACTS:
Bandian was seen going into a thicket and later emerging with her clothes stained with blood,
while she was staggering and visibly having problems supporting herself. Aguilar supported her
and helped her go up into her house. When Comcom tried to gather bamboo leaves to be used to
stop Bandian’s bleeding, he came upon the body of a newborn baby, which later on Bandian
confirmed as hers, near the path where Bandian was last seen. Dr. Nepomuceno went to Bandian’s
house and later deduced that Bandian gave birth inside her house, but later went into the thicket to
kill the child since it was fathered by a man other than her lover.

The lower court held Bandian guilty of infanticide, while the Solicitor-General believes she is
guilty of abandoning a minor under subsection 2 of Art. 276, RPC.

ISSUE:
Whether or not Bandian is guilty of infanticide or abandonment of a minor.

RULING:
She is not guilty of any of the two. For infanticide or abandonment of a minor to be punishable, it
must have been willfully committed by the offender. In this case, however, it was not shown that
Bandian, in causing the death of the newborn baby, did so willfully. She, together with her lover,
who affirmed that the child was his, were eagerly awaiting the birth of their child, as such, there is
no reason for her to be ashamed of the baby’s father.

Due to the fact that not only was she uneducated, but also had not experienced childbirth before,
were the reason why she did not notice that she had given birth while answering the call of nature.
And that by reason of the sudden dizziness she had experienced after giving birth, she was not able
to recover her newborn baby, leading to its demise.
She had, therefore, committed an injury by mere accident under Art. 12(4) of the RPC, which has
4 requisites:

1. A person is performing a lawful act;


2. With due care;
3. He causes an injury by mere accident; and
4. Without fault or intention of causing it.
As a result, she was acquitted.

222
CRIMINAL LAW 1 CASE DIGESTS
ATTY. LYAN DAVD JUANICO
BLOCK 1-M A.Y. 2020 - 2021

24. RAYMUND MADALI and RODEL MADALI v. PEOPLE OF THE PHILIPPINES


G. R. No. 180380; August 4, 2009
Chico-Nazario, J.

FACTS:
At around 5:30 in the afternoon of 13 April 1999, BBB, who made a living by selling goods aboard
ships docked at the Romblon Pier, and who was constantly assisted by her 15-year-old son AAA,
was on a ship plying her wares. AAA, together with Jovencio and petitioner Raymund, was there
helping his mother. Sometime later, Raymund and AAA left the ship. Jovencio stayed a little
longer. At about 9:00 p.m. of the same day, Jovencio and another friend named Michael Manasan
sat beside the Rizal monument in the Poblacion of Romblon, located between the Roman Catholic
Church and Lover’s Inn. Michael had just left Jovencio when Raymund, Rodel, Bernardino and
the victim AAA arrived. After meandering around, the group proceeded to climb the stairs, atop
of which was the reservoir just beside the Romblon National High School. The victim, AAA,
ascended first; behind him were Rodel, Raymund, Bernardino and witness Jovencio. As soon as
they reached the reservoir, Bernardino blindfolded AAA with the handkerchief of Raymund.
Bernardino at once blurted out, "Join the rugby boys." AAA replied, "That’s enough." Bernardino
then struck AAA thrice with a fresh and hard coconut frond. AAA lost his balance and was made
to stand up by Raymund, Rodel and Bernardino. Raymund took his turn clobbering AAA at the
back of his thighs with the same coconut frond. AAA wobbled. Before he could recover, he
received punches to his head and body from Rodel, who was wearing brass knuckles. The
punishments proved too much, as AAA lost consciousness. Not satisfied, Raymund placed his
handkerchief around the neck of AAA, with its ends tied to a dog chain. With the contraption, the
three malefactors pulled the body up a tree.
Stunned at the sight of his cousin being ill-treated, Jovencio could only muster a faint voice saying
"Enough" every single-time AAA received the painful blows. Bernardino, who seemed to suggest
finishing off the victim, remarked, "Since we’re all here, let’s get on with it." Before leaving the
scene, the three assailants warned Jovencio not to reveal the incident to anyone, or he would be
next. Jovencio hardly slept that night, and did not divulge the incident to anyone for the next few
days. BBB, the victim’s mother, was worried when her son did not come home. It was three days
later that a certain Eugenio Murchanto reported to the police authorities about a dead man found
in Barangay ZZZ near the Romblon National High School. When the policemen went there, they
found the cadaver emitting a foul odor, with maggots crawling all over, hanging from a tree with
a handkerchief tied around the neck and a dog chain fastened to the handkerchief. Also found in
the area were paraphernalia for inhaling rugby, as well as empty bottles of gin and a coconut frond.
The provincial hospital refused to conduct an autopsy, since AAA’s corpse was already
decomposing and stank so badly. It was through the intercession of the NBI that the body was
223
CRIMINAL LAW 1 CASE DIGESTS
ATTY. LYAN DAVD JUANICO
BLOCK 1-M A.Y. 2020 - 2021

eventually exhumed and examined by medico-legal experts. Dr. Floresto P. Arizala, Jr., who
conducted the examination, opined that the victim died due to head injuries and not to asphyxiation
by hanging. He declared that the victim was already dead when he was tied to the tree, and that
the variety of injuries sustained by the victim could be attributed to more than one assailant.
Jovencio narrated the incident and pointed to Raymund, Rodel and Bernardino as the perpetrators
of the crime. However, because of the threat made on him by a certain Wilson, an uncle of
Raymund and Rodel, Jovencio executed a second affidavit repudiating his first affidavit. On 28
May 1999, Jovencio made his third sworn statement substantially reverting to his first affidavit.
The accused, on the other hand, advanced the defense of denial and alibi, claiming that they had
nothing to do with the death of AAA, and that they were nowhere near the locus criminis when
the killing occurred. According to Rodel, 16 years old, he was with his father Rodolfo Madali in
the house of a friend named Noel Mindoro. Rodel’s testimony was corroborated by his father and
Noel Mindoro. Raymund, 14 years of age, and Bernardino declared that they were in their
respective houses on the night in question. Bernardino’s testimony was supported by his father
Bernardino Maestro, Sr. and by his neighbor Diana Mendez. Raymund’s friend, Pastor Mario
Fajiculay backed up the former’s alibi.
The Regional Trial Court of Romblon, Romblon ruled that the three accused were guilty of
homicide, but appreciated the privilege mitigating circumstance of minority in favor of them. Court
of Appeals affirmed the said decision. However, pursuant to Section 64 of Republic Act No. 9344,
otherwise known as the "Juvenile Justice and Welfare Act of 2006," Raymund’s case was
dismissed. Rodel’s conviction was sustained, but the imposition of said penalty was suspended
pursuant to Republic Act No. 9344. Petitioners contended.

ISSUE:
Whether or not petitioners Raymund and Rodel Madali are criminally liable.

RULING:
NO. Raymund Madali is exempt from criminal liability because he was only 14 years of age at the
time he committed the crime, he should be exempt from criminal liability and should be released
to the custody of his parents or guardian pursuant to Sections 6 and 20 of Republic Act No. 9344,
which provides that “a child fifteen years of age or under at the time of the commission of the
offense shall be exempt from criminal liability. However, the child shall be subjected to an
intervention program pursuant to Section 20 of this Act.” However, the exemption from criminal
liability herein established does not include exemption from civil liability, which shall be enforced

224
CRIMINAL LAW 1 CASE DIGESTS
ATTY. LYAN DAVD JUANICO
BLOCK 1-M A.Y. 2020 - 2021

pursuant to the second paragraph of Section 6, Republic Act No. 9344. He is exempt from criminal
liability, but his civil liability is not extinguished.
Moreover, although the crime was committed on 13 April 1999 and Republic Act No. 9344 took
effect only on 20 May 2006, the said law should be given retroactive effect in favor of Raymund
who was not shown to be a habitual criminal.
However, as to Rodel Madali, he cannot be exempt from both criminal and civil liabilities. It must
be borne in mind that he was 16 years old at the time of the commission of the crime. A
determination of whether he acted with or without discernment is necessary pursuant to Section 6
of Republic Act No. 9344, which provides that “a child above fifteen years but below eighteen 18
years of age shall likewise be exempt from criminal liability and be subjected to an intervention
program, unless he/she has acted with discernment, in which case, such child shall be subjected to
the appropriate proceedings in accordance with this Act.” Discernment is that mental capacity of
a minor to fully appreciate the consequences of his unlawful act. Such capacity may be known and
should be determined by taking into consideration all the facts and circumstances afforded by the
records in each case.
The Court of Appeals could not have been more accurate when it opined that Rodel acted with
discernment. Rodel, together with his cohorts, warned Jovencio not to reveal their hideous act to
anyone; otherwise, they would kill him. Rodel knew, therefore, that killing AAA was a
condemnable act and should be kept in secrecy. He fully appreciated the consequences of his
unlawful act. However, his criminal liability is mitigated pursuant to Article 68 of the Revised
Penal Code which states that: “the penalty to be imposed upon a person under 18 but above 15
shall be the penalty next lower than that prescribed by law, but always in the proper period.”
However, the sentence to be imposed against Rodel should be suspended pursuant to Section 38
of Republic Act No. 9344, which states that “once the child who is under eighteen years of age at
the time of the commission of the offense is found guilty of the offense charged, the court shall
determine and ascertain any civil liability which may have resulted from the offense committed.
However, instead of pronouncing the judgment of conviction, the court shall place the child in
conflict with the law under suspended sentence, without need of application. Provided, however,
that suspension of sentence shall still be applied even if the juvenile is already eighteen 18 years
of age or more at the time of the pronouncement of his/her guilt.”
Raymund Madali is exempted from criminal liability. With respect to Rodel Madali, being a child
in conflict with the law, his sentence is suspended and remanded his case to the court a quo for
further proceedings in accordance with Section 38 of Republic Act No. 9344.

225
CRIMINAL LAW 1 CASE DIGESTS
ATTY. LYAN DAVD JUANICO
BLOCK 1-M A.Y. 2020 - 2021

25. PEOPLE OF THE PHILIPPINES v. FLORENCIO AGACER, EDDIE AGACER,


ELYNOR AGACER, FRANKLIN AGACER, AND ERIC AGACER
G. R. No. 177751; January 7, 2013
Del Castillo, J.

FACTS:
The victim Cesario Agacer was clearing a section of his farm in Cagayan. He was also preparing
the beddings for the rice seedlings for the coming planting season. Genesis and Roden, witnesses
for the prosecution, claimed that it was at this moment when the respondents suddenly emerged
and surrounded the victim. Visibly intimidated, he moved backwards and retreated to where the
farm laborers were working. Franklin Agacer set afire the rice straws covered the victim’s
seedlings. As a result, he returned to put out the fire. Together with Eric Agacer, Franklin threw
stones at the victim which forced the latter to retreat again. Florencio Agacer, while standing side
by side with Eric, signaled the victim to come closer, to which the letter obliged. However, when
he was five meters away from the group, Eddie Agacer suddenly pulled out a gun concealed inside
a sack, and without warning, shot the victim hitting him in the left portion of his chest. Almost
simultaneously, Elynor Agacer took aim at the victim with his bow and arrow but missed. As the
victim fell, respondents fled towards the irrigation canal, where another gunshot rang. Thereafter,
a short firearm was thrown from where the respondents ran towards. Respondents then
immediately left the scene of the crime on board a hand tractor and a tricycle.
The witnesses and the other farm laborers scattered in different directions. Genesis reached
Barangay Capakian, and informed the victim’s son, Neldison Agacer. During the autopsy, a total
of eight entrance wounds were found, mostly on the chest. Fatal gunshots were inflicted by the use
of a firearm capable of discharging several slugs simultaneously.
The Regional Trial Court of Aparri, Cagayan held that respondents acted in conspiracy and found
them guilty beyond reasonable doubt of the crime of murder qualified by treachery. Court of
Appeals affirmed the said ruling in all aspects. The Supreme Court also affirmed the said decision
on December 14, 2011.
Respondents filed a motion for reconsideration asserting that their mere presence at the scene of
the crime is not evidence of conspiracy; that there was no treachery since a heated argument
preceded the killing of the victim; and that even assuming that their guilt was duly established, the
privileged mitigating circumstance of minority should have been appreciated in favor of appellant
Franklin who was only 16 years and 106 days old at the time of the incident, having been born on
December 21, 1981.

226
CRIMINAL LAW 1 CASE DIGESTS
ATTY. LYAN DAVD JUANICO
BLOCK 1-M A.Y. 2020 - 2021

ISSUE:
Whether or not the mitigating circumstance of minority be appreciated in favor of Franklin
Agacer?

RULING:
YES. As a Minor, Franklin Agacer is entitled to the privileged mitigating circumstance of
minority. The Supreme Court agrees with respondents that Franklin is entitled to the privileged
mitigating circumstance of minority. Franklin’s Certificate of Live Birth shows that he was born
on December 20, 1981, hence, was merely 16 years old at the time of the commission of the crime
on April 2, 1998. He is therefore entitled to the privileged mitigating circumstance of minority
embodied in Article 68 (2) of the Revised Penal Code, which provides that when the offender is a
minor over 15 and under 18 years, the penalty next lower than that prescribed by law shall be
imposed on the accused but always in the proper period. The rationale of the law in extending such
leniency and compassion is that because of his age, the accused is presumed to have acted with
less discernment. This is regardless of the fact that his minority was not proved during the trial and
that his birth certificate was belatedly presented for our consideration, since to rule accordingly
will not adversely affect the rights of the state, the victim and his heirs.
Respondent Franklin Agacer is sentenced to suffer the penalty often years of prision mayor in its
medium period, as minimum, to seventeen years and four months of reclusion temporal in its
medium period, as maximum.

227
CRIMINAL LAW 1 CASE DIGESTS
ATTY. LYAN DAVD JUANICO
BLOCK 1-M A.Y. 2020 - 2021

26. VICKY C. TY v. PEOPLE OF THE PHILIPPINES


G. R. No. 149275; September 27, 2004
Tinga, J.

FACTS:
The evidence for the prosecution shows that Ty’s mother Chua Lao So Un was confined at the
Manila Doctors’ Hospital from October 30, 1990 until June 4, 1992. Being the patient’s daughter,
Ty signed the "Acknowledgment of Responsibility for Payment" in the Contract of Admission
dated October 30, 1990. As of June 4, 1992, the Statement of Account shows the total liability of
the mother in the amount of ₱657,182.40. Ty’s sister, Judy Chua, was also confined at the hospital
from 13 May 1991 until May 2, 1992, incurring hospital bills in the amount of ₱418,410.55. The
total hospital bills of the two patients amounted to ₱1,075,592.95. On June 5, 1992, Ty executed
a promissory note wherein she assumed payment of the obligation in installments. To assure
payment of the obligation, she drew several postdated checks against Metrobank payable to the
hospital. The seven checks, each covering the amount of ₱30,000.00, were all deposited on their
due dates. But they were all dishonored by the drawee bank and returned unpaid to the hospital
due to insufficiency of funds, with the "Account Closed" advice. Soon thereafter, the complainant
hospital sent demand letters to Ty by registered mail. As the demand letters were not heeded,
complainant filed the seven Informations subject of the instant case.
Ty claimed that she issued the checks because of "an uncontrollable fear of a greater injury." She
averred that she was forced to issue the checks to obtain release for her mother whom the hospital
inhumanely and harshly treated and would not discharge unless the hospital bills are paid. She
alleged that her mother was deprived of room facilities, such as the air-condition unit, refrigerator
and television set, and subject to inconveniences such as the cutting off of the telephone line, late
delivery of her mother’s food and refusal to change the latter’s gown and bedsheets. She also
bewailed the hospital’s suspending medical treatment of her mother. The "debasing treatment,"
she pointed out, so affected her mother’s mental, psychological and physical health that the latter
contemplated suicide if she would not be discharged from the hospital. Fearing the worst for her
mother, and to comply with the demands of the hospital, Ty was compelled to sign a promissory
note, open an account with Metrobank and issue the checks to effect her mother’s immediate
discharge.
The Regional Trial Court of Manila rendered a decision finding Ty guilty of seven counts of
violation of B.P. 22 and sentencing her to a prison term. Ty interposed an appeal from the Decision
of the trial court. Before the Court of Appeals, Ty reiterated her defense that she issued the checks
"under the impulse of an uncontrollable fear of a greater injury or in avoidance of a greater evil or
injury." She also argued that the trial court erred in finding her guilty when evidence showed there
was absence of valuable consideration for the issuance of the checks and the payee had knowledge
228
CRIMINAL LAW 1 CASE DIGESTS
ATTY. LYAN DAVD JUANICO
BLOCK 1-M A.Y. 2020 - 2021

of the insufficiency of funds in the account. She protested that the trial court should not have
applied the law mechanically, without due regard to the principles of justice and equity. Court of
Appeals affirmed the judgment of the trial court with modification.

ISSUE:
Whether or not petitioner Ty is criminally liable.

RULING:
YES. Petitioner Vicky Ty is criminally liable. For the exempting circumstance of uncontrollable
fear to be invoked successfully, the following requisites must concur: (1) existence of an
uncontrollable fear; (2) the fear must be real and imminent; and (3) the fear of an injury is greater
than or at least equal to that committed. It must appear that the threat that caused the uncontrollable
fear is of such gravity and imminence that the ordinary man would have succumbed to it. It should
be based on a real, imminent or reasonable fear for one’s life or limb. A mere threat of a future
injury is not enough. It should not be speculative, fanciful, or remote. A person invoking
uncontrollable fear must show therefore that the compulsion was such that it reduced him to a mere
instrument acting not only without will but against his will as well. It must be of such character as
to leave no opportunity to the accused for escape.
In the case at bar, the first requisite is absent because to begin with, there was no showing that the
mother’s illness was so life-threatening such that her continued stay in the hospital suffering all its
alleged unethical treatment would induce a well-grounded apprehension of her death. Secondly, it
is not the law’s intent to say that any fear exempts one from criminal liability much less petitioner’s
flimsy fear that her mother might commit suicide. In other words, the fear she invokes was not
impending or insuperable as to deprive her of all volition and to make her a mere instrument
without will, moved exclusively by the hospital’s threats or demands. Also, the fear, if any,
harbored by Ty was not real and imminent. Ty claims that she was compelled to issue the checks,
a condition the hospital allegedly demanded of her before her mother could be discharged, for fear
that her mother’s health might deteriorate further due to the inhumane treatment of the hospital or
worse, her mother might commit suicide. This is speculative fear; it is not the uncontrollable fear
contemplated by law.
Ty has also failed to convince the Court that she was left with no choice but to commit a crime.
She did not take advantage of the many opportunities available to her to avoid committing one. By
her very own words, she admitted that the collateral or security the hospital required prior to the
discharge of her mother may be in the form of postdated checks or jewelry. And if indeed she was

229
CRIMINAL LAW 1 CASE DIGESTS
ATTY. LYAN DAVD JUANICO
BLOCK 1-M A.Y. 2020 - 2021

coerced to open an account with the bank and issue the checks, she had all the opportunity to leave
the scene to avoid involvement. Moreover, petitioner had sufficient knowledge that the issuance
of checks without funds may result in a violation of B.P. 22. She even testified that her counsel
advised her not to open a current account nor issue postdated checks "because the moment I will
not have funds it will be a big problem." Besides, apart from petitioner’s bare assertion, the record
is bereft of any evidence to corroborate and bolster her claim that she was compelled or coerced
to cooperate with and give in to the hospital’s demands.
As to the contention with regard to the justifying circumstance of avoidance of greater evil or state
of necessity, the Court also disagrees. For this justifying circumstance to be invoked successfully,
the following requisites must concur: (1) that the evil sought to be avoided actually exists; (2) that
the injury feared be greater than the one done to avoid it; (3) that there be no other practical and
less harmful means of preventing it.
In the case at bar, the first requisite is absent because the evil sought to be avoided is merely
expected or anticipated. If the evil sought to be avoided is merely expected or anticipated or may
happen in the future, this defense is not applicable. Ty could have taken advantage of an available
option to avoid committing a crime. By her own admission, she had the choice to give jewelry or
other forms of security instead of postdated checks to secure her obligation. Moreover, for the
defense of state of necessity to be availing, the greater injury feared should not have been brought
about by the negligence or imprudence, more so, the willful inaction of the actor.34 In this case,
the issuance of the bounced checks was brought about by Ty’s own failure to pay her mother’s
hospital bills.

230
CRIMINAL LAW 1 CASE DIGESTS
ATTY. LYAN DAVD JUANICO
BLOCK 1-M A.Y. 2020 - 2021

27. PEOPLE OF THE PHILIPPINES v. HENRY ARPON y JUNTILLA


G. R. No. 183563; December 14, 2011
Leonardo-De Castro, J.

FACTS:
In one afternoon when AAA was only eight years old, she stated that the respondent Henry Arpon
raped her inside their house. The respondent stripped off her shorts, panties and shirt and went on
top of her. He had his clothes on and only pulled down his zipper. He then pulled out his organ,
put it in her vagina and did the pumping motion. AAA felt pain but she did not know if his organ
penetrated her vagina. When he pulled out his organ, she did not see any blood. She did so only
when she urinated. AAA also testified that the respondent raped her again in July 1999 for five
times on different nights. The respondent was then drinking alcohol with BBB, the stepfather of
AAA, in the house of AAA’s neighbor. He came to AAA’s house, took off her panty and went on
top of her. She could not see what he was wearing as it was nighttime. He made her hold his penis
then he left. When asked again how the respondent raped her for five nights in July of the said
year, AAA narrated that he pulled down her panty, went on top of her and pumped. She felt pain
as he put his penis into her vagina. Every time she urinated, thereafter, she felt pain. AAA said that
she recognized the respondent as her assailant since it was a moonlit night and their window was
only covered by cloth. He entered through the kitchen as the door therein was detached. AAA
further related that the respondent raped her again twice in August 1999 at nighttime. He kissed
her and then he took off his shirt, went on top of her and pumped. She felt pain in her vagina and
in her chest because he was heavy. She did not know if his penis penetrated her vagina. She related
that the respondent was her uncle as he was the brother of her mother. AAA said that she did not
tell anybody about the rapes because the respondent threatened to kill her mother if she did. She
only filed a complaint when he proceeded to also rape her younger sister, DDD.
Upon the other hand, the defense called the respondent to the witness stand to deny the
informations filed against him and to refute the testimony of AAA. He testified that when the first
incident of rape allegedly happened in 1995, he was only 13 years old as he was born on February
23, 1982. In 1995, he worked in Sagkahan, Tacloban City as a houseboy for a certain Gloria
Salazar and he stayed there up to 1996. He stated that he was working in Tacloban City when the
alleged rapes happened in the municipality of XXX. When he would go home from Tacloban, he
would stay at the house of a certain Fred Antoni. He did not go to the house of AAA as the latter’s
parents were his enemies. He said that he had a quarrel with AAA’s parents because he did not
work with them in the ricefields. He further recounted that in July 1999, he was also living in
Tacloban City and worked there as a dishwasher at a restaurant. He worked there from 1998 up to
September 1999. The respondent likewise stated that in August 1999, he was still working at the
same restaurant in Tacloban City. While working there, he did not go home to XXX as he was

231
CRIMINAL LAW 1 CASE DIGESTS
ATTY. LYAN DAVD JUANICO
BLOCK 1-M A.Y. 2020 - 2021

busy with work. He denied that he would have drinking sprees with AAA’s stepfather, BBB,
because they were enemies.
The Regional Trial Court of Tacloban City rendered a decision convicting respondent of one count
of statutory rape and seven counts of rape. Respondent filed a Motion for Reconsideration of the
said Decision, asserting that the trial court failed to consider his minority as a privileged mitigating
circumstance. Court of Appeals promulgated the assailed decision.
ISSUE:
Whether or not respondent Henry Arpon is criminally liable.
RULING:
YES. Respondent Henry Arpon is not fully exempt from criminal liability. However, he is exempt
from the first instance of rape, and his criminal liability for the second and third instances are
mitigated. Pertinently, the first paragraph of Section 7 of Republic Act No. 9344 provides for the
rule on how to determine the age of a child in conflict with the law, stating that “the child in conflict
with the law shall enjoy the presumption of minority. He/She shall enjoy all the rights of a child
in conflict with the law until he/she is proven to be eighteen years of age or older.” Furthermore,
in Sierra v. People, the Court clarified that, in the past, the Court deemed sufficient the testimonial
evidence regarding the minority and age of the accused provided the following conditions concur,
namely: "(1) the absence of any other satisfactory evidence such as the birth certificate, baptismal
certificate, or similar documents that would prove the date of birth of the accused; (2) the presence
of testimony from accused and/or a relative on the age and minority of the accused at the time of
the complained incident without any objection on the part of the prosecution; and (3) lack of any
contrary evidence showing that the accused's and/or his relatives' testimonies are untrue."
In the case at bar, the respondent testified that he was born on February 23, 1982 and that he was
only 13 years old when the first incident of rape allegedly happened in 1995. Other than his
testimony, no other evidence was presented to prove the date of his birth. However, the records of
this case show neither any objection to the said testimony on the part of the prosecution, nor any
contrary evidence to dispute the same. Thus, the Regional Trial Court and the Court of Appeals
should have appreciated the respondent’s minority in ascertaining the appropriate penalty.
Although the acts of rape in this case were committed before Republic Act No. 9344 took effect
on May 20, 2006, the said law is still applicable given that Section 68 thereof specifically provides
for its retroactive application. People v. Sarcia further stressed that "with more reason, the Act
should apply to a case wherein the conviction by the lower court is still under review."

232
CRIMINAL LAW 1 CASE DIGESTS
ATTY. LYAN DAVD JUANICO
BLOCK 1-M A.Y. 2020 - 2021

Thus, in the matter of assigning criminal responsibility, Section 6 of Republic Act No. 9344 is
explicit in providing that “a child fifteen years of age or under at the time of the commission of
the offense shall be exempt from criminal liability. However, the child shall be subjected to an
intervention program pursuant to Section 20 of the Act. A child above fifteen years but below
eighteen years of age shall likewise be exempt from criminal liability and be subjected to an
intervention program, unless he/she has acted with discernment, in which case, such child shall be
subjected to the appropriate proceedings in accordance with this Act.” The exemption from
criminal liability herein established does not include exemption from civil liability, which shall be
enforced in accordance with existing laws. The above provision effectively modified the minimum
age limit of criminal irresponsibility in paragraphs 2 and 3 of the Revised Penal Code, as amended.
Accordingly, for the first count of rape, the testimony of the respondent sufficiently established
that he was only 13 years old at that time. In view of the failure of the prosecution to prove the
exact date and year of the first incident of rape, any doubt therein "should be resolved in favor of
the accused, it being more beneficial to the latter." The Court, thus, exempts the respondent from
criminal liability for the first count of rape pursuant to the first paragraph of Section 6 of Republic
Act No. 9344. The respondent, nevertheless, remains civilly liable therefor.
For the second and third counts of rape that were committed in the year 1999, the respondent was
already 17 years old. We likewise find that in the said instances, the respondent acted with
discernment. In Madali v. People, the Court had the occasion to reiterate that “discernment is that
mental capacity of a minor to fully appreciate the consequences of his unlawful act. Such capacity
may be known and should be determined by taking into consideration all the facts and
circumstances afforded by the records in each case." In the case at bar, the fact that the respondent
acted with discernment was satisfactorily established by the testimony of AAA, which we had
already found to be credible. Verily, AAA testified that she at first did not tell anybody about the
sexual assault she suffered at the hands of the respondent because the latter told her that he would
kill her mother if she did so. That the respondent had to threaten AAA in an effort to conceal his
dastardly acts only proved that he knew full well that what he did was wrong and that he was aware
of the consequences thereof.
In accordance with the second paragraph of Article 68 of the Revised Penal Code, as amended,
and in conformity with our ruling in Sarcia, when the offender is a minor under eighteen years of
age, "the penalty next lower than that prescribed by law shall be imposed, but always in the proper
period. However, for purposes of determining the proper penalty because of the privileged
mitigating circumstance of minority, the penalty of death is still the penalty to be reckoned with."
Thus, for the second and third counts of rape, the proper penalty imposable upon the respondent
is reclusion perpetua for each count. Had the trial court correctly appreciated in favor of the
respondent the circumstance of his minority, the latter would have been entitled to a suspension of

233
CRIMINAL LAW 1 CASE DIGESTS
ATTY. LYAN DAVD JUANICO
BLOCK 1-M A.Y. 2020 - 2021

sentence for the second and third counts of rape under Section 38 of Republic Act No. 9344, which
provides for the rules on automatic suspension of sentence. Upon suspension of sentence and after
considering the various circumstances of the child, the court shall impose the appropriate
disposition measures as provided in the Supreme Court Rule on Juvenile in Conflict with the Law.
Be that as it may, the suspension of sentence may no longer be applied in the instant case given
that the respondent is now about 29 years of age and Section 40 of Republic Act No. 9344 puts a
limit to the application of a suspended sentence, namely, when the child reaches a maximum age
of 21.
Nonetheless, the disposition set forth under Section 51 of Republic Act No. 9344 is warranted in
the instant case, which provides for the confinement of convicted children. Additionally, the civil
liability of the respondent for the second and third incidents of rape shall not be affected by the
above disposition and the same shall be enforced in accordance with law and the pronouncements
in the prevailing jurisprudence.
For the first count of rape herein established, the accused-appellant Henry Arpon y Juntilla is
hereby exempted from criminal liability. For the second and third counts of rape, the respondent
is found guilty beyond reasonable doubt of two counts of qualified rape and is hereby sentenced
to suffer the penalty of reclusion perpetua for each count.

234
CRIMINAL LAW 1 CASE DIGESTS
ATTY. LYAN DAVD JUANICO
BLOCK 1-M A.Y. 2020 - 2021

28. PEOPLE OF THE PHILIPPINES v. REY MONTICALVO y MAGNO


G. R. No. 193507; January 30, 2013
Perez, J.

FACTS:
In the afternoon of 9 December 2002, AAA and her friend, Analiza, were in front of the sari-sari
store of AAA’s mother, BBB, while respondent was inside the fence of their house adjacent to the
said sari-sari store. Shortly, thereafter, respondent invited AAA to go with him to the kiln at the
back of their house. AAA acceded and went ahead. Upon seeing respondent and AAA going to
the kiln, Analiza, pretending to look for her one peso coin, followed them until she reached a
papaya tree located three and a half meters away from the place. Analiza hid under the papaya tree
and from there she saw respondent undress AAA by removing the latter’s shorts and panty.
Respondent, however, glanced and saw Analiza. Frightened, Analiza ran away and went back to
the sari-sari store of BBB without telling BBB what she saw. Respondent proceeded to satisfy his
bestial desire. After undressing AAA, respondent made her lie down. He then placed himself on
top of AAA and made push and pull movements. Afterwards, respondent stopped, allowed AAA
to sit down for a while and then sent her home. When AAA arrived at their house around 7:30
p.m., she was asked by her mother, BBB, where she came from and why she came home late. AAA
replied that she was at the back of their house as respondent brought her there and had sexual
intercourse with her.
The following day, BBB brought AAA to the police station and then to the Northern Samar
Provincial Hospital where AAA was examined by Dr. Nochete. Dr. Nochete explained that AAA
could have possibly sustained those complete healed hymenal lacerations more than a month prior
to the date of the examination, but also clarified that even though AAA has no fresh hymenal
laceration it does not necessarily mean that no sexual intercourse was committed on her on 9
December 2002. It is possible that AAA did not sustain any fresh hymenal laceration because the
vaginal canal has become loose. He did not also find any trace of spermatozoa on AAA’s vagina.
AAA was also examined by Dr. Belicena, who found that AAA is suffering from moderate to
severe mental retardation, meaning, AAA is suffering from the specific form of below average
intelligence that has a low reproduction functioning resulting in impaired functioning. Dr.
Belicena, however, recommended a full battery of psychological testing to determine AAA’s exact
mental age.
Respondent denied having raped AAA, claiming that on December 9, 2002, at around 1:00 p.m.,
he, together with Pio and a certain Dinnes Samson, was having a drinking spree in the house of
one Adolfo Congayao. They finished drinking at around 6:00 p.m. As he was too drunk, Pio
assisted him in going home. He went to sleep and woke up only at 12:00 midnight as he needed to

235
CRIMINAL LAW 1 CASE DIGESTS
ATTY. LYAN DAVD JUANICO
BLOCK 1-M A.Y. 2020 - 2021

urinate. He went back to sleep and woke up at 6:00 a.m. of the following day. He was surprised
that AAA charged him with rape. He was then arrested at around 3:00 p.m. of 10 December 2002.
Alexander, a defense witness, presented respondent’s Certificate of Live Birth to prove that the
latter was only 17 years old during the commission of the crime.
The Regional Trial Court of Catarman, Northern Samar found the respondent guilty beyond
reasonable doubt of the crime of rape of a demented person. The Court of Appeals affirmed the
said decision in toto.
ISSUE:
Whether or not respondent Rey Monticalvo is criminally liable.
RULING:
YES. Respondent Rey Monticalvo is not fully exempt from criminal liability. However, his
criminal liability is mitigated because the Court finds merit in respondent’s assertion that he was
a minor during the commission of the crime charged. Respondent’s Certificate of Live Birth,
containing the facts of his birth, shows that he was born on February 23, 1985. Indeed, at the time
of the commission of the crime charged, respondent was only 17 years old, a minor. Thus, he is
entitled to the privileged mitigating circumstance of minority pursuant to Article 68 (2) of the
Revised Penal Code, as amended.
On 20 May 2006, Republic Act No. 9344 took effect. Section 68 thereof specifically provides for
its retroactive application. The said law is applicable in this case even though the crime was
committed four years prior to its enactment and effectivity. Parenthetically, with more reason
should Republic Act No. 9344 apply to this case as the 2005 conviction by the lower courts was
still under review when the law took effect in 2006.
Moreover, Section 38 of Republic Act No. 9344 warrants the suspension of sentence of a child in
conflict with the law notwithstanding that he/she has reached the age of majority at the time the
judgment of conviction is pronounced. Upon suspension of sentence and after considering the
various circumstances of the child, the court shall impose the appropriate disposition measures as
provided in the Supreme Court Rule on Juveniles in Conflict with the Law. However, while
Section 38 of Republic Act No. 9344 provides that suspension of sentence can still be applied even
if the child in conflict with the law is already eighteen years of age or more at the time of the
pronouncement of his/her guilt, Section 40 of the same law limits the said suspension of sentence
until the said child reaches the maximum age of 21.
At present, appellant is already 27 years of age, and the judgment of the trial court was promulgated
prior to the effectivity of Republic Act No. 9344. Therefore, the application of Sections 38 and 40

236
CRIMINAL LAW 1 CASE DIGESTS
ATTY. LYAN DAVD JUANICO
BLOCK 1-M A.Y. 2020 - 2021

of the said law is already moot and academic. Be that as it may, to give meaning to the legislative
intent of Republic Act No. 9344, the promotion of the welfare of a child in conflict with the law
should extend even to one who has exceeded the age limit of 21 years, so long as he/she committed
the crime when he/she was still a child. The offender shall be entitled to the right to restoration,
rehabilitation and reintegration in accordance with Republic Act No. 9344 in order that he/she is
given the chance to live a normal life and become a productive member of the community. The
age of the child in conflict with the law at the time of the promulgation of the judgment of
conviction is not material. What matters is that the offender committed the offense when he/she
was still of tender age. The appellant, therefore, shall be entitled to appropriate disposition under
Section 51 of Republic Act No. 9344, which provides for the confinement of convicted children.
To conform to this Court’s ruling in People v. Sarcia, the case shall be remanded to the court of
origin to effect appellant’s confinement in an agricultrual camp or other training facility.
Respondent is found guilty of rape under subparagraph (b) of Article 266-A of the Revised Penal
Code, as amended, and not under subparagraph (d) thereof; and in view of the privileged mitigating
circumstance appreciated in favor of respondent, the penalty of reclusion perpetua is reduced to
reclusion temporal.

237
CRIMINAL LAW 1 CASE DIGESTS
ATTY. LYAN DAVD JUANICO
BLOCK 1-M A.Y. 2020 - 2021

MODULE 7: MITIGATING CIRCUMSTANCES

1. THE PEOPLE OF THE PHILIPPINES v. NICOLAS JAURIGUE and AVELINA


JAURIGUE
C. A. No. 384; February 21, 1946
De Joya, J.

FACTS:
Herein respondent appellant Avelina Jaurigue and her family lived in the barrio of Sta. Isabel, City
of San Pablo, Laguna. The deceased Amado Capina, which was stabbed by the respondent on
September 20, 1942, also lived in the said barrio. Prior to the said stabbing incident, Capina had
been courting the respondent in vain; and about a month before the fatal night, he snatched a
handkerchief belonging to the respondent, bearing her nickname “Aveling,” while it was being
washed by respondent’s cousin, Josefa Tapay.
On September 13, 1942, Capina approached respondent while she was feeding a dog under her
house. The former then spoked of his love, and he thereupon embraced and kissed her. He also
touched respondent’s breast. Due to this, respondent, a resolute and quick-tempered girl, slapped
Capina, gave him fist blows and kicked him. The following morning, she informed her mother
about it. Since then, she armed herself with a long fan knife whenever she went out, for purposes
of self-protection.
On September 15, 1942, Capina climbed up to the house of the respondent and entered her room
while the latter is asleep. The respondent felt her forehead with the intention of raping her; she
immediately screamed for help, which awakened her parents and brough them to her side. Capina
then came out from under the bed, where he was hiding. He then kissed the hand of the
respondent’s father to beg for forgiveness.
On the night of September 20, 1942, respondent’s family went to the local Church where it was
quite bright. Respondent was left alone in the bench because her father tended to do some business.
Capina then sat beside the respondent and placed his hand on top of her thigh. Observing this
highly inappropriate conduct, respondent stabbed Capina in the neck, causing a single wound from
which he died. Respondent thereafter surrendered without question.
Nicolas Jaurigue and Avelina Jaurigue were prosecuted in the Court of First Instance of Tayabas,
for the crime of murder, of which Nicolas Jaurigue was acquitted, but respondent Avelina Jaurigue
was found guilty of homicide.

238
CRIMINAL LAW 1 CASE DIGESTS
ATTY. LYAN DAVD JUANICO
BLOCK 1-M A.Y. 2020 - 2021

ISSUES:
1. Whether or not the respondent’s act shall be justified based on complete self-defense.
2. Whether or not there are mitigating circumstances present in the case at bar.
3. Whether or not there are aggravating circumstances present in the case at bar.

RULING:
1. NO. The respondent’s act does not constitute the justifying circumstance of self-defense; therefore,
her act shall not be justified. There are three requisites before the justifying circumstance of self-
defense shall be granted: (1) unlawful aggression on the part of the victim; (2) reasonable necessity
of the means employed by the accused; and (3) lack of sufficient provocation on the part of the
accused.

With regard to the first requisite, as long as there is actual danger of being raped, unlawful
aggression is present. Therefore, a woman is justified in killing her aggressor in the defense of her
honor. However, in the case at bar, when the deceased sat by the side of defendant and appellant
on the same bench, near the door of the barrio chapel and placed his hand on the upper portion of
her right thigh, without her consent, the said chapel was lighted with electric lights, and there were
already several people, about 10 of them in number, inside the chapel, including her own father
and the barrio lieutenant. Under the circumstances, there was and there could be no possibility of
her being raped. Therefore, unlawful aggression on the part of the victim, the first requisite, is not
present.

Moreover, unlawful aggression is a conditio sine qua non for self-defense to be appreciated.
Without unlawful aggression, the accused has nothing to prevent or repel, and there is then no
basis for appreciating the other two requisites. Therefore, there can be no self-defense, complete
or incomplete, in the absence of unlawful aggression.

However, if the respondent had killed Capina when the latter climbed up her house late at night
and surreptitiously entered her bedroom, she could have been justified for killing him if the means
she used was reasonable.

2. YES. The Supreme Court granted three mitigating circumstances in favor of the accused:
(1) voluntary surrender; (2) vindication of a grave offense; and (3) praeter intentionem.

For voluntary surrender to be appreciated as a mitigating circumstance, the following requisites


should be present: (1) the offender has not been actually arrested; (2) the offender surrendered

239
CRIMINAL LAW 1 CASE DIGESTS
ATTY. LYAN DAVD JUANICO
BLOCK 1-M A.Y. 2020 - 2021

himself to a person in authority or the latter's agent; and (3) the surrender was voluntary. All the
three requisites are present in the case at bar. This is due to the fact that respondent immediately
and voluntarily and unconditionally surrendered to the barrio lieutenant (second requisite) in said
chapel, admitting having stabbed the deceased, immediately after the incident (first requisite), and
agreed to go to her house shortly thereafter and to remain there subject to the order of the said
barrio lieutenant, an agent of the authorities (third requisite).

With regard to vindication of a grave offense for it to be appreciated as a mitigating circumstance,


the following requisites must be present: (1) victim committed a grave offense; (2) grave offense
was committed against the offender or his spouse, ascendants, descendants, legitimate,
illegitimate, or adopted brothers or sisters, or his relative by affinity within the same degree; and
(3) offender committed the crime in proximate vindication of such grave offense. All the three
requisites are also present in the case at bar. First, the deceased committed a grave offense in three
instances: (1) embracing, kissing, and touching of breasts; (2) trespassing with intention to rape;
and (3) placing of hand in thigh. Second, the grave offense was done against the respondent. Lastly,
respondent committed the act of killing on proximate vindication of the offenses because of the
further fact that she had acted in the immediate vindication of a grave offense committed against
her a few moments before, and upon such provocation as to produce passion and obfuscation, or
temporary loss of reason and self-control. Even in the absence of the third incident, vindication
still allows for a specific lapse of time.

Lastly, praeter intentionem shall also be considered because she had not intended to kill the
deceased but merely wanted to punish his offending hand with her knife, as shown by the fact that
she inflicted upon him only one single wound.

3. NO. There are no aggravating circumstances present in the case at bar. The claim of the
prosecution that the offense was committed by the defendant and appellant, with the aggravating
circumstance that the killing was done in a place dedicated to religious worship, cannot be legally
sustained; as there is no evidence to show that the defendant and appellant had murder in her heart
when she entered the chapel that fatal night. Avelina is not a criminal by nature. She happened to
kill under the greatest provocation. She is a God-fearing young woman, typical of our country
girls, who still possess the consolation of religious hope in a world where so many others have
hopelessly lost the faith of their elders and now drifting away, they know not where.
Respondent committed the crime of homicide, with no aggravating circumstance whatsoever,
but with at least three mitigating circumstances of a qualified character to be considered in her
favor; and, in accordance with the provisions of article 69 of the Revised Penal Code, she is entitled
to a reduction by one or two degrees in the penalty to be imposed upon her. The law prescribes the
penalty of reclusion temporal for the crime of homicide; and if it should be reduced by two degrees,
the penalty to be imposed in the instant case is that of prision correccional; and pursuant to the
240
CRIMINAL LAW 1 CASE DIGESTS
ATTY. LYAN DAVD JUANICO
BLOCK 1-M A.Y. 2020 - 2021

provisions of section 1 of Act No. 4103 of the Philippine Legislature, known as the
Indeterminate Sentence Law, herein defendant and appellant should be sentenced to an
indeterminate penalty ranging from arresto mayor in its medium degree, to prision correccional in
its medium degree.

241
CRIMINAL LAW 1 CASE DIGESTS
ATTY. LYAN DAVD JUANICO
BLOCK 1-M A.Y. 2020 - 2021

2. PEOPLE OF THE PHILIPPINES v. MAMERTO NARVAEZ


G.R Nos. 33466-67; April 20, 1983
Makasiar, J.

FACTS:
At about 2:30 in the afternoon of August 22, 1968, Graciano Juan, Jesus Verano and Cesar Ibanez
together with the two deceased Davis Fleischer and Flaviano Rubia, were fencing the land of
George Fleischer, father of deceased Davis Fleischer. The place was in the boundary of the
highway and the hacienda owned by George Fleischer. This is located in the municipality of
Maitum, South Cotabato. At the place of the fencing is the house and rice drier of respondent
Mamerto Narvaez. At that time, respondent was taking his rest, but when he heard that the walls
of his house were being chiselled, he arose and there he saw the fencing going on. If the fencing
would go on, respondent would be prevented from getting into his house and the bodega of his
ricemill. So he addressed the group, saying 'Pare, if possible you stop destroying my house and if
possible we will talk it over what is good,' addressing the deceased Rubia, who is respondent's
compadre. The deceased Fleischer, however, answered: 'No, gademit, proceed, go ahead.'
Respondent apparently lost his equilibrium and he got his gun and shot Fleischer, hitting him. As
Fleischer fell down, Rubia ran towards the jeep, and knowing there is a gun on the jeep, respondent
fired at Rubia, likewise hitting him. Both Fleischer and Rubia died as a result of the shooting'.
Respondent surrendered to the police thereafter, bringing with him a shotgun, and claiming he shot
two persons.
It appears, however, that this incident is intertwined with the long-drawn-out legal battle between
the Fleischer and Co., Inc. of which deceased Fleischer was the secretary-treasurer and deceased
Rubia the assistant manager, on the one hand, and the land settlers of Cotabato, among whom was
respondent.
The Court of First Instance of South Cotabato convicted the respondent with the crime of Murder
qualified by treachery with the aggravating circumstance of evident premeditation offset by the
mitigating circumstance of voluntary surrender.

ISSUES:
1. Whether or not the respondent’s act shall be justified based on complete self-defense.
2. Whether or not there are mitigating circumstances present in the case at bar.

242
CRIMINAL LAW 1 CASE DIGESTS
ATTY. LYAN DAVD JUANICO
BLOCK 1-M A.Y. 2020 - 2021

RULING:
1. NO. The respondent’s act does not constitute the justifying circumstance of self-defense; therefore,
his act shall not be justified. There are three requisites before the justifying circumstance of self-
defense shall be granted: (1) unlawful aggression on the part of the victim; (2) reasonable necessity
of the means employed by the accused; and (3) lack of sufficient provocation on the part of the
accused.

The second requisite of self-defense is absent in the case at bar. The reasonableness of the
resistance is also a requirement of the justifying circumstance of self-defense or defense of one's
rights under paragraph 1 of Article 11, Revised Penal Code. When the respondent fired his shotgun
from his window, killing his two victims, his resistance was disproportionate to the attack.

Be that as it may, respondent’s act in killing the deceased was not justifiable, since not all the
elements for justification are present. He should therefore be held responsible for the death of his
victims, but he could be credited with the special mitigating circumstance of incomplete defense,
pursuant to paragraph 6, Article 13 of the Revised Penal Code.

2. YES. The Supreme Court granted three mitigating circumstances in favor of the accused:
(1) incomplete self-defense; (2) voluntary surrender; and (3) passion and obfuscation.

Pursuant to paragraph 6, Article 13 of the Revised Penal Code, respondent’s act in killing the
deceased was not justifiable, since not all the elements for justification are present. He should
therefore be held responsible for the death of his victims, but he could be credited with the special
mitigating circumstance of incomplete defense. The aggression referred to by respondent is the
angry utterance by deceased Fleischer of the following words: "Hindi, sigue, gademit, avante", in
answer to his request addressed to his compadre, the deceased Rubia, when he said, "Pare, hinto
mona ninyo at pag-usapan natin kung ano ang mabuti." This was in reaction to his having been
awakened to see the wall of his house being chiselled. The verbal exchange took place while the
two deceased were on the ground doing the fencing and the respondent was up in his house looking
out of his window. According to respondent, Fleischer's remarks caused this reaction in him: "As
if, I lost my senses and unknowingly I took the gun on the bed and unknowingly also I shot Mr.
Fleischer, without realizing it, I shot Mr. Fleischer." The third element of defense of property is
present, lack of sufficient provocation on the part of respondent who was defending his property.
As a matter of fact, there was no provocation at all on his part, since he was asleep at first and was
only awakened by the noise produced by the victims and their laborers. His plea for the deceased
and their men to stop and talk things over with him was no provocation at all. The first requisite,
which is a conditio sine qua non, and third requisite of self-defense are present. This means that

243
CRIMINAL LAW 1 CASE DIGESTS
ATTY. LYAN DAVD JUANICO
BLOCK 1-M A.Y. 2020 - 2021

majority of the requisites are present in the case at bar. Therefore, respondent is entitled to a
mitigating circumstance of incomplete self-defense.

For voluntary surrender to be appreciated as a mitigating circumstance, the following requisites


should be present: (1) the offender has not been actually arrested; (2) the offender surrendered
himself to a person in authority or the latter's agent; and (3) the surrender was voluntary. All the
three requisites are present in the case at bar. The trial court has properly appreciated the presence
of the mitigating circumstance of voluntary surrender, it appearing that appellant surrendered to
the authorities soon after the shooting.

Lastly, for passion and obfuscation to be appreciated as a mitigating circumstance, the following
requisites should be present: (1) there be an act both unlawful and sufficient to produce a condition
of the mind; (2) said act which produced the obfuscation was not far removed from the of the crime
by a considerable period of time, during which the perpetrator might recover his equanimity; and
(3) passion or obfuscation must arise from lawful sentiments. The appellant awoke to find his
house being damaged and its accessibility to the highway as well as of his rice mill bodega being
closed. Not only was his house being unlawfully violated; his business was also in danger of
closing down for lack of access to the highway. These circumstances, coming so near to the time
when his first house was dismantled, thus forcing him to transfer to his only remaining house, must
have so aggravated his obfuscation that he lost momentarily all reason causing him to reach for his
shotgun and fire at the victims in defense of his rights. Considering the antecedent facts of this
case, where appellant had thirty years earlier migrated to this so-called "land of promise" with
dreams and hopes of relative prosperity and tranquility, only to find his castle crumbling at the
hands of the deceased, his dispassionate plea going unheeded-all these could be too much for any
man-he should be credited with this mitigating circumstance.
Consequently, respondent is guilty of two crimes of homicide only, the killing not being
attended by any qualifying nor aggravating circumstance but extenuated by the privileged
mitigating circumstance of incomplete defense-in view of the presence of unlawful aggression on
the part of the victims and lack of sufficient provocation on the part of the appellant-and by two
generic mitigating circumstance of voluntary surrender and passion and obfuscation.
Respondent is guilty beyond reasonable doubt of only two homicides, mitigated by
privileged extenuating circumstance of incomplete self-defense as well as by two mitigating
circumstances of voluntary surrender and passion and obfuscation, without any aggravating
circumstance.

244
CRIMINAL LAW 1 CASE DIGESTS
ATTY. LYAN DAVD JUANICO
BLOCK 1-M A.Y. 2020 - 2021

3. People v. Ulep
G.R. no. 132547; 20 September 2000
J. Bellosillo

FACTS:
On December 22, 1995, at around 2:00 am, A man named Buenaventura Wapili was in his room,
talking to himself and experiencing high fever. When his brother-in-law, Dario Leydan heard
Wapili, the former tried to convince the latter to go out of his room and talk with the former. Wapili
told Leydan that he could not understand himself and went back to his room.

Moments later, Leydan heard loud noises coming from Wapili’s room. The sounds were akin to
smashing furniture. Leydan tried to calm down Wapili but was unable to do so. Leydan called
Pastor Bonid from the Alliance Church of Kidapawan to help him pacify Wapili. They were unable
to enter the room because Wapili was already too wild and violent. While this was happening,
Wapili emerged from his room naked and chased Leydan. With the help of two neighbors, Leydan
tried to tie Wapili using a rope to pacify the latter. This attempt was unsuccessful due to Wapili’s
size and strength. Wapili suddenly ran away from them and kept on running toward no particular
direction.

Norma Plando, a policewoman and their neighbor, asked for assistance from her fellow policemen.
She contacted SP01 Ernesto Ulep, SPO1 Edilberto Espadera and SPO2 Crispin Pillo using a hand-
held radio to secure the premises of the nearby area. The police officers arrived at around 4:00 in
the morning of the same day. SP01 Ulep, SPO1 Espadera and SPO2 Pillo rode an Anfra police
service jeep. They went down the vehicle when they saw Wapili running towards them. According
to the police officers, Wapili was armed with a bolo and a rattan stool. This was disputed by the
other witnesses, since he was only holding a rattan stool.

In order to stop Wapili, SPO1 Ulep fired a warning shot in the air and told him to put down his
weapon or else he will be shot. Wapili retorted “pusila” or “fire” and continued advancing towards
Ulep and the police officers. SPO1 shot the Wapili when he was about two or three meters away
from them. Wapili stopped in his tracks since he was hit in various parts of his body. It was in this
position that SPO1 Ulep came closed to him and shot him in the head.

ISSUE:
Whether or not the presence of ½ of the requisites shall constitute a majority in accordance with
Article 69 and entitles him to a mitigating circumstance under Article 13(1)

245
CRIMINAL LAW 1 CASE DIGESTS
ATTY. LYAN DAVD JUANICO
BLOCK 1-M A.Y. 2020 - 2021

RULING:
Yes, The court held that the incomplete justifying circumstance of fulfillment of a duty or lawful
exercise of right in Article 11 paragraph 5 of the Revised Penal Code can be appreciated in favor
of the victim.

The requisites of the justifying circumstance of fulfillment of a duty or lawful exercise of right are
1) That the accused acted in the performance of a duty or in the lawful exercise of a right or office
and; 2) That the injury caused or the offense committed be the necessary consequence of the due
performance of duty or the lawful exercise of such right or office.

In the case at bar, The second requisite is wanting. The moment Ulep shot the victim, hitting and
disabling him. The aggression had already ceased. There was no necessity for the accused to shoot
the victim in the head after he fell to the ground. Only one out of two requisites needed was
fulfilled.

Article 69 of the Revised Penal Code provides that for a penalty to be lowered by one or two
degrees, the majority of the conditions be present. Since there are only two requisites in Article 11
paragraph 5, one out of two requisites already constitute a majority. Thus, this constitutes
Incomplete justification as a special or privileged mitigating circumstance.

In addition to incomplete justification as a mitigating circumstance, The court also credits


Voluntary surrender as a mitigating circumstance since immediately after killing the victim,
Wapili, Ulep himself reported his acts and voluntarily surrendered.

246
CRIMINAL LAW 1 CASE DIGESTS
ATTY. LYAN DAVD JUANICO
BLOCK 1-M A.Y. 2020 - 2021

4. Guillermo v. People,
G.R. No. 153287; 20 January 2009
J. Brion

FACTS:
On July 21, 1996 at 5:40 pm in the afternoon, Winnie Alon, Wilfredo Cabison, Eddie Roque and
Vicente Alon were at Melecio Heyres’ restaurant. The accused, Noel Guillermo, Arnel Socias and
Joemar Palma were also in the same restaurant drinking beer. Guillermo’s group invited Winnie’s
group over to sit with them since they knew each other from childhood. Before going to the
restaurant, Both Vicente and Winnie were already drunk and intoxicated.

An altercation between Arnel and Winnie took place when they discussed the manner of cutting
of wood. Noel took hold of Winnie and stabbed him in the neck three times. Joemar, on the other
hand, went to the kitchen and got a knife. Arnel hit Vicente’s head with a beer bottle and the latter
fell down and lost consciousness.

According to the accused, it was Winnie who started the fight. Winnie challenged Arnel to a wood-
cutting contest, to which the latter refused. Arnel refused the challenge. Winnie told arne; “You
are already old in that business, but your finished product is still crooked. You are all dumb." Arnel
replied, "If the wood itself is crooked, you cannot have a straight lumber. You are dumb if you
insist you can." Winnie took offense at this remark and grabbed Arnel by the collar. Accused Noel
tried to stop the fight and told them to settle things peacefully. It was then when Winnie hit the
petitioner with a beer bottle three times. He stood up and Winnie struck him with a broken bottle.
Noel stepped back and pulled out his knife and struck Winnie. Noel justifies his act of stabbing
Winnie as self-defense.

ISSUE:
Whether or not petitioner Noel Guillermo is entitled to the privileged mitigating circumstance of
incomplete self defense.

RULING:
Yes. As a justifying circumstance, self-defense may be complete or incomplete. It is considered as
a complete self defense when all the requisites are present. On the other hand, it is considered
incomplete self-defense when unlawful aggression is present along with any one of the two
essential requisites.

247
CRIMINAL LAW 1 CASE DIGESTS
ATTY. LYAN DAVD JUANICO
BLOCK 1-M A.Y. 2020 - 2021

In the case at bar, It was undisputed that it was Winnie Alon, the deceased, who started the fight.
Winnie was the unlawful aggressor in attacking the accused by striking him with beer bottles. The
accused act to pacifying the victim and his friend was lawful. Thus, the requisite of unlawful
aggression is present. The second requisite is also present since it was established that there was
no provocation made by the accused because he was only trying to pacify the quarrel between
Winnie and Arnel.

The third element of self-defense, which is the reasonableness of the means to repel the aggression,
was lacking. The victim attacked the accused with beer bottles and his fists while the latter
defended himself with a knife. There is an obvious disproportion between the weapons employed
by the parties, more so in the hands of the drunk victim, who held it clumsily. There was
no rational equivalence between the means of the attack and the means of defense employed by
the accused.

Since the only lacking requisite is the rational equivalence of the means employed. The petitioner
is entitled to the privileged mitigating circumstance of incomplete self-defense.

248
CRIMINAL LAW 1 CASE DIGESTS
ATTY. LYAN DAVD JUANICO
BLOCK 1-M A.Y. 2020 - 2021

5. People v. Ural
G.R. No. L-30801, 27 March 1974
J. Aquino

FACTS:
On July 31, 1966, Brigido Alberto saw Domingo Ural boxing Felix Napola. Napola collapsed on
the floor after taking too much of the blows. Ural stepped on Napola after boxing him and he went
out of the cell.

He returned with a bottle and poured its contents on Napola’s body, who was still on the floor.
Then, he lit it with a match and left Napola. Napola screamed in agony and shouted for help but
nobody came to help him

Alberto left the building. He was disturbed by the events he saw. Before he departed, Ural
cautioned him not to tell anyone what he saw. Ural was allegedly drunk when he committed the
acts.

ISSUE:
Whether or not Article 13 par 3 may be credited as a mitigating circumstance.

RULING:
Yes. The trial court failed to appreciate the mitigating circumstance “that the offender had no
intention to commit so grave a wrong as that committed” as stated in Par 3, Article 13 of the
Revised Penal Code.

In the case at bar, Ural had no intention of killing Napola. It was manifest from the facts that he
only intended to maltreat him. When Ural realized that Napola had caught fire, He allowed Napola
to secure medical treatment.

However, The court ruled that this is covered by Article 4 of the revised Penal code which provides
that “criminal liability shall be incurred by any person committing a felony although the wrongful
act done be different from which he intended.” Ural is still liable for the consequences of his
actions, that is the death of Felix Napola.

Thus, the mitigating circumstance of “no intent to commit so grave a wrong as that committed”
may be credited in favor of Ural.

249
CRIMINAL LAW 1 CASE DIGESTS
ATTY. LYAN DAVD JUANICO
BLOCK 1-M A.Y. 2020 - 2021

6. People v. Gonzales,
G.R. No. 139542, 21 June 2001
J. Gonzaga-Reyes

FACTS:
On October 31, 1998, The families of Noel Andres and Inocencio Gonzales went to Loyola
Memorial park. After they finished paying their respects to their loved ones, they went on their
way home at around 2:30pm. At the intersection near the Garden of Remembrance, Their vehicles
families almost collided because Noel Andres was driving straight while Gonzales was turning left
towards the exit. Noel stepped on the breaks to prevent a crash from happening.

Noel knocked on Innocencio’s car window and told him to be careful with driving. Gonzales
answered back ad when Noel saw the former turning red, he went back to his car. It was here when
Innocencio’s son Dino blocked Noel’s way to confront him. Andres felt threatened so he
immediately boarded his car, leaving his window open to talk to Dino.

While Noel was arguing with Dino, Innocencio took out the gun he had in his car compartment.
Trisha, his daughter tried to stop him from joining the fight. He tried to free his hand, which held
the gun and weighed down by his daughter. In the process of grappling with his daughter for the
gun, Innocencio lost his balance and accidentally fired his gun. The shot hit the tinted last window
on the left side of the Noel’s Tamaraw FX.

One of his passengers said, “Binaril kami.” Noel turned around and saw his wife, bloodied and
unconscious; and his son and his nephew wounded. Andres did not hear the shot because both
Dino and him were shouting while arguing. He brought his family to the hospital to get sufficient
treatment. His son, Kenneth and his nephew, Kevin were treated but his pregnant wife, Feliber
Andres died from the gunshot. Gonzales was charged with the complex crime of Murder with
Double Frustrated Murder and Attempted Murder

ISSUE:
Whether or not there are mitigating circumstances present in the case at bar.

RULING:
No, the court did not appreciate any mitigating circumstances.

The accused, Innocencio, argues that the lower court erred in not crediting the following mitigating
circumstances: Voluntary surrender, passion and obfuscation, incomplete defense of a relative and

250
CRIMINAL LAW 1 CASE DIGESTS
ATTY. LYAN DAVD JUANICO
BLOCK 1-M A.Y. 2020 - 2021

lack of intent to commit so grave a wrong. However, the court agrees with the lower court that it
should not be appreciated.

Voluntary Surrender
Witness testimonies contradicts the presence of this mitigating circumstance. According to
Witness Ramos, Innocencio Gonzales tried to escape by driving away towards the exit. He was
prevented only by Andres and the other onlookers blocking his path.

Passion and Obfuscation


Passion and Obfuscation was not present in the case at bar. The court held that Noel Andres’ act
of shouting at Dino was not sufficient to produce passion and obfuscation. It is important to not
that Dino was answering back and was not completely helpless and oppressed. Noel’s aggressive
behavious towards them may be demeaning or humiliating but it could not be considered as
sufficiently provoking for them to shoot at his vehicle.

Incomplete Defense of a Relative


For Incomplete Defense of a relative the following requisites should be present: 1) unlawful
aggression and either 2) reasonable necessity of the means employed to prevent or repeal it; or
3)In case the provocation was given by the person attacked, the one making the defense had no
part therein. In the case at bar, Noel Andres’ act of cursing and shouting at Innocencio and Dino
Gonzales do not constitute an unlawful aggression towards them. Since unlawful aggression is an
indispensable element in self-defense, whether complete or incomplete, the absence of which
means that there is no self-defense to speak of.

Lack of intent to commit so grave a wrong


The court did not credit the presence of this mitigating circumstance. Lack of intent to commit so
grave a wrong is present when there is a notable disparity between the means employed by the
accused to commit a wrong and the resulting crime committed. In the case at bar, Innocencio had
a gun. His use of a gun was sufficient to produce the resulting crimes committed and there was no
notable disparity between his use of a gun and the injuries caused to Noel’s family.
However, the court also held that the killing was not attendant with treachery, since what had
happened was a chance encounter between the two parties. The court took into consideration that
the shooting was preceded by a heated argument. In such cases, There is no treachery present since
the accused could not have deliberately employed a treacherous mode of attack. Hence, The crime
was homicide and two counts of slight physical injuries.
In sum, The court finds that there are no mitigating circumstances present in the case at bar but
modifies the lower court’s decision since the crime was homicide and two counts of slight physical
injuries.

251
CRIMINAL LAW 1 CASE DIGESTS
ATTY. LYAN DAVD JUANICO
BLOCK 1-M A.Y. 2020 - 2021

7. People v. Pagal,
G.R. No. L-32040, 25 October 1977
J. Concepcion Jr.

FACTS:
Pagal and Torcelino were the employees of the victim, Gau Guan. Gau Guan allegedly maltreated
his two employees. On the evening of December 26, 1969, Pagal and Torcelino tried to rob their
employer, Guan, but the latter did not open the safe containing the money. They tortured Gau Guan
by stabbing him with an ice pick and clubbing him with an iron pipe on different parts of his body.
Gau Guan died as a result of his wounds.

The accused confessed to their crimes and information was filed against them for the crime of
robbery with homicide. They pleaded guilty during their arraignment and contend that they did the
acts due to passion and obfuscation resulting from the victim’s maltreatment of them. The lower
court did not appreciate the presence of the mitigating circumstance and found the accused guilty
of Robbery with Homicide and sentenced them to death.

ISSUE:
What are the Modifying circumstances present in the case at bar?

RULING:
The court held that the Mitigating circumstance of Plea of Guilty and the Aggravating
circumstance of Nocturnity are present in the case at bar.

PASSION AND OBFUSCATION


The court held that the circumstance of passion and obfuscation cannot be mitigating in a crime
that is planned and calmly meditated before its execution. In People v. Daos. The court did not
heed the appellant’s claim that passion and obfuscation should be credited in ther favor, because
the death of the victim took place on the occasion of a planned and calmly meditated robbery made
by the appellants. Similar to the case at bar, Pagal and Torcelino planned and calmly meditated the
robbery they committed before proceeding to torture and kill the victim.

PROVOCATION
Pagal and Torcelino also aver that the root cause of their Provocation was due to the maltreatment
they received from the victim. The court does not consider this as provocation a mitigating
circumstance. The court held that Provocation must be sufficient and immediately preceeding the
act. In the case at bar, the maltreatment that both Pagal and Torcelino experience in the hands of
Gau Guan occurred much earlier than the date of the commission of the crime.

252
CRIMINAL LAW 1 CASE DIGESTS
ATTY. LYAN DAVD JUANICO
BLOCK 1-M A.Y. 2020 - 2021

DISRESPECT OF AGE and NOCTURNITY


The aggravating circumstance of disrespect of age is not present in the case at bar. The mitigating
circumstance of disrespect of age is only applicable in crimes against persons. In the case at bar,
Pagal and Torcelino committed Robbery with Homicide. Robbery with homicide is a crime against
property and as such, disrespect of age cannot be appreciated in the case at bar.

The trial court correctly appreciated Nocturnity as an aggravating circumstance. Pagal and
Torcelino purposely and deliberately sought the cover of the night perpetuate the crime of Robbery
with homicide.

The court finds that the mitigating circumstance of Plea of Guilty and the aggravating circumstance
of Nocturnity present in the case at bar.

253
CRIMINAL LAW 1 CASE DIGESTS
ATTY. LYAN DAVD JUANICO
BLOCK 1-M A.Y. 2020 - 2021

8. Urbano v. People,
G.R. No. 182750, 20 January 2009
J. Velasco, Jr.

FACTS:
On September 28, 1993, Brigido Tomelden, Rodel Urbano and their co-workers were drinking
beer in a restaurant in the compound of the Lingayen Water District. During the course of their
drinking session, Tomelden and Urbano had a heated argument. Tomelden insulted Urbano and
the latter asked him why he was always being made fun of when the former was drunk.

The argument quickly escalated into a fistfight between the two. Their co-workers was successful
in temporarily pacifying the two. After a while, Tomelden and Urbano started with their fist fight
again. It was at this moment that Urbano delivered a “lucky punch” which made Tomelden, who
was bigger than Urbano, topple down. Tomelden almost hit his head on the ground had not his co-
workers prevented his fall. The blow was not without effect. The lucky punch delivered by Urbano
made Tomelden’s nose bleed and rendered him unconscious.

Upon being brought home, He told his wife of the fight and complained of pain in his nape, head,
and ear. He was immediately brought to the hospital. He went in and out of the hospital for the
next few days complaining of dizziness, headache, and other pains. The doctors observed that
Tomelden was in a state of drowsiness and frequent vomiting. Upon his last discharge on October
10, 1993, He again complained of extreme head pain. His wife brought him back to the hospital.
Things took a turn for the worst and Brigido Tomelden died as a result.

ISSUE:
Whether or not the lower court erred in not appreciating the mitigating circumstances of sufficient
provocation on the part of the victim and lack of intent to commit so grave a wrong in favor of the
petitioner

RULING:
Yes the lower court erred in not appreciating the mitigating circumstances.

PROVOCATION
When the law speaks of provocation either as a mitigating circumstance or as an element of self-
defense, the reference is to an unjust or improper conduct of the offended party that is capable of
exciting anyone to do wrong and should immediately precede the act.

254
CRIMINAL LAW 1 CASE DIGESTS
ATTY. LYAN DAVD JUANICO
BLOCK 1-M A.Y. 2020 - 2021

In the case at bar, Tomelden’s insults directed at Urbano, uttered before the fistfight constituted
sufficient provocation. Based on the witnesses accounts, It was Tomelden who challenged Urbano
to fight. Urbano tried to avoid fighting with Tomelden since the latter was bigger than he was.

LACK OF INTENTION TO COMMIT SO GRAVE A WRONG


The mitigating circumstance of lack of intention to commit so grave a wrong should also be
appreciated in favor of Urbano. While intent to kill may be presumed from the death of the victim,
this mitigating circumstance may still be considered in view of attendant facts and circumstance
surrounding the case.

In the case at bar, Urbano tried to avoid fighting the victim on account of their varying statures.
He tried to parry Tomelden’s blows although he was able to land a lucky punch on the latter. There
is also that fact that he was one of those who helped carry the unconscious Tomelden to the office
of the general manager. Urbano’s acts were far from trying to harm or kill the victim.

Thus, the mitigating of circumstances of sufficient provocation on the part of the victim and lack
of intent to commit so grave a wrong should be appreciated in favor of the accused Urbano.

255
CRIMINAL LAW 1 CASE DIGESTS
ATTY. LYAN DAVD JUANICO
BLOCK 1-M A.Y. 2020 - 2021

9. People v. Benito,
G.R. No. L-32042, 13 February 1975
J. Aquino

FACTS:
Alberto Benito was a former employee of the Civil Service Commission Main Branch. He was
suspended for Dishonesty. After two months , he was reinstated but was criminally charged with
QUALIFIED THEFT, MALVERSATION OF PUBLIC FUNDS, ESTAFA and
FALSIFICATION OF PUBLIC DOCUMENTS and administratively charged for
"DISHONESTY" culminating in his dismissal.

Prior to the dismissal of Benito, It was reported to Pedro Moncayo that Benito may be stealing the
money from the Civil Service Commission by stealing the money orders. Moncayo allegedly
confronted Benito about the matter and the latter admitted to malversing an amount between 4,000
and 5,000 from his sales of examination fee stamps. Moncayo later wrote a report on it, leading to
the dismissal of Benito.

Benito then went to Pedro Moncayo to ask for help with his dismissal and his cases. Moncayo
refused to help him and threatened him saying, “"UMALIS KA NGA DIYAN BAKA MAY
MANGYARI PA SA IYO AT BAKA IPAYARI KITA DITO".

Benito returned the following morning at 11:00 in the morning of December 12, 1969. When
Moncayo saw him, the former remarked in front of many people, “Nagiistambay pala dito ang mga
magnanakaw” (Hangout of thieves). Benito was humiliated and left.

At around 5:25 pm of the same day, Benito went back to the Civil Service Commission. He waited
for Moncayo outside the compound. Moncayo arrived in his car and Benito followed him. When
Moncayo’s car stopped, Benito shot him eight (8) times in the head and several parts of his body.
Moncayo died on the spot.

ISSUE:
Whether or not sufficient provocation attendant in the case at bar.

RULING:
No there was no sufficient provocation present. Sufficient provocation is present when the
following requisites are present: 1) the provocation must be sufficient 2) The provocation
originates from the offended party 3) the provocation must be immediate to the act. In the case at

256
CRIMINAL LAW 1 CASE DIGESTS
ATTY. LYAN DAVD JUANICO
BLOCK 1-M A.Y. 2020 - 2021

bar, the remark was made at 11 o clock in the morning and the murder happened at 5’o clock in
the afternoon. There was a six hour interval between the alleged grave offense and the murder.
The court held that six hours was enough time for Benito to calm down but instead of pulling
himself together, Benito used the six-hour interval to plan the victim’s demise. Furthermore, there
was no showing that the victim said the remark directly to Benito, since he was the presence of a
lot of people. The facts of the case stongly suggest that it was not the remark that pushed Benito
to murder Moncayo but the latter’s refusal to help Benito with the cases filed against him. Thus,
Sufficient Provocation was not present in the case at bar.

257
CRIMINAL LAW 1 CASE DIGESTS
ATTY. LYAN DAVD JUANICO
BLOCK 1-M A.Y. 2020 - 2021

10.Bacabac v. People
G.R. No. 149372, 11 September 2007
CARPIO-MORALES, J:

FACTS:
On the night of December 23, 1990, the victim, Hernani Quidato, was at a dance hall in Purok 4,
San Joaquin, Iloilo City in the company of Eduardo Selibio and Melchor Selibio. And so were
Jonathan Bacabac and Edzel Talanquines who later on left the dance hall. Not long after, the victim
and his companions also left and on their way home, they encountered Jonathan and Edzel. There,
it appears that the two groups had a misunderstanding.

On the same night, Jesus Delfin Rosadio, who was also at the dance hall and on his way home,
noticed a commotion. He soon saw that Melchor was "hugging" Edzel, and later "tying" Jonathan
"with his hands." Still later, he saw the victim hit Edzel with a "stick." Jesus then told the victim
and his companions that Edzel is the son of Councilor Jose Talanquines, Jr. However, Eduardo
ignored this and threaten Jesus to go away for they might shoot him. Jesus therefore left and went
to Edzel's residence to report to his father what he had witnessed.

Meanwhile, Edzel and Jonathan managed to flee. Then the victim and his companions left headed
for home. On their way they met the petitioner, Pat. Ricardo Bacabac, together with Edzel and
Jonathan who are his nephews, and Edzel's father, Jose, his mother, and two sisters at the corner
of M.H. Del Pilar and Sto. Domingo Streets. The petitioner and Jose were carrying M-16
armalites, while Jonathan and Edzel were carrying a piece of wood and a revolver. Jesus thereupon
pointed to the victim and his companions as the ones who had manhandled Jonathan and Edzel.

The victim apologized and explained that he and his companions mistook Jonathan and Edzel for
other persons. However, Jesus blurted out that he victim and his companions are just bragging that
they are brave and was only bullying small children. At that moment, the petitioner, fired his
armalite into the air, while Jose fired his armalite at the victim and Eduardo, even hitting Jonathan
in the thigh as he was on the move to strike with a piece of wood. Eduardo fell and so did the
victim who was in a kneeling position, and as he was raising his hands in surrender, Jose shot him
again. Melchor was able to escape.

The victim, Eduardo, and Jonathan were brought to the hospital. The victim was pronounced dead
on arrival. Eduardo died two hours later.

On April, 30, 1993 the RTC of Iloilo held that the petitioner and his co-accused are guilty of
murder qualified by treachery.

258
CRIMINAL LAW 1 CASE DIGESTS
ATTY. LYAN DAVD JUANICO
BLOCK 1-M A.Y. 2020 - 2021

ISSUES:
Whether or not the mitigating circumstances of immediate vindication of a grave offense is present
in the case at bar.

RULING:
No, the mitigating circumstances of immediate vindication of a grave offense is not present in the
case at bar.

Likewise, the court stated that the petitioner's invocation of the mitigating circumstance of
"immediate vindication of a grave offense," it fails.

Article 13, paragraph 5 of the Revised Penal Code states that:

“5. That the act was committed in the immediate vindication of a grave offense to the one
committing the felony (delito), his spouse, ascendants, descendants, legitimate, natural, or adopted
brothers or sisters, or relatives by affinity within the same degrees.”

Based on the statements of Edzel and Jesus, the former was hit on his ear and not on his head.
Thus, it will not constitute and classified as a “grave offense”

Furthermore, Edzel, is the petitioners nephew, and such is not a relative by affinity within the same
degree as declared in Article 13 of the RPC shown above.

259
CRIMINAL LAW 1 CASE DIGESTS
ATTY. LYAN DAVD JUANICO
BLOCK 1-M A.Y. 2020 - 2021

11. US v. Hicks
G.R.No.4971, 23 September 1909
TORRES, J:

FACTS:
September, 1902, to November, 1907, Augustus Hicks and Agustina Sola, illicitly lived together
in the municipality of Parang, Cotabato, Moro Province. After about 4 years of relationship trouble
arise between then and on 1907, Agustina left Hick’s house and went to live with her brother-in-
law.

On the 21st of December, Augustus Hicks together with a soldier named Lloyd Nickens called at
said house, and from the sala called out to his old mistress, Agustina, who was in her room with
Corporal Current, and after conversing with her in the Moro dialect for a few minutes, asked the
corporal to come out of said room in which the corporal appeared at the door of the room, and after
a short conversation, the exchange of word got heated leading to a situation where Hicks draw his
revolver from his trousers' pocket. However, Corporal Current was able to see it soon enough to
stop him but Hicks snatched his hand away causing the Corporal Current to hide himself behind
the partition, just as Hicks drew his revolver and fired at Agustina Sola who was close by in the
sala of the house. The bullet struck her in the left side of the breast and died a little more than an
hour later

Hicks immediately fled from the house and gave himself up to the chief of police of the town.
Later he was charged with the crime of murder and during trial, he pleaded not guilty.

ISSUE:
Whether or not there are mitigating circumstances present in the case at bar.

RULING:
No, the court held that there are no mitigating circumstances present in this case.

The accused, Augustus Hicks, cannot invoke loss of reason and self-control produced by jealousy
(passion or obfuscation, Art 13, paragraph 6 of the RPC) as alleged by the defense, inasmuch as
the only causes which mitigate the criminal responsibility for the loss of self-control are such as
originate from legitimate feelings, not those which arise from vicious, unworthy, and immoral
passions.

This is because evidence was found that before the day of the crime, Hicks was heard repeatedly
saying that the time of Agustina has and that he would rather see her dead than in the arms of

260
CRIMINAL LAW 1 CASE DIGESTS
ATTY. LYAN DAVD JUANICO
BLOCK 1-M A.Y. 2020 - 2021

another man. And later, on the after the crime the police found on a table in the culprit’s house
several loaded cartridges, a bottle of oil and a piece of cloth used undoubtedly for cleaning the
revolver. Proving that Hicks deliberately and after due reflection had resolved to kill the woman
who had left him for another man.

The court provided that Hicks “was already provided with a clean and well-prepared weapon and
carried other loaded cartridges besides those already in his revolver, he entered the house, greeting
everyone courteously and conversed with his victim, in what appeared to be a proper manner,
disguising his intention and calming her by his apparent repose and tranquility, doubtless in order
to successfully accomplish his criminal design, behaving himself properly as he had planned to do
beforehand.”

Furthermore, the allegations of the accused that he only pulled out his revolver because the
Corporal Current was the first to do so was proven in variance with the testimony of the four
witnesses further proving that the shot that struck the Agustina in the breast and caused her death
was not due to an accident but to a willful and premeditated act on the part of the aggressor with
intent to deprive the victim of her life.

261
CRIMINAL LAW 1 CASE DIGESTS
ATTY. LYAN DAVD JUANICO
BLOCK 1-M A.Y. 2020 - 2021

12.US v. Dela Cruz


G.R.No.7094, 29 March 1912
CARSON, J:

FACTS:
In the case at bad, the appellant, Hilario Dela Cruz, was convicted of the crime of homicide and
was sentenced to fourteen years eight months and one day of reclusion temporal, the medium
degree of the penalty prescribed by the code.

Based in the evidence presented it discloses that the convict, in the heat of passion, killed the
deceased, who had theretofore been his querida (concubine or lover) upon discovering her in
flagrante in carnal communication with a mutual acquaintance.
However, the court believed that the mitigating circumstance stated now in Article 13, subsection
6 of the Revised Penal Code should have been taken into consideration.

ISSUE:
Whether or not the accused can invoke the mitigating circumstance of passion and obfuscation.

RULING:
Yes, Hilario Dela Cruz may invoke the mitigating circumstance of passion and obfuscation.
Article 13, paragraph 6 of the RPC states that:

“6. That of having acted upon an impulse so powerful as naturally to have produced passion or
obfuscation.”

The evidence disclosed that in the heat of passion Dela Cruz killed the deceased, who had
theretofore been his querida, upon discovering her in flagrante in carnal communication with a
mutual acquaintance.

The court also distinguished this case from U.S. vs. Hicks (G.R.No.4971). In which the alleged
passion and obfuscation of the aggressor was the convict's vexation, disappointment and anger by
the refusal of the woman to continue to live in illicit relations with him. It was also held in the case
of U.S. vs. Hicks that the "causes which mitigate the criminal responsibility for the loss of self-
control are such as originate from legitimate feelings, not those which arise from vicious,
unworthy, and immoral passions.”

However, in the case at bar, the actions of Dela Cruz resulted by his impulse and which naturally
"produced passion and obfuscation" was not by the fact that his querida has declined to have illicit

262
CRIMINAL LAW 1 CASE DIGESTS
ATTY. LYAN DAVD JUANICO
BLOCK 1-M A.Y. 2020 - 2021

relations with him, but the sudden revelation that she was untrue to him, and his discovery of her
in flagrante in the arms of another. Which concurs to a case, with a similar state of facts, decision
by the Supreme Court of Spain that the court cited.

The penalty to Hilario Dela Cruz was modified and reduced due to the mitigating circumstance of
passion and obfuscation.

263
CRIMINAL LAW 1 CASE DIGESTS
ATTY. LYAN DAVD JUANICO
BLOCK 1-M A.Y. 2020 - 2021

13.People v. Gelaver
G.R. No. 95357, 9 June 1993
QUIASON, J:

FACTS:
On March 24, 1988, the appellant, Eduardo Gelaver killed his lawfully wedded wife, Victoria
Gelaver y Pacinabao in Barangay Poblacion, Municipality of Sto. Niño, Province of South
Cotabato, Philippines.

A witness presented by the prosecution, testified that on the day of crime he heard shouts coming
from the house of Tessie Lampedario in Barangay Poblacion, Municipality of Sto. Niño, South
Cotabato and saw the appellant, Gelaver, and a woman having a heated argument then appellant
held the neck of the victim, dragged her and with a knife on his right hand, stabbed the latter three
times on the breast, went out of the gate and fled.

Eduardo Gelaver admitted killing his wife and mother of his four children but claimed that he did
so after catching her having carnal act with her paramour.

Gelaver stated that Victoria left their conjugal home and abandoned them on July 3, 1987. He also
stated that he did not know the name of his wife's paramour nor the name of the owner of the house
where his wife and her paramour had lived together. And on March 24, 1988, their daughter,
Sheryl, informed him that his wife and paramour were living at a house in front of the Sto. Niño
Catholic Church. Causing him to go to that place and upon entering he saw his wife lying on her
back and her paramour on top of her, having sexual intercourse

Furthermore, Gelaver alleges that when his wife saw him, she pushed her paramour aside. Her
paramour immediately stood up, took a knife placed on top of the bedside table and attacked
appellant. The latter was able to wrest possession of the knife and then used it against the
paramour, who evaded the thrusts of the appellant by hiding behind the victim. Thus, it was the
victim who received the stab intended for the paramour. He claimed that his mind had been
dimmed or overpowered by passion and obfuscation by the sight of his wife having carnal act with
her paramour causing him to further stab his wife resulting the death of the latter.

ISSUES:
1. Whether or not the mitigating circumstance of voluntary surrender to the authorities is present in
the case at bar?

264
CRIMINAL LAW 1 CASE DIGESTS
ATTY. LYAN DAVD JUANICO
BLOCK 1-M A.Y. 2020 - 2021

2. Whether or not the trial court erred in finding the presence of the mitigating circumstance of
passion or obfuscation?

RULINGS:
1. Yes, the court held that the mitigating circumstance stated in paragraph 7 of Article 13 of the
Revised Penal Code is present in the case at bar.
The trial court was correct in finding the presence of the mitigating circumstance of voluntary
surrender to the authorities.

Gelaver, immediately after committing the offense, voluntarily placed himself at the disposal of
the police authorities as evidenced by the entry in the official police blotter. Thus, allows him the
benefit of the mitigating circumstance of voluntary surrender to the authorities.

2. Yes, the trial court erred in finding the presence of the mitigating circumstance of passion
or obfuscation,

First, the court found out that the testimonies of the appellant is tainted with inconsistencies which
leads the court to believe his allegation that he caught his wife and her paramour in sexual
intercourse.

Second, if there was a naked man with the victim, he would have had no time to get dressed
because he was then under attack by appellant. There would then have been the spectacle of a man
in the nude running in the streets.

And lastly, Sheryl, the daughter that Gelaver claims to be the one who informed him where her
mother was stated in her testimony that she did not know the house where the crime was committed
and she had not gone to that place.

Therefore, the trial court erred in finding the presence of the mitigating circumstance of passion or
obfuscation "as a result of his (appellant's) wife leaving their home and their children."

265
CRIMINAL LAW 1 CASE DIGESTS
ATTY. LYAN DAVD JUANICO
BLOCK 1-M A.Y. 2020 - 2021

14. People v. Bello


G.R. No. L-18792, 28 February 1964
REYES, J.B.L., J:

FACTS:
On September 17, 1954 the accused Guillermo Bello, a widower and at that time Is 54 years old
took a young peasant lady name Alicia Cervantes, who was about 24 years old, as his common
law wife without the benefit of marriage.

That on May 15, 1958, with the livelihood of Bello as ah “kaingin” maker being not enough, he
induced Alicia to accept an employment as entertainer in a bar and restaurant known as Maring's
Place situated at the corner of Aguinaldo and Bonifacio Streets, Gumaca, Quezon. Alicia then
started working as a public hostess and that Bello who was so infatuated with his young bride used
to watch her movements in Maring's Place every day.

Later Bello saw Alicia enter the Gumaca theater with a man whom the accused found later was
caressing his common-law wife inside the movie house and that he took Alicia out of the theater
and warned her to be more discreet in her personal conduct in Gumaca. However, despite that
Alicia continued to serve at Maring's Place as a public hostess.

On May 20, 1958, Bello went to Maring's Place to ask for some money from Alicia but was turned
down by Maring and Alicia. Maring the owner of the place even told him to forget Alicia
completely because he was already an old man, an invalid besides and should stop bothering
Alicia. Bello then left and went home.

On his way home, Bello he met the brothers Justo Marasigan, and Luis Marasigan who greeted
him and he heard Luis saying to his brother Justo that he, Bello, is the man whose wife is being
used by Maring for white slave trade. Due to his grief, Bello went to Paty's place in Gumaca where
he drank 5 glasses of tuba. Later that day he went to Realistic Studio which is in front of Maring's
Place and from there watched the movements of Alicia. Then that night at about nine in the
evening, he entered Maring's Place and without much ado held Alicia from behind with his left
hand and with his right hand stabbed Alicia several times with a balisong. When Bello saw Alicia
fall on the ground and believing her to be mortally wounded, he fled and went to the municipal
building and there surrendered himself to the police of Gumaca.

ISSUES:
1. Whether or not the mitigating circumstance of voluntary surrender to the authorities is present in
the case at bar.

266
CRIMINAL LAW 1 CASE DIGESTS
ATTY. LYAN DAVD JUANICO
BLOCK 1-M A.Y. 2020 - 2021

2. Whether or not the accused is entitled the benefit of the mitigating circumstance of passion or
obfuscation.

RULINGS:
1. Yes, the mitigating circumstance of voluntary surrender to the authorities is present in the case at
bar.

The court held that the mitigating circumstance of voluntary surrender to the authorities is present
and that even both the prosecution and the defense recognizes this. The fact that after committing
the crime at Maring’s Place, fled and went to surrender himself to the police allows him the benefit
of the mitigating circumstance as stated in Article 13, paragraph 7 of the Revised Penal Code.

2. Yes, the accused, Bello, is entitled to the benefit of another mitigating circumstance of
passion or obfuscation.

The insulting comments on him to leave Alicia be since he is old and invalid, and the statement he
heard on his way home of being the husband "whose wife was being used by Maring for purposes
of prostitution", was recognized by the court to have deeply wounded the appellant's feelings that
he was driven to consume a large amount of tuba (wine) before going to Alicia to ask her to quit
her job at Maring’s place and her loose conduct. However, his plea was rejected.

Such rejection showed that Alicia was not motivated by any desire to lead a chaste life henceforth,
but showed her determination to pursue a lucrative profession. Thus, the court held that they cannot
see how the accused's insistence that she live with him again, and his rage at her rejection of the
proposal, cannot be properly termed as arising from immoral and unworthy passions, and therefore
the accused in the case at bar can be given the benefit of the mitigating circumstance of having
acted on a provocation sufficiently strong to produce passion and obfuscation.

267
CRIMINAL LAW 1 CASE DIGESTS
ATTY. LYAN DAVD JUANICO
BLOCK 1-M A.Y. 2020 - 2021

15. People v. Amaguin


G.R. Nos. 54344-45, 10 January 1994
BELLOSILLO, J:

FACTS:
In the afternoon of May 24, 1977, Hernando Oro recounted that he and his brothers Diosdado and
Danilo, brother-in-law Rafael Candelaria, and first cousin Sergio Argonzola were invited by their
eldest brother Pacifico to the latter's house in the interior of Divinagracia Street, La Paz, Iloilo
City, for a small gathering to celebrate the town fiesta. Later at five in the afternoon they went to
the plaza where they could get a ride. While they were in Divinagracia Street, Pacifico was called
by accused Celso Amaguin asking Pacifico to come to him that was rejected by the former since
he was entertaining his guest. Then suddenly, Celso, with a butcher's knife in hand, rushed towards
Pacifico. From there on a brawl between the two sides started which resulted to the death of the
Oro brothers Pacifico and Diosdado.

The statements of Hernando Oro were supported by both witnesses Danilo Oro, the youngest of
the Oros, and Rafael Candelaria, a brother-in-law of the Oros.

The defense on the other side alleges that it was the Oro brothers instigated the fight. Accused
Gildo Amaguin recounted that day that he say Pacifico talking to Celso, the a companion of
Pacifico came forward, held Celso by the shoulder and said: "This is the bravest man in
Divinagracia Street, the Amaguin." Meanwhile, another companion of Pacifico gave Celso a flying
kick that sent him reeling. Gildo then went down the house shouting: "Don't fight." However, the
attackers drew their knives and slingshots. Thus, starting the fight.

The story of was confirmed by Vicente Belicano and Nilda Tagnong, long-time residents of
Divinagracia Street, and Nenita Amaguin, mother of the accused brothers.

Afer the joint trial, the then Court of First Instance of Iloilo found the version of the prosecution
more credible held the accused Gildo Amaguin, guilty of murder and Willie Amaguin, guilty as
accomplice.

ISSUE:
Whether or not the court erred in not appreciating the mitigating circumstance of voluntary
surrender.

268
CRIMINAL LAW 1 CASE DIGESTS
ATTY. LYAN DAVD JUANICO
BLOCK 1-M A.Y. 2020 - 2021

RULING:
Yes, the mitigating circumstances of voluntary surrender is present in the case at bar and should
be appreciated.

For such mitigating circumstance to be appreciated the following elements must be present:
a. The offender has not been actually arrested;
b. The offender surrendered himself to a person in authority; and,
c. The surrender must be voluntary

And the court found that all requisites appear to have attended their surrender. While it may have
taken both Willie and Gildo a week before turning themselves in, the fact is, they voluntarily
surrendered to the police authorities before arrest could be effected.

In the part of Gildo, his uncle brought him to the authorities in Fort San Pedro for "safe-keeping"
and turned over to the local police after a week. While, Willie, turned himself in after five days,
upon learning that law enforcers were looking for him. Allowing both of them the benefit of the
mitigating circumstances of voluntary surrender.

269
CRIMINAL LAW 1 CASE DIGESTS
ATTY. LYAN DAVD JUANICO
BLOCK 1-M A.Y. 2020 - 2021

16. People v. Dela Cruz


G.R. No. L-45284, 29 December 1936
AVANCEÑA, C.J.:

FACTS:
On May 30, 1936 in Manila, Philippines, the accused, Francisco de la Cruz, Fernando Legaspi and
other three unknown persons acted together with intent to gain, attacked and assaulted Yu Wan.
The accused, after attacking, robbed Yu Wan, 26 pesos without his consent.

The blows in the face and other parts of the body of Yu Wan resulted to him to require medical
attendance for a period of more than one but less than nine days and have prevented him to go to
his work in the same period of time.

The accused pleaded not guilty. However, during trial and after two witnesses have testified the
accused withdrew their plea of not guilty, substituting it by that of guilty.

The court sentenced Francisco de la Cruz to six months and one day of prision correctional and,
considering him a habitual delinquent, sentenced him furthermore to the additional penalty of six
years and one day of prision mayor. The other accused Fernando Legaspi was sentenced to ten
months of prision correccional. Francisco de la Cruz appealed from this sentence.

ISSUE:
Whether or not the accused can invoke the mitigating circumstance of surrender or plea of guilt.

RULING:
No, the accused cannot invoke the mitigating circumstance stated in Article 13, paragraph 7 of the
Revised Penal Code which states:

“7. That the offender had voluntarily surrendered himself to a person in authority or his agents, or
that he had voluntarily confessed his guilt before the court prior to the presentation of the evidence
for the prosecution.”

The court held that in order for such to be cited it requires that plea be spontaneous and that it be
made prior to the presentation of evidence by the prosecution. Furthermore, confession of guilt,
although subsequent to the consummation of the crime and entirely alien to its development,
constitutes a cause for the mitigation of the penalty, not because it is a circumstance modifying
criminal responsibility already incurred and in the evolution of which it has not intervened

270
CRIMINAL LAW 1 CASE DIGESTS
ATTY. LYAN DAVD JUANICO
BLOCK 1-M A.Y. 2020 - 2021

absolutely, but because, as an act of repentance and respect for the law, it indicates a moral
disposition in the accused favorable to his reform.

It is clear that based on the facts stated above, the accused, Francisco de la Cruz, does not deserve
the benefits of Article 13, paragraph 7 of the RPC who pleaded not guilty at first then pleaded the
opposite during trial and after the presentation of some evidence for the prosecution witnesses.

271
CRIMINAL LAW 1 CASE DIGESTS
ATTY. LYAN DAVD JUANICO
BLOCK 1-M A.Y. 2020 - 2021

17. Canta v. People


G.R. No. 140937; 28 February 2001
MENDOZA, J.

FACTS:
On March 14, 1986 in Malitbog Southern Leyte, Canta was accused of having feloniously, take,
steal and carry away a black female cow belonging to Narciso Gabriel without his consent. The
aforesaid cow belongs to his half-sister which he acquired and became under the care of Gardenio
Agapay. One afternoon, Agapay took the cow to graze in a mountain 40 meters away from his hut.
The following day he found it gone where he left it, but left hoof prints which led to the house of
Vallejos. He was told that Canta took it. Gardenio and Tura, who took care of the cow before, were
instructed to claim it, but the petitioner’s wife told them that he delivered the cow to his father, the
barangay captain. They met Canta on the way to the Captain’s house, and he told them that the
owner should claim it. Nevertheless they identified the cow and was told that Canta would call
them the next day so he could talk to his father which he never did. Gabriel reported the matter to
the police and after investigation Canta admitted that he took the cow, but he said that he owned
it and showed two certificates of ownership. Petitioner’s certificate of ownership was denied by
the municipal treasurer and stated that he has no Certificate of Ownership of large cattle in the
records of the municipality. It was later found out by the testimony of the janitor that he helped
the petitioner make the fraudulent certificate and antedated such a document. On January 24,
1997, the trial court rendered its decision finding petitioner guilty of the offense charged. The
Court of Appeals affirmed the trial court's decision and denied the petitioner's motion for
reconsideration. Hence, this petition.

ISSUE:
Whether or not mitigating circumstances analogous to voluntary surrender is present.

RULING:
Yes. The Trial Court correctly found the petitioner guilty of P.D. No 533 otherwise known as the
Anti-Cattle Rustling Law of 1974. However, he should be given the benefit of the mitigating
circumstance analogous to voluntary surrender. The circumstance of voluntary surrender has the
following elements: (1) the offender has not actually been arrested; (2) the offender surrenders to
a person in authority or to the latter's agent; and (3) the surrender is voluntary. In the present case,
petitioner Exuperancio Canta had not actually been arrested. In fact, no complaint had yet been
filed against him when he surrendered the cow to the authorities. It has been repeatedly held that
for surrender to be voluntary, there must be an intent to submit oneself unconditionally to the
authorities, showing an intention to save the authorities the trouble and expense that his search and
capture would require. In the petitioner's case, he voluntarily took the cow to the municipal hall of

272
CRIMINAL LAW 1 CASE DIGESTS
ATTY. LYAN DAVD JUANICO
BLOCK 1-M A.Y. 2020 - 2021

Padre Burgos to place it unconditionally in the custody of the authorities and thus saved them the
trouble of having to recover the cow from him. This circumstance can be considered analogous to
voluntary surrender and should be considered in favor of the petitioner.

273
CRIMINAL LAW 1 CASE DIGESTS
ATTY. LYAN DAVD JUANICO
BLOCK 1-M A.Y. 2020 - 2021

18. Mariano v. People


GR No. 178145; 7 July 2014
MUÑOZ PALMA, J:

FACTS:
The Office of the Provincial Prosecutor of Bulacan charged the petitioner with frustrated murder
for hitting and bumping Ferdinand de Leon while overtaking the latter's jeep.

At about 6:30 in the evening of September 12, 1999, Ferdinand de Leon was driving his owner
type jeep along Barangay Engkanto, Angat, Bulacan. With him were his wife, Urbanita, and their
two-year old son, as they just came from a baptismal party. Luis de Leon, an uncle of Ferdinand,
also came from the baptismal party and was driving his owner type jeep. Accused-appellant
Reynaldo Mariano was driving his red Toyota pick-up with his wife, Rebecca, and their helper,
Rowena Años, as passengers. They had just attended a worship service in Barangay Engkanto.

The Toyota pick-up overtook the jeep of Ferdinand de Leon and almost bumped it. Ferdinand got
mad, overtook the pick-up and blocked its path. Reynaldo Mariano stopped the pick-up behind the
jeep. Ferdinand alighted from his jeep and approached Reynaldo. Ferdinand claimed that he and
Reynaldo had an altercation. However, Reynaldo insisted that he just stayed inside the pick-up and
kept quiet while Ferdinand hurled invectives at him. Urbanita tried to pacify Ferdinand and sought
the assistance of Luis de Leon. Luis intervened and told Ferdinand and Reynaldo
"magpasensiyahan na lamang kayo at pagpasensiyahan mo si Ferdinand." Ferdinand and Reynaldo
heeded the advice of Luis and they went their separate ways.

Instead of proceeding to his house in Norzagaray, Ferdinand decided to drop by his mother's house
in San Roque, Angat to pick up some items. He parked his jeep in front of the house of his mother
and alighted therefrom. However, he was bumped by a moving vehicle, thrown four (4) meters
away and lost consciousness. Urbanita shouted, "Mommy, Mommy, nasagasaan si Ferdie ." She
identified the fast moving vehicle that bumped Ferdinand as the same red Toyota pick-up driven
by Reynaldo.

On the other hand, Reynaldo and his wife, Rebecca, tried to show that the jeep of Ferdinand
stopped on the road in front of the house of the latter's mother about five (5) to six (6) meters away
from their pick-up. Reynaldo stopped the pick-up as he saw an oncoming vehicle, which he
allowed to pass. Thereafter, Reynaldo made a signal and overtook the jeep of Ferdinand. However,
Ferdinand suddenly alighted from his jeep, lost his balance and was sideswiped by the
overtaking pick-up. Reynaldo did not stop his pick-up and he proceeded on his way for fear that
the bystanders might harm him and his companions. After bringing his companions to their house
in Marungko, Angat, Bulacan, Reynaldo proceeded to Camp Alejo S. Santos in Malolos, Bulacan
to surrender and report the incident.

RTC- Frustrated Homicide


CA- Reckless Imprudence resulting to Serious Physical Injuries

274
CRIMINAL LAW 1 CASE DIGESTS
ATTY. LYAN DAVD JUANICO
BLOCK 1-M A.Y. 2020 - 2021

ISSUE:
Whether or not the mitigating circumstance of voluntary surrender should be considered
in the case.

RULING:
As aptly observed by the court a quo, only a vehicle that is moving beyond the normal rate of
speed and within the control of the driver's hands could have caused Ferdinand's injuries. The very
fact of speeding is indicative of imprudent behavior, as a motorist must exercise ordinary care and
drive at a reasonable rate of speed commensurate with the conditions encountered, which will
enable him or her to keep the vehicle under control and avoid injury to others using the
highway.

Thus, had Reynaldo not driven his pick-up at a fast speed in overtaking the jeep of Ferdinand, he
could have easily stopped his pick-up or swerved farther to the left side of the road, as there was
no oncoming vehicle, when he saw that Ferdinand alighted from his jeep and lost his balance, in
order to avoid hitting the latter or, at least, minimizing his injuries.

In its decision, 14 the CA found that Ferdinand had sustained multiple facial injuries, a fracture of
the inferior part of the right orbital wall, and subdural hemorrhage secondary to severe head
trauma; that he had become stuporous and disoriented as to time, place and person. It was also on
record that he had testified at the trial that he was unable to attend to his general merchandise store
for three months due to temporary amnesia; and that he had required the attendance of caregivers
and a masseur until October 31, 1999.

Contrary to the petitioner's insistence, the mitigating circumstance of voluntary surrender cannot
be appreciated in his favor. Paragraph 5 of Article 365, Revised Penal Code, expressly states that
in the imposition of the penalties, the courts shall exercise their sound discretion, without regard
to the rules prescribed in Article 64 of the Revised Penal Code.

275
CRIMINAL LAW 1 CASE DIGESTS
ATTY. LYAN DAVD JUANICO
BLOCK 1-M A.Y. 2020 - 2021

19. People v. Ignas


GR No. 140514-15 September 30, 2003
STREET, J.:

FACTS:
Appellant is an elementary school graduate. He resided at Cruz, La Trinidad, Benguet, where he
operated a bakery. He is married to Wilma Grace Ignas, by whom he has a son of minor age.
Wilma Grace used to be the cashier of Windfield Enterprise, which is owned by Pauline Gumpic.
Pauline had a brother, Nemesio Lopate. It was he whom appellant fatally shot.

Sometime in September 1995, appellant's wife, Wilma Grace Ignas, confided to her close friend,
Romenda Foyagao, that she was having an affair with Nemesio Lopate.

On the evening of October 16, 1995, Wilma Grace, Romenda, and Nemesio went to Manila.
Romenda and Nemesio were sending off Wilma Grace at the Ninoy Aquino International Airport
as she was leaving for Taiwan to work as a domestic helper. Upon arrival in Manila, the trio
checked at Dangwa Inn, with Nemesio and Wilma Grace sharing a room. All three of them stayed
at the inn until October 18, 1995, when Wilma Grace left for Taiwan.

Thereafter, Romenda received from Taiwan four letters written by Wilma Grace on various dates.
Although all the letters were addressed to Romenda, two of them were meant by Wilma Grace to
be read by her paramour, Nemesio. In the other two letters, Wilma Grace instructed Romenda to
reveal to appellant her affair with Nemesio.

It was only sometime late in February 1996 that Romenda, following her bosom friend's written
instructions, informed appellant about the extramarital affair between Wilma Grace and Nemesio.
Romenda informed him that the two had spent a day and a night together in a room at Dangwa Inn
in Manila. Appellant became furious. He declared "Addan to aldaw na dayta nga Nemesio, patayek
dayta nga Nemesio" (There will be a day for that Nemesio. I will kill that Nemesio). Appellant
then got all the letters of Wilma Grace from Romenda.

Elders of the tribe had a talk to confirm if the gossip that Nemesio and Wilma Grace had an affair.
Appellant confirmed the gossip. The elder advised Nemesio to stay in Mountain trail until things
had cooled down.

Shortly after their talk, appellant closed down his bakeshop and offered his equipment for sale.
Among the potential buyers he approached was Mayamnes, but the latter declined the offer.
Sometime during the first week of March, Mayamnes saw appellant load his bakery equipment on
board a hired truck and depart for Nueva Vizcaya.

Bayanes turned towards the place where the sound of the gunshots came from. She testified that
she saw a person falling to the ground. Standing behind the fallen individual, some 16 inches away,
was another person who tucked a handgun into his waistband and casually walked away.

276
CRIMINAL LAW 1 CASE DIGESTS
ATTY. LYAN DAVD JUANICO
BLOCK 1-M A.Y. 2020 - 2021

Initially, she only saw the gunman's profile, but when he turned, she caught a glimpse of his face.
29 She immediately recognized him as the appellant June Ignas. She said she was familiar with
him as he was her townmate and had known him for several years. Witness Bayanes was five or
six meters away from the scene, and the taillight of a parked jeepney, which was being loaded with
vegetables, plus the lights from the roof of the bagsakan, aided her recognition of appellant.

According to her, given the blackened edges of the gunshot wound at the victim's back, Nemesio
was shot from a distance of less than three (3) feet.

Witness Bayacsan testified that shortly after they arrived from Kayapa, he had an opportunity to
talk with appellant at the La Trinidad Police Station. There, appellant disclosed to this witness that
he shot and killed Nemesio. Bayacsan, however, did not inform the police about appellant's
revelation as he considered appellant his good friend.

Prosecution witness Pauline Gumpic, the victim's sister, testified that she and appellant had a
private talk, while the latter was in police custody, and appellant admitted to her that he killed her
brother. Gumpic declared that appellant revealed to her that he shot Nemesio for having illicit
relations with appellant's wife and failing to ask for his forgiveness.

SPO4 Arthur Bomagao of the La Trinidad police, who headed the team that investigated the fatal
shooting of Nemesio, declared on the stand that appellant voluntarily admitted to him that he shot
the victim with a .38 caliber handgun. Bomagao further testified that appellant surrendered to him
the letters of Wilma Grace, wherein the latter admitted her affair with Nemesio.

RTC-Murder AC of treachery, nighttime, special AC of use of unlicensed firearms—NO MC

ISSUE:
Whether or not the appellant is entitled to benefit from any mitigating circumstance.

RULING:
Murder or Homicide
The 2000 Revised Rules of Criminal Procedure requires that the qualifying and aggravating
circumstances must be specifically alleged in the information. Although the Revised Rules of
Criminal Procedure took effect only on December 1, 2000 or long after the fatal shooting of
Nemesio Lopate, as a procedural rule favorable to the accused, it should be given retrospective
application. Hence, absent specific allegations of the attendant circumstances of treachery, evident
premeditation, and nocturnity in the amended information, it was error for the trial court to
consider the same in adjudging appellant guilty of murder. As worded, we find that the amended
information under which appellant was charged and arraigned, at best indicts him only for the
crime of homicide. Any conviction should, thus, fall under the scope and coverage of Article 249
of the Revised Penal Code.

As for the separate case for illegal possession of firearm, we agree with the trial court's order to
dismiss miss the information for illegal possession of firearm and ammunition in Criminal Case

277
CRIMINAL LAW 1 CASE DIGESTS
ATTY. LYAN DAVD JUANICO
BLOCK 1-M A.Y. 2020 - 2021

No. 97-CR-2753. 68 Under R.A. No. 8294, which took effect on July 8, 1997, where murder or
homicide is committed with the use of an unlicensed firearm, the separate penalty for illegal
possession of firearm shall no longer be imposed since it becomes merely a special aggravating
circumstance.

Crime and Punishment


As appellant can only be convicted of homicide, it follows that he cannot, under the provisions of
RA No. 7659, be sentenced to suffer the death penalty. The penalty for homicide under Article
249 of the Revised Penal Code is reclusion temporal. Our task now is to determine whether there
are aggravating or mitigating circumstances which could modify the penalty.

More specifically, may the special aggravating circumstance of use of an unlicensed firearm be
taken against the appellant?

We find merit in the appellant's contentions. It is not enough that the special aggravating
circumstance of use of unlicensed firearm be alleged in the information, the matter must be proven
with the same quantum of proof as the killing itself. Thus, the prosecution must prove: (1) the
existence of the subject firearm; and (2) the fact that the accused who owned or possessed it does
not have the corresponding license or permit to own or possess the same. The records do not show
that the prosecution presented any evidence to prove that appellant is not a duly licensed holder of
a caliber .38 firearm. The prosecution failed to offer in evidence a certification from the Philippine
National Police Firearms and Explosives Division to show that appellant had no permit or license
to own or possess a .38 caliber handgun. Nor did it present the responsible police officer on the
matter of licensing as a prosecution witness. Absent the proper evidentiary proof, this Court cannot
validly declare that the special aggravating circumstance of use of unlicensed firearm was
satisfactorily established by the prosecution. Hence such special circumstance cannot be
considered for purposes of imposing the penalty in its maximum period.

Coming now to the obverse side of the case, is the appellant entitled to benefit from any mitigating
circumstance?

Appellant, firstly contends that assuming without admitting that he is guilty, the lower court should
have considered at least the mitigating circumstance of immediate vindication of a grave offense
as well as that of passion and obfuscation. Appellant points out that the victim's act of maintaining
an adulterous relationship with appellant's wife constituted a grave offense to his honor, not to
mention the shame, anguish, and anxiety he was subjected to. Even the mere sight of the victim
must have triggered an uncontrollable emotional outburst on appellant's part, so that even a chance
meeting caused in him an irresistible impulse powerful enough to overcome all reason and
restraint. Secondly, appellant points out that the trial court failed to consider his voluntary
surrender as a mitigating circumstance.

The Solicitor General counters that there was literally no "immediate vindication" to speak of in
this case. Appellant had sufficient time to recover his serenity following the discovery of his wife's
infidelity. Nor could passion and obfuscation be appreciated in appellant's favor because the killing

278
CRIMINAL LAW 1 CASE DIGESTS
ATTY. LYAN DAVD JUANICO
BLOCK 1-M A.Y. 2020 - 2021

was not proximate to the time of the offense. Appellant became aware of the treatment offensive
to his dignity as a husband and to the peace and tranquility of his home two weeks earlier. This
interval between the revelation of his wife's adultery and the fatal shooting was ample and
sufficient for reason and self-control to reassert themselves in appellant's mind.

As to the mitigating circumstance of voluntary surrender, the OSG stresses that his supposed
surrender at Kayapa, Nueva Vizcaya was actually due to the efforts of law enforcers who came
looking for him. There he did not resist, but lack of resistance alone is not tantamount to voluntary
surrender, which denotes a positive act and not merely passive conduct.

According to the OSG, for the mitigating circumstance of vindication of a grave offense to apply,
the vindication must be "immediate." This view is not entirely accurate. The word "immediate" in
the English text is not the correct translation of the controlling Spanish text of the Revised Penal
Code, which uses the word "proxima." The Spanish text, on this point, allows a lapse of time
between the grave offense and the actual vindication. Thus, in an earlier case involving the
infidelity of a wife, the killing of her paramour prompted proximately — though not immediately
— by the desire to avenge the wrong done, was considered an extenuating circumstance in favor
of the accused. The time elapsed between the offense and the suspected cause for vindication,
however, involved only hours and minutes, not days. Hence, we agree with the Solicitor General
that the lapse of two (2) weeks between his discovery of his wife's infidelity and the killing of her
supposed paramour could no longer be considered proximate. The passage of a fortnight is more
than sufficient time for appellant to have recovered his composure and assuaged the unease in his
mind. The established rule is that there can be no immediate vindication of a grave offense when
the accused had sufficient time to recover his serenity. Thus, in this case, we hold that the
mitigating circumstance of immediate vindication of a grave offense cannot be considered in
appellant's favor.

We likewise find the alleged mitigating circumstance of passion and obfuscation inexistent. The
rule is that the mitigating circumstances of vindication of a grave offense and passion and
obfuscation cannot be claimed at the same time, if they arise from the same facts or motive. In
other words, if appellant attacked his victim in proximate vindication of a grave offense, he could
no longer claim in the same breath that passion and obfuscation also blinded him. Moreover, for
passion and obfuscation to be well founded, the following requisites must concur: (1) there should
be an act both unlawful and sufficient to produce such condition of mind; and (2) the act which
produced the obfuscation was not far removed from the commission of the crime by a considerable
length of time, during which the perpetrator might recover his moral equanimity. To repeat, the
period of two (2) weeks which spanned the discovery of his wife's extramarital dalliance and the
killing of her lover was sufficient time for appellant to reflect and cool off.

Appellant further argues that the lower court erred in failing to consider voluntary surrender as a
mitigating circumstance. On this point, the following requirements must be satisfied: (1) the
offender has not actually been arrested; (2) the offender surrendered himself to a person in
authority; and (3) the surrender was voluntary. Records show, however, that leaflets and posters
were circulated for information to bring the killer of Nemesio to justice. A team of police

279
CRIMINAL LAW 1 CASE DIGESTS
ATTY. LYAN DAVD JUANICO
BLOCK 1-M A.Y. 2020 - 2021

investigators from La Trinidad, Benguet then went to Kayapa, Nueva Vizcaya to invite appellant
for questioning. Only then did he return to Benguet. But he denied the charge of killing the victim.
Clearly, appellant's claimed surrender was neither spontaneous nor voluntary.

280
CRIMINAL LAW 1 CASE DIGESTS
ATTY. LYAN DAVD JUANICO
BLOCK 1-M A.Y. 2020 - 2021

20. Bongalon v. People


GR No. 169533 March 20, 2013
BERSAMIN, J.:

FACTS:
The Prosecution showed that on May 11, 2002, Jayson dela Cruz (Jayson) and Roldan, his older
brother, both minors, joined the evening procession for the Santo Niño at Oro Site in Legazpi City;
that when the procession passed in front of the petitioner's house, the latter's daughter Mary Ann
Rose, also a minor, threw stones at Jayson and called him "sissy"; that the petitioner confronted
Jayson and Roldan and called them names like "strangers" and "animals"; that the petitioner struck
Jayson at the back with his hand, and slapped Jayson on the face; 4 that the petitioner then went to
the brothers' house and challenged Rolando dela Cruz, their father, to a fight, but Rolando did not
come out of the house to take on the petitioner; that Rolando later brought Jayson to the Legazpi
City Police Station and reported the incident; that Jayson also underwent medical treatment at the
Bicol Regional Training and Teaching Hospital; 5 that the doctors who examined
Jayson issued two medical certificates attesting that Jayson suffered the following contusions, to
wit: (1) contusion .5 x 2.5 scapular area, left; and (2) +1 x 1 cm. contusion left zygomatic area and
contusion .5 x 2.33 cm. scapular area, left.

Petitioner’s Defense:
the petitioner denied having physically abused or maltreated Jayson. He explained that he only
talked with Jayson and Roldan after Mary Ann Rose and Cherrylyn, his minor daughters, had told
him about Jayson and Roldan's throwing stones at them and about Jayson's burning Cherrylyn's
hair. He denied shouting invectives at and challenging Rolando to a fight, insisting that he only
told Rolando to restrain his sons from harming his daughters.

RTC- Guilty of Child Abuse (RA 7610)


CA- Guilty of Child Abuse
The petitioner asserts that he was not guilty of the crime charged; and that even assuming that he
was guilty, his liability should be mitigated because he had merely acted to protect her two minor
daughters.

ISSUE:
Whether or not the petitioner was guilty of the crime charged and that even assuming that he was
guilty, his liability should be mitigated because he had merely acted to protect her two minor
daughters.

RULING:
He mentions instances attendant during the commission of the crime that he claims were really
constitutive of justifying and mitigating circumstances; and specifies reasons why he believes
Republic Act No. 7610 favors his innocence rather than his guilt for the crime charged. The errors
he thereby underscores in the petition concerned only the CA's appreciation and assessment of the
evidence on record, which really are errors of judgment, not of jurisdiction.

281
CRIMINAL LAW 1 CASE DIGESTS
ATTY. LYAN DAVD JUANICO
BLOCK 1-M A.Y. 2020 - 2021

Although we affirm the factual findings of fact by the RTC and the CA to the effect that the
petitioner struck Jayson at the back with his hand and slapped Jayson on the face, we disagree with
their holding that his acts constituted child abuse within the purview of the above-quoted
provisions. The records did not establish beyond reasonable doubt that his laying of hands on
Jayson had been intended to debase the "intrinsic worth and dignity" of Jayson as a human being,
or that he had thereby intended to humiliate or embarrass Jayson. The records showed the laying
of hands on Jayson to have been done at the spur of the moment and in anger, indicative of his
being then overwhelmed by his fatherly concern for the personal safety of his own minor daughters
who had just suffered harm at the hands of Jayson and Roldan. With the loss of his self-control,
he lacked that specific intent to debase, degrade or demean the intrinsic worth and dignity of a
child as a human being that was so essential in the crime of child abuse.

SC- Slight Physical Injuries


In imposing the correct penalty, however, we have to consider the mitigating circumstance of
passion or obfuscation under Article 13 (6) of the Revised Penal Code, because the petitioner lost
his reason and self-control, thereby diminishing the exercise of his will power. Passion or
obfuscation may lawfully arise from causes existing only in the honest belief of the accused. It is
relevant to mention, too, that in passion or obfuscation, the offender suffers a diminution of
intelligence and intent. With his having acted under the belief that Jayson and Roldan had thrown
stones at his two minor daughters, and that Jayson had burned Cherrlyn's hair, the petitioner was
entitled to the mitigating circumstance of passion.

282
CRIMINAL LAW 1 CASE DIGESTS
ATTY. LYAN DAVD JUANICO
BLOCK 1-M A.Y. 2020 - 2021

21. Nizurtado v. Sandiganbayan


GR No. 107838 December 7, 1984
VITUG, J.:

FACTS:
Felix Nizurtado (Barangay Captain) of having committed the complex crime of malversation of
public funds through falsification of public document.

When arraigned by the Sandiganbayan, Nizurtado pleaded "not guilty" to the charge. That
sometime in 1983 and 1984, accused Felix Nizurtado was the Barangay Captain of Barangay
Panghulo of Malabon, Metro Manila and discharged his functions as such; That sometime in 1983,
the Ministry of Human Settlements, the Metro Manila Commission and Kilusang Kabuhayan at
Kaunlaran (KKK) undertook a Livelihood Program for Barangays in Metro Manila consisting of
loans in the amount of P10, 000.00 per barangay. That as Barangay Captain of Barangay Panghulo,
accused received a check in the amount of P10,000.00 for said barangay's livelihood program;
That the check, to be encashed, had to be supported by a project proposal to be approved by the
KKK; That the accused encashed the check received by him in the amount of P10,000.00 with the
Land Bank of the Philippines; and That the accused distributed the amount of P10,000.00 in the
form of loans of P1,000.00 each to members of the barangay council.

"Since Romero and Gomez had not borrowed any amount from the said fund, they told Sandel to
ask Nizurtado if he had any proof of their alleged loans. So Sandel wrote Nizurtado

RTC- malversation of public funds through falsification of public documents (2 mitigating


circumstance)

ISSUE:
Whether or not the mitigating circumstances are applicable to the accused.

RULING:
Petitioner was able to encash the check on 18 October 1988 on the basis of a resolution of the
Barangay Council, submitted to the KKK Secretariat, to the effect that a livelihood project, i.e.,
"T-shirt manufacturing," had already been identified by the council. The money, however, instead
of its being used for the project, was later lent to, along with petitioner, the members of the
Barangay Council. Undoubtedly, the act constituted "misappropriation" within the meaning of the
law.

The Sandiganbayan has considered the mitigating circumstances of voluntary surrender and
restitution in favor of Nizurtado.

Voluntary surrender (Art. 13, par. 7, Revised Penal Code), therefore, may thus be treated as a
modifying circumstance independent and apart from restitution of the questioned funds by
petitioner (Art. 13, par. 10, Revised Penal Code). We are convinced, furthermore, that petitioner

283
CRIMINAL LAW 1 CASE DIGESTS
ATTY. LYAN DAVD JUANICO
BLOCK 1-M A.Y. 2020 - 2021

had no intention to commit so grave a wrong as that committed. (Art. 13, par. 3, Revised Penal
Code), entitling him to three distinct mitigating circumstances.

The actual attendance of two separate mitigating circumstances of voluntary surrender and
restitution, also found by the Sandiganbayan and uncontested by the Solicitor General, entitles the
accused to the penalty next lower in degree. For purposes of determining that next lower degree,
the full range of the penalty prescribed by law for the offense, not merely the imposable penalty
because of its complex nature, should, a priori, be considered. It is our considered view that the
ruling in People vs. Gonzales, 73 Phil. 549, as opposed to that of People vs. Fulgencio, 92 Phil.
1069, is the correct rule and it is thus here reiterated. In fine, the one degree lower than prision
mayor maximum to reclusion temporal minimum is prision mayor minimum to prision mayor
medium (being the next two periods in the scale of penalties [see Art. 64, par 5, in relation to Art.
61, par 5, Revised Penal Code]) the full range of which is six years and one day to ten years. The
one degree lower penalty should, conformably with Article 48 of the Code (the penalty for complex
crimes), be imposed in its maximum period or from eight years, eight months and one day to ten
years. The presence of the third mitigating circumstance of praeter intentionem (lack of intention
to commit so grave a wrong as that committed) would result in imposing a period the court may
deem applicable. Considering, however, that the penalty has to be imposed in the maximum period,
the only effect of this additional mitigating circumstance is to impose only the minimum portion
of that maximum period, that is, from eight years, eight months and one day to nine years, six
months and ten days, from which range the maximum of the indeterminate sentence shall be taken.

284
CRIMINAL LAW 1 CASE DIGESTS
ATTY. LYAN DAVD JUANICO
BLOCK 1-M A.Y. 2020 - 2021

MODULE 8: AGGRVATING CIRCUMSTANCES

1. People v. Legaspi
G.R. Nos. 136164-65; 20 April 2001
MELO, J.

Facts:
On February 11, 1997 at around 2:00 in the morning, Honorata Ong, who was then sleeping
inside her house with her three daughters was awakened by the sound of their door opening. She
initially thought that it was her husband coming home from work. As she opened her eyes, an
armed man standing by her feet with his pants and briefs already down on his knees was pointing
at her eldest daughter. As she was alarmed that her daughter might be harmed, she told the man
not to hurt her daughter, thus she was told to stand up with a knife poked at her. The man made
her lie down at the adjacent sofa and had carnal knowledge with her against her will with a knife
pointed at her neck. Honorata smelled that the man reeked of alcohol. After that, the man asked
for money but all she could give is a few bills amounting to P500.00. She was threatened not to
tell anyone of what happened. But later that day, she mustered her courage and reported the said
incident to the barangay Captain. Her sister in law identified the description of the assailant in the
person of Edgar Legaspi. Legaspi denied the allegations against him and stated that he was sleeping
at his residence during that incident. His alibi was not given credence and was found by the trial
court guilty of rape aggravated by dwelling and nighttime, and of robbery aggravated by dwelling
sentencing him the supreme penalty of death for rape and penalty of 6mos to 9yrs for robbery.
Thus this appeal.

ISSUE:
Whether or not the RTC erred in appreciating the aggravating circumstance of nighttime and
dwelling.

RULING:
Yes, The lower court erred in appreciating the aggravating circumstances. Upon a cursory
examination of the information filed against Legaspi, it shows that the aggravating circumstances
of nighttime and dwelling are not specified therein. But at the time the trial court rendered its
decision, the non-allegation of generic aggravating circumstances in the information was

immaterial, since the rule then prevailing was that generic aggravating circumstances duly proven
in the course of the trial could be taken into account by the trial court in determining the proper

285
CRIMINAL LAW 1 CASE DIGESTS
ATTY. LYAN DAVD JUANICO
BLOCK 1-M A.Y. 2020 - 2021

imposable penalty even if such circumstances were not alleged in the information. Nonetheless, it
is to be noted that the appreciation by the trial court of the aggravating circumstances of dwelling
and nighttime, despite the non-allegation thereof in the Information, resulted in the imposition of
the supreme penalty of death upon him. Consequently, it was held that due to their non-allegation
in the Information for rape filed against accused-appellant, the aggravating circumstances of
nighttime and dwelling cannot be considered in raising the penalty imposable upon accused-
appellant from reclusion perpetuato death. Thus, the Rules now require qualifying as well as
aggravating circumstances to be expressly and specifically alleged in the Complaint or
Information, otherwise the same will not be considered by the court even if proved during the trial.
And this principle is applicable in all criminal cases, not only in cases where the aggravating
circumstance would increase the penalty to death. With this, the Court gives fair warning to
prosecutors that henceforth, they must prepare well-crafted information that allege the
circumstances qualifying and aggravating the crimes charged, otherwise the same will not be
considered by the court in determining the proper imposable penalty.

286
CRIMINAL LAW 1 CASE DIGESTS
ATTY. LYAN DAVD JUANICO
BLOCK 1-M A.Y. 2020 - 2021

2. People v. Capalac
G.R.No. L-38297; 23 October 1982
FERNANDO, C.J.:

FACTS:
On September 20, 1970 at around 2:00 o'clock in the afternoon, while inside a cockpit arena,
Jimmy Magaso without provocation stabbed Moises Capalac. As Magaso was attempting to
escape, tried to board a jeepney but was unsuccessful in his attempt and was shot twice as he was
confronted by Jesus and Mario Capalac. Knowing that he was completely at the mercy of the two
brothers, he raised his hands as a sign of surrender, but they were not to be appeased. He was
pistol-whipped by appellant Mario Capalac, being dealt several blows on the head and the face.
After he had fallen to the ground, Jesus Capalac stabbed the deceased on the chest three or four
times. He was brought to the hospital where he died due to hemorrhagic shock due to a wound of
the heart. Mario Capalac was convicted of murder. The lower court found that the crime was
committed with evident premeditation and treachery. The lower court also held that appellant took
advantage of his position as a police officer and employed means or brought about circumstances
which added ignominy to the natural effects of his act. It sentenced him to suffer the death penalty.
Hence, this appeal.

ISSUE:
Whether or not the Trial court correctly appreciated the qualifying and aggravating circumstances
in this case.

RULING:
The Trial court correctly appreciated the qualifying circumstance of treachery in this case.
According to the Revised Penal Code “There is treachery when the offender commits any of the
crimes against the person, employing means, methods, or forms in the execution thereof which
tend directly and specially to insure its execution, without risk to himself arising from the defense
which the offended party might make” Magaso's situation was hopeless. Any defense he could
have put up would be futile and unavailing. His hands were raised in surrender. That
notwithstanding, he was pistol-whipped. When lying prostrate on the ground, he was stabbed. It
must be remembered that, according to the testimonial evidence, there were two other persons
assisting the brothers Capalac. If they were not included in the information, the explanation would
appear to be that they managed to elude capture. There was no risk, therefore, to the aggressors,
no hope for the victim. The trial court committed no error then in appreciating the circumstance
of treachery as being present.

287
CRIMINAL LAW 1 CASE DIGESTS
ATTY. LYAN DAVD JUANICO
BLOCK 1-M A.Y. 2020 - 2021

However, the lower court erred in appreciating the aggravating circumstances of evident
premeditation, ignominy and of the crime being committed with the offender taking advantage of
his official position as having attended the commission of the crime. The brothers were prompted
by their desire to avenge Moises. They went after Jimmy, assaulted him, and relied on the weapons
Jesus had already died and so he was not included as an accused in the information filed. The two
companions were not named and were not included in the information that they carried. There was
no evidence that they deliberately employed means to add ignominy to the natural effects of the
act. Also, The mere fact that appellant Mario is a member of the police force did not by itself
justify the aggravating circumstance of taking advantage of public office/position. He acted as
brother of Moises, instinctively reacting to what was undoubtedly a vicious assault on his kin. He
pistol-whipped the deceased because he had a pistol with him. It came in handy and he acted
accordingly. That he was a policeman is of no relevance in assessing his criminal responsibility.

288
CRIMINAL LAW 1 CASE DIGESTS
ATTY. LYAN DAVD JUANICO
BLOCK 1-M A.Y. 2020 - 2021

3. People v. Gapasin
G.R. No. 73489; 25 April 1994
QUIASON, J.:

FACTS:
On October 1979, in the province of Isabela, accused Loreto Gapasin, Nicanor Saludares, Lorenzo
Soriano alias Olit, Amor Saludare, Frank Saludares, Bel Saludares, and Nick Saludares, conspiring
and confederating together and all helping one another, with evident premeditation and treachery,
did then and there wilfully, unlawfully, criminally and feloniously, with intent to kill, attack and
shoot Jerry Calpito, with an Armalite rifle. After almost 6 years, trial ensued. According to the
prosecution witness Alberto, he and Ballad left the house of Teppang after attending “pamisa” for
Teppang’s deceased father. Calpito followed them, and while walking on barangay road, Calpito
shot the appellant with an armalite rifle. When Calpito fell on the ground, appellant fired more
shots at him. Then Amor Saludares planted a .22 revolver on left hand of Calpito. Faustina Calpito
ran to her fallen husband after hearing shots, but Nicanor Saludares pointed his gun at her, and
Soriano fired his gun upwards. Saludares warned that he would kill any relative of Calpito who
would come near him. Faustina and relatives scampered away as the group chased after them.
Autopsy showed Calpito sustained 4 gunshot wounds - Dr. Layugan opined that the victim was in
standing position when he was shot by someone positioned to his right. Appellant invoked self-
defense and testified that mission order on September 23, 1979 to investigate a report regarding
presence of unidentified men in Barrio San Jose, Isabela. Instructed by Sgt. Ignacio to get in touch
with Nicanor Saludares, who may give him information on the identities of persons with
unlicensed firearms. Saludares informed Gapasin that Calpito had an unlicensed firearm. Saludares
informed Gapasin that it would be best to see Calpito during burial of his relative Attended
“pamisa” in house of Teppang, but did not confront Calpito to avoid creating a disturbance at the
“pamisa” Waited in yard of Saludares’s house for Calpito to pass by. When Calpito was about 3
meters away, the appellant asked him what was bulging in his waist. Calpito took a step back, drew
his firearm and fired twice at appellant - missed because appellant dropped to the ground while
firing his armalite.

ISSUE:
Whether or not the RTC erred in appreciating qualifying and aggravating circumstances in this
case.

RULING:
No. The Trial Court correctly appreciated the aggravating circumstances in this case. The trial
court correctly ruled that the crime of murder was indeed committed. Treachery attended the

289
CRIMINAL LAW 1 CASE DIGESTS
ATTY. LYAN DAVD JUANICO
BLOCK 1-M A.Y. 2020 - 2021

commission of the crime. The two conditions to constitute treachery were present in the case at
bench, to wit: (a) the employment of means of execution that gives the person who is attacked no
opportunity to defend himself or to retaliate; and (b) the means of execution were deliberately or
consciously adopted. Appellant deliberately executed the act in such a way that his quarry was

unaware and helpless. Evident premeditation was indubitably proven by the evidence showing that
the execution of the criminal case was preceded by cool thought and reflection. Appellant's
resolution to carry out the criminal intent during the space of time sufficient to arrive at a clear
judgment was shown. The information alleged three other generic aggravating circumstances:
ignominy, abuse of superior strength and taking advantage of public position. The trial court
correctly ruled out ignominy on the strength of the autopsy conducted by the doctor who failed to
find any other injuries such as bruises and contusions which may indicate that the victim was
kicked by his assailants. It also correctly held that treachery absorbed abuse of superior strength.
The trial court properly appreciated taking advantage of public position as an aggravating
circumstance. Appellant, a member of the Philippine Constabulary, committed the crime with an
armalite which was issued to him when he received the mission order. Voluntary surrender may
be considered in appellant's favor but this is offset by the aggravating circumstance of taking
advantage of public position. Therefore, only the generic aggravating circumstance of evident
premeditation may be appreciated against appellant.

290
CRIMINAL LAW 1 CASE DIGESTS
ATTY. LYAN DAVD JUANICO
BLOCK 1-M A.Y. 2020 - 2021

4. People v. Tiongson
G.R. No. L-35123-24; 25 July 1984
CONCEPCION JR., J.:

FACTS:
On October 1971, at 5:30 PM Rudy TIongson escaped from jail, with de la Cruz and Santiago.
While escaping, Tiongson killed Zosimo Gelera, a member of the police force guarding them and
PC constable Aurelio Canela who went in pursuit of them. Tiongson was charged with murder
qualified by treachery in two separate information. Upon arraignment, accused assisted by counsel
pleaded guilty to both informations but the trial court did not render judgment outright, and ordered
prosecution to present evidence. After which, accused was sentenced to suffer death penalty. He
was found guilty, with the following aggravating crcumstances: evident premeditation, in
contempt or with insult to public authorities, crime was committed in an uninhabited place, and
abuse of superior strength, which resulted in the imposition of the death penalty. Thus, the cases
are now under mandatory review.

ISSUE:
Whether or not the Qualifying and aggravating circumstances are present.

RULING:
No. Treachery which would qualify the killing of Pat. Gelera and PC Constable Canela to Murder,
was not present, the crimes may only be punished as Homicide. It may be true that a judicial
confession of guilt admits all the material facts alleged in the information, including the

aggravating circumstances listed therein, as stated by the trial judge, yet where there has been a
hearing and such circumstances are disproven by the evidence, they should be disallowed in the
judgment. It was also held that the aggravating circumstances of (1) evident premeditation, (2) in
contempt of or with insult to public authorities, (3) uninhabited place, and (4) abuse of superior
strength were not present in the commission of the crimes. Evident premeditation must be ruled
out in view of the absence of sufficient proof that a plan to kill the victims existed, the execution
of which was preceded by deliberate thought and reflection. The aggravating circumstance that the
crimes were committed in contempt of or with insult to the public authorities cannot also be
appreciated since Pat. Gelera and PC Constable Canela were the very ones against whom the
crimes were committed. Besides, Pat. Gelera and PC Constable Canela are not persons in authority,
but merely agents of a person in authority. The lower court also erred in finding that the killing of
PC Constable Canela was committed in an uninhabited place. In order that the aggravating
circumstance of the commission of a crime in an uninhabited place may be considered, it is
necessary that the place of occurrence be where there are no houses at all, a considerable distance

291
CRIMINAL LAW 1 CASE DIGESTS
ATTY. LYAN DAVD JUANICO
BLOCK 1-M A.Y. 2020 - 2021

from the village or town, or where the houses are a great distance apart. Besides, the record does
not show that the place was intentionally sought by the accused to facilitate the commission of the
crime. The accused was trying to evade his pursuers, PC Constable Canela among them, and their
encounter was purely by chance. Finally, the aggravating circumstance of abuse of superior
strength must also be ruled out since there is no direct evidence that the accused employed superior
strength in the killing of Pat. Gelera. The accused was then a detainee and was unarmed.

292
CRIMINAL LAW 1 CASE DIGESTS
ATTY. LYAN DAVD JUANICO
BLOCK 1-M A.Y. 2020 - 2021

5. People v. Magdueno
G.R. no. L-68699; 22 September 1986
PER CURIAM:

FACTS:
On October 15 1980, in the City of Puerto Princesa Palawan, a few minutes past 8:00 o'clock in
the morning, as soon as the late Fiscal Fernando M. Dilig had placed himself at the driver's seat
inside his jeep parked near his house at the corner Roxas and D. Mendoza Streets, Puerto Princesa
City, all of a sudden, two successive gunshots burst into the air, as the gunman coming from his
left side aimed and poured said shots into his body, inflicting two fatal wounds that instantaneously
caused his death. Three witnesses positively identified the assailant as accused Hermogenes
Magdueño. Magdueño also executed an extra-judicial confession wherein he admitted that he
killed Fiscal Dilig for a price or reward and implicated Leonardo Senas and Mauricio de Leon to
the commission of the crime. However, both Senas and de Leon were later dropped from the
amended information for lack of a prima facie case against them. However, all the other accused
were acquitted for insufficiency of evidence. The Trial Court rendered its judgement rendering

Magdueno guilty of murder qualified by treachery and evident premeditation, and aggravated by
price or reward and that the crime was committed in contempt of/or with insult of public authority.

ISSUE:
Whether or not the court correctly appreciated the qualifying and aggravated circumstances.

RULING:
Yes, the trial court correctly appreciated treachery as qualifying circumstace. Treachery in the
commission of the crime is clearly established by the record. The appellant fired two successive
shots at the defenseless Fiscal Dilig while the latter was still seated in his jeep, hitting him at the
neck and lumbar region. According to Dr. Rufino P. Ynzon, who performed the autopsy, on the
victim; both wounds were fatal and that "death will definitely occur." Immediately after the
shooting, the appellant fled still holding his firearm. The manner of the execution was such that
the appellant deliberately and consciously adopted means and ways of committing the crime and
insured its execution without risk to himself arising from any defense Fiscal Dilig might make.
The two conditions necessary for treachery to exist are present. Therefore, The attendant
circumstance of treachery qualifies the crime to murder.

However, the aggravating circumstance of commission of a crime with insult to public authority
does not seem to be borne by the records. For this aggravating circumstance to be considered it
must not only be shown that the crime was not committed in the presence of the public authority
293
CRIMINAL LAW 1 CASE DIGESTS
ATTY. LYAN DAVD JUANICO
BLOCK 1-M A.Y. 2020 - 2021

but also that the crime was not committed against the public authority himself. In the instant case
Fiscal Dilig, the public authority involved in the crime, was the victim. Hence, the lower court,
erred in including commission of the crime with insult to public authority as an aggravating
circumstance.

294
CRIMINAL LAW 1 CASE DIGESTS
ATTY. LYAN DAVD JUANICO
BLOCK 1-M A.Y. 2020 - 2021

6. People v. Tac-an
G.R. nos. 76338-39; 26 February 1990
FELICIANO, J.:

FACTS:
On December 14, 1984, in the City of Tagbilaran, accused Renato Tac-an while acting under the
influence of drugs with an unlicensed firearm shot Francis Ernest Escano III hitting and inflicting
upon the latter multiple gunshot wounds which directly caused his death. Renato Tac-an and
Francis Escano were close friends being classmates in high school and members of the local Bronx
gang. Francis withdrew from the gang on the advice of his mother who saw that Renato carried a
handgun on his visits to their home. Things started turning sour between the two, and came to a
head on Dec 14, 1984. After an earlier altercation on that day, Renato went home and got his gun.
He entered the Mathematics class under Mr. Damaso Pasilbas in Rm15 and shouted for Francis.
After locating the victim he fired at him but missed. He was later able to hit him in the head as he
was running to the door with his classmates to escape. After this, Renato paced outside in the
hallway. A teacher unknowing that Renato was the culprit, asked him for help unwittingly
informing him that Francis was still alive. Renato immediately re- entered the room and saying
"So, he is still alive. Where is his chest?" Standing over Francis sprawled face down on the
classroom floor, Renato aimed at the chest of Francis and fired once more. The bullet entered
Francis' back below the right shoulder, and exited on his front chest just above the right nipple.
2 separate informations were filed against Tac-an, illegal possesion of unlicensed firearm under
Section 1, paragraph (2), of Presidential Decree No. 1866, and murder. Appellant entered a plea
of not guilty in both cases. On 31 July 1986, the trial court rendered a decision convicting appellant
under both information. Hence, this appeal.

ISSUE:
Whether or not the Trial Cours correctly appreciated the qualifying and aggravating
circumstances.

RULING:
Yes. the Trial Court correctly appreciated treachery qualifying the death of the victim to murder.
Accused Renato fired three (3) shots before hitting Francis with the fourth shot, can only be
ascribed to the indifferent marksmanship of Renato and to the fact that Francis and the other
students were scurrying from one part of the room to the other in an effort to evade the shots fired
by Renato. The cumulative effect of the circumstances underscored by the trial court was that the
attack upon Francis had been carried out in a manner which disabled Francis from defending
himself or retaliating against Renato. Finally, the circumstance that Renato, having been informed
that Francis was still alive, re-entered Room 15 and fired again at Francis who lay on the floor and
bathed with his own blood, manifested Renato's conscious choice of means of execution which

295
CRIMINAL LAW 1 CASE DIGESTS
ATTY. LYAN DAVD JUANICO
BLOCK 1-M A.Y. 2020 - 2021

directly and especially ensured the death of his victim without risk to himself. 22 We are compelled
to agree with the trial court that treachery was here present and that, therefore, the killing of Francis
Ernest Escaño III was murder. However, the aggravating circumstances of evident premeditation
and of having acted with contempt of or insult to the public authorities shall be deleted and not
taken into account; and (b) the special aggravating circumstances of acting while under the
influence of dangerous drugs and with the use of an unlicensed firearm shall similarly be deleted
and not taken into account. There being no generic aggravating nor mitigating circumstances
present, the appellant shall suffer the penalty of reclusion perpetua.

296
CRIMINAL LAW 1 CASE DIGESTS
ATTY. LYAN DAVD JUANICO
BLOCK 1-M A.Y. 2020 - 2021

7. PEOPLE OF THE PHILIPPINES vs FRANCISCO DIAZ and GERARDO DIAZ


No. L-24002; January 21, 1974
BELLOSILLO, J.:

FACTS:
About two o’clock in the afternoon of September 4, 1963, Remegia Carasos, a fourteen year old
girl, and her first cousin, Anita Pacaira, eleven years old, were gathering camotes in a farm located
at a place fittingly called Sitio Camotian, Barrio Perito, municipality of Sta. Margarita, Western
Samar. Francisco Diaz, a twenty-four year old unmarried farmer of that place, whom Remegia and
Anita had known for many years suddenly appeared. Without any preliminaries, he embraced
Remegia from behind and against her will and held her breast. He knelt behind her while she was
gathering Camotes. She shouted for help and reacting to Remegia’s cry for help, Anita, with a
bolo, struck Francisco on the head and hands. Francisco released Remegia and fled. Francisco
suffered some injuries in consequence of the blows. The two girls left the camote farm and went
to the house of their grandfather, Quintin Tadia and informed him that Francisco Diaz had
embraced and abused Remegia. Tadia reported the incident to the barrio lieutenant.

The morning of the following day, Tadia and her granddaughters were on their way to the
poblacion to file the complaint. He was unarmed and carrying an oblong basket on his back
containing boiled Camotes. While they were ascending a hill or cliff, Francisco Diaz and his
younger brother Gerardo, appeared on the crest of the hill. Gerardo was armed with a locally made
shotgun called bardog about fifty inches long. He immediately fired sidewise at Tadia while about
four meters from the latter, hitting him in the neck which felled Tadia. He rolled down the lower
part of the cliff near the Alao Creek and lay there flat on his back with his catopis. Then, the
brothers jumped to the lower part of the cliff. Gerardo told his brother to stab him. Francisco placed
his foot on the prostrate body of Quintin Tadia, bent over him and repeatedly stabbed him in
different parts of his body. Francisco was armed with a bolo commonly called utak which is used
in gathering firewood. After witnessing the assault, Remegia Carasos ran in the direction of her
house. Anita Pacairo hid herself among the bushes or tall grasses sitting, crouching and peeping
and seeing all that was happening. Tadia died on the spot where he fell. Gerardo placed his bardog
on a moss-covered stone called palanas about three brazas from Tadia’s body. Remegia informed
her father and the inhabitants of the barrio about the ambuscade and the killing of her grandfather.
Gerardo Diaz went home while Francisco surrendered to the authorities. Gerardo Diaz was arrested
and did not resist arrest.

ISSUE:
Whether or not the following aggravating circumstances are present:
1. Treachery

297
CRIMINAL LAW 1 CASE DIGESTS
ATTY. LYAN DAVD JUANICO
BLOCK 1-M A.Y. 2020 - 2021

2. Abuse of Superior Strength


3. Disregard of Age
4. Evident Premeditation in the case of Francisco Diaz

RULING:
1. Treachery is appreciated as a qualifying circumstance in this case.
There was treachery (alevosia) because the brothers made a deliberate surprise or
unexpected assault on Tadia. They literally ambushed him by waiting for him on the cliff, a high
ground which rendered it difficult for him to flee or maneuver in his defense. Tadia was shot
sidewise while he was ascending the hill or cliff burdened by his catopis or food basket. That was
another circumstance which handicapped him in resisting the assault. The initial attack was
successful. Tadia fell and rolled down the cliff and was stabbed by Francisco Diaz.
The appellants resorted to means of execution which directly and specially insured the
killing without any risk to themselves arising from any defense which the victim could have made.
Actually, he was not able to make any defense, unarmed and attacked unawares as he was. The
treacherous mode of attack is incontrovertible.

2. Abuse of Superior Strength is present but absorbed by Treachery.


It was held in this case that the attack was also attended with abuse of superiority. Two
armed young men unexpectedly assaulted an unarmed sexagenarian. However, abuse of superior
strength is merged with treachery.

3. The circumstance of old age cannot be considered aggravating.


There was no evidence that the accused deliberately intended to offend or insult the age of
the victim.

4. As to Francisco Diaz, Evident Premeditation should be appreciated.


Evident premeditation was appreciated as a generic aggravating circumstance. It may
reasonably be assumed that Francisco Diaz became aware that same afternoon that Tadia, who was
his neighbor, was going to the poblacion to lodge a complaint against him. That would explain
why early in the morning of the next day, September 5th, at about seven o’clock, he and his brother
were already in the hill or cliff waiting for Tadia who was on his way to town. Thus, there was a
sufficient interval of time, more than one-half day, within which appellant Francisco Diaz had full
opportunity for meditation and reflection and to allow his conscience to overcome the resolution
of his will had he desired to hearken to its warnings. However, with respect to Gerardo Diaz,
evident premeditation should not be appreciated. Obviously, he participated in the assault in order

298
CRIMINAL LAW 1 CASE DIGESTS
ATTY. LYAN DAVD JUANICO
BLOCK 1-M A.Y. 2020 - 2021

to help his elder brother who exercised some moral ascendancy over him and who was the
one directly affected by the embracing incident which preceded the killing.

299
CRIMINAL LAW 1 CASE DIGESTS
ATTY. LYAN DAVD JUANICO
BLOCK 1-M A.Y. 2020 - 2021

8. PEOPLE OF THE PHILIPPINES vs. CLARITO ARIZOBAL (at large), ERLY


LIGNES and TWO (2) JOHN DOES
PER CURIAM:

FACTS:
On 12 August 1994 two separate Informations were filed before the Regional Trial Court of
Cataingan, Masbate, charging Clarito Arizobal, Erly Lignes, Rogelio Gemino and two (2) John
Does with Robbery in Band with Homicide for robbing and slaying Laurencio Gimenez and his
son Jimmy Gimenez.

Clementina Gimenez, wife of victim Laurencio Gimenez, testified that on 24 March 1994 she
together with her husband Laurencio Gimenez and a grandchild were sound asleep in their house
in Tuybo, Cataingan, Masbate when Laurencio roused her from sleep and told her to open the door
because there were persons outside the house. Since it was pitch-dark she lit a kerosene lamp and
stood up to open the door when three (3) armed men confronted her pointing their guns at her. She
recognized two of them as Clarito Arizobal and Erly Lignes but failed to recognize the third person
who was wearing a maskara. Clarito asked Laurencio about his gun but he interjected that he has
no gun and if he wants, he can look around for it. While the man in maskara stood guard at the
door, Clarito and Lignes barged into the master’s bedroom and forcibly opened the
aparador, ransacked their cabinet and scattered everything on the floor until they found P8,000.00
among sheets of paper. Before leaving with their loot they ordered Laurencio to go with them to
Jimmy’s house because they have something to talk about Against his will, Laurencio went with
them. Clementina recalled that shortly after the group left she heard a volley of shots. Her
grandchild, as if sensing what befell her grandfather, could only mutter in fear that her lolo is dead.

Erlinda Gimenez, wife of Jimmy Gimenez, narrated that on 24 March 1994, after she and her son
had taken supper, her husband Jimmy with one Francisco Gimenez arrived. Jimmy informed
Erlinda that they had already bought a carabao. After he handed her skinning a chicken for their
supper, three (3) men suddenly appeared and ordered them to lie face down. One of them pushed
her to the ground while the others tied Francisco and Jimmy as they whipped the latter with an
armalite rifle. She noticed one of them wearing a mask, another a hat, and still another, a bonnet.
Realizing the utter helplessness of their victims, the robbers took the liberty of consuming the food
and cigarettes Erlinda was selling in her sari-sari store. Finding no softdrinks to complete their
snack, two (2) of the intruders ordered Erlinda to buy coke for them at the neighboring store. But
they warned her not to make any noise, much less alert the vendor. When they returned to the
house of Jimmy, the robbers proceeded to ransack the household in search for valuables. They
took around P1,000.00 from her sari-sari store and told them to produce P100,000.00 in exchange
for Jimmy’s life. Since the couple could not produce such a big amount in so short a time, Erlinda

300
CRIMINAL LAW 1 CASE DIGESTS
ATTY. LYAN DAVD JUANICO
BLOCK 1-M A.Y. 2020 - 2021

offered to give their certificate of large cattle. The culprits however would not fall for the ruse and
threw the document back to her. Three masked men then dragged Jimmy outside the house and
together with Laurencio brought them some fifty meters away while leaving behind Clarito
Arizobal and Erly Lignes to guard Francisco and Erlinda’s son. Moments later she heard a burst
of gunfire which reverberated through the stillness of the night. When the masked men returned to
Jimmy’s house, one of them informed Erlinda that her husband and father-in-law had been killed
for trying to escape. Upon hearing this, Erlinda, as if the heavens had fallen on her, slowly lost
consciousness.

The Trial court gave full credence to the testimony of the prosecution witnesses and rejected the
alibi of accused-appellant Erly Lignes and found both accused Clarito Arizobal and Erly Lignes
guilty of robbery with homicide.

ISSUE:
Whether or not the following aggravating circumstances are present:
1. Dwelling
2. Treachery
3. By a Band
4. Nighttime

RULING:

1. Dwelling is appreciated as an aggravating circumstance in this case.


Generally, dwelling is considered inherent in the crimes which can only be committed in the abode
of the victim, such as trespass to dwelling and robbery in an inhabited place. However, in robbery
with homicide the authors thereof can commit the heinous crime without transgressing the sanctity
of the victim’s domicile. In the case at bar, the robbers demonstrated an impudent disregard of the
inviolability of the victims’ abode when they forced their way in, looted their houses, intimidated
and coerced their inhabitants into submission, disabled Laurencio and Jimmy by tying their hands
before dragging them out of the house to be killed.

2. Treachery does not apply in this case.


Treachery was incorrectly considered by the trial court. The accused stand charged with, tried and
convicted of robbery with homicide. This special complex crime is primarily classified in this
jurisdiction as a crime against property, and not against persons, homicide being merely an incident
of robbery with the latter being the main purpose and object of the criminals. As such, treachery
cannot be validly appreciated as an aggravating circumstance.

301
CRIMINAL LAW 1 CASE DIGESTS
ATTY. LYAN DAVD JUANICO
BLOCK 1-M A.Y. 2020 - 2021

3. Band cannot be considered aggravating in this case.


While it appears that at least five malefactors took part in the commission of the crime, the
evidence on record does not disclose that “more than three” persons were armed, and robbery in
band means “more than three armed malefactors united in the commission of robbery.” Nowhere
in the records can we gather that more than three (3) of the robbers were armed. Hence, “band”
cannot be aggravating where no proof is adduced that at least four of the five perpetrators involved
in this case were armed.

4. Nighttime was also not appreciated in this case.


The fact that the offense was committed at 9:30 in the evening does not suffice to sustain
nocturnidad for, by itself, nighttime is not an aggravating circumstance. To be properly so
considered, it must be shown that nocturnidad was deliberately and intentionally sought by
accused-appellants to help them realize their evil intentions. Nowhere can we infer from the
records that the malefactors sought the cover of darkness to facilitate the accomplishment of their
devious design. On the contrary, the locus criminis was well lighted and nighttime was merely an
incidental element to the whole drama.

The offenders did not commit two separate counts of robbery with homicide but only a delito
continuado, as the ransacking of the two houses and the killing of the victims were not entirely
disconnected and distinct acts of depredation. They arose from a single criminal impulse and
intent, “there being unity of purpose and of right violated.”

302
CRIMINAL LAW 1 CASE DIGESTS
ATTY. LYAN DAVD JUANICO
BLOCK 1-M A.Y. 2020 - 2021

9. PEOPLE OF THE PHILIPPINES vs. AMADO DANIEL alias “AMADO ATO”


G.R. No. L-40330; November 20, 1978
MUÑOZ PALMA, J:

FACTS:
Margarita Paleng was temporarily boarding at a house located at Pinsao, Guisad, Baguio City, as
she was then a first year high school student at the Baguio Eastern High School. She had
just arrived in the City from Tublay in a Dangwa bus. Because it was then raining and the bus was
parked several meters away from the bus station, she waited inside the bus and after about three
minutes of waiting, the accused came and started molesting her by inquiring her name and getting
hold of her bag. She called the attention of the bus driver and the conductor about the actuation of
the accused, but it seemed that the former were also afraid of him. She rode a jeep where the
accused also followed her. When she alighted the jeep, the accused also alighted and tried to carry
her bag.

When she reached her boarding house, the accused dashed in and closed the door behind him and
also entered the room. He pulled a dagger eight inches long and threatened her: that if she talks,
she will kill her. Margarita was stunned into silence because of her fear. Thereupon, the accused
held her hair with his left hand and forced her to lie down in bed, placed his left hand with a
handkerchief in Margarita’s mouth, at the same time holding the dagger and her neck with his right
hand. She was forcibly made to lie down and, at this moment, the accused removed the buttons of
his pants. Her attempts to extricate herself from the accused was to no avail as, she was only 4 ft.
and 8 inches tall and weighed about 95 to 100 pounds while the accused was 5 ft. and 7 inches tall
and weighed about 126 pounds. He then held his penis used his thigh to separate the legs of
Margarita tried, but failed, to remove her panty. He guided his penis inside the vagina of the
complainant after prying open the part of her panty covering her private parts. Then he succeeded
in having carnal knowledge of the offended party then MArgarita lost consciousness. She was
brought to the Baguio General Hospital by his father the next day where she was examined.

The appellant claimed that he and Margarita were acquainted with each other since 1963, and there
were occasions when they rode together in a bus; that the incident of September 20, 1965 inside
the room of Margarita was with the latter’s consent, and in fact it was the second time he had
carnal knowledge with her, the first time having occurred inside a shack; that he promised
Margarita that he would marry her, but to his surprise, she filed the instant complaint against him.

ISSUE:
Whether or not the aggravating circumstance of Dwelling is applicable in this case.

303
CRIMINAL LAW 1 CASE DIGESTS
ATTY. LYAN DAVD JUANICO
BLOCK 1-M A.Y. 2020 - 2021

RULING:

Yes, dwelling is applicable in this case.

The crime committed by the appellant is rape with the use of a deadly weapon with the aggravating
circumstance of having been committed in the dwelling of the offended party. Although Margarita
was merely renting a bedspace in a boarding house, her room constituted for all intents and
purposes a “dwelling” as the term is used in Article 14(3), Revised Penal Code. It is not necessary,
under the law, that the victim owns the place where he lives or dwells. Be he a lessee, a boarder,
or a bedspacer, the place is his home the sanctity of which the law seeks to protect and uphold.

304
CRIMINAL LAW 1 CASE DIGESTS
ATTY. LYAN DAVD JUANICO
BLOCK 1-M A.Y. 2020 - 2021

10. THE PEOPLE OF THE PHILIPPINES vs. APOLONIO APDUHAN, JR. alias JUNIOR,
ET AL., APOLONIO APDUHAN, JR. alias JUNIOR
G.R. No. L-19491; August 30, 1961
CASTRO, J.:

FACTS:
On or about the 23rd day of May, 1961, at about 7:00 o'clock in the evening, in the municipality
of Mabini, province of Bohol, Philippines, Apduhan and five (5) other persons who are still at
large all of them armed with different unlicensed firearms, daggers, and other deadly weapons,
conspired, confederated and helped one another, with intent to gain did then and there willfully,
unlawfully and feloniously enter, by means of violence, the dwelling house of the spouses
Honorato Miano and Antonia Miano, which was also the dwelling house of their children, the
spouses Geronimo Miano and Herminigilda de Miano; and, once inside the said dwelling
house. The accused attacked, hacked and shoot Geronimo Miano and another person by the name
of Norberto Aton, who happened to be also in the said dwelling house that resulted in their death.
The accused also took three hundred twenty two pesos belonging to the Miano spouses. During
trial, Apduhan changed his former plea of not guilty to a plea of guilty. After the trial judge had
repeatedly apprised him of the severity of the offense, and the strong possibility that the capital
penalty might be imposed upon him despite the plea, Apduhan persisted to plead guilty with the
request that death penalty be not imposed.

The trial court convicted Apolonio Apduhan Jr. of robbery with homicide and sentencing him to
death. The prosecution contends that the act was committed with the special aggravating
circumstance that the crime was committed by a band with the use of unlicensed firearms, and
other aggravating circumstances being (1) That the crime was committed in the dwelling of the
offended parties without any provocation from the latter; (2) That the nighttime was purposely
sought to facilitate the commission of the crime; and (3) That advantage was taken of superior
strength.

ISSUE:
Whether or not the following aggravating circumstances are present:
1. The act was committed with the special aggravating circumstance that crime was
committed by a band with the use of unlicensed firearms that would determine the penalty
imposable.
2. That the crime was committed in the dwelling of the offended parties without any
provocation from the latter
3. That the nighttime was purposely sought to facilitate the commission of the crime; and

305
CRIMINAL LAW 1 CASE DIGESTS
ATTY. LYAN DAVD JUANICO
BLOCK 1-M A.Y. 2020 - 2021

4. That advantage was taken of superior strength.

RULING:
1. The special aggravating circumstance that crime was committed by a band with the use of
unlicensed firearms cannot be appreciated in this case.
Art. 295 provides that if any of the classes of robbery described in subdivisions 3, 4, and 5
of art. 294 is committed by a band, the offender shall be punished by the maximum period of the
proper penalty. Correspondingly, the immediately following provisions of art. 296 define the term
"band", prescribe the collective liability of the members of the band, and state that "when any of
the arms used in the commission of the offense be an unlicensed firearm, the penalty to be imposed
upon all the malefactors shall be the maximum of the corresponding penalty provided by law."
Viewed from the contextual relation of articles 295 and 296, the word "offense" mentioned in the
above-quoted portion of the latter article logically means the crime of robbery committed by a
band, as the phrase "all the malefactors" indubitably refers to the members of the band and the
phrase "the corresponding penalty provided by law" relates to the offenses of robbery described in
the last three subdivisions of art. 294 which are all encompassed within the ambit of art. 295.
Evidently, therefore, art. 296 in its entirety is designed to amplify and modify the provision on
robbery in band which is nowhere to be found but in art. 295 in relation to subdivisions 3, 4, and
5 of art. 294. Verily, in order that the aforesaid special aggravating circumstance of use of
unlicensed firearm may be appreciated to justify the imposition of the maximum period of the
proper penalty, it is a condition sine qua non that the offense charged be robbery committed by a
band within the contemplation of art. 295. To reiterate, since art. 295 does not apply to subdivisions
1 and 2 of art. 294, then the special aggravating factor in question, which is solely applicable to
robbery in band under art. 295, cannot be considered in fixing the penalty imposable for robbery
with homicide under art. 294(1), even if the said crime was committed by a band with the use of
unlicensed firearms.

2. Dwelling is appreciated in this case.


The settled rule is that dwelling is aggravating in robbery with violence or intimidation of
persons, like the offense at bar. The rationale behind this pronouncement is that this class of
robbery could be committed without the necessity of transgressing the sanctity of the home.
Morada is inherent only in crimes which could be committed in no other place than in the house
of another, such as trespass and robbery in an inhabited house.

3. Nighttime is appreciated in this case.


Nocturnity is aggravating when it is purposely and deliberately sought by the accused to
facilitate the commission of the crime or to prevent their being recognized or to insure unmolested
escape. Nocturnidad must concur with the intent and design of the offender to capitalize on the

306
CRIMINAL LAW 1 CASE DIGESTS
ATTY. LYAN DAVD JUANICO
BLOCK 1-M A.Y. 2020 - 2021

intrinsic impunity afforded by the darkness of night. In the case at bar, the affidavit of the accused
Apduhan shows that he and his co-malefactors took advantage of the nighttime in the perpetration
of the offense as they waited until it was dark before they came out of their hiding place to
consummate their criminal designs.

4. Abuse of Superiority was withdrawn by the prosecution but the court believes that the said
withdrawal was ill-advised.
The circumstance of abuse of superiority was, however, withdrawn by the prosecution on
the ground that since the offense of robbery with homicide was committed by a band, the element
of cuadrilla necessarily absorbs the circumstance of abuse of superior strength. The court believes
that said withdrawal was ill-advised since the circumstances of band and abuse of superiority are
separate and distinct legal concepts. The element of band is appreciated when the offense is
committed by more than three armed malefactors regardless of the comparative strength of the
victim or victims. Hence, the indispensable components of cuadrilla are (1) at least four
malefactors and (2) all of the four malefactors are armed. On the other hand, the gravamen of abuse
of superiority is the taking advantage by the culprits of their collective strength to overpower their
relatively weaker victim or victims. Hence, in the latter aggravating factor, what is taken into
account is not the number of aggressors nor the fact that they are armed, but their relative physical
might vis-a-vis the offended party.

307
CRIMINAL LAW 1 CASE DIGESTS
ATTY. LYAN DAVD JUANICO
BLOCK 1-M A.Y. 2020 - 2021

11. PEOPLE OF THE PHILIPPINES vs. MARTIN MANDOLADO and JULIAN


ORTILLANO
G.R. No. L-51304-05; June 28, 1983
GUERRERO, J.:

FACTS:
Julian Ortillano, Martin Mandolado, Conrado Erinada and Anacleto Simon, trainees/draftees of
the Armed Forces of the Philippines and assigned to the 3rd Infantry Battalion of the Philippine
Army, were passengers of a bus bound for Midsayap, North Cotabato. They alighted at the bus
terminal in Midsayap. They decided to drink ESQ rum, at the said bus terminal. Mandolado got
drunk and went inside the public market. Subsequently, he returned, grabbed his .30 caliber
machine gun and started firing. His companions tried to dissuade him but he nonetheless continued
firing his gun. Erinada and Simon sensed trouble which is why they ran away and hailed and
boarded a Ford Fiera. Appellants followed and boarded the vehicle. The soldiers forced the driver
of the Ford Fiera to bring them to the Midsayap crossing. Mandolado got a knife and attacked the
driver. The Ford Fiera sped away when the appellants alighted in the said crossing but Mandolado
fired his gun at the said car hitting the right side of the back of the driver’s sister who was then on
board said vehicle. They boarded another jeep driven by Herminigildo Tenorio which was bound
for Cotobato City. On the way, both appellants kept firing their guns prompting Tenorio to say that
if they will not stop firing their gun, he will ram the jeep into something. Upon learning that the
jeep was bound for Cotabato City and not Pikit, North Cotabato, appellant Mandolado got angry,
“cocked” his gun and ordered the driver to stop. While the jeep was coming to a full stop, Conrado
Simon and Anacleto Erinada immediately jumped off the jeep and ran towards their detachment
camp. Appellants also got off the jeep, Mandolado fired his .30 caliber machine gun at and hit the
occupants of the jeep, while Ortillano likewise, fired his armalite, not at the occupants of said jeep
but downwards hitting the ground. These bursts of gunfire were heard by both Conrado Erinada
and Anacleto Simon who were then already about fifty meters away from the jeep while running
towards their detachment camp.

The appellants ran away from the scene and boarded another vehicle, alighting at Pinaring
crossing. Mandolado proceeded to a house where he left his belongings and changed his wet
uniform. They boarded a bus and as the bus was not proceeding Pikit, Cotobato, appellants alighted
at the Midsayap crossing and waited for a bus bound for Pikit. They were able to riddle in a truck
and arrived at their camp. They returned their firearms but did not report the incident. While the
appellants went to see a movie the next day, they saw a certain Sgt. Villanueva who was then
leaving for Luzon. Sgt. Villanueva informed the appellants that they were suspects in the Tenorio
and Mendoza killings. Immediately thereafter, appellant Mandolado purchased two passenger

308
CRIMINAL LAW 1 CASE DIGESTS
ATTY. LYAN DAVD JUANICO
BLOCK 1-M A.Y. 2020 - 2021

tickets for Manila. However, before appellants could board the ship bound for Manila, they were
apprehended.

Appellant Mandolado admitted the killing of Tenorio and Mendoza whereas appellant Ortillano
admitted his presence at said killings and of his having fired his armalite downwards after appellant
Mandolado fired upon the killed the afore-named victims. Mandolado was found guilty by the
RTC of the crime of Murder for the killing of Tenorio and for the killing of Mendoza with the
aggravating circumstances of (1) advantage was taken of his being a draftee in the Philippine
Army, and (2) abuse of confidence or obvious ungratefulness.

ISSUE:
Whether or not the following aggravating circumstances are present:
1. Treachery as a qualifying circumstance.
2. Taking advantage of public position
3. Abuse of confidence or Obvious Ungratefulness

RULING:
1. The killing of the two victims in the case at bar is correctly qualified as murder there being present
the qualifying circumstance of treachery.
There is treachery when the offender commits any of the crimes against the person, employing
means, methods or forms in the execution thereof which tend directly and specially to insure its
execution, without risk to himself arising from the defense which the offended party might make.
It is quite clear and explicit that when the appellants alighted from the jeep, the accused Mandolado
immediately fired his .30 caliber machine gun at the occupants of the jeep, the victims Nolasco
Mendoza and Herminigildo Tenorio, and both of them died instantaneously on the spot, and from
this sudden means or manner of attack, it can reasonably be concluded that it tended directly to
insure its execution without risk to the appellant-assailant and also deprive the victims of any
chance or opportunity to defend themselves. We also rule that the particular means or manner
employed by the appellant-assailant was consciously or deliberately sought and not a mere
accidental circumstance resorted to on the spur of the moment on the basis of the evidence that the
appellant had previously and repeatedly fired his .30 caliber machine gun at the bus terminal in
Midsayap and had also fired the machine gun at the Ford Fiera which took them to Midsayap
junction and that appellants waited for sometime riding on board the jeep driven by Tenorio before
they ordered the jeep to stop, alight therefrom and then shoot the occupants therein.

2. Taking advantage of public position was not appreciated in this case.


While it may be true that a soldier in the Armed Forces of the Philippines is deemed as one who
holds public position, there is no persuasive showing that herein appellants being draftees of the

309
CRIMINAL LAW 1 CASE DIGESTS
ATTY. LYAN DAVD JUANICO
BLOCK 1-M A.Y. 2020 - 2021

Army, in full military uniform and carrying their high-powered firearms, facilitated the
commission of the crimes they were charged. It may be conceded that as draftees, the accused
could easily hitchhike with private vehicles, as in the case of the deceased Tenorio’s owner-type
jeep, but there is no evidence that when they stopped the jeep the accused already intended to shoot
the occupants of the vehicle.

3. Abuse of Confidence and Obvious Ungratefulness was not appreciated in this case.
In order that abuse of confidence be deemed as aggravating, it is necessary that there exists
a relation of trust and confidence between the accused and one against whom the crime was
committed and the accused made use of such a relationship to commit the crime. It is also essential
that the confidence between the parties must be immediate and personal such as would give that
accused some advantage or make it easier for him to commit the crime; that such confidence was
a means of facilitating the commission of the crime, the culprit taking advantage of the offended
party’s belief that the former would not abuse said confidence. In the instant case, there is
absolutely no showing of any personal or immediate relationship upon which confidence might
rest between the victims and the assailants who had just met each other then. Consequently, no
confidence and abuse thereof could have facilitated the crimes.
Similarly, there could have been no obvious ungratefulness in the commission of the crime
for the simple reason that the requisite of trust of the victims upon the accused prior to the criminal
act and the breach thereof as contemplated under Article 14, par. 4 of the Revised Penal Code are
manifestly lacking or non-existent. In all likelihood, the accused Army men in their uniforms and
holding their high-powered firearms cowed the victims into boarding their jeep for a ride at a
machine gun point which certainly is no source of gratefulness or appreciation.

310
CRIMINAL LAW 1 CASE DIGESTS
ATTY. LYAN DAVD JUANICO
BLOCK 1-M A.Y. 2020 - 2021

12. PEOPLE OF THE PHILIPPINES vs. JEFFREY GARCIA y CARAGAY and THREE
JOHN DOES JEFFREY GARCIA y CARAGAY
G.R. NO. 141125; February 28, 2002
ABAD SANTOS, J.:

FACTS:
The victim, Cleopatra Changlapon, was nineteen years old and a sophomore student of B.S.
Physical Therapy at the Baguio Central University. On July 14, 1998, she left school at 6:30 p.m.
to go home to Km. 3, La Trinidad, Benguet. As she was crossing Bonifacio Street, Baguio City,
she saw a white van approaching so she stopped to let it pass. Suddenly, the van stopped in front
of her. The rear door slid open and Cleopatra was pulled by the arms into the van. Something was
sprayed on her face which made her eyes sting and feel dizzy. She shouted, then she felt a fist blow
on her stomach and she fell unconscious. When she came to, she was already in a room totally
undressed, lying flat on her back on a bed. There were four men who took turns in having sexual
intercourse with her. The accused-appellant took his turn and went on top of her. One of the men
sat on her right leg and pinned it down, while another held her left leg. Cleopatra tried to punch
the accused-appellant with her right hand, but the Bombay-looking man held her right arm.
Accused-appellant then had sexual intercourse with her while holding her left arm.

One of the men again sprayed something on Cleopatra’s face which made her vision blurred. She
heard somebody say that it was 1:30. After that, she blacked out. When she regained
consciousness, she was lying by the roadside somewhere between Tam-awan and Longlong. It was
still dark. She already had her clothes on. She felt pain all over her body and was unable to move.
A taxi passed by and picked her up. Although she was afraid to ride the taxi, she boarded it just to
get home. The taxi brought her to her house.

The cartographic sketches were published in the Sun-Star newspaper. Police officers saw the
sketches and noticed that one of the suspects depicted in the cartographic sketch bore a striking
resemblance to accused-appellant, who was in their custody. Cleopatra was summoned to the
police station and positively identified the accused appellant.

The Court Finds the Accused Jeffrey Garcia guilty beyond reasonable doubt of the complex crime
of Forcible Abduction with Rape and likewise of the three (3) crimes of rape in conspiracy with
three (3) others whose identities and whereabouts are yet unknown. The trial court appreciated the
aggravating circumstances of nighttime, superior strength and motor vehicle.

311
CRIMINAL LAW 1 CASE DIGESTS
ATTY. LYAN DAVD JUANICO
BLOCK 1-M A.Y. 2020 - 2021

ISSUE:
1. Whether or not the crime committed by the appellant were the Complex Crime of Forcible
Abduction with Rape and three counts of Rape.
2. Whether or not the trial court correctly appreciated the aggravating circumstances.

HELD:
1. The crime committed were the Complex Crime of Forcible Abduction with Rape and three counts
of Rape.
In the case at bar, the information sufficiently alleged the elements of forcible abduction.
It was likewise alleged that the accused-appellant and his three co-accused conspired, confederated
and mutually aided one another in having carnal knowledge of the complainant by means of force
and intimidation and against her will. Aside from alleging the necessary elements of the crimes,
the prosecution convincingly established that the carnal knowledge was committed through force
and intimidation. Moreover, the prosecution sufficiently proved beyond reasonable doubt that
accused-appellant succeeded in forcibly abducting the complainant with lewd designs, established
by the actual rape. Hence, accused-appellant is guilty of the complex crime of forcible adduction
with rape. He should also be held liable for the other three counts of rape committed by his three
co-accused, considering the clear conspiracy among them shown by their obvious concerted efforts
to perpetrate, one after the other, the crime.
There can only be one complex crime of forcible abduction with rape. The crime of forcible
abduction was only necessary for the first rape. Thus, the subsequent acts of rape can no longer be
considered as separate complex crimes of forcible abduction with rape. They should be detached
from and considered independently of the forcible abduction. Therefore, accused-appellant should
be convicted of one complex crime of forcible abduction with rape and three separate acts of rape.

2. The trial court erred in appreciating nighttime, superior strength and motor vehicle in this
case.
These were not alleged in the information. Under the amended provisions of Rule 110,
Sections 8 and 9 of the Revised Rules on Criminal Procedure, which took effect on December 1,
2000, aggravating as well as qualifying circumstances must be alleged in the information,
otherwise, they cannot be considered against the accused even if proven at the trial. Being
favorable to the accused-appellant, this rule should be applied retroactively in this case.

13. PEOPLE OF THE PHILIPPINES vs.ARMANDO RODAS and JOSE RODAS, SR.,
G.R. No. 175881; August 28, 2007
CHICO-NAZARIO, J.:

312
CRIMINAL LAW 1 CASE DIGESTS
ATTY. LYAN DAVD JUANICO
BLOCK 1-M A.Y. 2020 - 2021

FACTS:
Titing Asenda, was helping his brother, Danilo Asenda in harvesting the latter’s corn. On the same
day, at around 8:00 in the evening, a benefit dance at Milaub, which was sponsored by Boboy
Raquilme, was being held. Among those roaming in the vicinity of the dance hall were Alberto
Asonda and Ernie Anggot. They stopped and hung out near the fence to watch the affair. Titing
Asenda was standing near them. They saw Charlito Rodas, Armando Rodas, Jose Rodas, Jr., and
Jose Rodas, Sr. surround Titing Asenda. Suddenly, without a word, Charlito Rodas, armed with a
hunting knife, stabbed Titing at the back. Armando Rodas then clubbed Titing with a chako hitting
him at the left side of the nape causing him to fall. Thereafter, Jose Rodas, Sr. handed to Jose
Rodas, Jr. a bolo which the latter used in hacking Titing, hitting him on the left elbow. Alberto
Asonda and Ernie Anggot tried to help Titing but Armando Rodas prevented them by pointing a
gun at them and firing it towards the sky. Titing died. The trial court found the accused-appellants
guilty of the crime of Murder. The appellants argue that assuming arguendo they are guilty, they
are liable only for the crime of homicide, not murder. They contend that treachery was absent since
they, together with Charlito and Jose Jr., met the victim casually in the dance hall.

ISSUE:
1. Whether or not treachery is present in this case as a qualifying circumstance.
2. Whether or not the following aggravating circumstances are present:
a. Evident Premeditation
b. Nocturnity
c. Abuse of Superior Strength

RULING:
1. Treachery attended the killing as a qualifying circumstance.
The essence of treachery is the sudden and unexpected attack by the aggressor on an
unsuspecting victim, depriving the latter of any real chance to defend himself, thereby ensuring its
commission without risk to the aggressor, and without the slightest provocation on the part of the
victim. In the case under review, the victim was completely unaware that he was going to be
attacked. He was not forewarned of any danger to himself as there was no altercation or
disagreement between the accused and the victim. If treachery may be appreciated even when the
victim was forewarned, more so should it be appreciated when the victim was not, as in the case
at bar. The suddenness of the attack, the number of the accused and their use of weapons against
the unarmed victim prevent the possibility of any defense or retaliation by the victim. The fact that
the victim was already sprawled on the ground and still Jose Jr. hacked him with a bolo clearly
constitutes treachery.

313
CRIMINAL LAW 1 CASE DIGESTS
ATTY. LYAN DAVD JUANICO
BLOCK 1-M A.Y. 2020 - 2021

2. Aggravating Circumstances present


a. Evident Premeditation is not applicable in this case.
For evident premeditation to be appreciated, the following elements must be established:
(1) the time when the accused decided to commit the crime; (2) an overt act manifestly indicating
that he has clung to his determination; and (3) sufficient lapse of time between decision and
execution to allow the accused to reflect upon the consequences of his act. Evident premeditation
must be established by clear and positive proof; that is, by proof beyond reasonable doubt. The
essence of premeditation is that the execution of the criminal act was preceded by cool thought
and reflection upon the resolution to carry out the criminal intent during a space of time sufficient
to arrive at a calm judgment. In the case at bar, the prosecution failed to show the presence of any
of these elements.

b. Nocturnity cannot be appreciated in this case.


This circumstance is considered aggravating only when it facilitated the commission of the
crime, or was especially sought or taken advantage of by the accused for the purpose of impunity.
The essence of this aggravating circumstance is the obscuridad afforded by, and not merely the
chronological onset of, nighttime. Although the offense was committed at night, nocturnity does
not become a modifying factor when the place is adequately lighted and, thus, could no longer
insure the offender’s immunity from identification or capture. In the instant case, the prosecution
failed to show that nighttime facilitated the commission of the crime, or was especially sought or
taken advantage of by the accused for the purpose of impunity. The crime scene was sufficiently
lighted by a Petromax which led to the identification of all the accused.

c. The aggravating circumstance of abuse of superior strength attended the killing but cannot be
separately appreciated because it is absorbed in treachery.

There was a glaring disparity of strength between the victim and the four accused. The
victim was unarmed while the accused were armed with a hunting knife, chako and bolo. It is
evident that the accused took advantage of their combined strength to consummate the offense.

314
CRIMINAL LAW 1 CASE DIGESTS
ATTY. LYAN DAVD JUANICO
BLOCK 1-M A.Y. 2020 - 2021

14. People v. Damaso


G.R. No. L-30116, November 20, 1978
MEDIALDEA, J.:

FACTS:
On November 21, 1959, Donata Rebolledo and her son-in-law, Victoriano De La Cruz heard the
barking’s of dog outside their outside when two men armed with guns entered and pointed weapon
at them, tied up the hands of Victoriano, covered him with a blanket and asked Donata of the
whereabouts of her Daughter Catalina Sabado. Out of fear she kept silent and blocked the door
leading to the room of her daughter. Donata was also ordered to open her aparador from which the
men took valuable jewelry, clothing, documents, and cutting instruments. Two men brought
Catalino down from the house and asked her where they could find Susana Sabado, Donata’s
another daughter who was then in her store. Thereafter, Donata heard the men open Susana’s store.
After the intruders left, Donata untied the hands of Victoriano and asked him to check on the stores
and to see if her daughters were there. When Donata’s two daughter could not be found, Donata
ordered Victoriano to report the incident to the barrio lieutenant.

The following morning the two women were found already dead with wounds in several parts of
their bodies. A few days after the incident, Donata singled out Fausto Damaso as one of the men
who went up to her house on that evening. She and Victoriano had recognized Damaso because of
the light coming from a kerosene lamp placed on a small table near the aparador. Damaso,
however, initially denied ever having been to Donata’s house that night. Later, four other suspects
in the persons of co-accused Gregorio, Eugenio Alviar and Espejo were rounded up by the police.
The suspects were convicted of special complex crime of robbery with double homicide.

ISSUE:
Whether or not the crime was attended by the aggravating circumstance of armed band, treachery,
and uninhabited place?

RULING:
The trial court considered separately the three circumstances of armed band, treachery and
uninhabited place where under other situations one may be considered absorbed or inherent in the
other. The elements of each circumstance subsist independently and can be distinctly perceived
thereby revealing a greater degree of perversity on the part of the accused.

The presence of an armed band is to be considered as a generic aggravating circumstance under


Article 14(6) of the Revised Penal Code. The aggravating circumstance of band exists whenever
more than three armed malefactors act together in the commission of an offense. In the case at bar,

315
CRIMINAL LAW 1 CASE DIGESTS
ATTY. LYAN DAVD JUANICO
BLOCK 1-M A.Y. 2020 - 2021

the counsel concedes that at least three of the accused-appellants, namely Eugenio, Alviar, and
Gregio, were armed during the commission of the crime. He doubts, however, whether accused
Damaso carried any weapon and whether the “two stones’ carried by accused Espejo fall under the
catergory of “arms.” However, in the case at bar, it was clear from the statements made by the
accused-appellants that Damaso was armed during the night of the commission of the crime, and
it is immaterial what kind of firearm he carried, the only important thing is that he was armed that
night.

As to the aggravating circumstance of treachery it is also present in this case. Treachery is present
if the victim is killed while bound in such a manner as to be deprived of the opportunity to repel
the attach or escape with any possibility of success. In the case at bar, the fact that the bodies of
Catalina and Susana were found dead with their arms tied behind their backs as well as the
admission of Gregorio in his statement that he killed the sisters while their arms where held by
Eugenio and Damaso lead this Court to conclude that the killing of two women was done under
treacherous circumstance.

As regards to the aggravating circumstance of uninhabited place, the Court considers a crime as
having been committed in an uninhabited place when the killing was done in a secluded place. The
uninhibitedness of a place is determined not by the distance of the nearest house to the scene of
the crime, but whether or not in the place of commission, there was reasonable possibility of the
victim receiving help. In the case at bar, the two women were killed in a sugarcane field to the
national highway and considering such killing was done during nighttime and the sugarcane in the
field was tall enough to obstruct the view of neighbors and passerby, there was no reasonable
possibility for the victims to receive any assistance. That the accused deliberately sought the
solitude of the place shown by the fact that they brought the victims to the sugarcane field although
they could have disposed of them right in the house of Donata where they were found.

Therefore, the crime committed by the appellants is robbery with homicide. The proper penalty to
imposed, considering the presence of three aggravating circumstances found by the trial court, is
reclusion perpetua to death, when by reason or on occasion of the robbery the crime of homicide
shall have been committed.

316
CRIMINAL LAW 1 CASE DIGESTS
ATTY. LYAN DAVD JUANICO
BLOCK 1-M A.Y. 2020 - 2021

15. People v. Baldera


G.R. No. L-2390, April 24, 1950
REYES, J.:

FACTS:
A Casa Manila bus loaded with passenger left Batangas bound for Manila. On the highway in
barrio of the same province, it was held up by a group of five or six armed men. One of these, later
identified as appellant Pedro Baldera, who was armed with a.45 caliber pistol, fired a shot and this
was followed by a hail of bullets coming from different directions. As a result, several passengers
were wounded. After the shooting incident, appellant got on the bus and threatened the passengers
with his gun and took P90 from Pastor and P34 from Villena. After the appellant alighted the bus,
the driver of the bus headed to municipal building to report the incident to the authorities.

Shortly after the commission of the crime, the appellant was arrested in the municipality of
Batangas in connection with the theft of a radio, and as his features tallied with the personal
description of one of the highway men given to the chief of police by some of the passengers of
the held-up bus, he was also investigated in connection with the hold-up bus, and he then made a
confession, admitting his participation in the crime as the one who, armed with pistol, boarded the
bus and through intimidation relieved Villena of her money. The appellant is prosecuted for the
crime of robbery with homicide and serious and less serious physical injuries.

ISSUE:
Whether or not the crime is committed by a band and aggravated by recidivism?

RULING:
The counsel of the appellant contends that the lower court erred in holding that the crime
committed is robbery in band, alleging that there no sufficient proof that the perpetrator thereof
was more than three armed men. The fact, however, that there were more than three armed men in
the group that held up the bus appears in appellant’s own confession and is also established by the
uncontradicted testimony of one of the witnesses. Further, the point is really not material because
in the crime of robbery with homicide, it is not essential that the robber be in band, although that
circumstance may be considered as an aggravation in the imposition of the penalty. That even if it
be not considered as such in this case, there would still remain the other aggravating circumstance
that the robbery was perpetrated by attacking a vehicle, which is not offset by any mitigating
circumstance.

As to the aggravating circumstance of recidivism, the lower court erred in appreciating such
aggravating circumstance by reason of his previous conviction for theft. The crime of theft was
committed on or about December 30, 1947 while the offense now charged took place seven days
317
CRIMINAL LAW 1 CASE DIGESTS
ATTY. LYAN DAVD JUANICO
BLOCK 1-M A.Y. 2020 - 2021

before that date. Therefore, the proper penalty for the crime charged with two aggravating
circumstances is life imprisonment.

318
CRIMINAL LAW 1 CASE DIGESTS
ATTY. LYAN DAVD JUANICO
BLOCK 1-M A.Y. 2020 - 2021

16.People v. Melendrez
G.R. No. L-33913; December 19, 1933
AVANCEÑA, C.J.:

FACTS:
The accused in this case is Ricardo Melendrez who is a habitual delinquent, he having been
previously convicted twice of the crime of theft and once of the crime of estafa and having been
last convicted of the crime of estafa. Melendrez forcibly open the door of the store located in Pasay
that was occupied by Tin Bun Boc, and once inside the store, took and stole Tin Bun Boc’s personal
properties. On the date of the trial, Melendrez pleaded guilty to the crime charged. Thus, the court
find him guilty of robbery.

ISSUE:
Whether or not the aggravating circumstance of recidivism should be considered in this case?

RULING:
Yes, the aggravating circumstance in this case should be appreciated. The fiscal contends that the
aggravating circumstance of recidivism should be taken into account against the appellant. This
claim of the fiscal is in accordance with the judgment rendered by this court in banc in the case
of People vs. Aguinaldo (47 Phil., 728) while the old Penal Code was in force. But the enforcement
of the Revised Penal Code has resulted in a difference of opinion regarding this point on the part
of the members of this court. For this reason, after reviewing all the decisions affecting the matter,
rendered by this court both in banc and in division, it is now held that the aggravating circumstance
of recidivism should be taken into account in imposing the principal penalty in its corresponding
degree, notwithstanding the fact that the defendant is also sentenced to suffer an additional penalty
as a habitual delinquent. Thus, the proper penalty prescribed for said crime is prision
correccional in its medium degree. Inasmuch as there is a concurrence therein of one mitigating
and one aggravating circumstance, this penalty should be imposed in its medium degree.

319
CRIMINAL LAW 1 CASE DIGESTS
ATTY. LYAN DAVD JUANICO
BLOCK 1-M A.Y. 2020 - 2021

17. US v. Manalinde
G.R. No. 5292, August 28, 1909
TORRES, J.:

FACTS:
Juan Igual, a Spaniard, was seated on a chair in the doorway of Sousa’s store when he suddenly
received a wound on the head delivered from behind and inflicted by Kris. Ricardo Doroteo, a
clerk in the said store, upon hearing the noise and the cry of the wounded man, ran to his assistance
and found him lying on the ground. The Moro Manalinde, the aggressor, approached Choa and as
the latter was putting down his load in front of the door of a store and was about to enter, attacked
him with the same weapon, inflicting a sever wound in the left shoulder which made him fall into
the ground. Manalinde had entered the town carrying his weapon wrapped up in banana leaves and
also escaped by running away from the town.

When Manalinde was arrested he pleaded guilty and confessed that he had perpetrated the crime
charged. Further, he stated that his wife had died about a hundred days before and that he had come
from his home and by order of Datto Rajamudah Mupuck, who directed him to kill somebody,
because the said Mupuck had certain grievances to avenge a certain lieutenant and sergeant. That
if he carries the plan successfully, he would give him a pretty woman on his return, but that in case
he was captured he was to say that he performed the killing by order of Matiayo, Datto Piang,
Tambal and Inug. He was charged with the crime of murder.

ISSUE:
Whether or not promise of reward and premeditation should be considered in this case?

RULING:
Yes, promise of reward and premeditation are present, which in the present case are held to be
generic. In the case at bar, since the crime has already been qualified as committed with the
treachery, because the accused confessed that he voluntarily obeyed the order given him by Datto
Mupuck to go juramentado and kill some one in the town of Cotabato, with the promise that if he
escaped punishment he would be rewarded with a pretty woman. Upon complying with the order
the accused undoubtedly acted of his own volition and with the knowledge that he would inflict
irreparable injury on some of his fellow-beings, depriving them of life without any reason
whatever, well knowing that he was about to commit a most serious deed which the laws in force
in this country and the constituted authorities could by no means permit. Datto Mupuck, who
ordered and induced him to commit the crimes, as well as the accused knew perfectly well that he
might be caught and punished in the act of committing them.

320
CRIMINAL LAW 1 CASE DIGESTS
ATTY. LYAN DAVD JUANICO
BLOCK 1-M A.Y. 2020 - 2021

As to the aggravating circumstance of premeditation, the fact that the arrangement between the
instigator and the tool considered the killing of unknown persons, the first encountered, does not
bar the consideration of the circumstance of premeditation. The nature and the circumstances
which characterize the crime, the perversity of the culprit, and the material and moral injury are
the same, and the fact that the victim was not predetermined does not affect nor alter the nature of
the crime. Moreover, even though in a crime committed upon offer of money, reward or promise,
premeditation is sometimes present, the latter not being inherent in the former, and there existing
no incompatibility between the two, premeditation can not necessarily be considered as included
merely because an offer of money, reward or promise was made, for the latter might have existed
without the former, the one being independent of the other. In the present case there can be no
doubt that after the crime was agreed upon by means of a promise of reward, the criminal by his
subsequent conduct showed a persistency and firm intent in his plan to carry out the crime which
he intentionally agreed to execute, it being immaterial whether Datto Mupuck did or did not
conceive the crime, once Manalinde obeyed the inducement and voluntarily executed it.

321
CRIMINAL LAW 1 CASE DIGESTS
ATTY. LYAN DAVD JUANICO
BLOCK 1-M A.Y. 2020 - 2021

18. People v. Ilaoa


G.R. No. 94308, June 16, 1994
BELLOSILLO, J.:

FACTS:
Appellant Ruben Ilaoa was engaged in a drinking session with the deceased Nestor De Loyola
together with several other. Ruben was heard arguing with Nestor and that Ruben mauled and
kicked the deceased with the help of their drinking companions just outside Ruben’s apartment.
After which, appellant dragged the deceased with the help of Julius Eligino to the apartment.
Further, to seal the case against him, Ruben borrowed Alex Villamil’s tricycle at two o’clock in
the morning on the pretext that a neighbor was about to give birth and had to be rushed to the
hospital. However, he was seen driving the tricycle alone with a sack placed in the sidecar. The
sack was described as if it contained a human body and the next morning, the tricycle was returned
with bloodstains on the floor.

Pfc. Reynaldo Angeles was dispatched where the decapitated body of a man, identified through
his voter’s identification card as Nestor De Loyola, was found in a grassy portion in Balibago,
Angeles City. The deceased also bore forty-three stab wounds in the chest as well as slight burns
all over the body. The head was found some two feet away from the corpse. Ruben, Rogelio, Rodel
Ilaoa, Julius Eligino and Edwin Tapang, were charged for the crime of murder of the deceased,
Nestor De Loyola. However, only the brother Rubern and Rogelio stoold trial since the other
accused escaped and were never apprehended.

For his defense, appellant Ruben Ilaoa claims that the sack contained buntot ng pusa, a local term
for marijuana, not a human body, which he delivered to his compadre, Nelson De Loyola whom
he could not refuse. Moreover, it was the vomit discharged by his drinking companions that was
being swept clean by his girlfriend at the entrance of their apartment, not blood as the witnesses
declared. Hence, the appellant was charged with murder.

ISSUE:
Whether or not the aggravating circumstance of abuse of superior strength, cruelty and evident
premeditation are present in this case?

RULING:
No. The aggravating circumstance of abuse of superior strength, cruelty and evident premeditation
were not sufficiently proved to be appreciated against appellants. In the case at bar, the fact that
appellant quarrelled with the deceased, then mauled and pulled him to the apartment where the
latter was last seen alive, in addition to borrowing a tricycle which was found with bloodstains

322
CRIMINAL LAW 1 CASE DIGESTS
ATTY. LYAN DAVD JUANICO
BLOCK 1-M A.Y. 2020 - 2021

when returned, sufficiently point to Ruben as the culprit responsible for the crime. The fact that
the deceased was his compadre, hence, presumably would have no motive to kill the latter, is not
enough to exculpate appellant. It is a matter of judicial knowledge that persons have been killed
or assaulted for no apparent reason at all, and that friendship or even relationship is no deterrent
to the commission of a crime. Further, Ruben himself could not explain away such testimony for
he belied the excuse that the tricycle was needed to rush a pregnant woman to the hospital, which
was the explanation he gave to Alex Villamil when he borrowed it. The Court cannot even consider
that the story about the blood on the tricycle was simply concocted by Alex Villamil to incriminate
Ruben because the latter was his friend, as Ruben himself has admitted. In fact, he could think of
no reason for Alex Villamil to testify falsely against him. Despite the foregoing, however, we hold
appellant liable only for homicide, not murder.

323
CRIMINAL LAW 1 CASE DIGESTS
ATTY. LYAN DAVD JUANICO
BLOCK 1-M A.Y. 2020 - 2021

19. People v. Bibat


G.R. No. 124319, May 13, 1998
PURISIMA, J.

FACTS:
Robles, a prosecution witness, testified that the accused frequently went to his place because his
neighbor was the president of an organization or fraternity the accused and his companions were
members of. Before the incident occurred, the president of the said organization or fraternity told
Robles that a rumble transpired in school whereby someone died. The accused and his companions
were planning to take revenge against the victim. Thereafter, another witness saw that while she
was at the funeraria, she saw a person talking to the accused and overheard that the person told the
accused, “O pare, anduon na. Puntahan mo na. Siguraduhin mo lang na itumba mo na.” to which
the accused answered and demonstrated how he would execute the felonious act. The latter witness
then entered an alley along with the accused. While the victim was going out of the gate, the
accused hurriedly went to the victim, stabbed the latter twice on his chest, and immediately ran
away. The victim shouted for help. Upon hearing this, the accused went back and struck the victim
again. The victim died upon arrival at the hospital. With these, the trial court found that the accused
was guilty of murder and was sentenced to a penalty of reclusion perpetua. The accused averred
that the trial court erred to appreciate the aggravating circumstance of evident premeditation
because Robles only testified that the accused and his companions met and that some guns and
“tusok” were hidden in the witness’ house. This implies that the time when the accused conceived
to commit the crime was not proven. Hence, the accused argued that he could only be convicted
of homicide, and not of murder.

ISSUE:
Whether or not alll the requisites of evident premeditation present in this case so as to warrant the
qualification of the conviction of the accused to murder?

RULING:
Yes, all the requisites of evident premeditation were present that warranted the conviction of the
accused of murder, and not of homicide. The following must be proven to prove the presence of
evident premeditation in the commission of the felonious act: (1) the time when the offender
determined (conceived) to commit the crime; (2) an act manifestly indicating that the culprit has
clung to his determination; and (3) a sufficient lapse of time between the determination and
execution to allow him to reflect upon the consequences of his act.

In this case, the conception of the plan to kill the victim was shown when, at 11:30 in the morning
of the day the victim was killed, two witnesses saw the accused with his companions at the

324
CRIMINAL LAW 1 CASE DIGESTS
ATTY. LYAN DAVD JUANICO
BLOCK 1-M A.Y. 2020 - 2021

funeraria and one of the witnesses overheard that the accused and his companions were planning
to kill someone. Around 1:30 in the afternoon of the same day, the accused was seen by one of the
aforesaid witnesses running towards the victim, taking a pointed tool from a notebook, and
stabbing the victim on the chest. This circumstance manifestly indicated that the accused had clung
to his determination to kill the victim. Between these two circumstances that happened in the same
day, there was a sufficient time lapse of time for the accused to reflect upon the consequences of
his acts. Therefore, evident premeditation was present which qualified the conviction of the
accused to murder.

325
CRIMINAL LAW 1 CASE DIGESTS
ATTY. LYAN DAVD JUANICO
BLOCK 1-M A.Y. 2020 - 2021

20. People v. Empacis


G.R. No. 95756, May 14, 1993
NARVASA, C.J.:

FACTS:
Fidel Saromines and his wife, Camila, were about to close their small store, located in their house
at Kanguha, Dumanjug, Cebu, two men came and asked to buy some sardines and rice. They were
Romualdo (or Maldo) Langomez and Crisologo Empacis. Camila served them and they proceeded
to make a meal of the rice and sardines. After they finished eating, Romualdo told Fidel to sell him
cigarettes. As Fidel was handing over the cigarettes, Romualdo announced a "hold-up" and
commanded Fidel to give up his money. A struggle followed in the, course of which Romualdo
stabbed Fidel about three times. Crisologo joined in and with his own knife also stabbed Fidel.
From his little sister's room, Fidel's thirteen-year old son, Peter, saw his father fighting for his life
with Romualdo and Crisologo Empacis. Heeding his father's cry for help, Peter took hold of a
"pinuti" (a long bolo), and rushed to his father's defense. He struck out at Crisologo and inflicted
two wounds on him, one at the right shoulder, and the other, in the neck, Romualdo and Crisologo
jumped out of the house and fled, with the sound of Peter's defiant shout trailing them, "Come
back, if you are brave!" Soon later, Fidel Saromines expired. The post-mortem examination
disclosed four (4) stab wounds on the deceased, all in the upper back. Two of these, which
penetrated the lungs and heart, were fatal. After trial, the trial court convicted Crisologo Empacis
alone and meted out on him death penalty considering the attendance of the four generic
aggravating circumstances of dwelling, nighttime, craft or fraud and superior strength. Hence, this
appeal.

ISSUE:
Whether or not the aggravating circumstances of dwelling, nighttime, craft or fraud and superior
strength correctly appreciated?

RULING:
Yes, the four generic aggravating circumstances of dwelling, nighttime, craft or fraud and superior
strength correctly appreciated by the trial court in convicting Empacis of the crime of robbery with
homicide. The aggravating circumstance of craft or fraud was properly appreciated against
Empacis. In the case at bar, he and Romualdo pretended to be bona fide customers of the victim's
store and on this pretext entered the latter's store and later, into another part of his dwelling.

Nighttime was likewise properly appreciated as an aggravating circumstance against the accused.
To be sure, nighttime is not per se aggravating. It must be shown that nocturnity was deliberately
and purposely sought to facilitate, or that it actually facilitated, the commission of the crime. In
326
CRIMINAL LAW 1 CASE DIGESTS
ATTY. LYAN DAVD JUANICO
BLOCK 1-M A.Y. 2020 - 2021

the case at bar, the lateness of the hour no doubt precluded the presence of other customers who
could have deterred the felons, or come to the aid of the victim. All things considered, there is
adequate showing that nocturnity was deliberately sought by the robbers and did in reality facilitate
the perpetration of the felony.

For the aggravating circumstance of superior strength to be deemed present in a case, it does not
suffice to prove superiority in number on the part of the malefactors; it must appear that they
purposely employed excessive force, force out of proportion to the means of defense available to
the person attacked. In this case, the evidence shows that Empacis helped his co-accused by also
stabbing the victim; he and his companion took advantage of their combined strength and their
bladed weapons to overcome their unarmed victim and assure the success of their felonious design
to make off with his money.

Lastly, dwelling was correctly appreciated by the trial court because pretended to be bona fide
customers of the victim's store and on this pretext entered the latter's store and later, into another
part of his dwelling. Taken all together, the trial court correctly appreciated the four generic
aggravating circumstances of dwelling, nighttime, craft or fraud and superior strength in
convicting Empacis of the crime of robbery with homicide under Article 294 (1), in relation to
Article 296, of the Revised Penal Code.

327
CRIMINAL LAW 1 CASE DIGESTS
ATTY. LYAN DAVD JUANICO
BLOCK 1-M A.Y. 2020 - 2021

21. People vs. Bigcas


G.R. No. 94534, 2 July 1992
REGALADO, J.:

FACTS:
In an information dated October 13, 1988, herein appellants Quiliano Butron and Rodrigo Bigcas
were charged with murder. The aggravating circumstances alleged therein were abuse of superior
strength, treachery, and nighttime.
Appellants pleaded not guilty when arraigned and, thereafter, trial on the merits ensured, wherein
the prosecution and the defense presented different versions of the circumstances which gave rise
to the alleged killing.
For its part, the prosecution presented two witnesses: Rosito Doydoy and Jesus Calape. According
to the two witnesses, they saw two people ganging up on someone, herein victim Ambrocio
Palapar. According to them, Butron hit Palapar twice with a piece of wood while Bigcas stabbed
him several times.
The defense, on the other hand, interposed the plea of self-defense, claiming that it was Palapar
who attacked them first. Butron claimed that he was suddenly stabbed by Palapar with a bolo on
the stomach, and that when he backtracked, he was given three stab thrusts which he all parried.
He was then able to take hold of the victim’s weapon and repeatedly stabbed him until he fell.
ISSUE:
Whether or not the aggravating circumstances alleged in the information shall be applied
RULING:
First and foremost, the Supreme Court gave credence to the testimonies of the prosecution
witnesses and did not give credence to the claim of self-defense, as it was not established by clear
and convincing evidence.
The aggravating circumstances alleged in this case were: treachery, nighttime, and abuse of
superior strength.
The Supreme Court ruled that treachery was absent in this case since there was no evidence that
the accused deliberately employed means, methods, or forms considered in law as treacherous. It
also held that the aggravating circumstance of nighttime was likewise nonattendant since during
the incident, the moon was shining brightly. Moreover, nor did it facilitate the commission of the
crime nor was it purposely sought by the appellants to afford impunity. Lastly, abuse of superior
strength was also not taken into consideration due to the flimsy evidence of the prosecution on

328
CRIMINAL LAW 1 CASE DIGESTS
ATTY. LYAN DAVD JUANICO
BLOCK 1-M A.Y. 2020 - 2021

what transpired before the commotion. Clearly, the victim waited for appellants to catch up with
him. It cannot therefore be said that appellants could have anticipated such an unexpected
contingency and had accordingly conceived of taking advantage of their combined strength and
weapons. For abuse of superior strength to be appreciated, it is not sufficient that there be
superiority in number or strength; it is necessary that the accused must have cooperated and
intended to use or secure advantage from such superior strength.

329
CRIMINAL LAW 1 CASE DIGESTS
ATTY. LYAN DAVD JUANICO
BLOCK 1-M A.Y. 2020 - 2021

22. People vs. Sangalang


G.R. No. L-32914, 30 August 1974
AQUINO, J.:

FACTS:
This is a murder case concerning the killing of Ricardo Cortez.
The victim was on top of a coconut tree gathering tuba when suddenly, he was struck with a volley
of shots. His wife, Fiora, heard the shots and went outside. She saw five men firing at her husband
from a distance of about twenty-five meters. When he fell to the ground, those 5 men went near
him until they were only about five meters apart. Fiora then approached the five men and shouted
at them, “Bakit niyo pinagbabaril ang asawa ko?” after which she herself was fired upon by the
assailants. Luckily, she was able to retreat to the nearby hut for cover.
Fiora was able to positively identify the five malefactors, one of which was Laureano Sangalang.
Only Sangalang was apprehended by the Police.
ISSUE:
Whether or not treachery attended the commission of the crime
RULING:
Yes, the victim was shot while he was gathering tuba on top of a coconut tree. He was unarmed
and defenseless. He did not give any immediate provocation. The deliberate, surprise attack
shows that Sangalang and his companions employed a mode of execution which insured the killing
without any risk to them arising from any defense which the victim could have made. The
qualifying circumstance of treachery (alevosia), which was alleged in the information, was duly
established. Hence, the killing can be categorized as murder. Treachery absorbs the aggravating
circumstance of band. Evident premeditation, which was alleged in the information, was not
proven.

330
CRIMINAL LAW 1 CASE DIGESTS
ATTY. LYAN DAVD JUANICO
BLOCK 1-M A.Y. 2020 - 2021

23. People vs. San Pedro


G.R. No. L-44274, 22 January 1980
PER CURIAM:

FACTS:
From the evidence, it can be culled that the victim, Felimon Rivera, was a jeepney driver.
According to the statement made by herein defendant-appellant Artemio Banasihan, in the
afternoon of June 2, 1970, he and Luisito San Pedro approached Rivera. On the pretext of hiring
Rivera’s jeep to haul coconuts, they proceeded to Puypuy in Bay, Laguna, where they were joined
by Salvador Litan and Rodrigo Esguerra, who was then carrying a water pipe wrapped in paper.
Upon reaching a river between the barrios of Mainit and Puypuy, San Pedro ordered Rivera to
stop. Thereafter, at Esguerra’s signal, Litan hit Rivera at the nape with the water pipe. Rivera
jumped out of the jeep but was chased by San Pedro and Litan who stabbed him at the back several
times with a dagger. Esguerra then drove the jeep and the group proceeded to Makati, Rizal, He
then joined Nelson Piso and Antonio Borja. The jeep was brought to Cavite City where it was sold
for P2,000.00. Four days later, Piso went to Los Baños and gave San Pedro, Litan and Banasihan
P50.00 each, with the promise that the balance would be given later. However, the promised
balance was not given them.
ISSUES:
Whether or not the aggravating circumstance of craft is absorbed by treachery
Whether the resulting single aggravating circumstance of treachery should be offset by the
mitigating circumstance of lack of instruction
RULING:
As to the first issue involved, the Court held that in this case, craft cannot be absorbed by treachery.
It ruled that while in a number of cases, craft had been declared so absorbed, the same is not true
in this case because of the difference in factual circumstances between such cases and this one. It
expounded that the craft herein employed was not actually directed towards the facilitation of the
killing, but rather, towards the taking of the jeep in the robbery scheme. On the other hand, from
the definition of treachery, it is manifest that the element of defense against bodily injury makes
treachery proper for consideration only in crimes against person as so explicitly provided by the
Revised Penal Code.
As to the second issue, the Court declared that with the presence of two aggravating circumstances,
craft and treachery, it would make no difference even if the mitigating circumstance of lack of
instruction were appreciated in appellant's favor which is even doubtful from the fact alone, as was
allegedly proven by the testimony of appellant that he cannot read and write but can only sign his
331
CRIMINAL LAW 1 CASE DIGESTS
ATTY. LYAN DAVD JUANICO
BLOCK 1-M A.Y. 2020 - 2021

name. Additionally, lack of instruction is not applicable to the crimes of theft and robbery, much
less to the crime of homicide. The reason is that robbery and killing are, by their nature, wrongful
acts, and are manifestly so to the enlightened, equally as to the ignorant.

332
CRIMINAL LAW 1 CASE DIGESTS
ATTY. LYAN DAVD JUANICO
BLOCK 1-M A.Y. 2020 - 2021

24. People vs. Castillo


G.R. No. 120282, 20 April 1998
DIZON, J.:

FACTS:
This is an appeal from the decision of the trial court on the murder case filed against Roberto
Castillo Mones for the killing of Antonio Dometita.
The witness presented by the prosecution, who was the floor manager of the Cola Pubhouse along
EDSA, stated that right after when Antonio “Tony” Dometita came out of the pubhouse, appellant
Roberto Castillo suddenly appeared and, without warning, stabbed Tony with a fan knife on his
left chest. As Tony pleaded for help, appellant stabbed him once more, hitting him on the left
hand. When Tony ran away, the appellant pursued him. The witness came to know later that Tony
had died; his body was found outside the fence of the Iglesia ni Cristo Compound, EDSA, Quezon
City.
Dr. Bienvenido Munoz, the medico-legal officer who autopsied Tony's cadaver, testified that the
proximate cause of Tony's death was the stab wound on his left chest. Tony also suffered several
incised wounds and abrasions, indicating that he tried to resist the attack.
On the other hand, the defense presented its own witness, who was a tricycle driver. According to
such witness, he saw two people ganging up on a third. The defense surmises that these two people
are the culprits and not Castillo.
ISSUE:
Whether or not the aggravating circumstances of evident premeditation, abuse of superior strength,
and treachery attended the commission of the crime
RULING:
The prosecution was unable to prove the aggravating circumstance of evident premeditation. For
this circumstance to be appreciated, there must be proof, as clear as the evidence of the crime itself,
of the following elements: 1) the time when the offender determined to commit the crime, 2) an
act manifestly indicating that he clung to his determination, and 3) a sufficient lapse of time
between determination and execution to allow himself time to reflect upon the consequences of
his act. These requisites were never established by the prosecution.
However, the killing was not qualified by abuse of superior strength. "To properly appreciate the
aggravating circumstance of abuse of superior strength, the prosecution must prove that the
assailant purposely used excessive force out of proportion to the means of defense available to the

333
CRIMINAL LAW 1 CASE DIGESTS
ATTY. LYAN DAVD JUANICO
BLOCK 1-M A.Y. 2020 - 2021

person attacked." The prosecution did not demonstrate that there was a marked difference in the
stature and build of the victim and the appellant which would have precluded an appropriate
defense from the victim. Not even the use of a bladed instrument would constitute abuse of superior
strength if the victim was adequately prepared to face an attack, or if he was obviously physically
superior to the assailant.
Nonetheless, it was qualified by treachery. "Treachery is committed when two conditions concur,
namely, that the means, methods, and forms of execution employed gave the person attacked no
opportunity to defend himself or to retaliate; and that such means, methods, and forms of execution
were deliberately and consciously adopted by the accused without danger to his person." These
requisites were evidently present in this case when the accused appeared from nowhere and swiftly
and unexpectedly stabbed the victim just he was bidding goodbye to his friend, Witness Velasco.
Said action rendered it difficult for the victim to defend himself. The presence of "defense wounds"
does not negate treachery because, as testified to by Velasco, the first stab, fatal as it was, was
inflicted on the chest. The incised wounds in the arms were inflicted when the victim was already
rendered defenseless.

334
CRIMINAL LAW 1 CASE DIGESTS
ATTY. LYAN DAVD JUANICO
BLOCK 1-M A.Y. 2020 - 2021

25. People vs. Arizobal


G.R. No. 135051-52, 14 December 2000
PER CURIAM:

FACTS:
On 12 August 1994 two (2) separate Informations were filed before the Regional Trial Court of
Cataingnan, Masbate, charging Clarito Arizobal, Erly Lignes, Rogelio Gemino and two (2) John
Does with Robbery in Band with Homicide for robbing and slaying Laurencio Gimenez and his
son Jimmy Gimenez.
This is the factual background:
The prosecution presented, among others, Clementina Gimenez, wife of victim Laurencio
Gimenez. She testified that on 24 March 1994 she together with her husband Laurencio Gimenez
and a grandchild were sound asleep in their house in Tuybo, Cataingan, Masbate. At around 9:30
in the evening, Laurencio roused her from sleep and told her to open the door because there were
persons outside the house. Since it was pitch-dark she lit a kerosene lamp and stood up to open the
door. She was suddenly confronted by three (3) armed men pointing their guns at her. She
recognized two (2) of them as Clarito Arizobal and Erly Lignes but failed to recognize the third
person who was wearing a maskara. She readily identified Clarito because she used to pass by his
house in San Rafael while Erly was also a familiar face as he was a regular habitue of the flea
market.
According to Clementina, Clarito asked her husband, "Tay, where is your gun." But she promptly
interjected, "We have no gun, not even a bolo. If you want, you can look around for it." While the
man in maskara stood guard at the door, Clarito and Lignes barged into the master's bedroom and
forcibly opened the aparador. The terrified couple could not raise a finger in protest but had to
leave their fate to the whims of their assailants. The intruders ransacked their cabinet and scattered
everything on the floor until they found ₱8,000.00 among sheets of paper. Before leaving with
their loot they ordered Laurencio to go with them to Jimmy's house because "we have something
to talk about." Against his will, Laurencio went with them. Clementina recalled that shortly after
the group left she heard a volley of shots. Her grandchild, as if sensing what befell her grandfather,
could only mutter in fear, "Lolo is already dead!"
Erlinda Gimenez, wife of Jimmy Gimenez, narrated that on 24 March 1994, after she and her son
had taken supper, her husband Jimmy with one Francisco Gimenez arrived. Jimmy informed
Erlinda that they had already bought a carabao. After he handed her the certificate of large cattle,
and while he was in the process of skinning a chicken for their supper, three (3) men suddenly
appeared and ordered them to lie face down. One of them pushed her to the ground while the others

335
CRIMINAL LAW 1 CASE DIGESTS
ATTY. LYAN DAVD JUANICO
BLOCK 1-M A.Y. 2020 - 2021

tied Francisco and Jimmy as they whipped the latter with an armalite rifle. She noticed one of them
wearing a mask, another a hat, and still another, a bonnet.
Realizing the utter helplessness of their victims, the robbers took the liberty of consuming the food
and cigarettes Erlinda was selling in her sari-sari store. Finding no softdrinks to complete their
snack, two (2) of the intruders ordered Erlinda to buy coke for them at the neighboring store. But
they warned her not to make any noise, much less alert the vendor. When they returned to the
house of Jimmy, the robbers proceeded to ransack the household in search for valuables. They
took around ₱1,000.00 from her sari-sari store and told them to produce ₱100,000.00 in exchange
for Jimmy's life. Since the couple could not produce such a big amount in so short a time, Erlinda
offered to give their certificate of large cattle. The culprits however would not fall for the ruse and
threw the document back to her. Three (3) masked men then dragged Jimmy outside the house and
together with Laurencio brought them some fifty (50) meters away while leaving behind Clarito
Arizobal and Erly Lignes to guard Francisco and Erlinda's son. Moments later she heard a burst of
gunfire which reverberated through the stillness of the night.
When the masked men returned to Jimmy's house, one of them informed Erlinda that her husband
and father-in-law had been killed for trying to escape. Upon hearing this, Erlinda, as if the heavens
had fallen on her, slowly lost consciousness.
ISSUE:
Whether or not the aggravating circumstances of dwelling, treachery, band, and nighttime attended
the commission of the crime
RULING:
The aggravating circumstance of dwelling is present in this case. Generally, dwelling is considered
inherent in the crimes which can only be committed in the abode of the victim, such as trespass to
dwelling and robbery in an inhabited place. However, in robbery with homicide the authors thereof
can commit the heinous crime without transgressing the sanctity of the victim's domicile. In the
case at bar, the robbers demonstrated an impudent disregard of the inviolability of the victims'
abode when they forced their way in, looted their houses, intimidated and coerced their inhabitants
into submission, disabled Laurencio and Jimmy by tying their hands before dragging them out of
the house to be killed.
However, treachery cannot be appreciated. The accused stand charged with, tried and convicted of
robbery with homicide. This special complex crime is primarily classified in this jurisdiction as a
crime against property, and not against persons, homicide being merely an incident of robbery
with the latter being the main purpose and object of the criminals. As such, treachery cannot be
validly appreciated as an aggravating circumstance under Art. 14 of The Revised Penal Code.

336
CRIMINAL LAW 1 CASE DIGESTS
ATTY. LYAN DAVD JUANICO
BLOCK 1-M A.Y. 2020 - 2021

While it appears that at least five (5) malefactors took part in the commission of the crime, the
evidence on record does not disclose that "more than three" persons were armed, and robbery in
"band" means "more than three armed malefactors united in the commission of robbery." Nowhere
in the records can we gather that more than three (3) of the robbers were armed. Hence, "band"
cannot be aggravating where no proof is adduced that at least four (4) of the five (5) perpetrators
involved in this case were armed.
Likewise, the aggravating circumstance of nighttime did not attend the commission of the crime.
he fact that the offense was committed at 9:30 in the evening does not suffice to
sustain nocturnidad for, by itself, nighttime is not an aggravating circumstance. To be properly so
considered, it must be shown that nocturnidad was deliberately and intentionally sought by
accused-appellants to help them realize their evil intentions. Nowhere can we infer from the
records that the malefactors sought the cover of darkness to facilitate the accomplishment of their
devious design. On the contrary, the locus criminis was well lighted and nighttime was merely an
incidental element to the whole drama. First. The houses of the victims were adequately lighted
by kerosene lamps when the robbers entered and went about their looting spree. Second. The
robbers, particularly referring to accused-appellant and his co-accused, lingered in the locus
criminis and even conversed with their intended victims for an appreciable period of time inside
the well-lit houses. As Erlinda Gimenez testified, the place where the victims were gunned down
was adequately illuminated by the moonlight, although for undisclosed reasons she did not see the
actual shooting. All these taken together belie the assumption that the culprits took advantage of
the intrinsic impunity afforded by the cover of darkness and made the same as an ally to accomplish
their nefarious plan. Nocturnity lures those who crave for blood to yield to their baser impulses
with the false courage borne out of the belief that their identity would not be brought in the open.
The Court did not discern any such intention in this case.

337
CRIMINAL LAW 1 CASE DIGESTS
ATTY. LYAN DAVD JUANICO
BLOCK 1-M A.Y. 2020 - 2021

26. People vs. Escote


G.R. No. 140756, 4 April 2003
DAVIDE, JR., C.J.:

FACTS:
This is a case of robbery with homicide.
Juan Escote and Victor Ochovillos boarded a bus. Inside the bus with them was a police officer.
When the bus was travelling along the highway in Plaridel, Bulacan, the two malefactors suddenly
stood up, whipped out their guns and announced holdup. Thereafter, they accosted the passengers
and divested them of their money and valuables. They then proceeded to where the policeman was
sitting, took his gun, and shot him, despite his pleas for mercy.
ISSUE:
Whether or not treachery is a generic aggravating circumstance in robbery with homicide
RULING:
Going by the letter of the law, treachery is applicable only to crimes against persons as enumerated
in Title Eight, Chapters One and Two, Book II of the Revised Penal Code. However, the Supreme
Court of Spain has consistently applied treachery to robbery with homicide, classified as a crime
against property. Citing decisions of the Supreme Court of Spain, Cuello Calon, a noted
commentator of the Spanish Penal Code says that despite the strict and express reference of the
penal code to treachery being applicable to persons, treachery also applies to other crimes such as
robbery with homicide.
Thus, treachery is a generic aggravating circumstance to robbery with homicide although said
crime is classified as a crime against property and a single and indivisible crime. Treachery is not
a qualifying circumstance because as ruled by the Supreme Court of Spain in its decision dated
September 11, 1878, the word homicide is used in its broadest and most generic sense.
Article 62, paragraph 1 of the Revised Penal Code provides that in diminishing or increasing the
penalty for a crime, aggravating circumstances shall be taken into account. However, aggravating
circumstances which in themselves constitute a crime specially punishable by law or which are
included by the law in defining a crime and prescribing a penalty therefor shall not be taken into
account for the purpose of increasing the penalty. Under paragraph 2 of the law, the same rule shall
apply with respect to any aggravating circumstances inherent in the crime to such a degree that it
must of necessity accompany the commission thereof.

338
CRIMINAL LAW 1 CASE DIGESTS
ATTY. LYAN DAVD JUANICO
BLOCK 1-M A.Y. 2020 - 2021

Treachery is not an element of robbery with homicide. Neither does it constitute a crime specially
punishable by law nor is it included by the law in defining the crime of robbery with homicide and
prescribing the penalty therefor. Treachery is likewise not inherent in the crime of robbery with
homicide. Hence, treachery should be considered as a generic aggravating circumstance in robbery
with homicide for the imposition of the proper penalty for the crime.
In fine, in the application of treachery as a generic aggravating circumstance to robbery with
homicide, the law looks at the constituent crime of homicide which is a crime against persons and
not at the constituent crime of robbery which is a crime against property. Treachery is applied to
the constituent crime of homicide and not to the constituent crime of robbery of the special complex
crime of robbery with homicide.
The crime of robbery with homicide does not lose its classification as a crime against property or
as a special complex and single and indivisible crime simply because treachery is appreciated as
a generic aggravating circumstance. Treachery merely increases the penalty for the crime
conformably with Article 63 of the Revised Penal Code absent any generic mitigating
circumstance.
In sum then, treachery is a generic aggravating circumstance in robbery with homicide when the
victim of homicide is killed by treachery.
However, even though treachery was attendant in the killing, the same cannot be appreciated
against the accused because it was not alleged in the information.

339
CRIMINAL LAW 1 CASE DIGESTS
ATTY. LYAN DAVD JUANICO
BLOCK 1-M A.Y. 2020 - 2021

27. People vs. Villonez


G.R. Nos. 122976-77, 16 November 1998
DAVIDE, JR., J.:

FACTS:
The witnesses for the prosecution were Edgar Jimenez and Dr. Ronaldo Mendez, a Medico-Legal
Officer of the National Bureau of Investigation; and the witnesses for the defense were the
accused-appellants, as well as Arthur Aquino and Conrado Gungon.
Edgar Jimenez testified that on 3 May 1994, at around 9:00 p.m., while he was resting inside his
store at Hulo, Malabon, Metro Manila, a certain Tonton informed him that his close friend
GERARDO LONGASA had a fistfight with one Rudy, alias Dede, at Liwayway Street, Baritan,
Malabon. Edgar proceeded to the area to mediate, since LONGASA and Rudy were both his
friends. Edgar passed through Javier II Street in going to Liwayway Street. At Javier II Street, a
group of seven armed men, including accused-appellants, attacked Edgar. RUEL hit Edgar on his
forehead and back with a bottle. Edgar was able to escape from his attackers. While fleeing, he ran
past LONGASA, who seemed drunk. When Edgar called LONGASA, the attackers were already
upon LONGASA.
While he was about eight arms length away from LONGASA, Edgar saw EMERLITO hit
LONGASA with a 2 x 2 inches piece of wood. Simultaneously, REGANDO and RUEL struck
LONGASA with bottles. Rudy Santos and Eddie Santos then stabbed LONGASA seven and eight
times, respectively, even as two other persons named Rey and Budda held LONGASAs arms.
LONGASA fell to the ground. Edgar saw all these because the scene of the incident was
illuminated by a big fluorescent lamp located about three arms length away. Edgar rushed to
LONGASAs house and reported the incident to the latter’s parents.
Dr. Ronaldo Mendez conducted an autopsy on LONGASAs corpse and found a number of
abrasions, two contusions, two lacerated wounds, and six stab wounds. He explained that abrasions
were caused by hard, rough surface, possibly cement or a piece of wood. The contusions and
lacerations were caused by a blunt object, which could have been a piece of wood, a bottle, a pipe,
or any other hard object. The incise wounds or stab wounds were caused by a sharp-bladed or
sharp-edged instrument. Of the six stab wounds suffered by LONGASA, stab wounds numbered
1 and 3 on LONGASAs chest caused the latter’s death.
ISSUE:
Whether or not treachery attended the killing

340
CRIMINAL LAW 1 CASE DIGESTS
ATTY. LYAN DAVD JUANICO
BLOCK 1-M A.Y. 2020 - 2021

RULING:
The Supreme Court ruled, in contrast with the decision of the trial court, that there was treachery
in this case. It stated that just because the victim had engaged in a fight previous to the killing and
was thus forewarned of an attack against him, does not automatically mean that there was no
treachery. Treachery may still be appreciated even when the victim was forewarned of danger to
his person. What is decisive is that the execution of the attack made it impossible for the victim to
defend himself or to retaliate. The overwhelming number of the accused, their use of weapons
against the unarmed victim, and the fact that the victim’s hands were held behind him preclude the
possibility of any defense by the victim.
The other qualifying circumstance of abuse of superior strength, which the trial court appreciated,
will no longer be taken against accused-appellants, for it is absorbed in treachery.

341
CRIMINAL LAW 1 CASE DIGESTS
ATTY. LYAN DAVD JUANICO
BLOCK 1-M A.Y. 2020 - 2021

28. People of the Philippines v. Nicolas Guzman y Bocbosila


G.R. No. 169246. January 26, 2007
Chico-Nazario, J

FACTS:
Ronald is a jeepney driver and resident of Barangay Commonwealth, Quezon City. He testified
that on 25 November 1999, at about 9:00 in the evening, he stopped by and ate at a carinderia
located at the corner of Sto. Nino Street and Mactan Street, Brgy. Commonwealth, Quezon City.
After eating, he sat on a bench just beside the carinderia and rested. He noticed appellant and two
other persons having a drinking spree in a nearby grocery store. He also saw Michael walking
towards the direction of the same grocery store. When Michael was passing in front of the grocery
store, appellant and his two companions suddenly approached and surrounded Michael. Appellant
positioned himself at the back of Michael while his two companions stood in front of Michael.
Suddenly, they grabbed the shoulders of Michael and overpowered the latter. One of appellant's
companions, whom he described as a male with long hair, drew out a knife and repeatedly stabbed
Michael at the stomach. Afterwards, the appellant's other companion, whom he described as a male
with Hat top hair, took the knife from the companion with long hair, and also stabbed Michael at
the stomach. Later, appellant went in front of Michael, took the knife from the companion with
Hat top hair, and likewise stabbed Michael at the stomach. Appellant also kicked Michael when
the latter was already lying on the ground. He witnessed this stabbing incident at a distance of five
arms' length.
On 12 November 2001, the RTC rendered its Decision convicting appellant of murder. It sustained
the "clear, direct and positive" testimony of the prosecution witnesses who all declared that they
saw appellant stab Michael. It found no ill-motive on the part of the prosecution witnesses in
testifying against appellant. It also ruled that there was treachery in the killing of Michael since
the latter was unarmed, unsuspecting and very young at the time of the attack.
Appellant filed a Notice of Appeal on 26 November 2001. 19 On 28 February 2005, the Court
Appeals promulgated its Decision affirming with modification the RTC Decision.
Appellant contends that even if he were held liable for the death of Michael, there was no treachery
which will qualify the killing as murder. According to him, there is no evidence to show that
appellant and his two companions had deliberately and consciously adopted their mode of attack
to ensure its execution without risk to themselves. The stabbing incident occurred in a place that
was properly lighted. There were many people in the area then walking in different directions. He
claims that if he and his two companions wanted to ensure that no risk would come to them, then
they could have chosen another time and place to attack Michael.

342
CRIMINAL LAW 1 CASE DIGESTS
ATTY. LYAN DAVD JUANICO
BLOCK 1-M A.Y. 2020 - 2021

ISSUE:
Did the qualifying circumstance of treachery attend the killing of the victim?
RULING:
As can be gleaned from Article 14 of the Revised Penal Code, two essential elements/conditions
are required in order that treachery may be appreciated: (1) The employment of means, methods
or manner of execution that would ensure the offender's safety from any retaliatory act on
the part of the offended party, who has, thus no opportunity for self-defense or retaliation;
(2) deliberate or conscious choice of means, methods or manner of execution. Further, it must
always be alleged in the information and proved in trial in order that it may be validly considered.
In the instant case, treachery was alleged in the Information against appellant. Moreover, all the
essential elements/conditions of treachery were established and proven during the trial.
As viewed from the foregoing, the suddenness and unexpectedness of the attack of appellant and
his two companions rendered Michael defenseless, vulnerable and without means of escape. It
appears that Michael was unarmed and alone at the time of the attack. Further, he was merely
seventeen years of age then. In such a helpless situation, it was absolutely impossible for Michael
to escape or to defend himself against the assault of appellant and his two companions. Being
young and weak, Michael is certainly no match against adult persons like appellant and his two
companions. Michael was also outnumbered since he had three assailants, and, was unarmed when
he was stabbed to death. Appellant and his two companions took advantage of their size, number,
and weapon in killing Michael. They also deliberately adopted means and methods in exacting the
cruel death of Michael by first surrounding him, then grabbing his shoulders and overpowering
him. Afterwards, each of them repeatedly stabbed Michael with a knife at the stomach until the
latter fell lifeless to the ground. The stab wounds sustained by Michael proved to be fatal as they
severely damaged the latter's large intestine.
The fact that the place where the incident occurred was lighted and many people were walking
then in different directions does not negate treachery. It should be made clear that the essence of
treachery is the sudden and unexpected attack on an unsuspecting victim without the slightest
provocation on his part. This is even more true if the assailant is an adult and the victim is a minor.
Minor children, who by reason of their tender years, cannot be expected to put up a defense. Thus,
when an adult person illegally attacks a minor, treachery exists. As we earlier found, Michael was
peacefully walking and not provoking anyone to a fight when he was stabbed to death by appellant
and his two companions. Further, Michael was a minor at the time of his death while appellant and
his two companions were adult persons.

343
CRIMINAL LAW 1 CASE DIGESTS
ATTY. LYAN DAVD JUANICO
BLOCK 1-M A.Y. 2020 - 2021

29. People v. Torriefel


CA-G.R. No. 659-R. 29 November 1947
HERMOSISIMA, JR., J.

FACTS:
December 17, 1942, 5:00 p.m. Torrefiel and Ormeo were on their way to the USSAFE
headquarters in the mountains. They passed by Eady’s residence and talked to him at the balcony
to ask for khakis. Eady had none except what he had on.
Ceferina Cordero also came to the balcony and inquired about their mission. She scolded Torrefiel
and Ormeo because all their belongings have been looted by USSAFE soldiers. Torrefiel
threatened her with slapping; brought out revolver. Eady and Cordero were charged with being
fifth columnists as they refused to give aid to them. Subsequently they were taken to the USSAFE
headquarters.
Torrefiel took charge of Eady and Ormeo took charge of Cordero. Their hands were free but were
blindfolded. Cordero called to Eady every now and then to know if he was following. After a while
Eady did not respond anymore so they stopped to wait for them. Torrefiel had taken the wrong way
so he went back to a guardhouse and left Eady there. He tried to find a way to overtake Ormeo and
Cordero but was unsuccessful. At the guardhouse, he discovers Eady had escaped. Torrefiel
followed a different route enabling him to find Ormeo and Cordero. Ormeo rushed back to the
guardhouse upon discovering that Eady had escaped; Cordero was left with Torrefiel.
As Cordero was about to urinate, Torrefiel pushed her and carried her to a log and laid her on it
and raped her. Torrefiel began to unbutton his pants and wound cogon leaves around his genitals.
It was visible to Cordero as her blindfold had fallen down a little. Pressing her neck so she would
remain silent, Torrefiel proceeded to have intercourse with her. Ormeo, taking advantage, also had
sex with her. The soldiers desisted from bringing Cordero to their headquarters and returned her
to their house. A servant informed Cordero that Eady had gone away. Upon Eady’s return, Cordero
informed him that she was abused by Torriefel.
ISSUE:
What aggravating circumstance is present in the case at bar?
RULING:
When the accused raped a woman after winding cogon grass around his genital organ, he thereby
augmented the wrong done by increasing its pain and adding ignominy thereto.
In conclusion, the aggravating circumstance of ignominy is present in the case at bar.

344
CRIMINAL LAW 1 CASE DIGESTS
ATTY. LYAN DAVD JUANICO
BLOCK 1-M A.Y. 2020 - 2021

30. The People of the Philippines v. Jaime Jose y Gomez, et al.


G.R. No. L-28232. February 6, 1971
PER CURIAM:

FACTS:
It was that at about 4:30 o'clock in the morning of June 26, 1967, Miss De la Riva, homeward
bound from the ABS Studio on Roxas Blvd., Pasay City, was driving her bantam car accompanied
by her maid Helen Calderon, who was also at the front seat. Her house was at No. 48, 12th Street,
New Manila, Quezon City. She was already near her destination when a Pontiac two-door
convertible car with four men aboard (later identified as the four appellants) came abreast of her
car and tried to bump it. She stepped on her brakes to avoid a collision, and then pressed on the
gas and swerved her car to the left, at which moment she was already in front of her house gate;
but because the driver of the other car (Basilio Pineda, Jr.) also accelerated his speed, the two cars
almost collided for the second time. This prompted Miss De la Riva, who was justifiably annoyed,
to ask: "Ano ba?" Forthwith, Pineda stopped the car which he was driving, jumped out of it and
rushed towards her.
Pineda opened the door of Miss De la Riva's car and grabbed the lady's left arm. The girl held on
tenaciously to her car's steering wheel and, together with her maid, started to scream. Her strength,
however, proved no match to that of Pineda, who succeeded in pulling her out of her car. Seeing
her mistress' predicament, the maid jumped out of the car and took hold of Miss De la Riva's right
arm in an effort to free her from Pineda's grip. The latter, however, was able to drag Miss De la
Riva toward the Pontiac convertible car, whose motor was all the while running.
When Miss De la Riva, who was being pulled by Pineda, was very near the Pontiac car, the three
men inside started to assist their friend: one of them held her by the neck, while the two others
held her arms and legs. All three were now pulling Miss De la Riva inside the car. Before she was
completely in, appellant Pineda jumped unto the driver's seat and sped away in the direction of
Broadway Street. The maid was left behind.
At the Swanky Hotel, Miss De la Riva was blindfolded and was led out of the car to one of the
rooms on the second floor of the hotel. Inside the room Miss De la Riva was made to sit on bed.
Her blindfold was removed. She saw Pineda and Aquino standing in front of her, and Jose and
Cañal sitting beside her, all of them smiling meaningfully. Pineda told the complainant:
"Magburlesque ka para sa amin." The other three expressed their approval and ordered Miss De la
Riva to disrobe. The complainant ignored the command. One of the appellants suggested putting
off the light so that the complainant would not be ashamed. The idea, however, was rejected by
the others, who said that it would be more pleasurable for them if the light was on. Miss De la Riva
was told to remove her stockings, in order, according to them, to make the proceedings more
exciting. Reluctantly, she did as directed, but so slowly did she proceed with the assigned task that
345
CRIMINAL LAW 1 CASE DIGESTS
ATTY. LYAN DAVD JUANICO
BLOCK 1-M A.Y. 2020 - 2021

the appellants cursed her and threatened her again with the Thompson and the acid. They started
pushing Miss De la Riva around. One of them pulled down the zipper of her dress; another
unhooked her brassiere. She held on tightly to her dress to prevent it from being pulled down, but
her efforts were in vain: her dress, together with her brassiere, fell on the floor.
The complainant was now completely naked before the four men, who were kneeling in front of
her and feasting their eyes on her private parts. This ordeal lasted for about ten minutes, during
which the complainant, in all her nakedness, was asked twice or thrice to turn around. Then Pineda
picked up her clothes and left the room with his other companions. The complainant tried to look
for a blanket with which to cover herself, but she could not find one.
Very soon, Jose reentered the room and began undressing himself. Miss De la Riva, who was
sitting on the bed trying to cover her bareness with her hands, implored him to ask his friends to
release her. Instead of answering her, he pushed her backward and pinned her down on the bed.
Miss De la Riva and Jose struggled against each other; and because the complainant was putting
up stiff resistance, Jose cursed her and hit her several times on the stomach and other parts of the
body. The complainant crossed her legs tightly, but her attacker was able to force them open. Jose
succeeded in having carnal knowledge of the complainant. He then left the room.
The other three took their turns. Aquino entered the room next. A struggle ensued between him
and Miss De la Riva, during which he hit her on different parts of the body. Like Jose, Aquino
succeeded in abusing the complainant. The girl was now in a state of shock. Aquino called the
others into the room. They poured water on her face and slapped her to revive her. Afterwards,
three or the accused left the room, leaving Pineda and the complainant. After some struggle during
which Pineda hit her, the former succeeded in forcing his carnal desire on the latter. When the
complainant went into a state of shock for the second time, the three other men went into the room,
again poured water on the complainant's face and slapped her several times. The complainant heard
them say that they had to revive her so she would know what was happening. Jose, Aquino and
Pineda then left the room. It was now appellant Cañal's turn. There was a struggle between him
and Miss De la Riva. Like the other three appellants before him, he hit the complainant on different
parts of the body and succeeded in forcing his carnal lust on her.
Mention must be made of the fact that while each of the four appellants was struggling with the
complainant, the other three were outside the room, just behind the door, threatening the
complainant with acid and telling her to give in because she could not, after all, escape, what with
their presence.
The victim was released at around 6:00 o’clock in a taxicab which was chosen by the malefactors.
Appellants were convicted of the complex crime of forcible abduction with rape.

346
CRIMINAL LAW 1 CASE DIGESTS
ATTY. LYAN DAVD JUANICO
BLOCK 1-M A.Y. 2020 - 2021

ISSUE:
What aggravating circumstances are present in the case at bar?
RULING:
The commission of said crimes was attended with the following aggravating circumstances: (a)
nighttime, appellants having purposely sought such circumstance to facilitate the commission of
these crimes; (b) abuse of superior strength, the crime having been committed by the four
appellants in conspiracy with one another; (c) ignominy, since the appellants in ordering the
complainant to exhibit to them her complete nakedness for about ten minutes, before raping her,
brought about a circumstance which tended to make the effects of the crime more humiliating; and
(d) use of a motor vehicle. With respect to appellants Jose, Aquino and Cañal, none of these
aggravating circumstances has been offset by any mitigating circumstance. Appellant Pineda
should, however, be credited with the mitigating circumstance of voluntary plea of guilty, a factor
which does not in the least affect the nature of the proper penalties to be imposed, for the reason
that there would still be three aggravating circumstances remaining. As a result, appellants should
likewise be made to suffer the extreme, penalty of death in each of these three simple crimes of
rape.

347
CRIMINAL LAW 1 CASE DIGESTS
ATTY. LYAN DAVD JUANICO
BLOCK 1-M A.Y. 2020 - 2021

31. The People of the Philippines v. Michael J. Butler


G.R. No. L-50276. January 27, 1983
Guerrero, J.

FACTS:
Accused-appellant narrated the facts in his extrajudicial confession, which stated that:
“During the evening hours of 7 August 1975, while on liberty, I went to Bob's Tailor Shop in
Olongapo City, R.P. While I was there I talked to a girl and drank some gin and beer and got drunk.
The girl's name was Victoria PEÑA. There was another girl in the tailor shop and she was making
eyes at me. I walked outside the tailor shop and she followed me and we spoke to each other. This
was sometime after 9 PM. She asked me if I wanted to go home with her and I said yes. We caught
a tricycle and went to her house. She paid the man one peso. When we got to the house another
girl let us in. After we got to the house, the girl that I was with showed me her health card, but I
couldn't read the name on it. I went upstairs and the girl that I was with showed me the bedroom
which was just to the left at the top of the stairs. I went in and sat down on the bed. She came in
and asked me for some money. She told me she was going to screw me. (By this I understood we
were going to engage in sexual intercourse). I gave her approximately 27 pesos. She left the room
and said that she was going to get some cigarettes and would be right back. She came back later
and came into the room, walked out of the room and said something to the girl in the next room.
The two of them came into the bedroom where I was and they were laughing about something.
The other girl then left and the two of us were in the bedroom alone. Both of us got undressed and
I laid down on the bed and went to sleep. I woke up sometime later and she was in bed with me.
At this point I rolled the girl over and made love to her. (By this I mean I engaged in sexual
intercourse with her from the rear). My intention was to screw her in the vagina. If I screwed her
in the rectum, I didn't intend to. After we finished, I rolled over and went back to sleep again.
Roosters started crowing and I woke up and it was starting to get daylight. The girl was already
awake. I thought that it was time for me to go back to the ship so I told her that I had to leave. I
couldn't find my watch and asked her where it was and she said that the girl in the next room had
it. I was sitting on the bed and I reached down to pull up my sock and I discovered that a five peso
note that I had in my sock was missing. I asked her about it and she said that she had gotten it. We
started arguing about my five pesos and she started saying something to me in the Filipino language
and I told her to speak English. I walked over and looked at her hard and she wanted to know what
I was looking at and I asked her why she took my money. I said 'Ah, fuck it,' and pushed her down
onto the bed. She got off the bed and smacked me and I smacked her back. She started tussling
and acting like she was going to hit me with a karate chop. I thought she was going to do something
dangerous to me so I grabbed her, and we started wrestling on the bed. She grabbed me by the
throat and I picked up a statue of Jesus Christ that was sitting on a bedside stand and I hit her in

348
CRIMINAL LAW 1 CASE DIGESTS
ATTY. LYAN DAVD JUANICO
BLOCK 1-M A.Y. 2020 - 2021

the head. She fell Fat on her face. I didn't intend to kill the girl but I was mad and wanted to hurt
her. She didn't say anything to me but she was making some kind of groaning noise. I went in the
next room and got my watch, came back in the bedroom, got dressed and left. I started walking
towards the base. I saw the lights of a vehicle coming so 1 stepped inside of a building so I wouldn't
be caught out after the curfew. As it turned out it was a Marine in a military truck, I'm not sure if
he was with the Armed Forces Police or the Shore Patrol. The Marine was white and bald headed
and wore a badge. He gave me a ride to the Armed Forces Police Station at the Main Gate, Subic
Bay, I then went from there to my ship. I was dressed in civilian clothing and I had on a pair of
burgundy trousers and a blue and white printed shirt. I left these items of clothing on the top of my
bunk located in the 2nd Division berthing area.”
Dr. Roxas later testified that anal intercourse was had with the victim after her death as indicated
by the partly opened anus and the presence of spermatozoa in it. He testified that the anus would
have automatically and completely closed had the intercourse occurred, while the victim was still
alive. He also categorically testified that the victim died of asphyxia due to suffocation when
extreme pressure was exerted on her head pushing it downward, thereby pressing her nose and
mouth against the mattress.
The trial court found accused guilty of the crime of murder qualified by abuse of superior strength,
aggravated by treachery and outraging or scoffing at the corpse of the deceased.
ISSUE:
Did the trial court erred in finding the accused guilty of the crime of murder qualified by abuse of
superior strength, with aggravating circumstances of treachery and scoffing at the corpse of the
victim?
RULING:
The Court held that there was an abuse of superior strength attending the commission of the
crime. It is not only the notorious advantage of height that the accused had over his helpless victim,
he being 6 feet tall and weighing 155 lbs. while the girl was only 4 ft. 11 inches tall, but also his
strength which he wielded in striking her with the figurine on the head and in shoving her head
and pressing her mouth and nose against the bed mattress, which pressure must have been very
strong and powerful to suffocate her to death and without risk to himself in any manner or mode
whatsoever that she may have taken to defend herself or retaliate since she was already struck and
helpless on the bed, that convinced the Court to find and rule that the crime committed is murder
with the qualifying circumstance of abuse of superior strength. The evidence on record, however,
is not sufficient to show clearly and prove distinctly that treachery attended the commission of
the crime since there was no eyewitness account of the killing.

349
CRIMINAL LAW 1 CASE DIGESTS
ATTY. LYAN DAVD JUANICO
BLOCK 1-M A.Y. 2020 - 2021

The aggravating circumstance of outraging or scoffing at the corpse of the deceased


applies against the accused since it is established that he mocked or outraged at the person or
corpse of his victim by having an anal intercourse with her after she was already dead. The fact
that the muscles of the anus did not close and the presence of spermatozoa in the anal region as
testified to by Dr. Roxas, the medico-legal officer, and confirmed to be positive in the Laboratory
Report, clearly established the coitus after death. This act of the accused in having anal intercourse
with the woman after killing her is, undoubtedly, an outrage at her corpse.

350
CRIMINAL LAW 1 CASE DIGESTS
ATTY. LYAN DAVD JUANICO
BLOCK 1-M A.Y. 2020 - 2021

32. The People of the Philippines v. Rafael Saylan


G.R. No. L-36941. June 29, 1984
Abad Santos, J.

FACTS:
Eutropia A. Agno, a married woman, and a resident of Barrio Malinao, Gingoog City, was a
classroom teacher of the Malinao Elementary School.
In the afternoon of January 23, 1971, Eutropia went to the public market in Gingoog City to buy
foodstuffs for her family and thereafter, she proceeded to the store of her mother to fetch her five-
year old daughter Nilsonita. On their way home, Eutropia and Nilsonita boarded a passenger
jeepney and while inside the vehicle she (Eutropia) noticed that the other passengers were Rudy
Gonzales, a grade I pupil of the Malinao Elementary School, the appellant, Rafael Saylan, and a
couple whom she did not know. The jeepney went only as far as Malinas citrus farm because the
road to Barrio Malinao was not passable by vehicles. It was almost 6:30 o'clock in the evening
when the jeepney arrived at the Malinas citrus farm and so all the passengers alighted and had to
walk all the way to Barrio Malinao which was about three and a half kilometers away. After
walking some distance and upon reaching a junction, the couple separated from the group and took
the road leading to their house while Eutropia's group took the opposite road. The appellant,
however, joined the group of Eutropia and when they reached the place where the road was plain,
appellant who was then walking side by side with Eutropia suddenly pulled out a dagger about
eight inches long and pointing it at the latter said, “Do not shout, Nang, I will kill you!”. At this
juncture, appellant placed his right arm around the neck of Eutropia with the dagger pointed at her
left breast, after which he dragged Eutropia at some distance. When they reached the junction of
the trail for men and a trail for carabaos, he ordered everybody to stop and told the children to stay
behind and threatened to kill them if they persisted in following them. Thereafter, appellant again
dragged Eutropia by her hand and brought her towards a creek near a coconut tree which was about
five meters away from where Nilsonita and Rudy Gonzales were. The appellant then ordered
Eutropia to remove her panty which she refused at first, but appellant threatened to kill her, so she
removed her panty after which appellant ordered her to lie down. Subsequently, appellant placed
himself on top of the victim and inserted his penis into her vagina and succeeded in having sexual
intercourse with her by moving his buttocks up and down
After the first sexual act, appellant ordered Eutropia to standup which the latter helplessly and
grudgingly followed. Appellant again inserted his penis into her vagina and then performed a push
and pull movement. Not satisfied with the second intercourse, appellant ordered Eutropia to lie
down again preparatory to a third intercourse. Appellant again performed the sexual act with her.

351
CRIMINAL LAW 1 CASE DIGESTS
ATTY. LYAN DAVD JUANICO
BLOCK 1-M A.Y. 2020 - 2021

After the third intercourse, appellant ordered Eutropia to stand up and then he bent her body
downwards with her hands and knees resting on the ground. When the latter was already in this
position, appellant then placed himself behind her, inserted his penis into her vagina and executed
a push and pull movement in the dog's way of sexual intercourse.
After performing this uncommon way of sexual intercourse, appellant ordered Eutropia to lie down
again which the latter reluctantly obeyed because appellant's dagger was always pointed at her and
thereafter, he had carnal knowledge of her for the fifth time.
The accused was found guilty by the Court of First Instance of the crime of rape aggravated by
despoblado, ignominy, and disregard to rank. The Court of First Instance imposed the death
penalty, hence this review.
ISSUE:
What are the aggravating circumstance present in the case at bar?
RULING:
The complaint alleges the following aggravating circumstances: abuse of superior strength,
nocturnity, despoblado, ignominy, and reiteracion.
The trial court disregarded superiority because it "is inherent in the crime of rape or is absorbed
in the element of force." It also did not consider nocturnity "there being no evidence that the
accused purposely sought it to facilitate the commission of this rape."
Despoblado/uninhabited place was present according to the trial court because: "The accused
dragged the offended party, at the point of a dagger, to the carabao trail, about 10 meters from the
junction, but 40 to 50 meters below to better attain his purpose without interference, and to better
secure himself from detection and punishment. Even the junction where the two children were left
is already 400 meters from the nearest house. While there maybe occasional passersby, this does
not destroy its being an uninhabited place.
The trial court held that there was ignominy because the appellant used not only the missionary
position, i.e. male superior, female inferior, but also "The same position as dogs do" i.e., entry
from behind. The appellant claims there was no ignominy because "The studies of many experts
in the matter have shown that this 'position' is not novel and has repeatedly and often been resorted
to by couples in the act of copulation." This may well be if the sexual act is performed by
consenting partners but not otherwise.
The trial court also held that "there is no reiteracion because one of the offenses, namely Robbery
in Band, for which the accused has been penalized, was committed after the commission of this

352
CRIMINAL LAW 1 CASE DIGESTS
ATTY. LYAN DAVD JUANICO
BLOCK 1-M A.Y. 2020 - 2021

rape case, and the penalty imposed on the other offense of Frustrated Homicide, is lighter than the
penalty for rape."
Although not alleged in the complaint, the trial court stated that the offense was aggravated by
disregard of rank because it was a fact known to the appellant that Mrs. Agno was a
schoolteacher. The appellant claims that this circumstance cannot be assigned to him because there
was no deliberate intent to offend or insult the rank of Mrs. Agno. The Solicitor General agrees
with the appellant for the same reason.

353
CRIMINAL LAW 1 CASE DIGESTS
ATTY. LYAN DAVD JUANICO
BLOCK 1-M A.Y. 2020 - 2021

33. People of the Philippines v. Fernando Sultan y Lato


G.R. No. 132470. April 27, 2000
Bellosillo, J.

FACTS:
The Information charges accused-appellant with the special complex crime of robbery with rape.
The record shows that the prosecution has established that he committed both robbery and rape
with the intent to take personal property of another preceding the rape. Under Art. 294, par. (1), of
the Revised Penal Code, "... any person guilty of robbery with the use of violence against or
intimidation of persons shall suffer: 1. The penalty of reclusion perpetua to death, . . . when the
robbery shall have been accompanied by rape . . ." Complaining witness Juditha Bautista was
raped twice on the occasion of the robbery. In this regard, this Court had declared in some cases
that the additional rapes committed on the same occasion of robbery would not increase the
penalty. There were also cases, however, where this Court ruled that the multiplicity of rapes
committed could be appreciated as an aggravating circumstance.
ISSUE:
Should the additional rape constitute as an aggravating circumstance to the crime committed?
RULING:
It should not be appreciated as an aggravating circumstance. In the recent case of People v. Regala
the Court held that the additional rapes committed should not be appreciated as an aggravating
circumstance despite a resultant "anomalous situation" wherein robbery with rape would be on the
same level as robbery with multiple rapes in terms of gravity. The Court realized that there was no
law providing for the additional rape/s or homicide/s for that matter to be considered as aggravating
circumstance. It further observed that the enumeration of aggravating circumstances under Art. 14
of the Revised Penal Code is exclusive, unlike in Art. 13 of the same Code which enumerates the
mitigating circumstances where analogous circumstances may be considered, hence, the remedy
lies with the legislature. Consequently, unless and until a law is passed providing that the additional
rape/s or homicide/s may be considered aggravating, the Court must construe the penal law in
favor of the offender as no person may be brought within its terms if he is not clearly made so by
the statute. Under this view, the additional rape committed by accused-appellant is not considered
an aggravating circumstance. Applying Art. 63, par. (2), of the Revised Penal Code which provides
that "(i)n all cases in which the law prescribes a penalty composed of two indivisible penalties, the
following rules shall be observed in the application thereof . . . 2. (w)hen there are neither
mitigating nor aggravating circumstances in the commission of the deed, the lesser penalty shall
be applied," the lower penalty of reclusion perpetua should be imposed on accused-appellant.

354
CRIMINAL LAW 1 CASE DIGESTS
ATTY. LYAN DAVD JUANICO
BLOCK 1-M A.Y. 2020 - 2021

34. People of the Philippines v. Walpan Ladjaalam y Mihajil


G.R. Nos. 136149-51. September 19, 2000
Panganiban, J.

FACTS:
The trial court convicted appellant of three crimes: (1) maintenance of a drug den, (2) direct assault
with attempted homicide, and (3) illegal possession of firearms.
The Supreme Court held that the trial court correctly convicted appellant of the crimes of
maintenance of a drug den and direct assault with attempted homicide.
The OSG disagrees with the trial court’s decision to convict appellant of the separate offense of
illegal possession of firearms under PD 1866, as amended by RA 8294, on the ground that the trial
court should not have applied the new law. It contends that under the facts of the case, the
applicable law should have been PD 1866, as worded prior to its amendment by RA 8294
The trial court's ruling and the OSG's submission exemplify the legal community's difficulty in
grappling with the changes brought about by RA 8294. Hence, before us now are opposing views
on how to interpret Section 1 of the new law, which provides as follows:
"SECTION 1. Section 1 of Presidential Decree No. 1866, as amended, is hereby further amended
to read as follows:
"Section 1. Unlawful Manufacture, Sale, Acquisition, Disposition or Possession of Firearms or
Ammunition Instruments Used or Intended to be Used in the Manufacture of Firearms or
Ammunition. — The penalty of prision correccional in its maximum period and a fine of not less
than Fifteen thousand pesos (P15,000) shall be imposed upon any person who shall unlawfully
manufacture, deal in, acquire, dispose, or possess any low powered firearm, such as rimfire
handgun, .380 or .32 and other firearm of similar firepower, part of firearm, ammunition, or
machinery, tool or instrument used or intended to be used in the manufacture of any firearm or
ammunition: Provided, That no other crime was committed.
"The penalty of prision mayor in its minimum period and a fine of Thirty thousand pesos (P30,000)
shall be imposed if the firearm is classified as high powered firearm which includes those with
bores bigger in diameter than .30 caliber and 9 millimeter such as caliber .40, .41, .44, .45 and also
lesser calibered firearms but considered powerful such as caliber .357 and caliber .22 centerfire
magnum and other firearms with firing capability of full automatic and by burst of two or three:
Provided, however, That no other crime was committed by the person arrested.
"If homicide or murder is committed with the use of an unlicensed firearm, such use of an
unlicensed firearm shall be considered as an aggravating circumstance. xxx”

355
CRIMINAL LAW 1 CASE DIGESTS
ATTY. LYAN DAVD JUANICO
BLOCK 1-M A.Y. 2020 - 2021

Citing People v. Jayson, the OSG argues that the foregoing provision does not cover the specific
facts of this case. Since another crime — direct assault with multiple unlawful homicide — was
committed, appellant cannot be convicted of simple illegal possession of firearms under the second
paragraph of the aforecited provision. Furthermore, since there was no killing in this case,
possession cannot be deemed as an aggravating circumstance under the third paragraph of the
provision. Based on these premises, the OSG concludes that the applicable law is not RA 8294,
but PD 1866 which, as worded prior the new law, penalizes simple illegal possession of firearms
even if another crime is committed at the same time.
Applying a different interpretation, the trial court posits that appellant should be convicted of
illegal possession of firearms, in addition to direct assault with multiple attempted homicide. It did
not explain its ruling, however. Considering that it could not have been ignorant of the proviso in
the second paragraph, it seemed to have construed "no other crime" as referring only to homicide
and murder, in both of which illegal possession of firearms is an aggravating circumstance. In
other words, if a crime other than murder or homicide is committed, a person may still be convicted
of illegal possession of firearms. In this case, the other crime committed was direct assault with
multiple attempted homicide; hence, the trial court found appellant guilty of illegal possession of
firearms.
ISSUE:
Which of the conflicting interpretations of the law must be accepted?
RULING:
We cannot accept either of these interpretations because they ignore the plain language of
the statute. A simple reading thereof shows that if an unlicensed firearm is used in the commission
of any crime, there can be no separate offense of simple illegal possession of firearms. Hence, if
the "other crime" is murder or homicide, illegal possession of firearms becomes merely an
aggravating circumstance, not a separate offense. Since direct assault with multiple attempted
homicide was committed in this case, appellant can no longer be held liable for illegal possession
of firearms.
Moreover, penal laws are construed liberally in favor of the accused. In this case, the plain meaning
of RA 8294's simple language is most favorable to herein appellant. Verily, no other interpretation
is justified, for the language of the new law demonstrates the legislative intent to favor the accused.
Accordingly, appellant cannot be convicted of two separate offenses of illegal possession of
firearms and direct assault with attempted homicide. Moreover, since the crime committed was
direct assault and not homicide or murder, illegal possession of firearms cannot be deemed an
aggravating circumstance.

356
CRIMINAL LAW 1 CASE DIGESTS
ATTY. LYAN DAVD JUANICO
BLOCK 1-M A.Y. 2020 - 2021

Republic Act No. 8294 penalizes simple illegal possession of firearms, provided that the person
arrested committed "no other crime." Furthermore, if the person is held liable for murder or
homicide, illegal possession of firearms is an aggravating circumstance, but not a separate offense.
Hence, where an accused was convicted of direct assault with multiple attempted homicide for
firing an unlicensed M-14 rifle at several policemen who were about to serve a search warrant, he
cannot be held guilty of the separate offense of illegal possession of firearms. Neither can such
unlawful act be considered to have aggravated the direct assault.

357
CRIMINAL LAW 1 CASE DIGESTS
ATTY. LYAN DAVD JUANICO
BLOCK 1-M A.Y. 2020 - 2021

35. CELINO V COURT OF APPEALS


G.R. NO. 170562, JUNE 29, 2007
CARPIO, MORALES, J.

FACTS:
Two separate informations were filed before the Regional Trial Court of Roxas City charging
petitioner with violation of Section 2(a) of COMELEC Resolution No. 6446 (gun ban), and Section
1, Paragraph 2 of Republic Act No. (R.A.) 82944 (illegal possession of firearm). Upon
arraignment, he pleaded not guilty on Gun Ban violation charge. He filed a motion to quash
contending that he cannot be prosecuted for illegal possession of firearms if he was also charged
of committing other crime violating the COMELEC gun ban.
RTC denied on the ground that “the other offense charged is not one of those enumerated in Illegal
possession law”. CA affirmed the decision.
ISSUE
Whether or not petitioner cannot be prosecuted for illegal possession of firearms since he was
charged of other crime (violation of gun ban)
RULING
NO. The court ruled that RA 8294 illegal possession of firearms talks about the crimes of rebellion
or insurrection, sedition, or attempted coup d’état.
The ruling in other cases relied to by the petitioner is of different circumstances. They are charged
with illegal possession because of their commission as shown by the conviction of some other
crime. In the case, the petitioner has only been accused of committing a violation of Gun Ban.
Accusation is different from guilt.
When the offense committed is one of those stated in RA 8294, illegal possession should be
quashed and tried together with such offense – qualify or absorbed element of such crimes. If the
offense involved is none of those crimes, a separate case for illegal possession should continue.

358
CRIMINAL LAW 1 CASE DIGESTS
ATTY. LYAN DAVD JUANICO
BLOCK 1-M A.Y. 2020 - 2021

36. People v. Sibbu


GR No. 214757; March 29, 2017
DEL CASTILLO, J.:

FACTS:
Victim: daughter, mother and father

Version of the Prosecution:


Bryan Julian (Bryan), the private complainant in Criminal Case No. 11722 and a common witness
to all the cases, testified that between 6:30 and 7:00 p.m. of December 6, 2004, he was with his
three-year old daughter, Trisha May Julian (Trisha), the victim in Criminal Case No. 11721; his
mother Ofelia Julian (Ofelia), the victim in Criminal Case No. 11723; and his father, Warlito Julian
(Warlito), the victim in Criminal Case No. 11724 in the azotea of his parents' house in Barangay
Elizabeth, Marcos, Ilocos Norte when he saw from a distance of about five meters a person in
camouflage uniform with a long firearm slung across his chest and a black bonnet over his head.
When the armed man inched closer to the house, he tried to fix his bonnet thereby providing Bryan
the opportunity to see his face; Bryan had a clear look at the armed man because there were
Christmas lights hanging from the roof of their porch. Bryan recognized the armed man as the
appellant. 6 Bryan also saw two men in crouching position at a distance of three meters away from
the appellant. Fearing the worst, Bryan shouted a warning to his family. Appellant then fired upon
them killing Trisha, Ofelia and Warlito.

Version of the Defense:


At home taking care of his sick son.

RTC- Murder and Attempted murder


CA- Murder and Attempted Murder

ISSUE:
Whether or not treachery, dwelling and use of disguise are properly appreciated as aggravating
circumstances.

RULING:
We find that Bryan was able to identify the appellant as the assailant in the shooting incident; there
is no reason to doubt his positive testimony.
Appellant also questions the RTC's appreciation of the aggravating circumstances of treachery,
dwelling, and use of disguise.

Treachery
Treachery was correctly appreciated as qualifying circumstance in the instant case. Treachery is
present "when the offender commits any of the crimes against person, employing means, methods,
or forms in the execution thereof which tend directly and specially to insure its execution, without
risk to himself arising from the defense which the offended party might make." In this case

359
CRIMINAL LAW 1 CASE DIGESTS
ATTY. LYAN DAVD JUANICO
BLOCK 1-M A.Y. 2020 - 2021

however, before the shooting incident, appellant was seen with a gun slung over his neck and a
bonnet covered his face to conceal his identity. It is clear that appellant's purpose is to harm and
kill his victims.

In this case, the evidence on record reveals that at the time of the shooting incident, Warlito, Ofelia,
Trisha, and Bryan were at the porch of their house totally unaware of the impending attack. In
addition, they were all unarmed thus unable to mount a defense in the event of an attack. On the
other hand, appellant and his cohorts were armed. They also surreptitiously approached the
residence of the victims. Appellant, in particular, wore camouflage uniform to avoid detection.
Although Bryan was able to warn his family about the impending attack, it was too late for the
victims to scamper for safety or to defend themselves. At the time Bryan became aware of
appellant's presence, the latter was already in the vicinity of about five meters. In fine, appellant
employed deliberate means to ensure the accomplishment of his purpose of killing his victims with
minimal risk to his safety. There can be no other conclusion than that the appellant's attack was
treacherous.

Dwelling
The aggravating circumstance of dwelling should be taken into account. Although the triggerman
fired the shot from outside the house, his victim was inside. For this circumstance to be considered
it is not necessary that the accused should have actually entered the dwelling of the victim to
commit the offense; it is enough that the victim was attacked inside his own house, although the
assailant may have devised means to perpetrate the assault from without

Use of Disguise
The use of disguise was likewise correctly appreciated as an aggravating circumstance in this case.
Bryan testified that the appellant covered his face with a bonnet during the shooting incident. There
could be no other possible purpose for wearing a bonnet over appellant's face but to conceal his
identity, especially since Bryan and appellant live in the same barangay and are familiar with each
other.

All told, appellant was correctly convicted of three counts of murder considering the qualifying
circumstance of treachery and one count of attempted murder. Since two aggravating
circumstances of dwelling and use of disguise attended the commission of the crime of murder,
appellant should be sentenced to death in accordance with Article 63 28 of the Revised Penal Code.
Under Article 248 29 of the Revised Penal Code, murder is punishable by reclusion perpetua to
death. Thus, under Article 63, the higher penalty should be imposed. However, because of the
passage of Republic Act No. 9346, or An Act Prohibiting the Imposition of Death Penalty in the
Philippines, the imposition of death penalty is now prohibited. The law provides that in lieu of the
death penalty, the penalty of reclusion perpetua shall be imposed with no eligibility for parole.

360
CRIMINAL LAW 1 CASE DIGESTS
ATTY. LYAN DAVD JUANICO
BLOCK 1-M A.Y. 2020 - 2021

36. People v. Nazareno


GR No. 196434; October 24, 2012
PEREZ, J.:

FACTS:
On November 10, 1993 David Valdez (David), Magallanes, and Francisco attended the wake of a
friend. While there, they drank liquor with accused Nazareno and Saliendra. A heated argument
ensued between Magallanes and Nazareno but their companions pacified them.

On the following day, November 11, David, Magallanes, and Francisco returned to the wake.
Accused Nazareno and Saliendra also arrived and told the three not to mind the previous night's
altercation. At around 9:30 in the evening, while David, Francisco, and their friend, Aida Unos
were walking on the street, Nazareno and Saliendra blocked their path. Nazareno boxed Francisco
who fled but Saliendra went after him with a balisong. Francisco, who succeeded in hiding saw
Nazareno hit David on the body with a stick while Saliendra struck David's head with a stone.
David ran towards a gasoline station but Nazareno and Saliendra, aided by some barangay tanods,
caught up with him. As David fell, the barangay tanods took over the assault. This took place as
Magallanes stood about five meters across the highway unable to help his friend. Afterwards, Unos
brought David to the hospital. Dr. Rebosa performed surgery on David's head but he died on
November 14, 1993 of massive intra-cranial hemorrhage secondary to depressed fracture on his
right temporal bone in a form of blunt trauma.

RTC- Murder qualified by abuse of superior strength and aggravated by treachery


CA- murder qualified by abuse of superior strength

ISSUE:
Whether or not a qualifying circumstance of abuse of superior strength attended the killing of
David.

RULING:
Abuse of Superior Strength
The CA held that the killing of David should be characterized as one of murder qualified by abuse
of superior strength. The Court finds no fault in this ruling. There is abuse of superior strength
when the aggressors purposely use excessive force rendering the victim unable to defend himself.
The notorious inequality of forces creates an unfair advantage for the aggressor.

Here, Nazareno and Saliendra evidently armed themselves beforehand, Nazareno with a stick and
Saliendra with a heavy stone. David was unarmed. The two chased him even as he fied from them.
And when they caught up with him, aided by some unnamed barangay tanods, Nazareno and
Saliendra exploited their superior advantage and knocked the defenseless David unconscious. He
evidently died from head fracture caused by one of the blows on his head.

361
CRIMINAL LAW 1 CASE DIGESTS
ATTY. LYAN DAVD JUANICO
BLOCK 1-M A.Y. 2020 - 2021

37. People v. Dulin


GR No. 171284; June 29, 2015
BERSAMIN, J.

FACTS:
Accused: Alfredo “Freddie” Dulin
Victim: Francisco Batulan

Tamayao was on Tamayao Street in Atulayan Norte, Tuguegarao at about 10:00 o'clock in the
evening of August 22, 1990 when a young man came running from the house of Vicente Danao
towards the house of Batulan, shouting that his Uncle Totoy (Batulan) had been stabbed. Tamayao
rushed towards Danao's house, which was about 30 meters from his own house, and there he saw
Dulin stabbing Batulan who was already prostrate face down. Dulin was on top of Batulan, as if
kneeling with his left foot touching the ground. Dulin was holding Batulan by the hair with his left
hand, and thrusting the knife at the latter with his right hand. Seeing this, Tamayao ran towards
Batulan's house to inform Estelita Batulan, the victim's wife who was his aunt, about the incident.
He went home afterwards.

Tamayao mentioned of the long standing grudge between Batulan and Dulin, and of seeing them
fighting in April 1990. He recalled Dulin uttering on two occasions: He will soon have his day and
I will kill him.

Version of the Defense:


that Alberto was not yet around, arriving only at about 8:00 o'clock in the evening to talk with
Raymund and Nicanor about the price of the fighting cocks; that after their transaction, Alberto
served Nicanor and Raymund food, and he (Dulin) and Jun Danao thereafter accompanied
Raymund and Nicanor to the highway to get a tricycle ride, but on their way, they passed Angel
Bancud who called out to him: that he (Dulin) asked the others to go ahead, and he would just
catch up with them; that as he (Dulin) approached Bancud, Batulan, the cousin of his (Dulin)
mother, stabbed him on the right side of his body and in the left hand; that he complained to
Batulan: Uncle, you hit me (Dinisgrasya nakun), but Batulan replied: I will really kill you; that he
(Dulin) ran to the upper level of Carolina Danao's house, pursued by Batulan who stabbed him
again several times; that they grappled for the weapon until he (Dulin) was able to wrest it from
Batulan; that he (Dulin) stabbed Batulan with the weapon, and they struggled until he (Dulin) felt
weak, eventually falling to the ground; and that he (Dulin) regained consciousness only the next
day at the hospital.

RTC- Murder PMC- incomplete self-defense, No AC


CA- Murder qualified by Treachery No AC No MC

ISSUE:
Whether or not the conviction of the CA of the crime of murder as the qualifying circumstance of
treachery was not established.

362
CRIMINAL LAW 1 CASE DIGESTS
ATTY. LYAN DAVD JUANICO
BLOCK 1-M A.Y. 2020 - 2021

RULING:
Homicide No AC no MC
No incomplete Self-Defense

In rejecting Dulin's argument, the CA observed that although Batulan had initiated the attack
against Dulin the unlawful aggression from Batulan effectively ceased once Dulin had wrested the
weapon from the latter.

Treachery
Treachery, which was alleged in the information, is one such qualifying circumstance. There is
treachery when the offender commits any of the crimes against persons, employing means and
methods or forms in the execution thereof which tend to directly and specially ensure its execution,
without risk to himself arising from the defense which the offended party might make.

They continued to grapple for the weapon inside the house of Danao, and it was at that point when
Dulin stabbed Batulan several times.Under the circumstances, treachery should not be appreciated
in the killing of Batulan because the stabbing by Dulin did not take Batulan by surprise due to his
having been sufficiently forewarned of Dulin's impending assault, 32 and being thus afforded the
opportunity to defend himself, or to escape, or even to recover control of the weapon from Dulin.
The essence of treachery is that the attack comes without warning, or is done in a swift, deliberate
and unexpected manner, affording the hapless, unarmed and unsuspecting victim no chance to
resist or to escape, without the slightest provocation on the part of the victim. 33 The mode of
attack must not spring from the unexpected turn of events.

363
CRIMINAL LAW 1 CASE DIGESTS
ATTY. LYAN DAVD JUANICO
BLOCK 1-M A.Y. 2020 - 2021

38. Fantastico v. Malicse Sr.


GR No. 190912; January 12, 2015
PERALTA, J.:

FACTS:
On the afternoon of June 27, 1993, Elpidio Malicse, Sr. (Elpidio) was outside the house of his
sister Isabelita Iguiron (Isabelita) in Pandacan, Manila when all of a sudden, he heard Isabelita's
son, Winston, throwing invectives at him. Thus, Elpidio confronted Isabelita but she also cursed
him, which prompted the former to slap the latter. On that occasion, Elpidio was under the
influence of alcohol.

The Barangay Chairman heard what transpired and went to the place where the commotion was
taking place in order to pacify those who were involved. Elpidio was eventually persuaded to go
home where he drank some coffee. Thereafter, Elpidio went back to the house of Isabelita to offer
reconciliation. On his way there, he passed by the house of Kagawad Andy Antonio and requested
the latter to accompany him, but was instead told to go back home, leaving Elpidio to proceed
alone.

Upon reaching Isabelita's house, Elpidio saw the former's son, Titus Iguiron (Titus) and her son-
in-law Gary Fantastico (Gary) and asked the two where he can find their parents. Titus and Gary
responded, "putang ina mo, and kulit mo, lumayas ka, punyeta ka."

In his anger with the response of Titus and Gary, Elpidio kicked the door open and saw Isabelita's
elder son, Salvador Iguiron (Salvador) behind the door holding a rattan stick or arnis. Salvador hit
Elpidio on the right side of his head that forced the latter to bow his head but Salvador delivered a
second blow that hit Elpidio on the right eyebrow. Salvador attempted to hit Elpidio for the third
time but the latter got hold of the rattan stick and the two wrestled on the floor and grappled for
the possession of the same rattan stick. Then Titus ran towards the two and sprayed something on
Elpidio's face. Not being able to free himself from the clutches of Salvador and to extricate himself,
Elpidio bit Salvador's head.

Gary hit Elpidio on the right side of his head with a tomahawk axe when the latter was about to go
out of the house. Elpidio tried to defend himself but was unable to take the tomahawk axe from
Gary. Elpidio walked away from Titus but Gary, still armed with the tomahawk axe and Salvador,
with his arnis, including Titus, chased him.

Roland (Rolly) Villanueva, without any warning, hit Elpidio on the back of his head with a lead
pipe which caused the latter to fall on the ground. Elpidio begged his assailants to stop, but to no
avail. Salvador hit him countless times on his thighs, legs and knees using the rattan stick. While
he was simultaneously being beaten up by Salvador, Titus, Gary, Rolly, Nestor, Eugene and
Tommy, he tried to cover his face with his arm. Gary hit him with the tomahawk axe on his right
leg, between the knees and the ankle of his leg, which caused the fracture on his legs and knees.
Rolly hit Elpidio's head with a lead pipe, while Tommy hit him with a piece of wood on the back
of his shoulder.

364
CRIMINAL LAW 1 CASE DIGESTS
ATTY. LYAN DAVD JUANICO
BLOCK 1-M A.Y. 2020 - 2021

Thereafter, a certain "Mang Gil" tried to break them off but Titus and Gary shouted at him: "Huwag
makialam, away ng mag-anak ito" and the two continued to maul Elpidio. The people who
witnessed the incident shouted "maawa na kayo" but they only stopped battering him when a
bystander fainted because of the incident. Elpidio then pretended to be dead. It was then that
concerned neighbors approached him and rushed him to the emergency room of the Philippine
General Hospital (PGH).

RTC- Attempted Murder


CA- Attempted Murder

ISSUE:
Whether or not the Court is correct in appreciating the aggravating circumstance of treachery and
abuse of superior strength.

RULING:
SC Attempted Murder (Qualified by Abuse of Superior Strength)
Treachery

Petitioners also claim that the prosecution was not able to prove the presence of treachery or any
other qualifying circumstance.

Treachery
In this particular case, there was no treachery. There is treachery when the offender commits any
of the crimes against persons, employing means, methods, or forms in the execution, which tend
directly and specially to insure. its execution, without risk to the offender arising from the defense
which the offended party might make. The essence of treachery is that the attack comes without a
warning and in a swift, deliberate, and unexpected manner, affording the hapless, unarmed, and
unsuspecting victim no chance to resist or escape. For treachery to be considered, two elements
must concur: (1) the employment of means of execution that gives the persons attacked no
opportunity to defend themselves or retaliate; and (2) the means of execution were deliberately or
consciously adopted. From the facts proven by the prosecution, the incident was spontaneous, thus,
the second element of treachery is wanting. The incident, which happened at the spur of the
moment, negates the possibility that the petitioners consciously adopted means to execute the
crime committed. There is no treachery where the attack was not preconceived and deliberately
adopted but was just triggered by the sudden infuriation on the part of the accused because of the
provocative act of the victim.

QUALIFYING CIRCUMSTANCE OF ABUSE OF SUPERIOR STRENGTH


The said injuries inflicted on the complainant after he went back to his sister Isabelita's house.
When he kicked the door, the melee began. And the sequence of the injuries is proven by victim's
testimony. But it was a lopsided attack as the victim was unarmed, while his attackers were all
armed (rattan stick, tomahawk and lead pipe). And the victim was also drunk. This establishes the
element of abuse of superior strength. The suddenness of the blow inflicted by Salvador on Elpidio

365
CRIMINAL LAW 1 CASE DIGESTS
ATTY. LYAN DAVD JUANICO
BLOCK 1-M A.Y. 2020 - 2021

when he entered the premises show that the former was ready to hit the victim and was waiting for
him to enter. It afforded Elpidio no means to defend himself. And Salvador consciously adopted
the said actuation. He hit Elpidio twice on the head. Treachery is present in this case and must be
considered an aggravating circumstance against Salvador Iguiron. Rolly Villanueva, Gary
Fantastico and Salvador Iguiron were all armed while Elpidio, inebriated, had nothing to defend
himself with. There is clearly present here the circumstance of abuse of superior strength.

Abuse of superior strength is present whenever there is a notorious inequality of forces between
the victim and the aggressor, assuming a situation of superiority of strength notoriously
advantageous for the aggressor selected or taken advantage of by him in the commission of the
crime." The appreciation of this aggravating circumstance depends on the age, size, and strength
of the parties.

366
CRIMINAL LAW 1 CASE DIGESTS
ATTY. LYAN DAVD JUANICO
BLOCK 1-M A.Y. 2020 - 2021

39. People v. Bokingco


GR No. 187536 August 10, 2011
PEREZ, J.:

FACTS:
The victim, Noli Pasionand his wife, Elsa, were residing in a house along Mac Arthur Highway in
Balibago, Angeles City. Pasion owned a pawnshop, which formed part of his house. He also
maintained two (2) rows of apartment units at the back of his house. The first row had six (6) units,
one of which is Apartment No. 5 a nd was being leased to Dante Vitalicio , Pasion's brother-in-
law, while the other row was still under construction at the time of his death. Appellants, who were
staying in Apartment No. 3, were among the 13 construction workers employed by Pasion.

The prosecution's evidence shows that at around 1:00 a.m. on 29 February 2000, Vitalicio was
spin-drying his clothes inside his apartment when Pasion came from the front door, passed by him
and went out of the back door. A few minutes later, he heard a commotion from Apartment No. 3.
He headed to said unit to check. He peeped through a screen door and saw Bokingco hitting
something on the floor. Upon seeing Vitalicio, Bokingco allegedly pushed open the screen door
and attacked him with a hammer in his hand. A struggle ensued and Vitalicio was hit several times.
Vitalicio bit Bokingco's neck and managed to push him away. Bokingco tried to chase Vitalicio
but was eventually subdued by a co-worker. Vitalicio proceeded to his house and was told by his
wife that Pasion was found dead in the kitchen of Apartment No. 3. Vitalicio went back to
Apartment No. 3 and saw Pasion's body lying flat on the kitchen floor. Pasion and Vitalicio were
brought to the hospital. Pasion expired a few hours later while Vitalicio was treated for his injuries.

Elsa testified that she was in the master's bedroom on the second floor of the house when she heard
banging sounds and her husband's moans. She immediately got off the bed and went down. Before
reaching the kitchen, Col blocked her way. Elsa asked him why he was inside their house but Col
suddenly ran towards her, sprayed tear gas on her eyes and poked a sharp object under her chin.
Elsa was wounded when she bowed her head to avoid the tear gas. Col then instructed her to open
the vault of the pawnshop but Elsa informed him that she does not know the combination lock.
Elsa tried offering him money but Col dragged her towards the back door by holding her neck and
pulling her backward. Before they reached the door, Elsa saw Bokingco open the screen door and
heard him tell Col: "tara, patay na siya." Col immediately let her go and ran away with Bokingco.
Elsa proceeded to Apartment No. 3. Thereat, she saw her husband lying on the floor, bathed in his
own blood.

ISSUE:
Whether or not the stated qualifying circumstances were properly appreciated to consider the case
as murder.

RULING:
We agree with appellants that treachery cannot be appreciated to qualify the crime to murder in
the absence of any proof of the manner in which the aggression was commenced. For treachery to
be appreciated, the prosecution must prove that at the time of the attack, the victim was not in a

367
CRIMINAL LAW 1 CASE DIGESTS
ATTY. LYAN DAVD JUANICO
BLOCK 1-M A.Y. 2020 - 2021

position to defend himself, and that the offender consciously adopted the particular means, method
or form of attack employed by him. 29 Nobody witnessed the commencement and the manner of
the attack. While the witness Vitalicio managed to see Bokingco hitting something on the floor,
he failed to see the victim at that time.

Bokingco admitted in open court that he killed Pasion. But the admitted manner of killing is
inconsistent with evident premeditation. To warrant a finding of evident premeditation, the
prosecution must establish the confluence of the following requisites: (a) the time when the
offender was determined to commit the crime; (b) an act manifestly indicating that the offender
clung to his determination; and (c) a sufficient interval of time between the determination and the
execution of the crime to allow him to reflect upon the consequences of his act. It is indispensable
to show how and when the plan to kill was hatched or how much time had elapsed before it was
carried out. In the instant case, no proof was shown as to how and when the plan to kill was devised.
Bokingco admitted in court that he only retaliated when Pasion allegedly hit him in the head.
Despite the fact that Bokingco admitted that he was treated poorly by Pasion, the prosecution failed
to establish that Bokingco planned the attack.

It was during the preliminary investigation that Bokingco mentioned his and Col's plan to kill
Pasion. Bokingco's confession was admittedly taken without the assistance of counsel in violation
of Section 12, Article III of the 1987 Constitution.

The finding that nighttime attended the commission of the crime is anchored on the presumption
that there was evident premeditation. Having ruled however that evident premeditation has not
been proved, the aggravating circumstance of nighttime cannot be properly appreciated. There was
no evidence to show that Bokingco purposely sought nighttime to facilitate the commission of the
offense.

Abuse of confidence could not also be appreciated as an aggravating circumstance in this case.
Considering that the fact that Bokingco works for Pasion, it may be conceded that he enjoyed the
trust and confidence of Pasion. However, there was no showing that he took advantage of said
trust to facilitate the commission of the crime.

368
CRIMINAL LAW 1 CASE DIGESTS
ATTY. LYAN DAVD JUANICO
BLOCK 1-M A.Y. 2020 - 2021

40. People v. Tabarnero


GR No. 168169; February 24, 2010
LEONARDO-DE CASTRO, J.:

FACTS:
Victim: Ernesto Canatoy

Late at night on October 23, 1999, Gary went to the house of the deceased Ernesto Canatoy
(Ernesto), where the former used to reside as the live-in partner of Mary Jane Acibar (Mary Jane),
Ernesto's stepdaughter. Gary and Ernesto had a confrontation during which the latter was stabbed
nine times, causing his death. The versions of the prosecution and the defense would later diverge
as regards the presence of other persons at the scene and other circumstances concerning Ernesto's
death.

Gary, a 22-year-old construction worker at the time of his testimony in June 2001, testified that he
stayed in Ernesto's house from 1997 to 1999, as he and Mary Jane were living together. Mary Jane
is the daughter of Teresita Acibar, the wife of Ernesto. However, Gary left the house shortly before
the October 23, 1999 incident because of a misunderstanding with Ernesto when the latter
allegedly stopped the planned marriage of Gary and Mary Jane, who was pregnant at that time.

RTC- Murder
CA- Murder

ISSUE:
Whether or not the Court is correct to appreciate the aggravating circumstance of treachery.

RULING:
SC- Murder qualified by Treachery

Treachery
Yes. The aggravating circumstance of treachery qualified the killing. The Solicitor General argues
that treachery was amply demonstrated by the restraint upon Ernesto, which effectively rendered
him defenseless and unable to effectively repel, much less evade, the assault.

Abuse of Superior Strength- Absorbed by treachery


The act of the accused in stabbing Ernesto while two persons were holding him clearly shows the
deliberate use of excessive force out of proportion to the defense available to the person attacked.

No mitigating circumstance.

369
CRIMINAL LAW 1 CASE DIGESTS
ATTY. LYAN DAVD JUANICO
BLOCK 1-M A.Y. 2020 - 2021

MODULE 9: ALTERNATIVE CIRCUMSTANCES

1. PEOPLE V ATOP
G.R NO. 104303-05, FEBRUARY 10, 1998
PANGANIBAN, J.

FACTS:
Regina, 11 was raped 3 times of her grandmother’s common law, Atop. The first instance was
sometime in 1991, she was 10 years old when Atop inserted his finger to her vagina. She said it to
her grandmother, but she said that he was just showing his fatherly concern.
In 1992, she was called by the accused to do something with him. When she approaches him, he
immediately removes her panty and inserted his penis on her vagina.
When she was 12, again, the accused violated her while her grandmother was attending a delivery.
Again, she did not believe her.
One time in 1994, while she and her nieces were sleeping, she saw the accused looking at her then
rushed towards her and about to lay on top of her, but she was able to kick him.
She reported the incidents in 1992, 1993 and 1994 only in 1995 because she was afraid.
ISSUE:
Whether or not nighttime and relationships should be considered as aggravating circumstance.
RULING:
No. Nighttime and relationship as aggravating circumstance should be proved by the prosecution
that the accused passed the objective or subjective test for nighttime. It was not proved that
nighttime was deliberately sought the crime.
Relationship cannot be appreciated as aggravating circumstance. Relationship covers spouses,
ascendant, descendant, legitimate, natural, or adopted bro or sis, relative by affinity on the same
degree.
Affinity are “in-laws” or stepfather/mother. Common-law relations are not covered by “affinity”.
In the case, there is no blood relation between Regina and atop for top was the common law
husband of her grandmother.

370
CRIMINAL LAW 1 CASE DIGESTS
ATTY. LYAN DAVD JUANICO
BLOCK 1-M A.Y. 2020 - 2021

2. US V MCMANN
G.R. NO. L-2229, JULY 1, 1905
WILLARD, J.

FACTS:
Mcmann and McKay were employed by the quarter’s department of the Army. One day, Mckay
passed by the house of a moro, Amay Pindolonan, with a revolver in his hand, He tried to enter
the house to use a match to light his cigarette however the owner refused to. Then Mcmann arrived
and get the bolo from another moro named Master and holding a hammer on the other hand. He
also tried to enter but the owner did not let him. Altercation took place which resulted to the cutting
of the finger of the defendant. This Moro left for the camp to report the matter to the authorities.
Soon after this McKay and the Moro Pindolonan, being seated side by side at a distance of from 3
to 6 feet from the defendant, who was either standing or sitting on the stairway which led into the
house, the latter raised his pistol and fired at McKay. The bullet struck him in the back of the head
and killed him instantly. The Moro at once jumped up, looked around to see where the shot came
from, and started to run, whereupon the defendant shot him. The exact nature of his injuries does
not appear, but it appears that at the time of the trial, about a month after the event, he was still in
the hospital. At some time, probably after the killing of McKay, although the defendant says it was
before, the latter killed a dog which was on the premises. The defendant and McKay were both
drunk at this time.
The accused claim for the alternative mitigating circumstance of intoxication to be appreciated in
his case.
ISSUE
Whether or not Intoxication is a mitigating circumstance can be availed of.
RULING
No. Intoxication as a mitigating circumstance is not present in this case. The court held that the
defendant was drunk at the time the act was committed but held also that drunkenness was habitual
with him and therefore his condition could not be taken into consideration for the purpose of
lessening the sentence. The testimony given in the court was enough to prove that the defendant
was a habitual drinker.
Treachery is also present in this case as the victim was shot at the back of the head, in an
unexpected situation. Thus, the court found him guilty of murder.

371
CRIMINAL LAW 1 CASE DIGESTS
ATTY. LYAN DAVD JUANICO
BLOCK 1-M A.Y. 2020 - 2021

3. PEOPLE V SAN PEDRO


G.R. NO. L-44274, JANUARY 22, 1980
QUISUMBING, J.

FACTS:
Rivera, a jeepney driver, found dead in Laguna with his jeepney missing. Esguerra and Banasihan
admitted his participation in the crime.
Banasihan and San Pedro approached the victim saying he will hire his jeep to haul coconuts. Then
they lure him in the different way. Esguerra was holding a water pipe at that time. When they
reached a river, San Pedro told Rivera to stop then hit his nape with the pipe. Rivera tried to run
but he was stabbed then died. The accused sold the jeep in a different province.
Rodrigo Esguerra, when apprehended and interviewed by the police, admitted his participation
and named his companions.
ISSUES:
1. Whether or not the aggravating circumstance of craft is absorbed by treachery.
2. Whether the resulting single aggravating circumstance of treachery should be offset by the
mitigating circumstance of lack of instruction.
RULING:
1. NO. the craft in this case was not used to facilitate treachery unlike nighttime and abuse of superior
strength. It was directed toward facilitating the taking of the jeep. In the instant case, craft was
employed not with a view to making treachery more effective as nighttime and abuse of superior
strength would in the killing of the victim. It was directed actually towards facilitating the taking
of the jeep in the robbery scheme as planned by the culprits. From the definition of treachery, it is
manifest that the element of defense against bodily injury makes treachery proper for consideration
only in crimes against person as so explicitly provided by the Revised Penal Code

2. Lack of instruction cannot be appreciated. Even though the accused cannot read and write
but can only sign his name, Lack of instruction cannot be appreciated in the crime of theft, robbery,
and homicide.
The reason is that robbery and killing are, by their nature, wrongful acts, and are manifestly so
to the enlightened, equally as to the ignorant
The criteria in Lack of Instruction is not illiteracy alone but rather lack of sufficient intelligence.
The accused was sentenced guilty of Robbery with Homicide imposing the death penalty.

372
CRIMINAL LAW 1 CASE DIGESTS
ATTY. LYAN DAVD JUANICO
BLOCK 1-M A.Y. 2020 - 2021

4. PEOPLE V YANSON-DUMAGCAS
G.R. NO. 133527-28, DECEMBER 13, 1999
MELO, J.

FACTS:
Acting upon the inducement of spouse Jeanette Yanson-Dumancas and Charles Dumancas, under
the direction, cooperation and undue influence, exerted by P/Col. Nicolas M. Torres, taking
advantage of his position as the Station Commander of the Philippine National Police, Bacolod
City Station, with the direct participation and cooperation of Police Inspector Adonis C. Abeto,
other police officers Vicente Canuday, Jr., Jose Pahayupan, Mario Lamis, civilian (police) agents
Rolando R. Fernandez, Edwin Divinagracia, Teody Delgado, Jaime Gargallano, also taking
advantage of their respective positions, and Dominador Geroche, concurring and affirming in the
said criminal design, with the use of motor vehicle abduct, kidnap and detain one Danilo
Lumangyao and shortly thereafter at around 11 oclock in the evening of August 7, 1993 (1992),
failing in their aforesaid common purpose to extort money and in furtherance of said conspiracy,
with evident premeditation and treachery nocturnity and the use of motor vehicle, did then and
there shot and kill the said victim, while being handcuffed and blindfolded, that accused Cesar
Pecha and Edgar Hilado, with knowledge that said Lumangyao was victim of violence, did then
and there secretly bury the corpse in a makeshift shallow grave for the purpose of concealing the
crime of murder in order to prevent its discovery.
ISSUE:
Whether or not the accused is principal by inducement.
RULING:
No. On the case of accused-appellant Jeanette Yanson-Dumancas, the information charged her of
the crime of kidnapping for ransom with murder as principal by induction together with her
husband, Charles, who was found by the trial court not guilty of the crime.
What the Court now must examine is whether or not sufficient evidence was adduced by the
prosecution to prove beyond reasonable doubt that Jeanette indeed performed any of the following
acts: (a) directly forcing the killers to commit the crime, or (b) directly inducing them to commit
the crime.
There are 2 ways of directly forcing another to commit a crime, namely: (i) by using irresistible
force, or (ii) by causing uncontrollable fear. Upon review of the testimony of all the witnesses of
the prosecution, we find nothing to conclude that Jeanette used irresistible force or caused

373
CRIMINAL LAW 1 CASE DIGESTS
ATTY. LYAN DAVD JUANICO
BLOCK 1-M A.Y. 2020 - 2021

uncontrollable fear upon the other accused-appellants. The record is entirely bereft of any evidence
to show that Jeanette directly forced the participants of the said meeting to come up with such a
plan, by either using irresistible force or causing uncontrollable fear. The only basis relied upon
by the trial court in arriving at its conclusion that Jeanette is guilty of the crime as principal by
inducement, is the supposed commands or order given by her to accused-appellant Dominador
Geroche on two occasions. By no stretch of the imagination may these so-called commands,
standing alone, be considered as constituting irresistible force or causing uncontrollable fear.
Likewise, there are 2 ways of directly inducing another to commit a crime, namely: (i) by giving
a price, or offering reward or promise, and (ii) by using words of command. The Court finds no
evidence, as did the trial court, to show that Jeanette offered any price, reward, or promise.
In order that a person may be convicted as principal by inducement, the following must be
present: (1) the inducement be made with the intention of procuring the commission of the crime,
and (2) such inducement be the determining cause of the commission by the material executor. To
constitute inducement, there must exist on the part of the inducer the most positive resolution and
the most persistent effort to secure the commission of the crime, together with the presentation to
the person induced of the very strongest kind of temptation to commit the crime. The utterance,
CHEwhich was supposedly the act of inducement, should precede the commission of the crime
itself (People vs. Castillo).
In the case at bar, the abduction, which is an essential element of the crime charged (kidnapping
for ransom with murder) has already taken place when Jeanette allegedly told accused appellant
Geroche to take care of the two. Said utterance could, therefore, not have been the inducement to
commit the crime charged in this case.
Accused-appellants Jeanette Yanson-Dumancas are hereby acquitted.

374
CRIMINAL LAW 1 CASE DIGESTS
ATTY. LYAN DAVD JUANICO
BLOCK 1-M A.Y. 2020 - 2021

5. PEOPLE V MALUENDA
G.R. NO. 115351, MARCH 27, 1998
PANGANIBAN, J.

FACTS:
Engr. And Dr. Resus arrived at their residence/clinic. Waiting for the Resus spouses at the clinic
which adjoins the Resus spouses' residence were three men who identified themselves as
Commander Bobong Gonzaga (who is actually Raul Mondaga), Commander Bongkoy (who is
actually Maluenda) and alias "Alex". Upon the arrival of the Resus spouses, Mondaga declared
that they came upon orders of a certain Father Simon, an alleged NPA Commander, with his
directive to solicit money and medicines needed for the victims of the recent military-NPA
encounter at Melale, Agusan del Sur. The trio demanded from the couple medicines and money in
the amount of P20,000.00, but when the couple told them that they did not have such an amount,
they lowered their demand to P10,000.00, and reduced it still to P5,000.00 when the couple still
could not produce the said amount. Finally, the demand was lowered to any amount the Resus
couple could provide. The latter gave the amount of P500.00 plus assorted medicines worth
P800.00. After they were given the money and medicines, the trio demanded that they be driven
by Engr. Resus in his Volkswagen car to San Roque but the spouses said that it has no sufficient
gasoline. Mondaga demanded for it to be done tomorrow instead.
The next day, Mondaga arrived at the residence of the Resus couple. Mondaga hurried up Engr.
Resus. Upon reaching Alegria, Mondaga ordered Engr. Resus that he had to go with them. Against
his will, Engr. Resus went with the three. They went to the mountain hiking for almost two (2)
hours between the boundary of Cardon and Alegria. Upon reaching a hut, Mondaga told him that
he had forgotten something and had to go back and that Engr. Resus had to stay there. So Engr.
Resus, Maluenda, Alex and Gil Bueno passed the night in the farmhut.
Mondaga told the midwife that he [would] come back. Mondaga arrived at the Resus clinic at
around 7:00 in the evening. Mondaga demanded from Dr. Resus the amount of P300,000.00 for
the release of Engr. Resus. Dr. Resus told Mondaga that she could only produce P10,000.00.
Mondaga told Dr. Resus to reserve the amount for he would get it the following morning. He also
instructed Dr. Resus to look for the firearm of her husband. Dr. Resus then searched for the gun
ang gave it to him. Mondaga demanded for the use of Engr. Resus' motorcycle, but Dr. Resus told
him that the motorcycle was out of order. So Mondaga instructed Dr. Resus to get the motorcycle
of Legarto, which Dr. Resus did. Shortly thereafter, Legarto also arrived in his motorcycle.
Mondaga demanded that Dr. Resus go with them but the latter made excuses, particularly her
health. Dr. Resus asked that her helper Maria Abne go instead to which Mondaga agreed.

375
CRIMINAL LAW 1 CASE DIGESTS
ATTY. LYAN DAVD JUANICO
BLOCK 1-M A.Y. 2020 - 2021

The three arrived at Alegria, San Francisco, Agusan del Sur at around 7:00 a.m. Legarto then
safely kept his motorcycle after which they walked to the forest for about 2 hours until they reached
a carabao crossing where Mondaga left Legarto and Maria Abne for 30 minutes. Mondaga went
to the hut where he left Engr. Resus with a note from Dr. Resus which stateD, "Daddy, I have
committed only P10,000.00". He gave the note to Engr. Resus but told Engr. Resus that "you can
afford P300,000.00".
Dr. Resus, together with Nora Gubantes, went to Lianga to secure money from the relatives of Dr.
Resus. Since Dr. Resus' cousins were out of town, the two proceeded to San Francisco, Agusan
del Sur to see Dr. Presentacion Manatad, the mayor of San Francisco. Dr. Resus informed Mayor
Manatad about the incident and asked the mayor to give her an amount of P150,000.00 in return
for a PNB Check Dr. Resus [would] issue. Dr. Resus gave the money to Nora Gubantes with the
instruction to give the same to Legarto. Upon reaching Diatogon, Nora Gubantes gave the money
to her husband with the instruction to give the money to Legarto. Legarto acknowledged receiving
the money from Mr. Gubantes on August 22, 1992.
Mondaga, Maluenda and Legarto were later arrested by the police.
RTC ruled them guilty of kidnapping. CA affirmed the decision.
ISSUE:
Whether or not Legarto was a co-principal by indispensable cooperation.
RULING:
No. Art. 17 defined principal by indispensable cooperation as: Those who cooperate in the
commission of the offense by another act without which it would not have been accomplished.
In the case at bar, Legarto cannot be convicted under this definition because the prosecution failed
to allege, much less prove, any overt act on his part showing direct participation in the kidnapping
itself, his participation in the incident being limited to acts committed after the abduction was
already consummated. He was not with the kidnappers (1) when they forcibly solicited money and
medicine from the Resus couple, (2) when they brought the kidnap victim to Alegria, and (3) when
Mondaga demanded ransom for the victim's release. Together with the Resus housemaid, he
accompanied Mondaga to the hideout in Alegria only upon Dr. Resus' request. In short, the
prosecution failed to piece together a clear story as to show Legarto figured in the kidnapping
caper.
The act of delivering the ransom money and keeping part of it do not prove conspiracy. The
averments are sufficiently rebutted by Legarto's allegation that, out of loyalty to his former boss,
the participated in the release of the kidnap victim, not in his detention.

376
CRIMINAL LAW 1 CASE DIGESTS
ATTY. LYAN DAVD JUANICO
BLOCK 1-M A.Y. 2020 - 2021

Legarto may not have had a direct hand in the kidnapping, but he received part of the ransom and
used it to pay off his arrears in his motorcycle loan. Thus, having knowledge of the kidnapping for
ransom and without having directly participated therein, he took part in the crime after its
commission by profiting from its effects. He may not be the devil with the face of an angel that
the trial court described, but he is definitely not a saint. He is criminally liable as an accessory to
the crime of kidnapping for ransom.

377
CRIMINAL LAW 1 CASE DIGESTS
ATTY. LYAN DAVD JUANICO
BLOCK 1-M A.Y. 2020 - 2021

6. PEOPLE V MONTEALEGRE
G.R. NO. L-67948, MAY 31, 1988
CRUZ, J.

FACTS:
Wille Abadilla was eating at a restaurant, he smelled marijuana coming from a nearby table. He
went out to report and saw a police officer. Police Camtigue went inside the restaurant and
approached the table of the accused appellant Capalad and confronted them. A mayhem began.
While Camantigue was holding the two, Montealegre with this right hand and Capalad with his
left hand, Capalad suddenly and surreptitiously pulled out a knife and stabbed Camantigue let
loose Montealegre to draw the gun from his holster but Montealegre, thus released, restrained
Camantigue's hand to prevent the latter from defending himself Montealegre used both his hands
for his purpose as Capalad continued stabbing the Victim. While they were thus grappling, the
three fen to the floor and Capalad, freed from Camantigue's grip, rose and scampered toward the
door. Camantigue fired and, continuing the pursuit outside, fired again. Capalad fled into a dark
alley. Camantigue abandoned the chase and asked to be brought to a hospital. Capalad was later
found slumped in the alley with a bullet wound in Ms chest. Neither Camantigue nor Capalad
survived, both expiring the following day. The accused-appellant, testifying on his behalf, only
succeeded in confinning his own guilt. He claimed he ran away before the stabbing but his
testimony, consisting of denials, evasions, contradictions, claims of ignorance and forgetfulness
and protestations of innocence, does not have the ring of truth.

ISSUE:
Whether or not the accuse was correctly considered as co-principal

RULING:
Yes. The accused-appellant was correctly considered a co-principal for having collaborated with
Capalad in the killing of the police officer. The two acted in concert, with Capalad actually
stabbing Camantigue seven times and the accused-appellant holding on to the victim's hands to
prevent him from drawing his pistol and defending himself. While it is true that the accused-
appellant did not himself commit the act of stabbing, he was nonetheless equally guilty thereof for
having prevented Camantigue from resisting the attack against him. The accused-appellant was a
principal by indispensable cooperation. The requisites of this provision are: (1) participating in the

378
CRIMINAL LAW 1 CASE DIGESTS
ATTY. LYAN DAVD JUANICO
BLOCK 1-M A.Y. 2020 - 2021

criminal resolution, that is, there is either anterior conspiracy or unity of criminal purpose and
intention immediately before the commission of the crime charged; and (2) cooperation in the
commission of the offense by performing another act without which it would not have been
accomplished.

379
CRIMINAL LAW 1 CASE DIGESTS
ATTY. LYAN DAVD JUANICO
BLOCK 1-M A.Y. 2020 - 2021

7. People v. Madali
G.R. No. L-67803, 30 July 1990
FERNAN, C.J.:

FACTS:
On October 31, 1979, Patrolman Ricarte Madali killed father and son Cipriano and Felix Gasang,
and seriously wounded Agustin Reloj and Cipriano's daughter, Merlinda. The crimes stemmed
from an altercation between the son of the Madali spouses, Ramon, and the group of Felix Gasan
That night, Ricarte Madali saw Agustin then dragged him towards the gate of his house and the
latter was clubbed by Annie Madali. Felix Gasang then arrived. He saw Annie beamed her
flashlight at Felix and she said, "Here comes another." Ciprian arrived then Annie beamed her
flashlight at Cipriano and she said, "Here comes, here comes another, fire upon him." Madali shot
Cipriano who fell to the ground.

ISSUE:
Whether Annie Madali is a principal by inducement

RULING:
No. Annie Madali is an accomplice. Annie's shouts of "here comes, here comes another, shoot"
can’t incite Ricarte to fire at the victims to make Annie a principal by inducement. Those inciting
words do not have great dominance and influence over Madali as to become the determining cause
of the crimes.
Annie's beaming of her flashlight and warning husband of the presence of other persons in the
vicinity merely facilitated the commission of the shooting. Considering that, according to both of
the Madali spouses, "it was not so dark nor too bright”. Ricarte Madali could have nevertheless
accomplished his criminal acts without Annie's cooperation and assistance.

380
CRIMINAL LAW 1 CASE DIGESTS
ATTY. LYAN DAVD JUANICO
BLOCK 1-M A.Y. 2020 - 2021

8. Abejuela v. People
G.R. No. 80130, 19 August 1991
FERNAN, C.J.:

FACTS:
Benjamin Abejuela, a businessman engaged in the manufacture and fabrication of hand tractors
and other agricultural equipment, had a savings deposit with Banco Filipino, Tacloban Branch. In
1978, petitioner was befriended by Glicerio Balo, Jr., an employee of Banco Filipino in the same
Tacloban Branch.
Balo asked Abejuela to borrow the latter's passbook. Balo assured Abejuela that there was nothing
wrong in allowing him to use his passbook and even reassured Abejuela that he would accompany
him to the bank to make the deposit. Balo's practice of depositing and withdrawing money using
Abejuela's passbook continued for quite some time.
Thereafter, the bank's accountant and interest bookkeeper discovered a discrepancy between the
interest reconciliation balance and the subsidiary ledger balance. The bank officials confronted
Balo, who feigned ignorance and initially denied the accusations, but later admitted having posted
the false deposits. Petitioner Abejuela was also implicated because he was the owner of the
passbook used by Balo in accomplishing his fraudulent scheme.

ISSUE:
Whether or not Abejuela is an accomplice.

RULING:
No. Knowledge of the criminal intent of the principal is essential in order that petitioner Abejuela
can be convicted as an accomplice in the crime of estafa thru falsification of commercial document.
To be convicted as an accomplice, there must be cooperation in the execution of the offense by
previous or simultaneous acts. However, the cooperation which the law punishes is the assistance
rendered knowingly or intentionally, which assistance cannot be said to exist without the prior
cognizance of the offense intended to be committed.
Abejuela is acquitted. However, his acquittal will not extinguish his civil liability. Although
Abejuela, was unaware of the crime, he contributed by recklessly entrusting his passbook to Balo
and by signing the withdrawal slips. Abejuela failed to exercise prudence and care. Therefore, he
must be held civilly accountable.

381
CRIMINAL LAW 1 CASE DIGESTS
ATTY. LYAN DAVD JUANICO
BLOCK 1-M A.Y. 2020 - 2021

9. People v. Doble
G.R. No. L-30028, 31 May 1982
DE CASTRO, J.:

FACTS:
On June 13, 1966, 10 men sailed off on a motor banca to Navotas, Rizal and robbed the beach-
bank Prudential Bank& Trust Co. Prudential bank had a peculiar banking hours which open from
12 midnight to 8:00 AM. Only 8 men disembarked from the banca and went to rob the bank. The
8 men returned with P10,439.95 and left. Many were killed and injured as a result of the shooting.
Five out of ten men were brought to trial, the rest remain at large. Two of the five accused were
acquitted. Cresencio Doble, Simeon Doble, and Antonio Romaquin appealed in the charge of bank
robbery committed by band with multiple homicide, multiple frustrated homicide and assault upon
persons of authority.

ISSUE:
Whether the three accused are accomplices of the crime

RULING:
No. Only Romaquin and Cresencio are accomplices, Simeon Doble is not.
First, as to appellant Simeon Doble, evidence shows that the malefactors met in his house to discuss
the plan to rob the bank. The facts do not show that he performed any act tending to the perpetration
of the robbery, nor that he took a direct part therein or induced other persons to commit, or that he
cooperated in its consummation by some act without which it would not have been committed.
Simeon was not a principal both by agreement and encouragement for his non-participation in the
commission of the crime. Nor was it clearly proven that he had received fruits of the looted money
as to make him an accessory. Simeon Doble is entitled to acquittal with no sufficient evidence to
establish his guilt beyond reasonable doubt.
Second, with regard to the liability of Romaquin and Cresencio, the two men who waited in the
banca, it shows that their liability is less than that of a co-principal conspiracy or by actual
participation. Furthermore, it wasn’t established by evidence that in the meeting held in the house
of Simeon that they all agreed to kill and not just rob. The finding that appellants are liable as mere
accomplices may appear too lenient but evidence fails to establish their conspiracy with the real
malefactors who actually robbed the bank and killed several people. Hence, Cresencio and
Romaquin are guilty for the crime of robbery in band.

382
CRIMINAL LAW 1 CASE DIGESTS
ATTY. LYAN DAVD JUANICO
BLOCK 1-M A.Y. 2020 - 2021

10. People v Doctolero


G.R. No. 34386, 7 February 1991
REGALADO, J.:

FACTS:
On the evening of November 8, 1970, Epifania Escosio and Lolita de Guzman were killed in the
house of Marcia Sagun in Sitio Binday, municipality of San Fabian, province of Pangasinan, where
they were living. Jonathan Oviedo, 1 1/2 year old child of Lolita de Guzman, was on the same
occasion, slightly injured while being fed on the breast of his mother.
The malefactors were identified Ludovico, Conrado and Virgilio, all surnamed Doctolero. This
review was for Conrado’s liability only since Virgilio did not appeal and later on died.

ISSUE:
Whether Conrado is an accomplice

RULING:
Yes. Conrado together with his brother, Virgilio, stood by as their brother murder Epifania and
Lolita.Their presence of gave their brother Ludovico the encouragement and reliance to proceed
as he did proceed. It has been held that where one goes with the principals, and in staying outside
of the house while the others went inside to rob and kill the victim, the former effectively supplied
the criminals with material and moral aid, making him guilty as an accomplice.
Conrado and Virgilio contended that Ludovico killed them out of temper because Epifania slapped
his neck, thus negating a common criminal design in their minds. This contention is wrong. One
can be an accomplice even if he did not know of the actual crime intended by the principal provided
he was aware that it was an illicit act.

383
CRIMINAL LAW 1 CASE DIGESTS
ATTY. LYAN DAVD JUANICO
BLOCK 1-M A.Y. 2020 - 2021

11. People v De Vera


G.R. No. 128966, 18 August 1999
PANGANIBAN, J.:

FACTS:
Edwin de Vera, together with Roderick Garcia, Kenneth Florendo and Elmer Castro, was charged
with Murder before the Regional Trial Court of Quezon City in connection with the killing of one
Frederick Capulong. De Vera and Garcia "leaded not guilty during arraignment. The other two
accused, Florendo and Castro, were at large. During trial, the prosecution "resented as witness one
Bernardino Cacao who testified that he saw De Vera in the car, where an altercation later occurred.
Thereafter, he saw Florendo drag out of the vehicle apparently disabled Capulong and shot him in
the head moments later.
Furthermore, the trial court found that it was indeed Florendo who actually shot the victim.
However, it convicted De Vera as a principal because the scientific and forensic findings on the
criminal incident directly and substantially confirmed the existence of conspiracy among the four
accused. Aggrieved, de Vera appealed his conviction before the Supreme Court.

ISSUE:
Whether the trial court erred in convicting De Vera as an accomplice

RULING:
Yes. The testimony of the prosecution eyewitness contained nothing that could inculpate De Vera.
Aside from the fact that he was inside the car, no other act was imputed to him. Mere presence
does not amount to conspiracy. Indeed, the trial court based its finding of conspiracy on mere
presumptions, and not on solid facts indubitably indicating a common design to commit murder.
Such suppositions do not constitute proof beyond reasonable doubt.
The fact that De Vera was at the locus criminis in order to aid and abet the commission of the
crime did not make him a conspirator; at most, he was only an accomplice. Moreover, the
prosecution evidence has not established that De Vera was "art of the conspiracy to kill Capulong.
De Vera ’s participation, as culled from his own statement, was made after the decision to kill was
already a fait accompli.

384
CRIMINAL LAW 1 CASE DIGESTS
ATTY. LYAN DAVD JUANICO
BLOCK 1-M A.Y. 2020 - 2021

12. Garces v People


G.R. No. 173858. July 17, 2007
PANGANIBAN, J.:

FACTS:
On August 2, 1992, between 8:00 and 9:00 o'clock in the evening, AAA was on her way to the
chapel when the five accused suddenly appeared and approached her. Rosendo Pacursa covered
her mouth with his hands and told her not to shout or she will be killed. He then brought her inside
a nearby tobacco barn while his four companions stood guard outside. He succeeded to have carnal
knowledge with AAA.
In this appeal, Garces contends that he should be acquitted along with Aurelio and Antonio Pira.
However, in the case of petitioner, complainant positively identi�ed him as one of the companions
of Pacursa who remained outside the barn and who eventually entered upon noting the presence
of AAA's relatives nearby. He thereafter covered complainant's mouth and led her out of the barn.
All these circumstances demonstrate petitioner's complicity.

ISSUE:
Whether or not Garces is an accessory.

RULING:
No. Garces is an accomplice.
The facts show that petitioner participated in the commission of the crime even before complainant
was raped. He was present when Pacursa abducted complainant and when he brought her to the
barn. He positioned himself outside the barn together with the other accused as a lookout. When
he heard the shouts of people looking for complainant, he entered the barn and took complainant
away from Pacursa.
Having known of the criminal design and thereafter acting as a lookout, petitioner is liable as an
accomplice, there being insufficient evidence to prove conspiracy, and not merely as an accessory.
As de�fined in the Revised Penal Code, accomplices are those who, not being included in Article
17, cooperate in the execution of the offense by previous or simultaneous acts. The two elements
necessary to hold petitioner liable as an accomplice are present: (1)community of criminal design,
that is, knowing the criminal design of the principal by direct participation, he concurs with the
latter in his purpose; and (2) performance of previous or simultaneous acts that are not
indispensable to the commission of the crime.

385
CRIMINAL LAW 1 CASE DIGESTS
ATTY. LYAN DAVD JUANICO
BLOCK 1-M A.Y. 2020 - 2021

13. People v Talingdan


L-32126, 6 July 1978, 84 SCRA 19
PER CURIAM:

FACTS:
Armed with long guns, the four male accused gunned down Bernardo from below the "batalan" of
his house as he was sitting by the supper table and his twelve-year old daughter Corazon was
watching him nearby. The accused then climbed the stairs and seeing Bernardo still alive, accused
Talingdan and Tobias fired at him again. Corazon tried to call for help but Bides threatened to kill
her. The assailants then fled. Corazon recognized and knew the four as they were residents of their
barrio, but her mother, Teresa, who came out of their "silid" after the shooting, warned Corazon
not to tell anyone that she recognized her father's killers threatening to kill her if she did. When
peace officers repaired to their house to investigate what happened, Teresa claimed that she had
no suspects in mind.
Teresa was known to have illicit relations with Talingdan and prior to this incident had been seen
by her daughter Corazon meeting with the other accused on two occasions. Judgment a�ffirmed
except that the four male appellants were sentenced to death and appellant Teresa was convicted
only as an accessory to the crime.

ISSUE:
Whether or not Teresa is an accessory.

RULING:
Yes. Teresa is an accessory. There is in the record morally convincing proof that she is at the very
least an accessory to the offense committed by her co-accused. She was inside the room when her
husband was shot. As she came out after the shooting, she inquired from Corazon if she was able
to recognize the assailants of her father. When Corazon identi�ed appellants Talingdan, Tobias,
Berras and Bides as the culprits, Teresa did not only enjoin her daughter not to reveal what she
knew to anyone, she went to the extent of warning her.
Later, when the peace officers who repaired to their house to investigate what happened, instead
of helping them with the information given to her by Corazon, she claimed she had no suspects in
mind. In other words, whereas, before the actual shooting of her husband, she was more or less
passive in her attitude regarding her co-appellants' conspiracy, known to her, to do away with him,
after Bernardo was killed, she became active in her cooperation with them. These subsequent acts
of her constitute "concealing or assisting in the escape of the principal in the crime" which makes
her liable as an accessory after the fact under paragraph 3 of Article 19 of the Revised Penal Code.

386
CRIMINAL LAW 1 CASE DIGESTS
ATTY. LYAN DAVD JUANICO
BLOCK 1-M A.Y. 2020 - 2021

14. Dizon-Pamintuan v. People


G.R. No. 111426, 11 July 1994
DAVIDE, JR., J.:

FACTS:
On February 12, 1988,Teodoro Encarnacion, Undersecretary for the DPWH testified that he just
arrived at his residence coming from the airport and immediately proceeded inside his house. As
the helpers were picking up his belongings, five unidentified masked armed persons poked their
guns to the driver, helpers and Teodoro. All of them were made to lie face down on the floor.
Thereafter, the robbers looted the house and took away jewelries, cash, and other personal
properties. The matter was immediately reported to the police after the intruders left. Teodoro was
later informed that some of the lost items were in the Chinatown area. With their participation, an
entrapment was made and the spoused posed as buyers and they identified the jewelry stolen
displayed at the stall tended by the accused (now petitioner) and the latter was brought to the
precinct for investigation.

The lower court court ruled that the petitioner should not incur civil liability because as testified
by Teodoro, the petitioner admitted that she got the items but she did not know they were stolen
and that she surrendered the items and gave them to Teodoro’s wife. Further, the petitioner failed
to rebut the presumption of Fencing under Sec.5 of P.D 1612. The appellate court affirmed the
decision of the trial court but set aside the penalty imposed.

ISSUE:
Whether or not the petitioner is liable for violation of the Anti-Fencing Law

RULING:

Yes. The court held that the presumption is reasonable for no other natural or logical inference can
arise from the established fact of her possession of the proceeds of the crime of robbery or theft.

One is deemed to know a particular fact if he has the cognizance, consciousness or


awareness thereof, is aware of the existence of something, or has the acquaintance with facts, or
if he has something within the mind’s grasp with certitude and clarity.

Since the court cannot penetrate the mind of an accused and state with certainty what is contained
therein, it must determine such knowledge with care from the overt acts of that person. And given
two equally plausible states of cognition or mental awareness, the court should choose the one
which sustains the constitutional presumption of innocence.

387
CRIMINAL LAW 1 CASE DIGESTS
ATTY. LYAN DAVD JUANICO
BLOCK 1-M A.Y. 2020 - 2021

Moreover, Section 5 of P.D. No. 1612 expressly provides that “mere possession of any good,
article, item, object, or anything of value which has been the subject of robbery or thievery shall
be prima facie evidence of fencing. It follows that the petitioner is presumed to have knowledge
of the fact that the items found in her possession were the proceeds of robbery or theft.

In the case at bar, the petitioner solely relied on the testimony of her brother which was insufficient
to overcome the presumption and her brother even disclosed that the petitioner was engaged in the
jewelry business and that she used to acquire jewelry from a certain Fredo.

The accessory in the crimes of robbery and theft could be prosecuted as such under the Revised
Penal Code or under P.D 1612. In P.D 1612 he becomes a principal, Hence the State may choose
to prosecute him either under the Revised Penal Code or P.D. No. 1612, although the preference
for the latter would seem inevitable considering that fencing is a malum prohibitum, and P.D.
No. 1612 creates a presumption of fencing and prescribes a higher penalty based on the value of
the property.

388
CRIMINAL LAW 1 CASE DIGESTS
ATTY. LYAN DAVD JUANICO
BLOCK 1-M A.Y. 2020 - 2021

MODULE 10: PLULARITY OF CRIMES

1. People v. Tabaco
G.R. No. 100382, 19 March 1997
HERMOSISIMA, JR., J.:

FACTS:
The accused-appellant, Mario Tabaco was charged with four counts of murder in four related
informations. On the evening of March 22, 1987, the group of Ex- Mayor Jorge Arreola arrived at
the cockpit arena in Aparri, Cagayan which held a cock derby sponsored by the 117th PC battalion
stationed at the same place. The group was seated on the 4th row of the north western part of the
arena while the accused was seated at the lower portion of the arena with a distance of about 4-5
meters. At about 10 o’clock, the accused-appellant without warning or provocation, using his rifle,
shot the mayor followed by a successive burst of gunfire which resulted in the death of the late
Mayor and three others.

The cases were prosecuted under one information and the trial court ruled that the four victims
were gun downed by successive/continuous gun fires. Thus, the trial court found the accused-
appellant guilty of a complex crime under Art. 248 of the revised penal code and sentenced him to
suffer the penalty of reclusion perpetua, in its maximum period. The lower court held:

“Paraphrasing a more recent decision of the Supreme Court, we say — as the deaths of Oscar
Tahulug, Jorge Arreola, Felicito Rigunan and Romeo Regunton, in Criminal Cases Nos. 259, 270,
284 and 317 respectively, were the result of one single act of the accused Mario Tabaco, the penalty
— is the penalty imposed for the more serious offense. The more serious offense is murder, the
killing have been attended by TREACHERY because the victims were completely taken by
surprise and had no means of defending themselves against Mario Tabaco's sudden attack. The
penalty is imposable in its maximum degree, but as the death penalty is no longer permitted the
same is hereby reduced to a single Penalty of RECLUSION PERPETUA for the four (4) murders.”

ISSUE:
Whether or not the crime committed is a complex crime

RULING:

No. The court ruled that the trial court erred in imposing only a single penalty of reclusion
perpetua. Furthermore, the trial court erred in appreciating the facts in People v. Dama. In the said

389
CRIMINAL LAW 1 CASE DIGESTS
ATTY. LYAN DAVD JUANICO
BLOCK 1-M A.Y. 2020 - 2021

case, one bullet killed two persons. Only a single act produced two crimes, resulting in a compound
crime.

In the case at bar, the weapon used was a submachine gun. There was more than one bullet that
killed the victims. When the firearm was recovered from the accused-appellant, the magazine that
held 20 bullets was empty and several empty shells were recovered from the crime scene.
Therefore the ruling in the aforementioned case cannot be applied.

In People vs. Desierto. The accused in that case killed five persons with a Thompson sub- machine
gun, an automatic firearm which, like the M-14, is capable of firing continuously. As stated
therein:

“In the case at bar, Article 48 of the Revised Penal Code is not applicable because the death of
each of the five persons who were killed by appellant and the physical injuries inflicted upon each
of the two other persons injured were not caused by the performance by the accused of one simple
act as provided for by said article. Although it is true that several successive shots were fired by
the accused in a short space of time, yet the factor which must be taken into consideration is that,
to each death caused or physical injuries inflicted upon the victims, corresponds a distinct and
separate shot fired by the accused, who thus made himself criminally liable for as many offenses
as those resulting from every single act that produced the same. Although apparently he perpetrated
a series of offenses successively in a matter of seconds, yet each person killed and each person
injured by him became the victim, respectively, of a separate crime of homicide or frustrated
homicide. Except for the fact that five crimes of homicide and two cases of frustrated homicide
were committed successively during the tragic incident, legally speaking there is nothing that
would connect one of them with its companion offenses. (emphasis ours)”

In that case, although the burst of shots was caused by one single act of pressing the trigger of the
thompson submachine gun, in view of its special mechanism, the person firing it has only to keep
pressing the trigger with his finger and it would fire continually. Thus, it is not the act of pressing
the trigger which should produce the several felonies, but the number of bullets which actually
produced them.

Therefore, the four deaths which resulted from a burst of gunfire cannot be considered a complex
crime. They are separate crimes.The accused-appellant must therefore be held liable for each and
every death he has caused, and sentenced accordingly to four sentences of reclusion perpetua.

390
CRIMINAL LAW 1 CASE DIGESTS
ATTY. LYAN DAVD JUANICO
BLOCK 1-M A.Y. 2020 - 2021

2. People v. Valdez
G.R. No. 127663, 11 March 1999
MELO, J.:

FACTS:
At around 8 o’clock in the evening, the victims, Ramon Garcia, Jean Marie Garcia, Sandra
Montano, Willie Acosta, Willam Montano and Randy Tibule were at the house of the latter in
Manaoag, Pangasinan, discussing the wedding party of Jean Marie’s cousin. Thereafter, they rode
a tricycle going to Sitio Cabaoangan. As they were approaching Sito Cabaoangan, they
encountered the accused-appellant Rolando Valdez and his companions. When the tricycle’s
headlight flashed on their faces, they pointed their guns and fired at the tricycle resulting in the
deaths of the victims but William Montano and Randy Tibule survived.

The trial court found the accused-appellant guilty of the complex crime of multiple murder with
double frustrated murder and sentenced him to death.

ISSUE:
Whether or not the accused should be convicted of the complex crime of four counts of murder
and two counts of frustrated murder

RULING:
Yes. The court ruled that the trial court erred when it allowed itself to be carried away by the
erroneous information filed charging the complex crime of multiple murder and double frustrated
murder. Also, the court held that the case at bar does not fall under any of the instances defined in
Art. 48 of the Revised Penal Code which states:

ART. 48. Penalty for complex crimes.—When a single act constitutes two or more grave or less
grave felonies or when an offense is a necessary means for committing the other, the penalty for
the most serious crime shall be imposed, the same to be applied in its maximum period.

Furthermore, based from the evidence, the four murders did not result from a single act but from
several individual and distinct acts. Since there was more than one gunman involved, the act of
each is distinct from that of the other. Each act by each gunman in pulling the trigger of their
respective firearms, aiming each particular moment at different persons constitute distinct and
individual acts which cannot give rise to the complex crime of multiple murder. Thus, the court
ruled that the accused-appellant is guilty of four counts of murder for the death of the four victims
and the two counts of frustrated murder.

391
CRIMINAL LAW 1 CASE DIGESTS
ATTY. LYAN DAVD JUANICO
BLOCK 1-M A.Y. 2020 - 2021

3. People v. Sanchez
G.R No. 131116, 27 August 1999
PARDO, J.:

FACTS:
At around 10 in the morning of April 13th, 1991, state witness Vivencio Malabanan was a team
leader of a group of policemen who went to the Bishop Compound in Calauan, Laguna as part of
the security force of then mayor Antonio Sanchez (Mayor). After a while, accused Peradillas
arrived and informed the mayor that there would be a birthday party that night at Dr. Velecina’s
house, who was a political opponent of the mayor. Peradillas assured the mayor of Nelson
Peñalosa’s (victim) presence at the said party. The Mayor told them "Bahala na kayo mga anak.
Ayusin lang ninyo ang trabaho," and left the premises. Thereafter, Peradillas called Corcolon and
Averion and relayed the order to kill Peñalosa, who was one of the political leaders of Dr.
Velecina.

After making arrangements to acquire two-way radios and a vehicle, Malabanan along with the
three accused went their separate ways and agreed to meet at the Mayor’s house. At around 7 in
the evening, Malabanan and the three accused went to a poultry farm near Dr. Velecina’s house.
Peradillas then walked to his house in order to confirm if the victim was present at the party.
Afterwards, using the two-way radio, Peradillas informed Malabanan and the others that
Peñalosa’s jeep was leaving the compound. The group then pursued the jeep, and as they were
passing Victoria farms, Corcolon ordered Averion to overtake the jeep and as they overtook the
jeep, Peradillas and Corcolon fired their rifles in automatic mode. The son of Peñalosa fell off the
jeep. However, the jeep continued running in a zigzag motion until it overturned in front of another
farm. After the incident the accused proceeded to the mayor’s house and reported that the victim
was dead.

The accused interposed the defense of alibi and denial. However, the trial court ruled that the
prosecution’s evidence clearly and convincingly established the participation of the four accused
in the killing. Moreover, the trial court ruled that there was treachery, which qualified the crime to
murder and that the accused conspired in the committing the crime. The trial court also appreciated
the aggravating circumstances of evident premeditation, nighttime and use of a motor vehicle.

The trial court considered the crime as a complex crime of double murder. However, at the time
of the commission of the offense on April 13, 1991, there was a constitutional prohibition on the
imposition of the death penalty. Thus, each of the accused were sentenced to reclusion perpetua,
and ordered to pay damages to the heirs of the victims.

392
CRIMINAL LAW 1 CASE DIGESTS
ATTY. LYAN DAVD JUANICO
BLOCK 1-M A.Y. 2020 - 2021

ISSUE:

Whether or not the crime committed is a complex crime of double murder

RULING:

No. The court ruled that the accused are criminally liable for two counts of murder.

Article 48 of the Revised Penal Code provides that when a single act constitutes two or more grave
or less grave felonies, or when an offense is a necessary means of committing the other, the penalty
for the more serious crime in its maximum period shall be imposed.

The court ruled in People v. Vargas, Jr. that "several shots from a Thompson sub-machine, in view
of its special mechanism causing several deaths, although caused by a single act of pressing the
trigger, are considered several acts. Although each burst of shots was caused by one single act of
pressing the trigger of the submachine gun, in view of its special mechanism the person firing it
has only to keep pressing the trigger of the submachine gun, with his finger and it would fire
continually. Hence, it is not the act of pressing the trigger which should be considered as producing
the several felonies, but the number of bullets which actually produced them."

In the case at bar, based from Malabanan’s testimony, he stated that he heard three bursts of gunfire
from two armalites used by Corcolon and Peradillas. Hence, the accused are criminally liable for
as many offenses resulting from pressing the trigger of the armalites.

Therefore, the accused are liable for two counts of murder committed against the victims, Nelson
Peñalosa and his son, instead of the complex crime of double murder.

393
CRIMINAL LAW 1 CASE DIGESTS
ATTY. LYAN DAVD JUANICO
BLOCK 1-M A.Y. 2020 - 2021

4. People v. Nelmida
G.R. No. 184500, 11 September 2012
PEREZ, J.:

FACTS:
On June 5, 2001, the Mayor of Salvador, Lanao del Norte and his escorts composed of some
members of the Philippine Army, PNP and civilian aides were in Tubod, Lanao del Norte. They
returned to Salvador in the afternoon. All of them rode in the yellow pick-up truck service vehicle
of the mayor, who was sitting at the passenger seat of the said vehicle. At around 3pm of the same
day, the accused-appellants together with their co-accused, brought Samuel (witness) to a waiting
shed located at the left side of the road going to Salvador, Lanao del Norte. Samuel was instructed
to wait in the waiting shed while the malefactors positioned themselves on both sides of the road.
When the yellow pick-up truck passed by the waiting shed, the malefactors, using their high-
powered firearms, immediately opened fired and rained bullets on the said vehicle which resulted
in the death of two of the security escorts while the others suffered injuries. The Mayor ordered
the driver to continue driving in order to avoid more casualties. They stopped upon reaching a
CAFGU detachment in Salvador.

After the ambush, the accused-appellants and their co-accused fled to the house of Samuel’s aunt
to get their bags. Afterwards, the malefactors hurriedly ran towards another municipality.

After their arrest, the accused-appellants interposed the defense of alibi and denial. However, the
trial court, after finding the testimonies of the prosecution witnesses, most of whom were victims
of the ambush, to be credible, categorical, straightforward, spontaneous and consistent, coupled
with their positive identification of the appellants as among the perpetrators of the crime and their
lack of ill-motive to falsely testify against them, vis-à-vis the defense of denial and alibi presented
by the latter, the trial court rendered its decision applying Art.48 of the RPC finding the appellants
guilty of double murder with multiple frustrated murder and double attempted murder and imposed
the penalty of reclusion perpetua. The CA affirmed the same.

ISSUE:
Whether or not the accused-appellants are guilty of a complex crime under Art. 48 of the RPC

RULING:
No. The court affirmed the decision of the lower court in finding the accused-appellants guilty,
however the court ruled that the lower court erred in convicting them of a complex crime. The
court said that the accused-appellant should be convicted not of a complex crime but of
separate crimes of two counts of murder and seven counts of attempted murder as the killing and

394
CRIMINAL LAW 1 CASE DIGESTS
ATTY. LYAN DAVD JUANICO
BLOCK 1-M A.Y. 2020 - 2021

wounding of the victims in this case were not the result of a single act but of several acts of the
appellants, thus, making Article 48 of the Revised Penal Code inapplicable.

Art. 48 states provides:

ARTICLE 48. Penalty for Complex Crimes. — When a single act constitutes two or more crimes,
or when an offense is a necessary means for committing the other, the penalty for the most serious
crime shall be imposed, the same to be applied in its maximum period.

In a complex crime, two or more crimes are actually committed, however, in the eyes of the law
and in the conscience of the offender they constitute only one crime, thus, only one penalty is
imposed. There are two kinds of complex crime. The first is known as compound crime, or when
a single act constitutes two or more grave or less grave felonies while the other is known as
complex crime proper, or when an offense is a necessary means for committing the other. The
classic example of the first kind is when a single bullet results in the death of two or more persons.
A different rule governs where separate and distinct acts result in a number killed.

In the case at bar, it can easily be gleaned that the killing and wounding of the victims were not
the result of a single discharge of firearms by the appellants and their co-accused. Accused-
appellants and their co-accused opened fire and rained bullets on the vehicle boarded by the Mayor
and his security escorts, which resulted in the death of two people. Moreover, more than one
gunman fired at the vehicle of the victims. As held in People v.Valdez each act by each gunman
pulling the trigger of their respective firearms, aiming each particular moment at different persons
constitute distinct and individual acts which cannot give rise to a complex crime. The accused-
appellants and their co-accused performed not only a single act but several individual and distinct
acts in the commission of the crime. Therefore, Article 48 of the RPC would not apply for speaks
only of a single act.

395
CRIMINAL LAW 1 CASE DIGESTS
ATTY. LYAN DAVD JUANICO
BLOCK 1-M A.Y. 2020 - 2021

5. People vs. Hernandez


G.R. Nos. L-6025-26, 18 July 1956
CONCEPCION, J.:

FACTS:
The accused, along with thirty one defendants were charged with rebellion with multiple murder,
arsons, and robberies. The said accused and their co-conspirators were officers and/or members
of, or otherwise associated the Congress of Labor Organizations (CLO), which was an agency,
organ and instrumentality of the Communist Party of the Philippines (P.K.P), which have
synchronized their activities and fully cooperates in the rebellious activities of the Hukbong
Magpalayang Bayan (HMB), formerly known as the “Hukbalahaps”. The HMB have risen
publicly and take arms against the Republic of the Philippines, for the purpose of removing the
country of its allegiance to the government. To attain the said purpose, the HMB made armed raids,
ambushes and attack against police, army detachments, innocent civilians, and as a necessary
means to commit the crime of rebellion, in connection and in furtherance thereof, have committed
acts of murder, pillage, looting, plunder, arson, and planned destruction of private and public
property to create and spread chaos, disorder, terror, and fear so as to facilitate the accomplishment
of their purpose.

The prosecution maintains that accused-appellant is charged with, and has been convicted of,
rebellion complexed with murders, arsons and robberies, for which the capital punishment, it is
claimed, may be imposed, although the lower court sentenced him merely to life imprisonment.
On the other hand, the defense asserts that rebellion cannot be complexed with murder, arson, or
robbery.

ISSUE: Whether or not the crime of rebellion may be complexed with murder, arson or robbery
pursuant to Art. 48 of the Revised Penal Code

RULING:
No. The court ruled that Art. 48 of the RPC cannot be applied.

Article 48 of the Revised Penal Code provides:

“When a single act constitutes two or more grave or less grave felonies, or when an offense is a
necessary means for committing the other, the penalty for the most serious crime shall be imposed,
the same to be applied in its maximum period.”

396
CRIMINAL LAW 1 CASE DIGESTS
ATTY. LYAN DAVD JUANICO
BLOCK 1-M A.Y. 2020 - 2021

The language of Art. 48 requires the commission of two or more crimes, thus it does not apply
when the culprit is guilty of only one crime. Moreover, under the allegations of the amended
information, the murders, arsons and robberies described therein are mere ingredients of the crime
of rebellion allegedly committed by the accused and as a necessary means for the perpetration of
said offense of rebellion. Hence, the crime charged in the amended information simple rebellion,
not the complex crime of rebellion with multiple murder, arsons and robberies.

397
CRIMINAL LAW 1 CASE DIGESTS
ATTY. LYAN DAVD JUANICO
BLOCK 1-M A.Y. 2020 - 2021

6. Enrile vs. Salazar


G.R. No. 92163, 5 June 1990
NARVASA, J.:

FACTS:
On February 27,1990, Senate Minority Leader Juan Ponce Enrile (herein Senator) was arrested by
virtue of a warrant of arrest issued on an information charging him with the crime of rebellion with
murder and multiple frustrated murder allegedly committed during the period of the failed coup
attempt from November 29 to December 10, 1990. Thereafter, the Senator was held overnight at
the NBI headquarters, without bail and none having been recommended in the information and
none fixed in the arrest warrant. He was then brought to Camp Karingal the following day.

On the same day, the Senator, through his counsel, filed a petition for habeas corpus, alleging that
he was deprived of his constitutional rights.

The Court issued the writ prayed for and set the plea hearing on March 6, 1990. On March 5th, the
Solicitor General filed a consolidated return for the respondents in this case and in another case
which had been filed separately by two of the co-accused and raised similar questions. The return
asserts that the case does not fall within the Hernandez ruling because in Hernandez charged
murders and other common crimes committed as a necessary means for the commission of
rebellion, whereas the information against Senator Enrile charged murder and frustrated murder
committed on the occasion, but not in furtherance, of rebellion.

ISSUE:
Whether or not Senator Enrile has committed a complex crime arising from an offense being a
necessary means for committing another, which is referred to in the second clause of Art. 48 of
the RPC

RULING:
No. The Court unanimously voted to reject the theory that Hernandez is, or should be, limited in
its application to offenses committed as a necessary means for the commission of rebellion and
that the ruling should not be interpreted as prohibiting the complexing of rebellion with other
common crimes committed on the occasion, but not in furtherance thereof. The ruling in the
Hernandez case provides:

There is one other reason-and a fundamental one at that-why Article 48 of our Penal Code cannot
be applied in the case at bar. If murder were not complexed with rebellion, and the two crimes
were punished separately (assuming that this could be done), the following penalties would be

398
CRIMINAL LAW 1 CASE DIGESTS
ATTY. LYAN DAVD JUANICO
BLOCK 1-M A.Y. 2020 - 2021

imposable upon the movant, namely: (1) for the crime of rebellion, a fine not exceeding P20,000
and prision mayor, in the corresponding period, depending upon the modifying circumstances
present, but never exceeding 12 years of prision mayor, and (2) for the crime of murder, reclusion
temporal in its maximum period to death, depending upon the modifying circumstances present.
in other words, in the absence of aggravating circumstances, the extreme penalty could not be
imposed upon him. However, under Article 48 said penalty would have to be meted out to
him, even in the absence of a single aggravating circumstance. Thus, said provision, if construed
in conformity with the theory of the prosecution, would be unfavorable to the movant.

Moreover, the court stated in the Hernandez case that “When two or more crimes are the result of
a single act, the offender is deemed less perverse than when he commits said crimes thru separate
and distinct acts. Instead of sentencing him for each crime independently from the other, he must
suffer the maximum of the penalty for the more serious one, on the assumption that it is less grave
than the sum total of the separate penalties for each offense.” Thus, Senator Enrile is liable only
for Simple Rebellion.

399
CRIMINAL LAW 1 CASE DIGESTS
ATTY. LYAN DAVD JUANICO
BLOCK 1-M A.Y. 2020 - 2021

7. People v. Garcia
G.R. No. 141125, 28 February 2002
PER CURIAM

FACTS:
Cleopatra Changlapon, was nineteen years old. She left school at 6:30pm to go home as she was
crossing the street at Baguio City, a white van stopped in front of her and pulled her inside the van.
Something was sprayed on her that made her eyes sting dizzy. As she receivedblows from the
accused-appellants she fell unconscious. When she regained consciousness, she noticed that she
was already undressed and lying flat on a bed. In the room with her were four men. Afterwhich,
four men successfully had sexual intercourse with Cleopatra. As the fourth men done raping her,
she felt dizzy. Then again, they boxed her and she blacked out. When she regained consciousness,
she was lying by the roadside and felt pain all over her body. A taxi passed by and brought her to
her house. After she regained composure, she told that she had been raped by four men. Hence this
petition.

ISSUE:
Whether or not the court erred in finding the accused-appellant guilty of complex crime of forcible
abduction with rape and for three counts of rape

RULING:
No. the court did not erred in finding the accused guilty. In the case at bar, the information
sufficiently alleged the elements of forcible abduction, i.e., the taking of complainant against her
will and with lewd design. It was likewise alleged that accused-appellant and his three co-accused
conspired, confederated and mutually aided one another in having carnal knowledge of
complainant by means of force and intimidation and against her will. Aside from alleging the
necessary elements of the crimes, the prosecution convincingly established that the carnal
knowledge was committed through force and intimidation. Moreover, the prosecution sufficiently
proved beyond reasonable doubt that accused-appellant succeeded in forcibly abducting the
complainant with lewd designs, established by the actual rape. Also, there can only be one complex
crime of forcible abduction with rape. The crime of forcible abduction was only necessary for the
First rape. Thus, the subsequent acts of rape can no longer be considered as separate complex
crimes of forcible abduction with rape. They should be detached from and considered
independently of the forcible abduction. Therefore, accused-appellant should be convicted of one
complex crime of forcible abduction with rape and three separate acts of rape. And the proper
penalty for complex crime is the penalty for the most serious crime which shall be imposed in the
maximum period.

400
CRIMINAL LAW 1 CASE DIGESTS
ATTY. LYAN DAVD JUANICO
BLOCK 1-M A.Y. 2020 - 2021

8. Batulanon v. People
G.R. No. 139857, 15 September 2006
YNARES-SANTIAGO||

FACTS:
Complainant Polomolok Credit Cooperative Incorporated (PCCI) employed Batulanon as its
Cashier/Manager from May 1980 up to December 22, 1982. She was in charge of receiving
deposits from and releasing loans to the member of the cooperative. she signed the loan application
and voucher of her son Dennis Batulanon because he was a minor but she clarified that she asked
Gopio, Jr.(certain individual that signature needed before the release of vouchers), to add his
signature on the documents to avoid suspicion of irregularity; that contrary to the testimony of
Gopio, Jr., minors are eligible for membership in the cooperative provided they are children of
regular members.

Batulanon admitted that she took out a loan in her son's name because she is no longer qualified
for another loan as she still has to pay off an existing loan; that she had started paying off her son's
loan but the cooperative refused to accept her payments after the cases were filed in court. She
also declared that one automatically becomes a member when he deposits money with the
cooperative. During an audit conducted in December 1982, certain irregularities concerning the
release of loans were discovered. Thereafter, four informations for estafa thru falsification of
commercial documents were filed against Batulanon.

ISSUE:
Whether or not Batulanon is liable for the crime estafa thru falsification of commercial documents

RULING:
The Court of Appeals correctly ruled that the subject vouchers are private documents and not
commercial documents because they are not documents used by merchants or businessmen to
promote or facilitate trade or credit transactions nor are they defined and regulated by the Code
of Commerce or other commercial law. Rather, they are private documents, which have been
defined as deeds or instruments executed by a private person without the intervention of a public
notary or of other person legally authorized, by which some disposition or agreement is proved,
evidenced or set forth.

In the case at bar, Batulanon for falsifying Dennis Batulanon's signature in the cash voucher based
on the Information charging her of signing the name of her 3 year old son, Dennis. The records,
however, reveal that in Cash Voucher No. 374A, petitioner Batulanon did not falsify the signature
of Dennis. What she did was to sign: "by: lbatulanon" to indicate that she received the proceeds of

401
CRIMINAL LAW 1 CASE DIGESTS
ATTY. LYAN DAVD JUANICO
BLOCK 1-M A.Y. 2020 - 2021

the loan in behalf of Dennis. Said act does not fall under any of the modes of falsification under
Article 171 because there in nothing untruthful about the fact that she used the name of Dennis
and that as representative of the latter, obtained the proceeds of the loan from PCCI. The essence
of falsification is the act of making untruthful or false statements, which is not attendant in this
case. As to whether, such representation involves fraud which caused damage to PCCI is a different
matter which will make her liable for estafa, but not for falsification. Therefore, the disturbance in
property rights caused by Batulaono's misappropriation is in itself sufficient to constitute injury
within the meaning of Article 315 or Estafa.

402
CRIMINAL LAW 1 CASE DIGESTS
ATTY. LYAN DAVD JUANICO
BLOCK 1-M A.Y. 2020 - 2021

24. People v. Madrigal-Gonzales


G.R. No.L-16688, 30 April 1963
PAREDES, J |||

FACTS:
The accused-appellee was charged together with her 7 co-accused with the crime of fasification of
public documents under 27 separate informations. Cash aids were given when no such aide were
indeed distributed to the persons named and at the time and place and in such amounts specified,
or by making and/or causing it to appear that certain relief supplies were purchased by the accused.
The distribution of 27 falsification cases before the 18 branches of the lower court, accused-
appelles filed motion to quash 3 criminal cases against them on grounds of: double jeopardy and
that the 27 separate informations for falsification constitute of product of one single criminal
impulse or intent, and the same-that the charges are “related to similar if not the same transactions
and the evidence of the prosecution being in great part the same for all said charges- are therefore,
in the nature of a continuing offense which should be alleged and prosecuted only under one
information

ISSUE:
Whether or not the twenty-seven (27) separate informations falsifications are continuing offense

RULING:
No, the 27 separate information falsifications are not continuing offenses. The Court cannot
assume that the purpose of committing the twenty-seven (27) falsifications was to conceal the
malversation. This is so because there is no showing that for every particular amount they had
malversed on a certain period, they had purposely perpetrated the corresponding falsification, to
cover up such amount, until the whole amount proposed to be malversed, shall have been
completely misappropriated. In the absence of such showing, it is to be presumed that in the
falsification of each document, the criminal intent was separate and distinct.

In the case at bar, the informations show different vouchers, dates and amounts. These undeniable
facts, alleged in the informations, evidently show that different acts of falsification were committed
on different vouchers and covering distinct amounts. Each information did not refer to all said acts
of falsification. Neither is there merit in the argument that said acts of falsification constituted a
continuing offense, so as to have them all prosecuted in only one information. Therefore, Each
falsification and each malversation constitute independent offenses which must be punished
separately. The acts being independent from each other and executed by different voluntary
actions, each constitutes an independent offense.

403
CRIMINAL LAW 1 CASE DIGESTS
ATTY. LYAN DAVD JUANICO
BLOCK 1-M A.Y. 2020 - 2021

10. Gamboa v. Court of Appeals


G.R. No. L-41054, 28 November 1975
MARTIN, J.

FACTS:
Private respondent was a former employee of petitioner company in its optical suppy business at
Sta. Cruz, Manila. 124 complaints of estafa filed against private respondent. After the procedural
preliminary investigation, the office of the city fiscal filed 75 cases of estafa against private
respondent. Having collected and received from customers of the said company the sum of in
payment for goods purchased from it, he did then and there misappropriate, misapply and convert
the said sum to his own personal use and benefit by depositing the said amount in his own name
and personal account with the Associated Banking Corporation- Hayco duped Lu Chiong Sun into
affixing his signature and thumbprint on a general power of attorney that caused closing of son’s
account thereafter, opened accounts in his own name with the same bank. Petitoner contends that
all the indictments narrated in the seventy-five (75) informations were mere components of only
one crime, since the same were only impelled by a single criminal resolution or intent

ISSUE:
Whether or not the accusations contained in the 75 informations against respondent constitute a
single crime of estafa

RULING:
No. The daily abstractions from and diversions of private respondent of the deposits made by the
customers of the optical supply company from October 2,1972 to December 30, 1972, excluding
Saturdays and Sundays, cannot be considered as proceeding from a single criminal act within the
meaning of Article48. The abstractions were not made at the same time and on the same occasion,
but on variable dates. Each day of conversion constitutes a single act with an independent existence
and criminal intent of its own. All the conversions are not the product of a consolidated or united
criminal resolution, because each conversion is a complete act by itself. Specifically, the
abstractions and the accompanying deposits thereof in the personal accounts of private respondent
cannot be similarly viewed as "continuous crime".

404
CRIMINAL LAW 1 CASE DIGESTS
ATTY. LYAN DAVD JUANICO
BLOCK 1-M A.Y. 2020 - 2021

11. People v. Mallari


G.R. No. L-58886, 13 December 1988
FERNAN, J.

FACTS:
Petitioner Consuelo E. Mallari, with three (3) others, was accused of the crime of Estafa thru
Falsification of Public Document before the then Court of First Instance of Manila. Informations
filed in the two cases against him as the findings of facts of the appellate court tells that they refer
to the same series of acts. These series of acts amount to what is known in law as a continued,
continuous or continuing offense. In addition, in her motion for reconsideration, petitioner
contended that the current decision would place her twice in jeopardy of being punished for the
same offense as she had previously been convicted, sentenced and probationed for the same
offense of other case

ISSUE:
Whether the said acts amount to continued, continuous or continuing offense.

RULING:
A continued crime is a single crime consisting of a series of acts but all arising from one criminal
resolution. It is a continuous, unlawful act or series of acts set on foot by a single impulse and
operated by an unintermittent force, however long a time it may occupy. Although there are series
of acts, there is only one crime committed. Hence, only one penalty shall be imposed.

In the case at bar, the crime of estafa thru falsification of public document committed by Consuelo
Mallari, although consummated through a series of acts, was "set on foot" by the single intent or
impulse to defraud Remegio Tapawan of a total amount of P3,000.00. And contrary to the appellate
court's observation, there was only one deceit practised by petitioner on the two (2) victims. The
singularity of the offense committed by petitioner is further demonstrated by the fact that the
falsification of the two (2) public documents as a means of committing estafa were performed on
the same date, in the same place, at the same time and on the same occasion. Petitioner having
already been convicted of the complex crime of estafa thru fasification of pubic document, she can
no longer be held liabe for the same crime in this case. The rule against double jeopardy protects
the accused not against the peril of second punishment but against being tried for the same offense.

405
CRIMINAL LAW 1 CASE DIGESTS
ATTY. LYAN DAVD JUANICO
BLOCK 1-M A.Y. 2020 - 2021

12. Santiago v. Garchitorena


G.R. No. 109266, 2 December 1993
QUIASON, J

FACTS:
The case arose from complaints filed by a group of employees of the Commission of Immigration
and Deportation against petitioner, then CID Commissioner, for alleged violation of the Anti-Graft
and Corrupt Practices Act. In Oct 1988, Santiago approved the application for legalization of the
stay of about 32 aliens or by her favoring “unqualified” aliens with the benefits of the Alien
Legalization Program. Her act was said to be illegal and was tainted with bad faith. Two other
criminal cases, one for violation of the provisions of Presidential Decree No. 46 and the other for
libel, were also filed with the Regional Trial Court of Manila. Hence, It bears noting that the public
prosecutor filed a total of 32 informations against the petitioner for the violation of such law-
legalization of the stay of about 32 aliens.

ISSUE:
Whether or not the prosecutors were correct in filing 32 amended informations instead of one.

RULING:
There was only one crime that was committed in petitioner's case, and hence, there should only be
one information to be filled against her. The 32 Amended Informations charge what is known a s
delito continuado or "continued crime" and sometimes referred to as "continuous crime."

In the case at bench, the original information charged petitioner with performing a single criminal
act — that of her approving the application for legalization of aliens not qualified under the law to
enjoy such privilege. The original information also averred that the criminal act: (i) committed by
petitioner was in violation of a law — Executive Order No. 324 dated April 1988, (ii) caused an
undue injury to one offended party, the Government, and (iii) was done on a single day. The 32
Amended Informations reproduced verbatim the allegation of the original information, except that
instead of the word "aliens" in the original information each amended information states the name
of the individual whose stay was legalized.

The 32 Amended Informations aver that the offenses were committed on the same period of time,
on or about October 17, 1988. The strong probability even exists that the approval of the
application for the legalization of the stay of the 32 aliens was done by a single stroke of the pen,
as when the approval was embodied in the same document. Likewise, the public prosecutors
manifested at the hearing of the motion for a bill of particulars that the Government suffered a

406
CRIMINAL LAW 1 CASE DIGESTS
ATTY. LYAN DAVD JUANICO
BLOCK 1-M A.Y. 2020 - 2021

single harm or injury. Therefore, the 32 amended informations was consolidated into one
information charging only one offense.

407
CRIMINAL LAW 1 CASE DIGESTS
ATTY. LYAN DAVD JUANICO
BLOCK 1-M A.Y. 2020 - 2021

13. Ivler v. San Pedro


G.R. No. 172716, 17 November 2010
CARPIO, J

FACTS:
Petitioner Jason Ivler (petitioner) was charged before the Metropolitan Trial Court of Pasig City,
Branch 71 (MeTC), with two separate offenses: (1) Reckless Imprudence Resulting in Slight
Physical Injuries (Criminal Case No. 82367) for injuries sustained by respondent Evangeline L.
Ponce (respondent Ponce); and (2) Reckless Imprudence Resulting in Homicide and Damage to
Property (Criminal Case No. 82366) for the death of respondent Ponce's husband Nestor C. Ponce
and damage to the spouses Ponce's vehicle. Petitioner posted bail for his temporary release in both
cases. The RTC ruled that light offenses (e.g., slight physical injuries) cannot be complexed under
Article 48 of the Revised Penal Code with grave or less grave felonies (e.g., homicide). Hence, the
prosecution was obliged to separate the charge in Criminal Case No. 82366 for the slight physical
injuries from Criminal Case No. 82367 for the homicide and damage to property. Hence this
petition. As petitioner, moved to quash the Information in Reckless Imprudence Resulting in
Homicide and Damage to Property for placing him in jeopardy of second punishment for the same
offense of reckless imprudence.

ISSUE:
Whether or not the court were correct in filing 2 separate informations filed against the petitioner
arising from the same facts.

RULING:
No. the court must only file single charge against the petitioner. Reckless Imprudence is a Single
Crime, its consequences on persons and property are material only to determine the Penalty. The
two charges against petitioner, arising from the same facts, were prosecuted under the same
provision of the Revised Penal Code, as amended, namely, Article 365 defining and penalizing
quasi-offenses- Reckless imprudence consists in voluntary, but without malice, doing or failing to
do an act from which material damage results by reason of inexcusable lack of precaution on the
part of the person performing or failing to perform such act, taking into consideration his
employment or occupation, degree of intelligence, physical condition and other circumstances
regarding persons, time, and place. Conceptually, quasi-offenses penalize "the mental attitude or
condition behind the act, the dangerous recklessness, lack of care or foresight " unlike willful
offenses which punish the intentional criminal act. In connection with the attempt of the MeTC to
harmonize Article 365 and Article 48 on complexing crimes. Whereas, Article 48 works to
combine in a single prosecution multiple intentional crimes falling under Titles 1-13, Book II of
the Revised Penal Code, when proper; Article 365 governs the prosecution of imprudent acts and

408
CRIMINAL LAW 1 CASE DIGESTS
ATTY. LYAN DAVD JUANICO
BLOCK 1-M A.Y. 2020 - 2021

their consequences. However, the complexities of human interaction can produce a hybrid quasi-
offense not falling under either models — that of a single criminal negligence resulting in multiple
damages to persons and property with varying penalties corresponding to light, less grave or grave
offenses.

Jurisprudence adopts 2 approaches: 1.) Article 48 by "complexing" one quasi- crime with its
multiple consequences 48 unless one consequence amounts to a light felony, in which case charges
were split by grouping, on the one hand, resulting acts amounting to grave or less grave felonies
and filing the charge with the second level courts and, on the other hand, resulting acts amounting
to light felonies and filing the charge with the first level courts (which the MeTC implied); and 2.)
Article 48 and sanctions a single prosecution of all the effects of the quasi-crime collectively
alleged in one charge, regardless of their number or severity, penalizing each consequence
separately.

Evidently, these approaches, while parallel, are irreconcilable.


Hence, prosecutions under Article 365 should proceed from a single charge regardless of the
number or severity of the consequences. In imposing penalties, the judge will do no more than
apply the penalties under Article 365 for each consequence alleged and proven. In short, there shall
be no splitting of charges under Article 365, and only one information shall be filed in the same
first level court. Article 365 is a stronger and simpler protection of constitutional right under the
Double Jeopardy Clause. They are thereby denied the beneficent effect of the favorable sentencing
formula under Article 48, but any disadvantage thus caused is more than compensated by the
certainty of non-prosecution for quasi-crime effects qualifying as "light offenses" (or, as here, for
the more serious consequence prosecuted belatedly).

409
CRIMINAL LAW 1 CASE DIGESTS
ATTY. LYAN DAVD JUANICO
BLOCK 1-M A.Y. 2020 - 2021

MODULE 11: PENALTIES


1. PEOPLE OF THE PHILIPPINES vs . ALFREDO BON
G.R. No. 166401. October 30, 2006
TINGA, J.:

Facts:
Upon learning the sexual abuses made by the appellant to her daughters, CCC immediately filed
several complaints charging the appellant, Alfredo Bon, Eight (8) Informations with the rape of
AAA and BBB, his nieces. AAA and BBB’s birth certificates were presented as documentary
evidence before the RTC to prove that they were in fact minors when the felony was consummated
making it qualified rape under Article 266-B of the Revised Penal Code, which is punishable by
death. The RTC convicted appellant on all eight (8) counts of rape and as the penalty imposed
consisted of eight (8) death sentences, the records of the case were automatically elevated to this
Court for review.

The Court of Appeals downgraded the convictions in Criminal Case Nos. 6906 and 6908 to
attempted rape. According to the appellate court, it could not find evidence beyond reasonable
doubt in those two (2) cases that appellant had accomplished the slightest penetration of AAA’s
vagina to make him liable for consummated rape. Accordingly, the Court of Appeals reduced the
penalties for the two (2) attempted rape from death to reclusion temporal.

The Court thus affirms the conclusions of the Court of Appeals that it has been established beyond
reasonable doubt that appellant is guilty of six (6) counts of rape and two (2) counts of attempted
rape. However, in light of Rep. Act. No. 9346, the appropriate penalties for both crimes should be
amended.

ISSUE:
1. Whether Section 1 of Rep. Act No. 9346 expressly repealed the imposition of “death” in the
graduated scale of penalties under Article 71 of the Revised Penal Code

2. Whether the Court of Appeals erred in imposing the penalty of reclusion temporal for the
two (2) counts of attempted qualified rape

RULING:
1. Yes, Section 1 of Rep. Act No. 9346 deleted the word “death” as expressly provided for in the
graduated scale of penalties under Article 71

410
CRIMINAL LAW 1 CASE DIGESTS
ATTY. LYAN DAVD JUANICO
BLOCK 1-M A.Y. 2020 - 2021

Yet in truth, there is no material difference between "imposition" and "application," for both
terms embody the operation in law of the death penalty. Since Article 71 denominates "death" as
an element in the graduated scale of penalties, there is no question that the operation of Article 71
involves the actual application of the death penalty as a means of determining the extent which a
person's liberty is to be deprived. Since Rep. Act No. 9346 unequivocally bars the application of
the death penalty, as well as expressly repeals all such statutory provisions requiring the
application of the death penalty, such effect necessarily extends to its relevance to the graduated
scale of penalties under Article 71. The amendatory effects of Rep. Act No. 9346 extend only to
the application of the death penalty but not to the definition or classification of crimes.

We cannot find basis to conclude that Rep. Act No. 9346 intended to retain the operative effects
of the death penalty in the graduation of the other penalties in our penal laws.

In application to the case at bar, the penalty for the six (6) counts of rape was downgraded
from death to reclusion perpetua with no eligibility for parole, pursuant to Rep. Act. No. 9346.

2. Yes.

The application of death penalty being expressly repealed by Rep. Act No. 9346. The next
highest imposable penalty would be reclusion perpetua. Under Article 51 of the Revised Penal
Code, the penalty for an attempted felony is the “penalty lower by two degrees than that prescribed
by law for the consummated felony.”

Hence, in applying Article 51 with relation to Article 71 graduated scale of penalties, the
penalty to be imposed for the two (2) counts of attempted rape should be prision mayor instead of
reclusion temporal.

411
CRIMINAL LAW 1 CASE DIGESTS
ATTY. LYAN DAVD JUANICO
BLOCK 1-M A.Y. 2020 - 2021

2.PEOPLE VS LUCAS
G.R. NO. 108172 – 73
DAVIDE, JR., J.:

FACTS:
Complainant Chanda Lucas, who was born on 2 June 1973, testified that their0 house at 23-X
Daropa Road, Baesa, Quezon City, has only one bedroom. On 26 November 1983, she was
sleeping in the bedroom with her brother and sisters. Their mother did not sleep in their house at
that time. At about 2:00 to 3:00 a.m., she awoke and realized that her father was removing her
panty and shorts. He cautioned her to keep quiet. Then, her father, who was already naked, went
on top of her and placed his sexual organ inside her vagina. She was hurt but did not resist because
her father threatened to kill her. Only her older sister Cynthia witnessed the incident. Chanda
reported the incident to her mother and her aunt but the former did nothing. When her aunt said
that her father should be jailed, her mother did not agree.

The 26 November 1983 incident was only the first of many atrocities. Since then, her father had
been repeatedly molesting her, especially when her mother was not around. The last assault on her
womanhood occurred on 12 February 1991 when she was already seventeen years old. Before he
had sex with her at 3:00 a.m. on 12 February 1991, he first moved her brothers and sisters, who
were sleeping in the same room with her, to another place. She did not resist because he had
a balisong with him and told her that he can take her life anytime. After the sexual assault, he stood
up holding his balisong and again said that she has only one life and that he can take it anytime.

On the morning of 16 February 1992, in the company of her mother and uncle, she reported the
incident to the police in their area. The police investigator questioned her and her sworn statement
(Exhibit "D") was taken. In the afternoon of that day, she submitted to a medical examination at
Camp Crame and a medical certificate was issued.

Cynthia Lucas Viado, the elder sister of Chanda, testified that she witnessed the incident of 26
November 1983. She was then thirteen years old while Chanda was only nine years old. She saw
his father on top of Chanda, then she closed her eyes and covered her face with a blanket. She
reported the incident and the fact that she saw blood on the underwear of Chanda to her aunt
Neneng and her mother; the former was very angry upon learning of the incident but the latter did
not believe her; at that time, her mother loved her father
dearly. On cross- examination, Cynthia declared that her father intended to sexually abuse her on
26 November 1983 but because she resisted, her father instead raped Chanda. She was not able to
help Chanda because she was afraid of her father. Their brother and another sister were not aware
of the incident and they did not wake them up because they were ashamed of their neighbors.

ISSUE:

Whether or not the presence of modifying circumstances would affect the penalty.

412
CRIMINAL LAW 1 CASE DIGESTS
ATTY. LYAN DAVD JUANICO
BLOCK 1-M A.Y. 2020 - 2021

RULING:

Prior to R.A. No. 7659, the presence of modifying circumstances would not affect the penalty
of reclusion perpetua prescribed for the crime of rape because such a penalty was then
indivisible and under Article 63 of the Revised Penal Code, when the law prescribes a single
indivisible penalty, it shall be applied by the courts regardless of any mitigating or aggravating
circumstances that may have attended the commission of the deed. However, pursuant to Section
21 of R.A. No. 7659, which amended Article 27 of the Revised Penal Code, reclusion
perpetua has now a defined duration, i.e., from twenty (20) years and one (1) day to forty (40)
years. There is, however, no corresponding amendment to Article 76 of the same Code for the
purpose of converting reclusion perpetua into a divisible penalty with three specific period —
minimum, medium, and maximum — and including it in the table provided therein showing the
duration and the time included in each of the periods.

It may thus be said that although the law has now fixed the duration of reclusion perpetua, it did
not make explicit its intention to convert it into a divisible penalty. In any event, Article 65 of the
Code which provides:

Art. 65. Rules in cases in which the penalty is not composed of three periods. —
In cases in which the penalty prescribed by law is not composed of three periods,
the courts shall apply the rules contained in the foregoing articles, dividing
into three equal portions of time included in the penalty prescribed, and forming
one period of each of the three portions.

may be applied. Accordingly, the time included in the penalty of reclusion


perpetua (twenty [20] years and one [1] days to forty [40] years) can be divided into three
equal portions, with each composing a period. The periods of reclusion perpetua would
then be as follows:

minimum — 20 years and 1 day to


26 years and 8 months

medium — 26 years,
8 months and 1 day to
33 years and 4 months

maximum — 34 years, 4 months and


1 day to 40 years

Taking into account the presence of the aggravating circumstance of relationship in Criminal
Case No. Q-91-18465, the accused may finally be sentenced to thirty-four (34) years, four (4)
months and one (1) day of reclusion perpetua.

413
CRIMINAL LAW 1 CASE DIGESTS
ATTY. LYAN DAVD JUANICO
BLOCK 1-M A.Y. 2020 - 2021

Considering again such aggravating circumstance, the accused may be sentenced in Criminal
Case No. Q-91-18466 to an indeterminate penalty ranging from four (4) years, two (2) months
and one (1) day of prision correccional maximum as minimum to ten (10) years and one (1) day
of prision mayor maximum as maximum.

414
CRIMINAL LAW 1 CASE DIGESTS
ATTY. LYAN DAVD JUANICO
BLOCK 1-M A.Y. 2020 - 2021

3. PEOPLE OF THE PHILIPPINES vs. CONRADO LUCAS Y BRIONES


G.R. Nos. 108172-73; January 9, 1995
DAVIDE, JR., J.:

FACTS:
On 16 February 1991, Chanda Lucas y Austria, then seventeen years old, charged her natural
father, accused Jose Conrado Lucas, of attempted rape committed against her on 12 February 1991.
She also filed a separate complaint for a rape committed on 26 November 1982 against her when
she was still 9 years old. The cases were jointly tried after the accused had pleaded not guilty upon
his arraignment. The prosecution presented as witness the complainant herself; her sister Cynthia,
and Dr. Emmanuel Aranas.

Examinations by Dr Emmanuel Aranas disclosed healed lacerations, but he could not determine
when the lacerations were inflicted or sustained. He concluded that the complainant has had several
sexual experiences and was no longer a virgin. The trial court promulgated its decision and found
the accused guilty beyond reasonable doubt of two crimes of rape imposing penalty upon him of
reclusion perpetua.

On 4 November 1992, the accused filed a notice of appeal. He alleges that the trial court erred in
convicting him of the crime of rape in Criminal Case No. Q-91-18466 inasmuch as the same is
more serious than the offense charged. The Court modified the conviction for Criminal Case No.
Q-91-18466 from consummated rape to attempted rape.

In the decision in this case, promulgated on 25 May 1994, the First Division touched on the nature
of the penalty of reclusion perpetua in the light of Section 21 of R.A. No. 7659 which amended
Article 27 of the Revised Penal Code by specifically fixing the duration of reclusion perpetua at
twenty (20) years and one (1) day to forty (40) years. It opined that since no corresponding
amendment to Article 76 of the Revised Penal Code as made, the said law has not made explicit
an intention to convert reclusion perpetua into a divisible penalty. Nevertheless, it applied Article
65 of the Revised Penal Code and stated:

"Accordingly, the time included in the penalty of reclusion perpetua (twenty [20] years and one
[1] day to forty [40] years) can be divided into three equal portions with each composing a period.
The periods of reclusion perpetua would then be as follows:

minimum — 20 years and 1 day to 26 years and 8 months


medium — 26 years, 8 months and 1 day to 33 years and 4 months
maximum — 34 years, 4 months and 1 day to 40 years.

415
CRIMINAL LAW 1 CASE DIGESTS
ATTY. LYAN DAVD JUANICO
BLOCK 1-M A.Y. 2020 - 2021

Taking into account the presence of the aggravating circumstance of relationship in Criminal Case
No. Q-91-18465, the accused may finally be sentenced to thirty-four (34) years, from (4) months
and one (1) day of reclusion perpetua."

In a motion for clarification seasonably filed by the appellee on 28 June 1994 which was not
opposed by the accused-appellant in his comment, the appellee asks the Court tocorrect the
duration of the maximum period of reclusion perpetua from thirty-four (34) years, four (4) months
and one (1) day to forty (40) years, as stated in the decision, to thirty-three (33) years, four (4)
months and one (1) day to forty (40) years.

Since the issue of whether the amendment of Article 27 of the Revised Penal Code by Section 21
of R.A. No. 7659 has made reclusion perpetua a divisible penalty is one of first impression and of
sufficient importance, the First Division referred the motion for clarification to the Court en banc.
The latter accepted the referral.

ISSUE:
Whether Rep. Act No. 7659 reclassified reclusion perpetua as a divisible penalty

RULING:
No, Rep. Act No. 7659 did not reclassify reclusion perpetua as a divisible penalty

Now then, if Congress had intended to reclassify reclusion perpetua as a divisible penalty, then it
should have amended Article 63 and Article 76 of the Revised Penal Code. The latter if the law on
what are considered divisible penalties under the Code and what should be the duration of the
periods thereof.

After deliberating on the motion and re-examining the legislative history of R.A. No. 7659, the
Court concludes that although Section 17 of R.A. No. 7659 has fixed the duration of reclusion
perpetua from twenty (20) years and one (1) day to forty (40) years, there was no clear legislative
intent to alter its original classification as an indivisible penalty. It shall then remain as an
indivisible penalty.

In application to the case at bar, the Court resolved to modify the decision of 25 May 1994 in this
case by DELETING therefrom the disquisition on whether reclusion perpetua is a divisible penalty
and SETTING ASIDE its division into three periods and, finally, AMENDING the dispositive to
read as follows:

416
CRIMINAL LAW 1 CASE DIGESTS
ATTY. LYAN DAVD JUANICO
BLOCK 1-M A.Y. 2020 - 2021

"WHEREFORE, the challenged decision of 28 October 1992 of Branch 104 of the Regional Trial
Court of Quezon City in Criminal Case No. Q-91-18465 and Criminal Case No. Q-91-18466 is
hereby AFFIRMED, subject to the modifications above indicated. As modified:

"(1) In Criminal Case No. Q-91-18465, in addition to the penalty of reclusion perpetua imposed
by the trial court, accused JOSE CONRADO LUCAS Y BRIONES is further ordered to indemnify
the offended party, Chanda Lucas y Austria, in the sum of Fifty Thousand Pesos (P5,000.00); and

"(2) In Criminal Case No. Q-91-18466, accused JOSE CONRADO LUCAS Y BRIONES is
hereby found GUILTY beyond reasonable doubt of the lesser offense of attempted rape and is
hereby sentenced to suffer an indeterminate penalty ranging from Four (4) Years, Two (2) Months
and One (1) Day of prision correccional as minimum to Ten (10) Years and One (1) Day of prision
mayor as maximum, and to indemnify the offended party, Chanda Lucas y Austria, in the sum of
Thirty Thousand Pesos (P30,000.00).

417
CRIMINAL LAW 1 CASE DIGESTS
ATTY. LYAN DAVD JUANICO
BLOCK 1-M A.Y. 2020 - 2021

4. PEOPLE OF THE PHILIPPINESvs. GERARDO LATUPAN y SIBAL, alias JERRY


G.R. Nos. 112453-56. June 28, 2001
PARDO, J.:

FACTS:
Jaime, his three-year old son at the time of the said incident, testified that Latupan stabbed his
mother, stepped on him, threw his brother Leo outside the window, and stabbed his other brother
Jose. It was also testified by Ceferino that moments later, he saw accused Gerardo Latupan walking
in his direction, carrying a thin, bloodied knife and started chasing Ceferino's wife, who was able
to run to a house nearby. Unable to catch Ceferino's wife, Latupan turned to Ceferino and attempted
to thrust the knife into him, however, he was able to parry it. Later on, Ceferino was able to
convince Latupan to go to the authorities.

Gerardo Latupan was charged with two counts of frustrated murder and two counts of murder. At
the arraignment, Latupan pleaded not guilty to the charge of frustrated murder. During the pre-
trial conference, accused offered to change his plea of not guilty to guilty of the complex crime of
double murder and frustrated murder.

Thereafter, the Regional Trial Court, Branch 11 of Tuao, Cagayan convicted Latupan of the
complex crime of murder and he was sentenced to "life imprisonment." He was also convicted of
physical injuries and the penalty of "ten days imprisonment" was imposed upon him. Hence, this
appeal.

ISSUES:
1.Whether the trial court erred in convicting the accused of complex crime of multiple murder
2.Whether reclusion perpetua is the same as life imprisonment, which is imposed upon the accused

RULINGS:
1. Yes.
Article 48 of the Revised Penal Code provides: "When a single act constitutes two or more grave
or less grave felonies or when an offense is a necessary means for committing the other, the penalty
for the most serious crime shall be imposed, the same to be applied in its maximum period."

The instant case does not fall under any of the two mentioned instances when a complex crime is
committed. The killing of Lilia Asuncion and Jose Asuncion and the wounding of Jaime and Leo
Asuncion resulted not from a single act but from several and distinct acts of stabbing. "Where the
death of two persons does not result from a single act but from two different shots, two separate
murders, and not a complex crime, are committed."

418
CRIMINAL LAW 1 CASE DIGESTS
ATTY. LYAN DAVD JUANICO
BLOCK 1-M A.Y. 2020 - 2021

Thus, accused-appellant is liable, not for a complex crime of double murder, but for two separate
counts of murder, and separate counts of physical injuries.

2. No, reclusion perpetua is not the same as life imprisonment.


They are distinct in nature, in duration and in accessory penalties.

First, "life imprisonment" is imposed for serious offenses penalized by special laws, while
reclusion perpetua is prescribed under the Revised Penal Code.

Second, "life imprisonment" does not carry with it any accessory penalty. Reclusion perpetua has
accessory penalties.

Third, "life imprisonment" does not appear to have any definite extent or duration, while reclusion
perpetua entails imprisonment for at least thirty (30) years after which the convict becomes eligible
for pardon, although the maximum period thereof shall in no case exceed forty (40) years.

Thus, the courts must employ the proper nomenclature specified in the Revised Penal Code, such
as "reclusion perpetua," not "life imprisonment" or "ten days of arresto menor, " not "ten days of
imprisonment."

Hence, the proper penalty for each murder committed in April 1991, considering the absence of
aggravating and mitigating circumstances, is reclusion perpetua, with its accessory penalties.
Further, accused-appellant is liable for two counts of slight physical injuries and must be sentenced
to twenty (20) days of arresto menor, each, likewise with its accessory penalties under the Revised
Penal Code.

419
CRIMINAL LAW 1 CASE DIGESTS
ATTY. LYAN DAVD JUANICO
BLOCK 1-M A.Y. 2020 - 2021

5. PEOPLE OF THE PHILIPPINES vs . RODEL LANUZA y BAGAOISAN


G.R. No. 188562; August 24, 2011
LEONARDO-DE CASTRO, J.:

FACTS:
According to the private complainant, he did not actually hand the shotgun to the accused. Instead,
he merely placed it, together with one bullet, on top of the security guard's table. Although he was
turning over six bullets to the accused, the private complainant asserted that the five others were
inside a drawer on the security guard's table at their office upstairs. While the private complainant
who was about to go home was asking why the accused did not report on his scheduled shift, the
latter got the shotgun, placed the ammunition inside it, and shot him. The private complainant fell
down on his buttocks. The accused went near the private complainant and pulled the trigger a
second time, but the shotgun did not fire and the private complainant heard only a click. The
accused ran upstairs, and the private complainant crawled to his motorcycle and drove it himself
to the provincial hospital.

During the preliminary conference, the parties admitted, among other facts, that accused-appellant
shot private complainant Joel G. Butay; that as a result of the shooting, private complainant
sustained a gunshot wound which caused his confinement at the provincial hospital for 12 days;
that accused-appellant voluntarily surrendered to the Philippine National Police (PNP), Laoag
City, surrendering a shotgun, five live bullets, and one empty shell; and that private complainant
suffered actual damages amounting to P70,000.00.

Accused-appellant, however, asserted that the shooting was accidental, as contemplated under
Article 12 (4) of the Revised Penal Code, which exempts from criminal liability "any person who,
while performing a lawful act with due care, causes injury by mere accident without fault or
intention of causing it."

RTC found him guilty of the crime frustrated homicide. Accused-apellant filed his Appellant’s
Brief before the Court of Appeals to assail the foregoing judgment of conviction rendered against
him by the RTC. Accused-appellant urged the appellate court to impose upon him a sentence in
accord with Article 67 of the same Code, which specifically provided for the "[p]enalty to be
imposed when not all the requisites of exemption of the fourth circumstance of Article 12 are
present." Court of Appeals affirmed the decision of the RTC.

420
CRIMINAL LAW 1 CASE DIGESTS
ATTY. LYAN DAVD JUANICO
BLOCK 1-M A.Y. 2020 - 2021

ISSUE:

Whether the accused-appellant’s acts were a mere accident under Article 12 par.4 and should only
be imposed the sentence in accord with Article 67 of the RPC

RULING:

No.
It is axiomatic that a person who invokes accident must prove that he acted with due care.
According to him, he knew that it was not proper for a person to receive a firearm from another by
immediately inserting a finger inside the trigger guard. Likewise, he knew that the hand-over of a
firearm with its barrel pointed towards the giver or any other person was not proper. That he did
these improper acts despite his training and experience as a security guard undermines any
notion that he had acted with due care during the subject incident.
There are other reasons for withholding credence from the claim of the accused. According to him,
after the private complainant was shot and rendered unconscious, he did not go near him. For ten
seconds before he finally decided to go upstairs, he did nothing. Moreover, he made no attempt to
check if he was still alive or if he could help him in any way. That he offered no help to the
wounded private complainant undermines his claim of accident. As observed in People v.
Reyes, "had [the shooting] really been accidental, then the natural tendency of the accused would
have been to immediately give help to his unfortunate victim and even to plead and express his
regret to the mother of the deceased."
The Court stresses that accused-appellant himself admitted the fact of shooting, and only disputed
any intent to kill private complainant. The conclusion of the RTC, as affirmed by the Court of
Appeals and this Court, that accused-appellant intended to kill private complainant was not based
entirely on accused-appellant deliberately loading the shotgun, but also on the existence of motive
on accused-appellant's part, the location and severity of private complainant's injury, and accused-
appellant's behavior immediately after the shooting.

In application to the case at bar, the accused-appellant’s acts were not mere accident but was
proven to be deliberate and inexcusable act with the intent to kill the private complainant. Hence,
he may not be subjected to the provisions of Article 67 relating to penalty to be imposed when not
all the requisite of exemption of the fourth circumstance of Article 12 are present.

421
CRIMINAL LAW 1 CASE DIGESTS
ATTY. LYAN DAVD JUANICO
BLOCK 1-M A.Y. 2020 - 2021

6. VIRGILIO TALAMPAS y MATIC vs. PEOPLE OF THE PHILIPPINES


G.R. No. 180219. November 23, 2011
BERSAMIN, J.:

FACTS:
Prosecution witness Jose Sevillo (Jose) who allegedly witnessed the incident in question, testified
that on July 5, 1995 at about 7:00 o'clock in the evening, he together with Eduardo Matic (Eduardo)
and Ernesto Matic (Ernesto) were infront of his house, along the road in Zona Siete, Wawa,
Malaban, Biñan, Laguna, repairing his tricycle when he noticed the appellant who was riding on a
bicycle passed by and stopped. The latter alighted at about three (3) meters away from him, walked
a few steps and brought out a short gun, a revolver, and poked the same to Eduardo and fired it
hitting Eduardo who took refuge behind Ernesto. The appellant again fired his gun three (3) times,
one shot hitting Ernesto at the right portion of his back causing him (Ernesto) to fall on the ground
with his face down. Another shot hit Eduardo on his nape and fell down on his back (patihaya) .
Thereafter, the appellant ran away, while he (Jose) and his neighbors brought the victims to the
hospital. On June 6, 1995, Jose executed a Sworn Statement at the Biñan Police Station.

Talampas interposed self-defense and accident. He insisted that his enemy had been Eduardo Matic
(Eduardo), not victim Ernesto Matic (Ernesto); that Eduardo, who was then with Ernesto at the
time of the incident, had had hit him with a monkey wrench, but he had parried the blow; that he
and Eduardo had then grappled for the monkey wrench; that while they had grappled, he had notice
that Eduardo had held a revolver; that he had thus struggled with Eduardo for control of the
revolver, which had accidentally fired and hit Ernesto during their struggling with each other; that
the revolver had again fired, hitting Eduardo in the thigh; that he had then seized the revolver and
shot Eduardo in the head; and that he had then fled the scene when people had started swarming
around.

The RTC, giving credence to the testimony of the eyewitness Jose Sevilla, found Talampas guilty
beyond reasonable doubt of homicide sentencing him to an indeterminate penalty of Ten (10) years
and one (1) day of prision mayor, as minimum, to Fourteen (14) years and Eight (8) Months of
reclusion temporal, as maximum. The CA affirmed the decision of the RTC. Hence, this appeal.

ISSUE:
Whether the penalty imposed upon the accused-appellant is erroneous

422
CRIMINAL LAW 1 CASE DIGESTS
ATTY. LYAN DAVD JUANICO
BLOCK 1-M A.Y. 2020 - 2021

RULING:

Yes. The penalty for homicide under Article 246 of the Revised Penal Code is reclusion temporal.
Under Section 1 of the Indeterminate Sentence Law, 15 the court, in imposing a prison sentence
for an offense punished by the Revised Penal Code, or its amendments, is mandated to prescribe
an indeterminate sentence the maximum term of which shall be that which, in view of the attending
circumstances, could be properly imposed under the rules of the Revised Penal Code, and the
minimum term shall be within the range of the penalty next lower to that prescribed by the Revised
Penal Code for the offense.

With the absence of aggravating or mitigating circumstances, the imposable penalty is reclusion
temporal in its medium period, or 14 years, eight months, and one day to 17 years and four months.
This is pursuant to Article 64 of the Revised Penal Code. It is such period that the maximum term
of the indeterminate sentence should be reckoned from.

Hence, limiting the maximum term of the indeterminate sentence at only 14 years and eight months
contravened the express provision of the Indeterminate Sentence Law, for such penalty was within
the minimum period of reclusion temporal. Accordingly, the Court must add one day to the
maximum term fixed by the lower courts.

The Court also finds the increment of one day as part of the minimum term of the indeterminate
sentence unnecessary. Hence, in order to simplify the computation of the minimum penalty of the
indeterminate sentence, the Court deletes the one-day increment from the minimum term of the
indeterminate sentence.

The amended dispositive portion of the decision is as follows:

WHEREFORE, the Court AFFIRMS the decision promulgated on August 16, 2007 finding
VIRGILIO TALAMPAS y MATIC guilty beyond reasonable doubt of the crime of homicide,
and IMPOSES the indeterminate sentence of 10 years of prision mayor, as minimum, to 14
years, eight months, and one day of reclusion temporal, as maximum.

423
CRIMINAL LAW 1 CASE DIGESTS
ATTY. LYAN DAVD JUANICO
BLOCK 1-M A.Y. 2020 - 2021

7. PEOPLE OF THE PHILIPPINES vs. BETH TEMPORADA,


G.R. No. 173473. December 17, 2008
YNARES-SANTIAGO, J.:

Facts:
From September 2001 to January 2002, accused Rosemarie "Baby" Robles, Bernadette Miranda,
Nenita Catacotan and Jojo Resco and appellant Beth Temporada, all employees of the Alternative
Travel and Tours Corporation (ATTC), recruited and promised overseas employment, for a fee, to
complainants Rogelio Legaspi, Jr. as technician in Singapore, and Soledad Atle, Luz Minkay,
Evelyn Estacio and Dennis Dimaano as factory workers in Hongkong. The accused and appellant
were then holding office at Dela Rosa Street, Makati City but eventually transferred business to
Discovery Plaza, Ermita, Manila. After complainants had submitted all the requirements consisting
of their respective application forms, passports, NBI clearances and medical certificates, the
accused and appellant, on different dates, collected and received from them placement fees in
various amounts, viz.: a) from Rogelio Legaspi, Jr. — 57,600.00; b) from Dennis Dimaano —
P66,520.00; c) from Evelyn Estacio — P88,520.00; d) from Soledad Atle — P69,520.00 and e)
from Luz Minkay — P69,520.00. As none of them was able to leave nor recover the amounts they
had paid, complainant lodged separate criminal complaints against accused and appellant before
the City Prosecutor of Manila.

On November 29, 2002, Assistant City Prosecutor Restituto Mangalindan, Jr. filed six (6)
Informations against the accused and appellant, one for Illegal Recruitment in Large Scale under
Article 38 (a) of the Labor Code as amended, and the rest for five (5) counts of estafa under Article
315 paragraph 2 (a) of the Revised Penal Code.

The RTC rendered judgment convicting appellant of all the charges. The CA affirmed with
modification the Decision of the RTC as follows:

WHEREFORE, with MODIFICATION to the effect that in Criminal Cases Nos. 02-208373, 02-
208375, & 02-208376, appellant is sentenced to suffer the indeterminate penalty of six (6) years
of prision correccional maximum, as minimum, to ten (10) years and one (1) day of prision mayor
maximum, as maximum; and in Criminal Case No. 02-208374, she is sentenced to suffer the
indeterminate penalty of eight (8) years and one (1) day of prision mayor medium, as minimum,
to twelve (12) years and one (1) day of reclusion temporal minimum, as maximum, the appealed
decision is AFFIRMED in all other respects.

424
CRIMINAL LAW 1 CASE DIGESTS
ATTY. LYAN DAVD JUANICO
BLOCK 1-M A.Y. 2020 - 2021

ISSUES:

1. Whether the incremental rule penalty is analogous to a modifying circumstance falling under the
“attending circumstances” of the ISL
2. Whether the claim of the dissenting opinion of CJ Puno that maximum term should only be one
degree away from the minimum term, correct?
3. Whether the indeterminate penalty for the five estafa cases modified by the CA correct

RULINGS:
1. Yes.

This unique characteristic of the incremental penalty rule does not pose any obstacle to interpreting
it as analogous to a modifying circumstance, and, hence, falling within the letter and spirit of
"attending circumstances" for purposes of the application of the ISL.

Under the wording of the ISL, "attending circumstances" may be reasonably interpreted as
referring to such circumstances that are applied in conjunction with certain rules in the Code in
order to determine the penalty to be actually imposed based on the prescribed penalty of the Code
for the offense. The incremental penalty rule substantially meets this standard. The circumstance
is the amount defrauded in excess of P22,000.00 and the incremental penalty rule is utilized to fix
the penalty actually imposed. At its core, the incremental penalty rule is merely a mathematical
formula for computing the penalty to be actually imposed using the prescribed penalty as starting
point.

Thus, it serves the same function of determining the penalty actually imposed as the modifying
circumstances under Articles 13, 14, and 160 of the RPC, although the manner by which the former
accomplishes this function differs with the latter. For this reason, the incremental penalty rule
may be considered as merely analogous to modifying circumstances. Besides, in case of doubt
as to whether the incremental penalty rule falls within the scope of "attending circumstances" under
the ISL, the doubt should be resolved in favor of inclusion because this interpretation is more
favorable to the accused following the time-honored principle that penal statutes are construed
strictly against the State and liberally in favor of the accused.

Thus, even if the Dissenting Opinion's interpretation is gratuitously conceded as plausible, as


between Gabres and the dissent's interpretation, Gabres should be sustained since it is the
interpretation more favorable to the accused.

425
CRIMINAL LAW 1 CASE DIGESTS
ATTY. LYAN DAVD JUANICO
BLOCK 1-M A.Y. 2020 - 2021

2. No.

The one-degree difference is merely the result of a general observation from the application of
generic mitigating and ordinary aggravating circumstances in the RPC in relation to the ISL.
Nowhere does the ISL refer to the one-degree difference as an essential requisite of an "attending
circumstance". If the application of the incremental penalty rule deviates from the one-degree
difference, this only means that the law itself has provided for an exception thereto.

Verily, the one-degree difference is a mere consequence of the generic mitigating and ordinary
aggravating circumstances created by the legislature. The difficulty of the dissent with the
deviation from its so-called one degree difference rule seems to lie with the inability to view these
"attending circumstances" as mere artifacts or creations of the legislature. It does not make sense
to argue that the legislature cannot formulate "attending circumstances" that operate differently
than these generic mitigating and ordinary aggravating circumstances, and that, expectedly, leads
to a different result from the one-degree difference — for it would be to say that the creator can
only create one specie of creatures. Further, it should be reasonably assumed that the legislature
was aware of these special circumstances, like the incremental penalty rule or privileged mitigating
circumstances, at the time it enacted the ISL as well as the consequent effects of such special
circumstances on the application of said law.

Thus, for as long as the incremental penalty rule is consistent with the letter and spirit of "attending
circumstances" under the ISL, there is no obstacle to its treatment as such.

3. No.

The prescribed penalty for estafa under Article 315, par. 2 (d) of the RPC, when the amount
defrauded exceeds P22,000.00, is prisión correccional maximum to prisión mayor minimum. The
minimum term is taken from the penalty next lower or anywhere within prisión correccional
minimum and medium (i.e., from 6 months and 1 day to 4 years and 2 months). Consequently, the
RTC correctly fixed the minimum term for the five estafa cases at 4 years and 2 months of prisión
correccional since this is within the range of prisión correccional minimum and medium.

To compute the maximum period of the prescribed penalty, prisión correccional maximum to
prisión mayor minimum should be divided into three equal portions of time each of which portion
shall be deemed to form one period in accordance with Article 65 of the RPC. Following this
procedure, the maximum period of prisión correccional maximum to prisión mayor minimum is
from 6 years, 8 months and 21 days to 8 years. The incremental penalty, when proper, shall thus
be added to anywhere from 6 years, 8 months and 21 days to 8 years, at the discretion of the court.

426
CRIMINAL LAW 1 CASE DIGESTS
ATTY. LYAN DAVD JUANICO
BLOCK 1-M A.Y. 2020 - 2021

WHEREFORE, the Decision of the Court of Appeals is MODIFIED with respect to the
indeterminate penalties imposed on appellant for the five (5) counts of estafa, to wit:

(1) In Criminal Case No. 02-208372, the accused is sentenced to an indeterminate penalty
of 4 years and 2 months of prisión correccional as minimum, to 9 years, 8 months and 21 days of
prisión mayor as maximum.

(2) In Criminal Case Nos. 02-208373, 02-208375, and 02-208376, the accused is sentenced
to an indeterminate penalty of 4 years and 2 months of prisión correccional as minimum, to 10
years, 8 months and 21 days of prisión mayor as maximum for each of the aforesaid three estafa
cases.

(3) In Criminal Case No. 02-208374, the accused is sentenced to an indeterminate penalty
of 4 years and 2 months of prisión correccional as minimum, to 12 years, 8 months and 21 days of
reclusión temporal as maximum.

Corona, J.: Separate Opinion


Jurisprudence shows that there are two schools of thought on the incremental penalty in estafa vis-
à-vis the Indeterminate Sentence Law. Under the first school of thought, the minimum term is
fixed at prision correccional while the maximum term can reach up to reclusion temporal. This is
the general interpretation. It was resorted to in People v. Pabalan, People v. Benemerito, People
v. Gabres and in a string of cases.
On the other hand, under the second school of thought, the minimum term is one degree away from
the maximum term and therefore varies as the amount of the thing stolen or embezzled rises or
falls. It is the line of jurisprudence that follows People v. De la Cruz . Among the cases of this
genre are People v. Romero, People v. Dinglasan and Salazar v. People. The Court is urged in
this case to adopt a consistent position by categorically discarding one school of thought.
Hence, our dilemma: which of the two schools of thought should we affirm? Justice Corona
subscribes to the first school of thought since it is more favorable to the accused.
The fact that the amounts involved in the instant case exceed P22,000.00 should not be considered
in the initial determination of the indeterminate penalty; instead, the matter should be so taken as
analogous to modifying circumstances in the imposition of the maximum term of the full
indeterminate sentence. This interpretation of the law accords with the rule that penal laws should
be construed in favor of the accused.

427
CRIMINAL LAW 1 CASE DIGESTS
ATTY. LYAN DAVD JUANICO
BLOCK 1-M A.Y. 2020 - 2021

If the circumstance is considered simply as a modifying circumstance (as in Gabres), it will be


disregarded in determining the minimum term of the indeterminate sentence. The starting point
will be prision correccional maximum to prision mayor minimum and the penalty next lower will
then be prision correccional in its minimum to medium periods, which has a duration of six months
and one day to four years and two months. From the foregoing, it is more favorable to the accused
if the circumstance (that more than P22,000 was involved) is to be considered as a modifying
circumstance, not as a qualifying circumstance. Hence, I submit that the Gabres rule is preferable.

428
CRIMINAL LAW 1 CASE DIGESTS
ATTY. LYAN DAVD JUANICO
BLOCK 1-M A.Y. 2020 - 2021

8. In re Petition for Habeas Corpus, Pete Lagran


G.R. No. 147270, 15 August 2001, 363 SCRA 275
PUNO, J.:

FACTS:
The case involves the conviction of Pete, the petitioner, by the Regional Trial Court for three
counts of violating the Anti-bouncing checks law or B.P. 22. The sentence provided for him is one
year imprisonment for each count and a fine amounting to PHP 125,000.00. The same case
became final and executory. Petitioner was currently serving his sentence when he fled the petition
for habeas corpus. He prayed for his immediate release as he alleges that he has completed his
sentence. The legal basis for his petitioner is Article 70 of the revised penal code. His argument is
if the penalties or sentences imposed on the accused are identical, and such penalties or sentences
emanated from one court and one complaint, the accused shall serve them simultaneously.

ISSUE:
Whether or not petitioner is entitled to the privilege of serving multiple penalties simultaneously.

RULING:
The Supreme Court has ruled against the petitioner and dismissed the case. The privilege granted
under Article 70 of the Revised Penal Code is only applicable if the nature of the penalties so
permit, as indicated in the same article. The Court has pointed out that the only penalties that can
be served out simultaneously are; perpetual absolute disqualification, perpetual special
disqualification, temporary absolute disqualification, temporary special disqualification,
suspension, public censure, fine and bond to keep the peace, civil interdiction, and confiscation
and payment of costs. The nature of a penalty involving the “deprivation of liberty” cannot be
served simultaneously. In the present case, the petitioner is serving three counts of one year
imprisonment. Since the nature of the penalties is a “deprivation of liberty”, he is not entitled to
serve all the penalties simultaneously.

429
CRIMINAL LAW 1 CASE DIGESTS
ATTY. LYAN DAVD JUANICO
BLOCK 1-M A.Y. 2020 - 2021

MODULE 12: EXTINCTION OF CRIMINAL LIABILITY

1. Sermonia v. Court of Appeals


G.R. No. 109454, 14 June 1994
BELLOSILLO, J.:

FACTS:
Petitioner, Jose Sermonia, was charged with the crime of bigamy before the regional trial court for
contracting a marriage with Ma. Lourdes while petitioner’s marriage with Virginia is still valid
and subsisting. Petitioner herein filed a motion to quash, in the regional trial court, the information
file on the basis that the pursuit to push his crmiminal liability for the said bigamy case is already
barred by prescription. Petitioner claims that the second marriage contract was registered with the
office of the civil registrar in 1975. Since the registration with the same office constitutes as a
constructive notice to the whole world of the marriage, the period of prescription should start at
the same time when the marriage contract was registered with the office of the civil registrar.
Following the said counting of the prescription, the case should have been filed on or before 1990
and not in 1992. On the side of the prosecution, they have argued that the period of prescription
should have started in July 1991, the time of the complainant’s discovery of the problem.

ISSUE:
Whether or not the rule of the constructive notice should apply in the counting of the prescription
period for a bigamy criminal case.

RULING:
The court has ruled against the petitioner. The rule on constructive notice should not apply to
bigamy cases since a bigamous marriage is generally entered into in a place where the offender is
not known to be still a married person. The said place is to hide the legal impediment to marry. To
compare it to property cases, where the rule on constructive notice and prescription is also applied,
any verification regarding a concerned property may be easily undertaken by any third party. For
marriage verifications, it is difficult because the place where the offender is not known to be
married. If the counting of the prescription period for the crime of bigamy starts at the time of the
registration of the second marriage, prosecution of the offender is almost to impossible. It is also
a very heavy task for the government agencies involved to undertake a routine to verify each and
every marriage taken and to make sure that there is no second marriage.

430
CRIMINAL LAW 1 CASE DIGESTS
ATTY. LYAN DAVD JUANICO
BLOCK 1-M A.Y. 2020 - 2021

2. Republic v. Desierto, G.R. No. 136506


23 August 2001, 363 SCRA 585
AUSTRIA-MARTINEZ, J.:

FACTS:
Respondent Eduardo was initially subject to a complaint involving the violation of the Anti Graft
Law (RA No. 3019) and Article 186 of the RPC. The Ombudsman dismissed the preliminaries of
the case but was ordered by the Supreme Court to continue the preliminary investigation. The case
seeks to conclude the motion for reconsideration filed by the private respondent Eduardo M.
Cojuangco, Jr. and the Republic of the Philippines, the petitioner herein. Said motion was filed
after the Supreme Court’s decision to set aside the resolution of the Ombudsman in OMB-0-90-
2811 dismissing the Republic’s complaint, and ordered the Ombudsman to proceed with the
preliminary investigation in said case. Respondent alleges that in the complaint filed with
Ombudsman, the case was just dismissed because of lack of evidence or probable cause and not
because the offense has prescribed or that LOI 926 and PD Nos. 961 and 1468 precluded
prosecution under RA No. 3019 and Article 186 of the RPC. Respondent also contends in his
motion that the Supreme Court has no evidentiary basis to conclude that the offense involved had
not been prescribed.

ISSUE:
Whether or not private respondents are still subject to prosecution for violating RA No. 3019 and
Article 186 of the RPC because of the validity of LOI 926 and PD Nos. 961.

RULING:
The Supreme Court has ruled against the motion. The court has explained while indeed the
Ombudsman dismissed the complaint under the declaration that there is no sufficient evidence to
to establish a well-founded belief that the Anti-Graft law has been violated, such decision was
based on the premise that the acquisition of the oil mills were done in accordance with existing
laws and not because there was no evidence of any wrongdoing. If the government were to follow
the rule that valid laws should be taken into accounting into determining the violation of other
valid laws, then there would be a “blanket shield” for public officers guilty of violating the Anti
Graft law and Article 186 of the RPC.

431
CRIMINAL LAW 1 CASE DIGESTS
ATTY. LYAN DAVD JUANICO
BLOCK 1-M A.Y. 2020 - 2021

3. Romualdez v. Marcelo
G.R. No. 165510, 28 July 2006
YNARES-SANTIAGO, J.:

FACTS:
Romualdez, the petitioner herein, alleges that the Office of the Ombudsman gravely abused its
discretion in recommending the filing of 24 informations against him for violation of Section 7 of
the Anti-Graft Law, despite that the same 24 were actually dismissed when it was under the
jurisdiction of the Sandiganbayan. Petitioner raised that the Ombudsman cannot raise the already
dismissed cases and that the defense of prescription may be raised even for the first time on appeal
and thus there is no necessity for the presentation of evidence thereon. The other side defended
that the dismissal of the said informations does not preclude the petitioner from being subject to
criminal prosecution; that new informations may be filed in the event that the Ombudsman has
found probable cause to proceed; The same has also raised the stand that the prescription period
was interrupted by the (1) filing of the complaint with the Presidential Commission on Good
Government (PCGG) in 1987 and (2) the filing of the information with the Sandiganbayan in 1989
and (3) the absence of the petitioner from the Philippines from 1986 until 2000 also interrupted
the aforesaid period based on Article 91 of the Revised Penal Code.

ISSUE:
Whether or not the preliminary investigation conducted by the Ombudsman is void and if the
prescription period for the offenses charged has already been completed.

RULING:
For the first issue, the Supreme Court has held that the investigation is valid because the dismissal
of the case filed with the sandiganbayan was because of the reason that the complaint is filed by
an unauthorized party. For the second issue, the supreme court ruled that the offenses have already
been prescribed. The court explained that to count the prescription period, we have to consider the
following: (1) the period of prescription for the offense charged; (2) the time the period of
prescription starts to run; and (3) the time the prescriptive period was interrupted. In the case at
bar, the absence of the petitioner from the Philippines does not actually interrupt the prescription
period since RA No. 3019 is silent regarding this. The law should be interpreted in favor of the
accused.

432
CRIMINAL LAW 1 CASE DIGESTS
ATTY. LYAN DAVD JUANICO
BLOCK 1-M A.Y. 2020 - 2021

4. CRUZ III vs. GO


G.R. NO. 223446; JULY 28, 2006

FACTS:

By virtue of the 4 November 1993 Decision of the RTC, Branch 168, Pasig City in Criminal Case
No. 87411, respondent Rolito T. Go was convicted of murder and sentenced to suffer the penalty
of reclusion perpetua. He began serving his sentence on 30 April 1996 at the New Bilibid Prison.

On 30 July 2008, in carrying out the Resolution and Certificate of Eligibility by then Bureau of
Corrections (BuCor) Director Oscar C. Calderon, the New Bilibid Prison Classiffication Board
granted Go, along with other 24 inmates, a colonist status. Accordingly, in view of his commuted
sentence, Go filed a petition for habeas corpus on 30 January 2014, pleading for his release. He
posits that his original prison sentence

which shall expire on 31 January 2022 instead should have expired on 21 August 2013 upon
deduction of lawful and proper allowances for good conduct, colonist status, and preventive
imprisonment based on the provisions of Act No. 2489, otherwise known as "An Act Authorizing
Special Compensation, Credits, and Modification in the Sentence of Prisoners as a Reward for
Exceptional Conduct and Workmanship and for Other Purposes."

In opposition to Go's release, petitioners maintained that Go's sentence neither has expired nor was
commuted. According to petitioners, the grant of colonist status on Go did not carry with it the
automatic commutation of his sentence from the indivisible penalty of reclusion perpetua to thirty
(30) years because only the President has the power to commute a sentence. Sans the signature of
the President, any commutation is ineffectual.

ISSUE:
Whether or not the grant of colonist status on Go did not carry with it the automatic commutation
of his sentence from the indivisible penalty of reclusion perpetua to thirty (30) years because only
the President has the power to commute a sentence.

RULING:

As correctly resolved by the trial court, while only the President can commute a prison sentence,
Articles 70 and 97 of the Revised Penal Code (RPC) recognize partial reduction or commutation
of sentences by providing that "for penal penalties, the duration shall be computed for 30 years
and the allowances of good conduct must be applied on top of the [good conduct time allowance]
accorded to an inmate with a colonist status."

Accordingly, to implement the provisions of Article 97, the law has granted the Director of Prisons
the power to grant good conduct allowances. The mandate of the Director of Prisons embodied in

433
CRIMINAL LAW 1 CASE DIGESTS
ATTY. LYAN DAVD JUANICO
BLOCK 1-M A.Y. 2020 - 2021

Article 99 of the RPC is clear and unambiguous. In fact, once granted, such allowances shall not
be revoked. Article 99 of the RPC explicitly states:

Art. 99. Who grants time allowances. — Whenever lawfully justified, the Director of Prisons shall
grant allowances for good conduct. Such allowances once granted shall not be revoked. (Emphasis
supplied)

Therefore, after crediting his preventive imprisonment of nine (9) months and sixteen (16) days,
and the regular Good Conduct Time Allowance (GCTA) under Act No. 3815 and Special Credit
Time Allowance (SCTA) under Act No. 2409 granted upon him, Go has completed serving his
sentence of thirty (30) years on 21 August 2013, which he commenced to serve on 30 April 1996.

The intent and spirit of the law in affording persons the remedy of writ of habeas corpus is to
devise a speedy and effective means to relieve persons from unlawful restraint. 11 To rule
otherwise would render Article 99 of the RPC as a mere surplusage and would unduly impose
excessive imprisonment on inmates in violation of the basic right to liberty.

434
CRIMINAL LAW 1 CASE DIGESTS
ATTY. LYAN DAVD JUANICO
BLOCK 1-M A.Y. 2020 - 2021

5. People v. Bayotas
G.R. No. 102007, 2 September 1994
ROMERO, J.:

FACTS:
The case starts with a charge of Rape against the accused Bayotas. He was eventually convicted.
Pending appeal of his conviction, Bayotas passed away in the National Bilib Hospital because of
various health implications. The Supreme Court dismissed the criminal aspect of the appeal.
However, the Solicitor General contends that the death of the accused did not extinguish his civil
liability. His appeal should still be pursued for purposes of reviewing his conviction.

ISSUE:
Whether or not the death of the accused pending appeal of his conviction extinguish his civil
liability?

RULING:
The Supreme Court has ruled that the civil liability of the Bayotas is extinguished by his death by
reason of the nature of the act complained of, which is rape. Claim of civil liability survives if the
same may also be predicated on a source of obligation other than delict. As provided for under
Article 1157, these are: (1) Law, (2) Contracts, (3) Quasi-Contracts, (4) Quasi Delicts. The same
may be pursued in a separate civil action enforced against the executor or administrator of the
estate of the deceased-accused.

435
CRIMINAL LAW 1 CASE DIGESTS
ATTY. LYAN DAVD JUANICO
BLOCK 1-M A.Y. 2020 - 2021

6. Securities and Exchange Commission v. Intraport Resources Corp


G.R. No. 135808, 6 October 2008
CHICO-NAZARIO, J.:

FACTS:
Board of Directors of respondent corporation Interport Resources Corporation (IRC), acquired the
entire capital stock of GEHI. Under the Agreement, GHB, a member of the Westmont Group of
Companies in Malaysia, shall extend or arrange a loan required to pay for the proposed acquisition
by IRC of PRCI. IRC claims that a press releqase announcing the approval of the agreement was
sent through facsimile to the PSE and the SEC but the SEC was not able to receive it. SEC
Chairman n Order finding that IRC violated the Rules on Disclosure of Material Facts, in
connection with the Old Securities Act of 1936, when it failed to make timely disclosure of its
negotiations with GHB. The issue was eventually raised to the court of appeals then the same ruled
that there is no statutory authority for SEC to initiate and file any suit be they criminal, civil, or
administrative against the respondent corporation and its directors. With respect to Scetion 30 and
36 of the revised securities act, under the reasons that the same sections require enactment of
implementing rules to make them effective and that the securities regulations code did not repeal
the same sections of the revised securities act. The Court of Appeals has also ruled that the case
has been prescribed.

ISSUE:
Relevant to the module at hand, the only issue to tackle is whether or not the case filed has already
been prescribed and thus, the respondent is not criminally liable under the special law.

RULING:
The Supreme Court has held that the instant case has not yet been prescribed. Respondents
positioned themselves by stating that the prescription period applicable to offenses punished under
special laws is twelve years and that since the offense was committed in 1994, prescription already
set in as early as 2006. However, the SC disagrees with this contention. A preliminary
investigation of the case is considered an interrpuptiopn of the prescription period. The SEC has
already started investigative proceedings against the respondent as early as 1994; These, however,
were interrupted due to the preliminary injunction issued by the CA. It is only after the Supreme
Court corrects the ruling of the CA in its decision. The nature of the SEC’s investigation is
equivalent to the preliminary investigation conducted by the DOJ, hence the same investigation
has interrupted the prescription period.

436
CRIMINAL LAW 1 CASE DIGESTS
ATTY. LYAN DAVD JUANICO
BLOCK 1-M A.Y. 2020 - 2021

7. People v. Consorte
G.R. No. 194068. November 26, 2014
PEREZ, J.:

FACTS:
For the resolution of the Court is the Motion for Reconsideration of our Decision dated 9 July
2014, which affirmed the conviction of accused-appellant Benjie Consorte y Franco for the murder
of Elizabeth Palmar.

Accused-appellant raises the incredibility of his identification as the perpetrator of the crime. He
avers that despite the alleged positive identification made by Rolando Visbe, the testimony of
prosecution witness Aneline Mendoza clearly shows the impossibility of the same. Moreover,
further casting doubt on the alleged identification of accused-appellant is Visbe's unbelievable and
inconsistent statements on how such identification was made.

On a letter dated September 21, 2014, the Court was informed the accused-appellant died on July
14, 2014.

ISSUE:
Whether or not the death of the accused-appellant has an effect to his pending appeal regarding his
criminal and civil liability.

RULING:
Article 89 (1) of the Revised Penal Code is illuminating:

Art. 89. How criminal liability is totally extinguished. — Criminal liability is totally extinguished:
By the death of the convict, as to the personal penalties; and as to pecuniary penalties, liability
therefor is extinguished only when the death of the offender occurs before final judgment;

In the case at bar, accused-appellant died before final judgment, as in fact, his motion for
reconsideration is still pending resolution by the Court. As such, it therefore becomes necessary
for us to declare his criminal liability as well as his civil liability ex delicto to have been
extinguished by his death prior to final judgment.

437

You might also like